PACKRAT PRACTICE (ALL 9 - 16) - USE ME!

¡Supera tus tareas y exámenes ahora con Quizwiz!

A 30 year-old male presents with sudden onset of chills, fever, chest pain and a cough productive of greenish-brown sputum. On examination his temperature is 102 degrees F. He appears acutely ill and his respirations are shallow. Chest x-ray demonstrates left lower lobe consolidation. Which of the following findings would most likely be present on examination of his left lower lung? A. Hyperresonance B. Vesicular breath sounds C. Increased tactile fremitus D. Wheezing

(u) A. Hyperresonance is an abnormal sound as a result of air trapping as in COPD. (u) B. Vesicular breath sounds are the description of normal lung sounds. (c) C. Increased tactile fremitus occurs in the presence of fluid or a lung consolidation such as lobar pneumonia. (u) D. Wheezing is a musical noise sounding like a squeak caused by high velocity air flow through a narrow or obstructed airway.

Which of the following physical examination findings would be consistent with a pleural effusion? A. Hyperresonance to percussion B. Increased tactile fremitus C. Unilateral lag on chest expansion D. Egophony

(u) A. Hyperresonance to percussion would be suggestive of emphysema or pneumothorax. (u) B. Increased tactile fremitus would be consistent with a consolidation. (c) C. A lag on chest expansion may be seen in the presence of a pleural effusion. (u) D. The presence of egophony would be consistent with a consolidation.

A 50 year-old male with history of alcohol abuse presents with complaint of worsening dyspnea. Physical examination reveals bibasilar rales, elevated jugular venous pressure, an S3 and lower extremity edema. Chest x-ray reveals pulmonary congestion and cardiomegaly. Electrocardiogram shows frequent ventricular ectopy. Echocardiogram shows left ventricular dilatation and an ejection fraction of 30%. Which of the following is the most likely diagnosis in this patient? A. Hypertrophic cardiomyopathy B. Dilated cardiomyopathy C. Restrictive cardiomyopathy D. Tako-Tsubo cardiomyopathy

(u) A. Hypertrophic cardiomyopathy is characterized by a hyperdynamic left ventricle with asymmetric left ventricular hypertrophy. (c) B. Dilated cardiomyopathy is often caused by chronic alcohol use. It is characterized by signs and symptoms of left-sided heart failure, a dilated left ventricle and decreased ejection fraction. (u) C. Restrictive cardiomyopathy is characterized more commonly by right-sided heart failure than by left-sided heart failure. There is rapid early filling with diastolic dysfunction. Patients with restrictive cardiomyopathy will have a small thickened left ventricle and a normal or near normal ejection fraction on echocardiogram. (u) D. Tako-Tsubo cardiomyopathy (broken heart syndrome) is characterized by signs and symptoms of acute coronary syndrome, ST segment elevation on ECG and left ventricular apical dyskinesia.

Acute adrenal insufficiency is characterized by which of the following laboratory abnormalities? A. hyperglycemia and glucosuria B. hyponatremia and hyperkalemia C. hypercortisolism and hypokalemia D. neutrophilia with increased bands

(u) A. Hypoglycemia is frequently seen in adrenal crisis, while hyperglycemia and glucosuria may suggest Cushing's syndrome. (c) B. Hyponatremia and hyperkalemia characterize acute adrenal insufficiency. (u) C. Hypercortisolism and hypokalemia are common findings of Cushing's syndrome. (u) D. Eosinophilia and lymphocytosis are characteristic of adrenal crisis, not neutrophilia and increased bands.

Which of the following lesions is most characteristic of psoriasis? A. hypopigmented macules on hands and forearms B. pruritic pustules on finger webs and wrist creases C. crusting vesicles around the mouth and face D. scaling plaques on knees and elbows

(u) A. Hypopigmented macules are more characteristic of vitiligo. (u) B. Pruritic pustules are more characteristic of scabies. (u) C. Crusting vesicles around the mouth and face are more characteristic of herpes or impetigo. (c) D. Psoriasis most commonly presents with scaling plaques and papules on the scalp, elbows, forearms, lumbosacral regions, knees, hands, and feet.

A 55 year-old secretary presents with ongoing pain and numbness in her hand. These symptoms are worse at night and she must shake her hand to regain feeling in it. Which of the following physical examination signs will be present? A. Hypothenar atrophy B. Weakness of finger abduction C. Inability to maintain wrist extension against resistance D. Weakness of thumb abduction

(u) A. Hypothenar atrophy may occur with aging and disuse but it is not part of the median nerve involvement that occurs with carpal tunnel syndrome. (u) B. Finger abduction weakness is associated with ulnar nerve injury, which does not occur with carpal tunnel syndrome. (u) C. Radial nerve injury causes weakness of wrist extension and this is not part of carpal tunnel syndrome. (c) D. Median nerve injury causes weakness of thumb abduction (measured by thumb opposition strength) along with thenar atrophy. Tinel's and Phalen's signs will also be positive with carpal tunnel syndrome.

A 56 year-old man is admitted to a hospital unit for evaluation of rectal bleeding and weight loss. He has a strong family history of cancer. Soon after admission, a barium enema is scheduled. The patient refuses the "prep" because he fears x-ray radiation. He states he has had previous x-rays, but becomes frightened at the thought of an x-ray and "can't face it." The most likely diagnosis is A. hysterical personality. B. dissociative state. C. conversion reaction. D. phobic neurosis.

(u) A. Hysterical personality consists of multiple physical complaints referable to several other organ systems. (u) B. Dissociative state is precipitated by an emotional event that produces fugue, amnesia, somnambulism, multiple personality, and depersonalization. (u) C. Conversion reaction is characterized by physical symptoms in parts of the body related to psychic conflict. (c) D. Phobic neurosis is a phobic ideation of displacement where the patient transfers feelings of anxiety from the object to one that can be avoided.

A 38 year-old female has a 10 year diagnosis of migraine headaches. She had been using ergotamine to abort her headaches, but is now having one or two headaches per week that are interfering with work. Which of the following is the most appropriate preventive therapy? A. Sumatriptan (Imitrex) B. Promethazine (Phenergan) C. Propranolol (Inderal) D. Ketorolac (Toradol)

(u) A. Imitrex is not approved for preventive therapy but is used as abortive therapy. (u) B. Phenergan is an antiemetic that helps with nausea and vomiting for people who get migraines. It is not a preventive medication. (c) C. Propanolol is one beta blocker that is frequently used as a first-line prophylaxis for migraines. (u) D. Ketorolac is not indicated for scheduled, daily use as a preventive for migraines.

A 35 year-old patient has recurrent seasonal rhinitis and a history of mild asthma. Which of the following should be included for first-line management? A. Immunotherapy B. Decongestants C. Corticosteroid inhalers D. Cromolyn sodium (Intal)

(u) A. Immunotherapy (desensitization) is indicated as a last resort in patients who fail to either respond to pharmaceutical management or face prolonged exposure to known allergens. (u) B. Decongestants have a limited role in helping to decrease edema, and are generally ineffective in relieving allergic symptoms. (c) C. Regular use of corticosteroid nasal spray and oral inhalers prior to the allergy season is among the best means of preventing allergies. (u) D. Cromolyn sodium has been found to be moderately effective for some patients with allergic symptoms, but it is not usually first-line management.

A 73 year-old female patient was diagnosed 3 years ago with Alzheimer's disease and heart failure. Her 80 year-old husband can no longer help feed and bathe her or manage her medications. Which of the following support services is most appropriate for this patient? A. hospice care B. senior center C. adult day care D. skilled nursing facility

(u) A. Hospice care is a program to provide palliative care to individuals who are terminally ill and projected to die within 6 months. (u) B. Senior centers are community-based facilities that provide recreational activities and mid-day meals for functional adults. (u) C. Adult day health care is designed to meet the needs of functionally impaired adults in a community setting, but does not usually provide for the higher level of care that is required to feed or bathe a patient. (c) D. A skilled nursing facility is appropriate for patients requiring assistance with activities of daily living (i.e., feeding and bathing) and a higher level of safety assurance.

Which of the following pathogens has been linked with the development of acute myocarditis? A. Human papilloma virus B. Rotavirus C. Human Herpes Virus 6 D. Coxsackie B virus

(u) A. Human papilloma virus most commonly is associated with venereal warts and not myocarditis. (u) B. Rotavirus is primarily responsible for acute diarrhea. (u) C. Human Herpes Virus 6 is the causative organism for Roseola. (c) D. Although associated with a number of infectious and systemic diseases, myocarditis is most frequently the result of a viral infection, with Coxsackie B virus and echovirus being the most frequently implicated in the infection.

A 28 year-old female comes to the office with fever, flank pain, and dysuria for the past two days. Which of the following urinalysis results are suggestive of acute pyelonephritis? A. Hyaline casts B. Red cell casts C. White cell casts D. Granular casts

(u) A. Hyaline casts may be found in concentrated urine and are not indicative of renal disease. (u) B. Red cell casts are indicative of glomerulonephritis. (c) C. White cell casts are indicative of renal infection or inflammation as seen in pyelonephritis or interstitial nephritis. (u) D. Granular casts are non-specific and may be seen in acute tubular necrosis.

When injecting a corticosteroid into a joint to help relieve pain, which of the following would most commonly be mixed in the syringe? A. Hyaluronic acid B. Ketorolac C. Chondroitin sulfate D. Lidocaine

(u) A. Hyaluronic acid is usually injected by itself in patients with osteoarthritis. (u) B. Ketorolac is an NSAID that is given orally and not intra-articularly. (u) C. Chondroitin sulfate is taken orally and thought to improve symptoms in osteoarthritis. (c) D. 1% lidocaine and corticosteroids are most commonly used together for joint injections.

Which of the following mechanisms of action is most commonly associated with meniscal tears? A. Hyperextension B. Axial loading and rotation C. Hyperflexion D. Valgus force to the lateral knee

(u) A. Hyperextension injuries usually result in ACL and PCL injuries. (c) B. Axial loading and rotation most likely result in meniscal injuries. (u) C. Hyperflexion injuries result in PCL injuries. (u) D. Valgus force to the lateral knee more than likely results in medial collateral ligament injuries.

A woman brings her 3 month-old son to the clinic. Upon examination, it is noted he has a round face, a large protruding tongue, dry skin, an umbilical hernia, and his weight gain is below average. He appears apathetic and the mother says the infant is usually constipated. Which of the following is the most likely diagnosis? A. hyperparathyroidism B. nephrotic syndrome C. phenylketonuria D. hypothyroidism

(u) A. Hyperparathyroidism results in abnormal bone development, nausea, vomiting, and anorexia. (u) B. Nephrotic syndrome is associated with proteinuria, with resultant edema and ascites. Anorexia, abdominal pain and diarrhea are common findings. (u) C. Phenylketonuria is associated with mental retardation, motor deficits, and convulsions. (c) D. Congential hypothyroidism presents gradually, and at 3-6 months findings include poor appetite and feeding, sluggishness, constipation, enlarged abdomen and umbilical hernia, enlarged tongue, and the child does not meet developmental milestones.

A 45 year-old male patient presents to the office with a complaint of dizziness and vertigo that occurs suddenly when he rolls out of bed in the morning. He denies previous illness or any medical problems. On physical exam you note the presence of lateral nystagmus after a few second latency period. What is your recommendation for this patient? A. Low dose diazepam B. Repositioning maneuvers C. Diuretics and a low salt diet D. MRI of the brain

(u) A. Low dose diazepam is used to treat acute vertigo. (c) B. This patient has benign paroxysmal positional vertigo (BPPV) and repositioning maneuvers are recommended to move endolymphatic debris out of the posterior semicircular canal. (u) C. Diuretics and a low salt diet are used to treat Meniere's disease. (u) D. An MRI of the brain should be ordered in a patient in whom a central etiology of vertigo is suspected.

Which of the following is an expected finding in a patient with a diagnosis of an arterial embolism? A. lower extremity edema B. stasis dermatitis C. palpable cord D. pulselessness

(u) A. Lower extremity edema is commonly associated with venous insufficiency, not arterial embolism. (u) B. Stasis dermatitis is commonly seen in patients with venous insufficiency, not arterial embolism. (u) C. A palpable cord is commonly found in a patient with a DVT, not arterial embolism. (c) D. Pulselessness is a sign of acute ischemia secondary to arterial embolism.

A 16 year-old female comes to the office because her acne has become more severe and she would like to discuss treatment. She was originally prescribed topical clindamycin gel 1 year ago to treat open and closed comedones on her face. Despite regular use of the medication, she now has several comedones and papulopustules across her nose, cheeks, chin and forehead. Which of the following would be an appropriate additional treatment for moderate acne? A. Intralesional steroids B. Oral doxycycline C. Benzoyl peroxide gel D. Metronidazole cream

(u) A. Intralesional steroids can be used as an adjunct treatment for large cystic lesions. (c) B. For moderate acne, oral antibiotics are added to the topical medications treatment regimen. (u) C. For mild acne, benzoyl peroxide gel can be used alone or in combination with topical antibiotics and/or topical retinoids. (u) D. Metronidazole cream is a treatment for rosacea.

A 68 year-old woman comes to the office for evaluation of urinary incontinence. For the past few months, she has had an intense urgency to urinate, followed by leakage of urine. Which of the following is the most appropriate intervention for this patient? A. Intravaginal estrogen cream (Premarin cream) B. Terazosin (Hytrin) C. Oxybutynin (Ditropan) D. Intravaginal miconazole cream (Monistat cream)

(u) A. Intravaginal estrogen is indicated for women with vaginal atrophy that is contributing to stress incontinence. (u) B. Terazosin is indicated for men with urinary incontinence caused by urethral obstruction due to benign prostatic hyperplasia. (c) C. Urge incontinence is caused by detrusor overactivity that causes urinary leakage. Antimuscarinics, such as oxybutynin, are prescribed to relax the pelvic muscles. (u) D. Duloxetine is effective in reducing the number of stress incontinence episodes in women.

You are counseling a newly diagnosed type 2 diabetic about the need for referral to ophthalmology for a dilated funduscopic exam. Which of the following best describes the rationale for referral? A. He can wait until next year when he goes to get his refraction B. He does not need to see an ophthalmologist if his Hemaglobin A1C is < 6.0 C. Retinopathy is present in 20% of patient with type 2 Diabetes Mellitus at time of diagnosis D. Your non-dilated exam can substitute for this referral

(u) A. In type 2 diabetes, retinopathy is present in up to 20% of patients at diagnosis and may be the presenting feature. Eye examination for vision usually does not require a dilated eye exam, and refraction is not calculated with ophthalmoscope or direct visualization but by refractometer which does not examine the retina where retinopathy occurs. (u) B. See C for explanation. (c) C. In type 2 diabetes, retinopathy is present in up to 20% of patients at diagnosis and may be the presenting feature. (u) D. See C for the explanation.

Which of the following is considered a risk factor for the development of malignant melanoma? A. male gender B. inability to tan C. Japanese ethnicity D. brown-haired individuals

(u) A. Incidence of malignant melanoma is equal in males and females. (c) B. Inability to tan and propensity to burn are risk factors for developing malignant melanoma. (u) C. Malignant melanomas are most common in Caucasians and are rarely seen in the Japanese population. (u) D. Red hair and freckling is one of the major risk factors for malignant melanoma, not brown hair.

Which of the following mediators is responsible for initiating the urticaric response? A.Cyclic AMP B. Prostaglandins C. Prednisone D. IgE

(u) A. Increased levels of Cyclic AMP inhibit the histamine response. (u) B. Prostaglandins inhibit the release of histamine. (u) C. Prednisone is used to treat urticaria. (c) D. IgE triggers the release of histamine from mast cells that leads to urticaria.

A 73 year-old male with chronic renal failure due to longstanding hypertension and diabetes is being discharged from the hospital today. Which dietary recommendation should be made to prevent further complications and progression of his renal disease? A. Increase salt intake B. Restrict dietary protein C. Add a daily potassium supplement D. Decrease gluten consumption

(u) A. Increased salt intake can lead to volume overload in patients with chronic renal disease. (c) B. Restricting protein in the diet may slow the progression of chronic kidney disease. (u) C. Hyperkalemia can develop in patients with chronic renal failure who are taking supplements or have a high potassium diet. (u) D. Decreasing gluten consumption has no effect on renal failure.

A patient with Type 2 diabetes uses a mixture of NPH and regular insulin twice daily. She consistently has midafternoon bouts of hypoglycemia, despite eating her meals as scheduled. Which modification is most appropriate to reduce her mid-afternoon hypoglycemic events? A. Increase carbohydrate content of lunchtime meal B. Increase fat content of breakfast meal C. Reduce morning dose of NPH insulin D. Reduce morning dose of regular insulin

(u) A. Increasing carbohydrate content of lunchtime meal would be inappropriate if the glycemic control is otherwise normal; might promote weight gain. (u) B. Increasing fat content of breakfast meal would not aid in symptoms as it would slow glucose absorption and potentially worsen the hypoglycemia. (c) C. Reducing the morning dose of NPH insulin would be most appropriate as it exerts its greatest effect on the noontime meal. (u) D. Reducing the morning dose of regular insulin would have more effect on noon blood sugar control.

A 9 year-old patient presents for follow up of his allergic rhinitis symptoms. He continues to complain of nasal congestion, sneezing, rhinorrhea, and eczema despite avoidance therapy and treatment with oral cetirizine (Zyrtec) and nasal flunisolide (Nasarel). Examination reveals pale, boggy nasal mucosa and eczema of the face and lower extremities. Which of the following is the most appropriate treatment at this time? A. Ipratropium bromide B. Montelukast C. Immunotherapy D. Cromolyn sodium

(u) A. Ipratropium bromide does not alleviate the sneezing and pruritus symptoms this patient is experiencing. (u) B. Montelukast is less effective than intranasal steroids in the management of allergic rhinitis. (c) C. Immunotherapy is recommended in patients with severe allergic rhinitis who fail to respond to drug therapy and allergen avoidance. This patient has failed avoidance therapy, as well as antihistamines and intranasal corticosteroids. (u) D. Cromolyn sodium is much less potent than intranasal steroids and will likely not improve the patient's symptoms.

Which of the following is a common presenting clinical manifestation of a patient with interstitial lung disease? A. Early inspiratory crackles B. Progressive dyspnea on exertion C. Productive cough with copious sputum D. Decreased breath sounds with hyperresonant percussion

(u) A. Late, not early, inspiratory crackles are associated with interstitial lung disease. (c) B. Patients with interstitial lung disease commonly present with progressive dyspnea on exertion and a cough with minimal sputum production. (u) C. A productive cough of copious amounts of sputum is most typical of a patient with chronic bronchitis. (u) D. Physical examination findings of decreased breath sounds with hyperresonant percussion is consistent with a diagnosis of chronic obstructive lung disease.

Which of the following is the best radiologic study to image the pituitary gland? A. Lateral skull films B. Ultrasound C. Magnetic resonance imaging (MRI) D. Computed tomography (CT) scan

(u) A. Lateral skull films will not show the pituitary gland due to location. (u) B. It is not possible to visualize the pituitary gland by ultrasound due to physical location. (c) C. The pituitary gland and specifically pituitary tumors are best visualized by an MRI. (u) D. CT scan is not the preferred test for imaging the pituitary gland.

A 28 year-old patient presents with complaint of chest pain for two days. The patient describes the pain as constant and sharp. It is worse with lying down, better with sitting up and leaning forward. Vital signs are BP 120/80, HR 80, regular, RR 14 and Temperature 100.1 degrees F. Which of the following would you expect to find on physical examination? A. lower extremity edema B. carotid bruit C. pericardial friction rub D. splinter hemorrhages

(u) A. Lower extremity edema is seen with heart failure or venous insufficiency, not pericarditis. (u) B. Carotid bruits are associated with carotid artery stenosis, not pericarditis. (c) C. This patient has signs and symptoms of pericarditis. A pericardial friction rub is characteristic of acute pericarditis. (u) D. Subungual (splinter) hemorrhages are characteristic of infective endocarditis, not pericarditis.

A 7-year-old child with a history of type 1 diabetes mellitus for 3 years presents for routine follow-up. The mother states that the child has been having nightmares and night sweats. Additionally, his average morning glucose readings have risen from an average of 100 mg/dL to 145 mg/dL over the past week. This child is most likely experiencing A. a growth spurt. B. emotional problems. C. the Somogyi effect. D. the dawn phenomenon.

(u) A. Nightmares and night sweats are not associated with growth spurts. (u) B. With this limited history, it is impossible to label the child as emotionally unstable. (c) C. This refers to nocturnal hypoglycemia, which stimulates counter-regulatory hormone release resulting in rebound hyperglycemia. (u) D. This refers to an early morning rise in plasma glucose due to reduced tissue sensitivity to insulin between 5 AM and 8 AM. It is not associated with nightmares and night sweats.

A 24 year-old patient presents after a recent vacation. He complains of left ear pain. Physical examination reveals an inflamed external auditory canal and the tympanic membrane can not be visualized. Which of the following is the most appropriate treatment? A. Oral penicillin B. Ciprodex otic drops C. VoSoL drops D. Oral prednisone

(u) A. Oral antibiotics are not indicated in acute otitis externa. (c) B. Topical steroid/antibiotic drops are most useful in acute bacterial otitis externa. (u) C. VoSoL drops are utilized as preventive treatment of otitis externa, but are not effective for treatment. (u) D. Oral prednisone is not indicated in the treatment of acute otitis externa.

A 60 year-old patient with a history of tobacco and alcohol abuse presents for a routine physical. Which of the following physical examination findings is suspicious for oral carcinoma? A. Painful creamy-white patches overlying erythema B. Red smooth surface tongue C. Small vesicles on an erythematous base D. White lesion that cannot be removed by rubbing

(u) A. Oral candidiasis is characterized by painful creamy white patches overlying erythema. (u) B. Red smooth tongue is characteristic of glossitis which is most commonly associated with nutritional deficiencies. (u) C. Small vesicles on erythematous bases are characteristic of herpetic stomatitis. (c) D. Leukoplakia, premalignant lesions, are white lesions that can not be removed by rubbing.

A 46 year-old G4P4 African American female presents to the clinic complaining of heavy and prolonged menstrual flow over the past 6 months. Gynecological history includes menarche age 12 and LMP 3 weeks prior. Pelvic exam reveals a 14-week size, irregular uterus. Pelvic ultrasound shows the presence of a large intramural fibroid with normal endometrial lining. Which of the following is the most appropriate management for this patient? A. Oral contraceptive pill B. Levonorgestrel-releasing IUD C. Hysterectomy D. Myomectomy

(u) A. Oral contraceptive pills do not treat the leiomyomas.. (u) B. A levonorgestrel-releasing IUD works for treatment of menorrhagia related to multiple smaller leiomyomas (c) C. Large leiomyomas are the most common indication for hysterectomy in this age group. (u) D. Myomectomy is not an option for this large of an intramural fibroid.

Which of the following is an indication for a pediatric patient to receive the 23-valent polysaccharide vaccine (Pneumovax)? A. Children at any age with a history of asthma B. All children at 2,4,6 and 12-18 months of age C. All children at 12-23 months of age in a two dose series D. Children age 24-59 months at high risk for invasive pneumococcal disease

(u) A. Pediatric patients with cystic fibrosis, not asthma, are included in the indications for vaccination with Pneumovax, however they must be at least 2 years old. (u) B. The 7-valent pneumococcal conjugate vaccine (Prevnar) is currently recommended to be given to children under the age of two on the schedule outlined. (u) C. While a two dose series is recommended for appropriate pediatric patients that receive Pneumovax, the recommended timing between doses is 3-5 years. (c) D. Pneumovax is licensed for use in children over the age of 23 months and is indicated for all pediatric patients at increased risk for pneumococcal disease.

A 13 year-old female presents to the office with right knee and thigh pain and the inability to bear weight since waking yesterday morning. The mother states the child had a fever of 100.9 degrees F this morning and continues to be non-weight bearing. Examination reveals a warm, erythematous, swollen knee. Which of the following tests would be most beneficial in the diagnosis and treatment of this patient? A. Plain x-ray of the femur B. Antistreptolysin O titer C. Magnetic resonance imaging (MRI) of the femur D. Culture of joint aspirate

(u) A. Plain x-rays may aid in diagnostic suspicion, usually after a week or two, but do not aid in treatment choice. (u) B. ASO titer is not indicated in a patient with osteomyelitis. (u) C. MRI will detect the early inflammation of osteomyelitis but does not aid in the treatment choice. (c) D. A culture of the joint fluid will confirm the diagnosis and offer information regarding infectious agent.

Polysaccharide pneumococcal vaccine should be given to a patient with which of the following diseases? A. asthma B. sickle cell disease C. acute renal failure D. osteoarthritis

(u) A. Pneumococcal vaccination is not currently recommended for patients with asthma, acute renal failure, or osteoarthritis. (c) B. Patients with sickle cell disease are functionally asplenic and, therefore, require pneumococcal vaccination to prevent invasive disease. (u) C. See A for explanation. (u) D. See A for explanation.

A 13 year-old male with known cystic fibrosis presents to the emergency department, accompanied by his parents, with increased coughing, wheezing and low grade fever. Rales are audible on auscultation of the lungs. Treatment should target which of the following organisms? A. Pneumocystis jiroveci B. Haemophilus influenza C. Mycoplasma pneumoniae D. Pseudomonas aeruginosa

(u) A. Pneumocystis jiroveci is common in HIV patients, not patients with cystic fibrosis. (u) B. Staphylococcus aureus and Haemophilus Influenzae often begin in the first few months of life even in asymptomatic infants with cystic fibrosis, but are less common in older children. (u) C. Mycoplasma pneumoniae is a less common cause of pneumonia in patients with cystic fibrosis. (c) D. Pseudomonas aeruginosa is the most predominant pathogen in patients with cystic fibrosis.

A 2 year-old child is brought to the office because of a cough and a fever of 102 degrees F for 2 days. The physician assistant notes the presence of hoarseness, a barking cough, and stridor. The ears and nose exam are unremarkable. Auscultation of the chest reveals decreased breath sounds without crackles or expiratory wheezes. Which of the following would be the initial diagnostic impression? A. pneumonia B. bronchiolitis C. croup D. asthma

(u) A. Pneumonia would not produce the stridor noted on physical exam. Pneumonia would more than likely produce crackles, also not noted in this patient. (u) B. Bronchiolitis would produce inspiratory wheezes which are absent in this patient. (c) C. Hoarseness, inspiratory stridor, and a barking cough are classic signs of croup, all of which are noted in this patient. (u) D. Asthma would not normally present with fever and stridor.

The neuromuscular hallmark of polymyalgia rheumatica is A. asymmetric muscle pain and stiffness. B. distal paresthesias. C. muscle weakness. D. proximal muscle pain and stiffness.

(u) A. Polymyalgia rheumatica is associated with pain and stiffness but typically occurs in a symmetric distribution. (u) B. Peripheral neurons are unaffected by polymyalgia rheumatica and is not associated with neuropathic symptoms. (u) C. Although patients may appear to have proximal muscle weakness, this is usually related to pain and is not true measurable weakness. (c) D. Proximal symmetric muscle pain and stiffness, particularly involving the shoulder, neck and pelvic girdle, is the musculoskeletal hallmark of polymyalgia rheumatica.

An afebrile 22 year-old female presents with 2 days of a painful rectal mass which worsens with defecation. She denies any blood with bowel movements. She admits to ongoing constipation and generally poor diet of fast food. She is otherwise healthy. On examination there is a tender, swollen, bluish, ovoid mass visible at the anal margin. Which of the following is the most likely diagnosis? A. Rectal prolapse B. Thrombosed hemorrhoid C. Internal Hemorrhoid D. Rectal polyp

(u) A. Rectal prolapse shows radiating folds and is typically painless. (c) B. A thrombosed hemorrhoid causes acute pain increased with defecation and sitting. (u) C. Internal hemorrhoids are an enlargement of the normal vascular cushions located above the pectinate line. They are not usually palpable and might cause bright red bleeding during defecation. (u) D. Polyps are soft and may be difficult or impossible to feel. Proctoscopy is usually required for diagnosis.

Which of the following typical findings would be revealed during a sigmoidoscopy on a patient with Crohn's disease of the intestine? A. Rectal pseudopolyps B. Diffuse ulceration and bleeding C. Sheets of WBCs with inflamed mucosa D. Intermittent longitudinal mucosal ulcers and fissures

(u) A. Rectal pseudopolyps are associated with ulcerative colitis rather than Crohn's. (u) B. Diffuse ulcerations and bleeding are more characteristic of ulcerative colitis than Crohn's disease. (u) C. Sheets of WBCs or "pseudomembranes" can be detected in patients with pseudomembranous colitis. (c) D. Ulcerations tend to be linear with transverse fissures in Crohn's disease. These skip lesions are common with Crohn's disease.

A 54 year-old woman with history of lupus comes to the office with increasing significant peripheral edema over the past four days. Laboratory findings include marked proteinuria, hypoalbuminemia and hyperlipidemia. Which of the following diagnostic studies is the best for determining the cause of the proteinuria? A. Renal ultrasound B. Renal biopsy C. Cystoscopy D. Computed tomography scan

(u) A. Renal ultrasound may identify hydronephrosis from a stone or other source of obstruction. (c) B. Renal biopsy is performed in adults with new onset of nephrotic syndrome to determine the cause of the proteinuria and to guide management decisions. (u) C. Cystoscopy can be used in the evaluation of hematuria to assess for bladder or urethral neoplasm, benign prostatic hyperplasia, and radiation or chemical cystitis. (u) D. CT scanning may identify neoplasms of the kidney or ureter as well as benign conditions such as urolithiasis.

A 56 year-old male with a 40 pack-year smoking history presents complaining of progressive shortness of breath. Spirometry reveals an FEV1 of 2 L (40% of predicted), an FVC of 4 L (80% of predicted) and an FEV1/FVC of 50%. These findings are most consistent with A. sarcoidosis. B. chronic bronchitis. C. interstitial lung disease. D. congestive heart failure.

(u) A. Sarcoidosis, interstitial lung disease and congestive heart failure most commonly produce a restrictive pattern on spirometry with a reduction in forced expiratory volume in one second (FEV1) and forced vital capacity (FVC) secondary to the decrease in total lung capacity (TLC), but the FEV1/FVC ratio is usually normal or increased, not decreased. (c) B. The reduced FEV1 and FEV1/FVC is characteristic of an obstructive pattern that is seen in chronic obstructive pulmonary diseases, such as chronic bronchitis. (u) C. See A for explanation. (u) D. See A for explanation.

A young child is brought to the clinic because the mother noticed a rash while bathing the child. There is a very red slightly raised eruption on the child's face across both cheeks. The child has been in good health and does not appear ill today. The most likely diagnosis is A. scarlet fever. B. rubella. C. roseola. D. erythema infectiosum.

(u) A. Scarlet fever is associated with fever, pharyngitis, and "sandpaper rash" over face, neck, trunk, and extremities. (u) B. Rubella presents with round or oval maculopapular lesions that begin on the neck or face and spread to the extremities. (u) C. Roseola presents with a high fever with few symptoms, followed by the appearance of a macular rash over the trunk and neck. (c) D. Erythema infectiosum is generally asymptomatic, presenting with red papules on the face that coalesce to give a "slapped cheek" appearance.

: (27) 64 161. Clinical Intervention/Urology/Renal According to Kidney Disease Outcomes Quality Initiative (KDOQI) guidelines, dialysis should be started in a nondiabetic patient when the glomerular filtration rate (GFR) reaches 10 mL/min or less, or when serum creatinine rises to which of the following levels? A. 10 mg/dL B. 8 mg/dL C. 6 mg/dL D. 4 mg/dL

(u) A. See B for explanation. (c) B. According to KDOQI guidelines dialysis should be started when a patient has a GFR of 10 mL/min or serum creatinine of 8 mg/dL. In diabetic patients the corresponding recommended numbers are GFR of 15 mL/min or serum creatinine of 6 mg/dL. (u) C. See B for explanation. (u) D. See B for explanation.

An otherwise healthy college student presents with complaints of cough, malaise, fever, yellow-green nasal discharge and headache for the past 3 weeks. She complains of facial pain over the cheeks and while chewing. On examination, the nasal turbinates are erythematous and edematous nearly obstructing the nares bilaterally. She has not been previously evaluated. What is the most appropriate antibiotic in this patient? A. Amoxicillin/clavulanate (Augmentin) B. Trimethoprim/sulfamethoxazole (Bactrim) C. Levofloxacin (Levaquin) D. Moxifloxacin (Avelox)

(u) A. See B for explanation. (c) B. Acute bacterial rhinosinusitis (sinusitis) is associated with S. pneumoniae, H. Influenzae, and less commonly S. Aureus or Moraxella catarrhalis. First line therapies for bacterial sinusitis include amoxicillin, Bactrim, or doxycycline. Second line therapy includes Augmentin, Moxifloxacin, or Telithromycin. (u) C. See B for explanation. (u) D. See B for explanation.

Which of the following otherwise healthy persons should receive annual influenza immunization? A. 15 year-old high school student B. 23 year-old physician assistant student C. 32 year-old seminary student D. 48 year-old doctoral (history) student

(u) A. See B for explanation. (c) B. Adults over 65, and adults and children who are at high risk for contracting influenza or suffering complications should be immunized. Health care providers, including health care students who come in contact with patients with influenza and who can become carriers and infect others should be immunized. (u) C. See B for explanation. (u) D. See B for explanation.

: (27) 156. Scientific Concepts/Obstetrics/Gynecology Which of the following is required for the initiation and maintenance of the menstrual cycle? A. Prolactin release by the anterior pituitary B. Pulsatile secretion of gonadotropin-releasing hormone (GnRH) C. Follicular phase of variable length D. Estrogen secretion by the ovary

(u) A. See B for explanation. (c) B. Gonadotropin-releasing hormone is secreted in a pulsatile fashion from the arcuate nucleus of the hypothalamus. The pulsatile secretion of GnRh stimulates and modulates pituitary gonadotropin secretion. (u) C. See B for explanation. (u) D. See B for explanation.

Intraarticular injection of hyaluronic acid has been approved for treatment of patients with which of the following conditions? A. Rheumatoid arthritis of the knee B. Osteoarthritis of the knee C. Olecranon bursitis D. Gouty arthritis

(u) A. See B for explanation. (c) B. Intraarticular injection of hyaluronic acid has been approved recently for treatment of patients with osteoarthritis of the knee that have failed other therapies. Although the onset of action of this medication is slower than injected glucocorticoids, it has a sustained length of activity outlasting the injected glucocorticoids. (u) C. Treatment of olecranon bursitis may include incision and drainage but not hyaluronic acid injections. (u) D. Gout is treated with anti-inflammatory medications.

: (8) 43 102. Health Maintenance/Cardiology A patient 10 years status-post deep vein thrombosis presents with chronic lower extremity edema. Which of the following prevention strategies is most appropriate for this patient? A. Catheter directed thrombolysis B. Compression stockings C. Warfarin (Coumadin) therapy D. Vein stripping

(u) A. See B for explanation. (c) B. Irreversible tissue damage changes and associated complications in the lower legs can be minimized through early and aggressive anticoagulation of acute DVT to minimize the valve damage and by prescribing stockings if chronic edema develops in subsequent years. (u) C. See B for explanation. (u) D. See B for explanation.

: (8) 152. Health Maintenance/Gastrointestinal/Nutritional An asymptomatic 30 year-old male with no significant past medical history presents to the office. He is concerned about his father being diagnosed with colon cancer at the age of 60. Which of the following is the most appropriate health counseling for this patient? A. Flexible sigmoidoscopy now B. Colonoscopy at age 40 C. Flexible sigmoidoscopy age 35 D. Colonoscopy age 50 61

(u) A. See B for explanation. (c) B. It is recommended that patients with a single first-degree relative with colon cancer diagnosed at or after age 60 should begin screening at age 40 preferably with a colonoscopy. (u) C. See B for explanation. (u) D. See B for explanation.

: (9) 49. Scientific Concepts/Pulmonology An intoxicated patient is brought to the ED with a two day history of acute onset of severe fever, chills and chest pain with frequent cough and dyspnea. His past history is significant for a 30 pack year history of tobacco use, alcoholism for the past 20 years and hypertension. A chest radiograph reveals a left lower lobe infiltrate. Which of the following is the most likely causative organism? A. Chlamydia B. Klebsiella C. Legionella D. Mycoplasma

(u) A. See B for explanation. (c) B. Klebsiella is the most likely organism based on the rapidity of onset, the patient's history of alcoholism and the chest radiograph findings. Chlamydia and Mycoplasma typically have a more subacute onset. Legionella's presentation can be variable but typically results in patchy infiltrates on radiograph. (u) C. See B for explanation. (u) D. See B for explanation.

What laboratory test must be monitored frequently in patients who are taking clozapine (Clozaril)? A. Thyroid stimulation hormone B. White blood cell count C. Platelet count D. Aspartate aminotransferase

(u) A. See B for explanation. (c) B. Leukopenia, granulocytopenia, and agranulocytosis occur in approximately 1% of patients on this medication, clozapine should not be dispensed without proof of monitoring. (u) C. See B for explanation. (u) D. See B for explanation.

A 26 year-old male was lifting a heavy object two weeks ago when he felt a sudden onset of low back pain. He describes pain in the low mid back at the belt line aggravated with movement. Radicular symptoms are noted in the left buttock down the leg to the dorsal aspect of the foot. He denies any urine or bowel complaints His examination demonstrates an inability to stand on his toes and a positive straight leg raise. Which of the following is most appropriate diagnostic study in this patient? A. Computed tomography (CT) B. Magneti resonance imaging (MRI) C. Discography D. Electromyelogram

(u) A. See B for explanation. (c) B. MRI is the diagnostic study of choice in a patient with suspected disc herniation. (u) C. See B for explanation. (u) D. See B for explanation.

The most common arrhythmia encountered in patients with mitral stenosis is A. atrial flutter. B. atrial fibrillation. C. paroxysmal atrial tachycardia. D. atrio-ventricular dissociation.

(u) A. See B for explanation. (c) B. Mitral stenosis leads to enlargement of the left atrium, which is the major predisposing risk factor for the development of atrial fibrillation. (u) C. See B for explanation. (u) D. See B for explanation.

: (28) 99. Diagnosis/Pulmonology A 22 year-old college student presents with a one-day history of left-sided, sharp and non-radiating chest pain. He describes a recent cold with dry cough, sore throat and rhinitis starting five days ago but all of the symptoms are now much improved. He denies fever, palpitations or dyspnea. Examination reveals normal heart sounds, clear lung fields and no rash or palpable areas of tenderness to the chest wall. A chest radiograph is normal. Which of the following is the most likely diagnosis? 42 A. Angina pectoris B. Costochondritis C. Pleuritis D. Herpes zoster

(u) A. See C for explanation. (u) B. See C for explanation. (c) C. A young, otherwise healthy patient with a recent viral syndrome followed by acute, sharp, focal chest pain not worsened by chest wall palpation is very commonly inflammation of the pleuritic membranes. (u) D. See C for explanation.

Which of the following is an absolute contraindication to thrombolytic therapy in a patient with an acute ST segment elevation myocardial infarction? A. history of severe hypertension presently controlled B. current use of anticoagulation therapy C. previous hemorrhagic stroke D. active peptic ulcer disease

(u) A. See C for explanation. (u) B. See C for explanation. (c) C. Absolute contraindications to thrombolytic therapy include a previous hemorrhagic stroke, a stroke within one year, a known intracranial neoplasm, active internal bleeding, and a suspected aortic dissection. Severe, but controlled hypertension, use of anticoagulation, and active peptic ulcer disease are relative contraindications in which the risk/benefit ratio must be weighed in each patient. (u) D. See C for explanation.

: (8) 4. History & Physical/Dermatology A 46 year-old male is brought to the emergency department after sustaining burns to his entire right arm and leg in a house fire. Which of the following percentages best classifies the extent of his burns? A. 12 B. 18 C. 27 D. 36

(u) A. See C for explanation. (u) B. See C for explanation. (c) C. According to the "rule of nines" the percentages can be calculated as follows: entire arm = 9% of body surface area (BSA). Anterior surface of each leg = 9% BSA, posterior surface of each leg = 9% BSA. The patient has a total of 27% BSA with burns. (u) D. See C for explanation.

: (30) 216. Diagnostic Studies/Pulmonology Which of the following is the most likely chest radiograph presentation of adenocarcinoma of the lung? A. Hilar mass B. Lobar infiltrate C. Peripheral lesion D. Pneumothorax

(u) A. See C for explanation. (u) B. See C for explanation. (c) C. Adenocarcinoma most commonly presents as a solitary, peripheral lesion on chest radiograph. It is sometimes associated with an ipsilateral pleural effusion. (u) D. See C for explanation.

: (27) 189. Clinical Intervention/Psychiatry/Behavioral Medicine A 21 year-old female graduate student is brought to the ED by her friend who found the patient sitting in her car in the garage with the motor running. The friend reports that since she broke up with her fiance 2 months ago the patient has become increasingly withdrawn, has had difficulty concentrating, and has lost 10 pounds. What is your recommendation for treatment of this patient? A. Outpatient treatment with medication B. Outpatient treatment with medication and psychotherapy C. Inpatient treatment with medication and psychotherapy D. No treatment is necessary

(u) A. See C for explanation. (u) B. See C for explanation. (c) C. Admission is indicated after suicide attempt or after aborted suicide attempt if patient still at increased risk for suicide. (h) D. See C for explanation.

A disulfiram-like reaction may occur when alcohol is consumed with which of the following antibiotics? A. Ampicillin B. Levofloxacin (Levaquin) C. Metronidazole (Flagyl) D. Erythromycin

(u) A. See C for explanation. (u) B. See C for explanation. (c) C. Agents that may elicit a disulfiram-like reaction include calcium carbamide, hypoglycemic sulfonylureas, chloramphenicol, furazolidone, metronidazole, quinacrine, and certain cephalosporins. (u) D. See C for explanation.

A patient is started on a new antipsychotic medication for his disorder. Three days later he develops altered consciousness, lead-pipe rigidity, diaphoresis and catatonia. Vital signs reveal respiratory rate of 20, temperature of 105.6 degrees F, and pulse oximetry of 95% room air. Which of the following would be the most appropriate initial intervention in this patient? A. Immediate oral SSRI's and Benadryl B. Supportive care with fluids and antipyretics C. IV antibiotics and naloxone D. Intubation and mechanical ventilation

(u) A. See B for explanation. (c) B. Neuroleptic malignant syndrome is characterized by extrapyramidal signs, blood pressure changes, altered consciousness, hyperpyrexia, muscle rigidity, dysarthria, cardiovascular instability, fever, pulmonary congestion and diaphoresis. Controlling fever and fluid support are the best initial management. With a normal pulse oximetry mechanical ventilation is not indicated. (u) C. See B for explanation. (u) D. See B for explanation.

er to a hand surgeon C. Prescribe a nonsteroidal antiinflammatory D. Apply a splint for 4 weeks

(u) A. See B for explanation. (c) B. Nonsurgical treatment is ineffective in reversing or halting Dupuytren contracture. The primary indication for surgery is a fixed contracture of more than 30 degrees at the metacarpophalangeal joint or any degree of flexion contracture at the proximal interphalangeal joint. (u) C. See B for explanation. (u) D. See B for explanation.

Which of the following is the most common cause of ascites? A. Nephrotic syndrome B. Portal hypertension C. Bacterial peritonitis D. Uterine malignancy

(u) A. See B for explanation. (c) B. Over 80% of patients with ascites have portal hypertension secondary to chronic liver disease. Infection, malignancy, and nephrotic syndrome are common causes of non-portal hypertensive ascites. (u) C. See B for explanation. (u) D. See B for explanation.

: (28) 38 88. Clinical Therapeutics/Orthopedics/Rheumatology A 40 year-old female is starting infliximab (Remicade) for the management of rheumatoid arthritis. This patient should be screened for which of the following disease processes prior to initiating this medication? A. Osteoporosis B. Tuberculosis C. Hypothyroidism D. Renal failure

(u) A. See B for explanation. (c) B. Patients on TNF inhibitors are at greater risk for developing infections, specifically TB. Screening should be done prior to initiation. (u) C. See B for explanation. (u) D. See B for explanation.

A patient is diagnosed with hypothyroidism and started on levothyroxine (Synthroid). When is it ideal to recheck the TSH level? A. Two weeks B. Three to four weeks C. Two to three months D. Six months

(u) A. See B for explanation. (c) B. Patients taking levothyroxine (Synthroid) for thyroid replacement will achieve peak levels of T4 within three to four weeks. The half-life of levothyroxine is 7 days so it will take three to four weeks in order to achieve a steady state which means that TSH levels or T4 levels should not be checked sooner than this recommended time of three to four weeks. (u) C. See B for explanation. (u) D. See B for explanation.

: (28) 121. Diagnosis/Cardiology A 65 year-old male 5 days status-post myocardial infarction is evaluated for anterior chest pain. The pain improves with sitting up. The patient has associated fever, leukocytosis and a pericardial friction rub. Which of the following is the most likely diagnosis? A. Tako-tsubo cardiomyopathy B. Dressler syndrome C. Rupture of papillary muscles D. Recurrent myocardial infarction

(u) A. See B for explanation. (c) B. Pericarditis may occur 2-5 days after infarction due to an inflammatory reaction to transmural myocardial 50 necrosis. This is known as postmyocardial infarction pericarditis or Dressler syndrome. (u) C. See B for explanation. (u) D. See B for explanation.

: (27) 35 80. Scientific Concepts/Neurology A 58 year-old truck driver is found to have a positive Romberg test and loss of vibratory sensation in his lower extremities. What anatomical structure is the most likely affected? A. Cerebellum B. Posterior column C. Sensory cortex D. Vestibular apparatus

(u) A. See B for explanation. (c) B. Posterior column is responsible for vibratory sensation and proprioception. The Romberg test is a general test of proprioception with disease of the cerebellum, vestibular apparatus or posterior column being the most likely source. (u) C. The bilateral lower extremity findings make a lesion of the sensory cortex unlikely. (u) D. See B for explanation.

When prescribing loop diuretics which of the following electrolytes should be most closely monitored? A. Calcium B. Potassium C. Sodium D. Chloride

(u) A. See B for explanation. (c) B. Potassium along with magnesium should be monitored when prescribing loop diuretics. (u) C. See B for explanation. (u) D. See B for explanation.

: (7) 148. Scientific Concepts/Obstetrics/Gynecology An 18 year-old female presents with dysmenorrhea associated with nausea and diarrhea for the last 3 years. She is not taking any medications at this time. Examination reveals a midline, mobile, and non-tender uterus of normal size. She has no cervical motion tenderness. Which of the following is the most likely pathophysiology for this patient's symptoms? A. Myometrial irritation from ectopic endometrial glands B. Excessive prostaglandin-induced myometrial contraction C. Pelvic congestion from dilated spiral arterioles D. Excessive endometrial proliferation from unopposed estrogen

(u) A. See B for explanation. (c) B. Primary dysmenorrhea is caused by an excess of prostaglandin F2 alpha produced in the endometrium. Prostaglandin production in the uterus normally increases under the influence of progesterone, reaching a peak at the start of menses. (u) C. See B for explanation. (u) D. See B for explanation.

Which of the following is the rationale for the utilization of beta blocker therapy in a patient with hyperthyroidism? A. Reduce goiter size B. Alleviate symptoms C. Regulate TSH D. Normalize free T4

(u) A. See B for explanation. (c) B. Propranolol is used for symptomatic relief until the hyperthyroidism is resolved. It relieves tachycardia, tremor, diaphoresis and anxiety. (u) C. See B for explanation. (u) D. See B for explanation.

: (8) 77 196. Health Maintenance/Obstetrics/Gynecology Current recommendations concerning antepartum treatment of antibody negative, Rh-negative women with Rhimmune globulin include which of the following? A. Administration at 28 and 36 weeks of gestation B. Administration at 28 weeks gestation to all Rh-negative women C. Administration within 72 hours of delivery of a Rh D negative infant D. Administration in first pregnancy only

(u) A. See B for explanation. (c) B. Rh negative women are screened on initial and 28 week lab panels for antibodies to the Rh antigen. If the mother is antibody negative she will receive a Rhogam injection at 28 weeks and within 72 hours of the birth of a Rh positive baby. Rhogam is also prophylactically provided when any event takes place in the pregnancy where there is risk of fetomaternal hemorrhage that could lead to maternal exposure to fetal red blood cells including: abortion, ectopic pregnancy, bleeding associated with previa or abruption, amniocentesis, chorionic villus sampling, abdominal trauma, or external cephalic version. (u) C. See B for explanation. (u) D. See B for explanation.

Household contacts of a patient with bacterial meningitis are best treated with which of the following? A. Amoxicillin (Amoxil) B. Ciprofloxin (Cipro) C. Tetracycline (Sumycin) D. Vancomycin (Vancocin)

(u) A. See B for explanation. (c) B. Rifampin, Cipro, Levaquin, Zithromax and Rocephin are the drugs of choice. (u) C. See B for explanation. (u) D. See B for explanation.

As a general rule, sutures in the face should be removed in A. 3 days. B. 5 days. C. 7 days. D. 10 days.

(u) A. See B for explanation. (c) B. Sutures of the face should be removed in 5 days in order to allow for adequate healing and to limit the amount of scarring. (u) C. See B for explanation. (u) D. See B for explanation.

A 17 year-old girl uses an albuterol inhaler to treat her asthma. She uses the inhaler as needed and reports symptoms occurring 3-4 days per week. She experiences symptoms at night no more than once a month. Her spirometry during her most recent office visit is normal. What is the appropriate medical management of this patient? A. Add an inhaled long-acting bronchodilator B. Add an inhaled steroid C. Add an inhaled long-acting bronchodilator and steroid D. No change to her medical regimen

(u) A. See B for explanation. (c) B. Symptoms occurring more than twice a week but less than daily meet severity criteria for mild persistent asthma. The appropriate next step in her therapy is to add an inhaled steroid. Adding an inhaled long-acting bronchodilator is only recommended for moderate persistent asthma and only after an inhaled steroid has been added. (u) C. See B for explanation. (u) D. See B for explanation.

Who is most likely to require subacute bacterial endocarditis (SBE) prophylaxis prior to a dental procedure? A. 22 year-old female with mitral valve prolapse B. 36 year-old male with a bio-prosthesic mitral valve C. 45 year-old female with an ASD closure 8 months ago with no residual defect D. 15 year-old male with a bicuspid aortic valve

(u) A. See B for explanation. (c) B. The AHA recommends that patients with prosthetic heart valves receive antibiotic prophylaxis. As should cardiac transplant recipients with valve disease, unrepaired cyanotic CHD, repaired CHD with prosthetic material or device during the first six months of the procedure and repaired CHD with residual defects at site of patch or prosthetic device. (u) C. See B for explanation. (u) D. See B for explanation.

As part of the long-term management of a patient with type 1 diabetes mellitus, the glycosylated hemoglobin (HgbA1C) level should be ideally maintained at A. less than 5%. B. 6 to 7%. C. 8 to 10%. D. greater than 10%.

(u) A. See B for explanation. (c) B. The HgbA1C in patients with type 1 diabetes mellitus should be maintained between 6 and 7%. Lower levels lead to an increased number of episodes of hypoglycemia, and higher levels lead to an increased risk of retinopathy, nephropathy, and neuropathy. (u) C. See B for explanation. (u) D. See B for explanation.

If a woman has a normal 28-day menstrual cycle what tissue and hormonal phase occurs during the last 14 days? A. Proliferative follicular phase under the influence of estrogen. B. Secretory luteal phase under the influence of estrogen and progesterone. C. Proliferative follicular phase under the influence of estrogen and progesterone. D. Secretory luteal phase under the influence of estrogen.

(u) A. See B for explanation. (c) B. The endometrial changes seen in the latter half of the cycle are under the influence of both estrogen and progesterone from the corpus luteum. During this phase, the endometrium becomes more vascularized and slightly edematous. (u) C. See B for explanation. (u) D. See B for explanation.

At what age does the first tooth usually erupt in an infant? A. 2-4 months B. 6-8 months C. 10-12 months D. 14-16 months

(u) A. See B for explanation. (c) B. The first tooth in an infant to erupt is the central incisor at the average age of 6-8 months. (u) C. See B for explanation. (u) D. See B for explanation.

A 22 year-old graduate student presents with a six-month history of abdominal pain relieved with defecation which seems to coincide with her starting PA school. She describes alternating constipation and diarrhea as well as bloating. She denies any recent weight changes. Her only medication is citalopram (Celexa) What is the most likely diagnosis for this patient? A. Crohn's disease B. Irritable bowel syndrome C. Thyroid storm D. Celiac disease

(u) A. See B for explanation. (c) B. The patient's symptoms are consistent with irritable bowel syndrome. More than 50% of patients who seek medical attention for symptoms of IBS also have a diagnosis of depression, anxiety, or somatization. (u) C. See B for explanation. (u) D. See B for explanation.

: (27) 109. Diagnosis/Gastrointestinal/Nutritional A 65 year-old woman is admitted to the hospital for severe abdominal pain and vomiting over the last 6 hours. Her laboratory findings include: WBC Count - 19,000 Serum triglyceride level - 1100 mg/dL Glucose - 280 mg/dL Which of the following is the most likely diagnosis? A. Acute cholecystitis B. Acute pancreatitis C. Hepatitis D. Diabetic ketoacidosis

(u) A. See B for explanation. (c) B. This patient has acute pancreatitis. Typical symptoms include abdominal pain and nausea and vomiting. Significant hypertriglyceridemia is present. Transient hyperglycemia is common, in addition to elevation of lipase and amylase, serum amino-transferase and alkaline phosphatase are elevated in 50% of patients with acute pancreatitis. (u) C. See B for explanation. (u) D. See B for explanation.

A cirrhotic patient presents with progressive drowsiness and delirium. Physical examination reveals asterixis and tremor. Which of the following medications would you initiate? A. Empiric antibiotic therapy B. Lactulose (Kristalose) C. Bolus fluid challenge with sodium chloride D. Hydrochlorothiazide (Diuril)

(u) A. See B for explanation. (c) B. This patient has hepatic encephalopathy. Increased ammonia levels contribute to the mental status changes associated with hepatic encephalopathy. Lactulose leads to a change in bowel flora so that fewer ammonia forming organisms are present and also helps in the acidification of colon contents which leads to a nonabsorbable ammonium ion creation. (u) C. See B for explanation. (u) D. See B for explanation.

: (12) 62 155. Diagnostic Studies/Neurology A 74 year-old female with mild hypertension is brought to the ED with right hemiparesis after a fall two days ago. The family reports that the patient has had some confusion this morning. Examination reveals left pupillary dilation and decreased strength of the upper and lower extremity. Which of the following diagnostic studies should be ordered first? A. MRI of the brain B. CT scan of the brain C. Radiograph of the head D. Lumbar puncture

(u) A. See B for explanation. (c) B. This patient presents with a history of minor trauma and progressive neurological abnormalities consistent with subdural hematoma. The diagnosis would be confirmed by CT scan, which is less expensive and more sensitive for blood than an MRI. (u) C. Skull radiographs would not be helpful because they evaluate bony, not soft tissue, injury. (h) D. A lumbar puncture is contraindicated because of the potential for brain herniation.

A 74 year-old female is being treated for mild hypertension. She is found at home with right hemiparesis and brought to the emergency department. Her daughter states that the patient fell in her kitchen 2 days ago, but had no complaints at that time. She did state that her mother sounded a little confused this morning. The patient's left pupil is dilated. Which of the following diagnostic studies should be ordered first? A. MRI of the brain B. CT scan of the brain C. Skull x-ray D. Lumbar puncture

(u) A. See B for explanation. (c) B. This patient presents with a history of minor trauma and progressive neurological abnormalities consistent with subdural hematoma. Diagnosis would be confirmed by CT scan, which is less expensive and more sensitive for blood than an MRI. (u) C. Skull x-rays would not be helpful because they evaluate bony, not soft tissue, injury. (h) D. A lumbar puncture is contraindicated because of the potential for brain herniation.

A 26 year-old male presents with a four week history of fatigue, night sweats, and a painless mass in his neck. Physical examination confirms the presence of an enlarged right posterior cervical lymph node. What is the next best step in the evaluation of this patient? A. bone marrow biopsy and aspiration B. lymph node biopsy C. chest CT scan D. reexamine in 2-4 weeks

(u) A. See B for explanation. (c) B. This patient presents with possible lymphoma. Diagnosis is made by lymph node biopsy. Bone marrow biopsy and CT scan of the chest are used for staging of the disease. (u) C. See B for explanation. (u) D. With the presence of B-symptoms (fever, night sweats, and weight loss) the patient needs a biopsy to evaluate for possible lymphoma.

A 41 year-old male with a history of intravenous drug abuse presents to your office with acute, nontraumatic right knee pain, chills, and sweats starting 2 days ago. On physical examination, his temperature is 102.9 degrees F. The right knee is erythematous, edematous and tender to palpation and range of motion. Plain knee x-ray reveals soft tissue swelling. Which of the following is most likely the diagnosis? A. Gouty arthritis B. Septic arthritis C. Rheumatoid arthritis D. Psoriatic arthritis

(u) A. See B for explanation. (c) B. This patient's signs and symptoms are most consistent with septic arthritis. IV drug abuse places this patient at even greater risk. (u) C. See B for explanation. (u) D. See B for explanation.

: (8) 13. History & Physical/Gastrointestinal/Nutritional An elderly man who recently emigrated from a war-torn area of Africa is brought to the clinic by his daughter. She explains that her father's diet was very limited in calories and protein and that he mostly ate corn and very little fresh foods. He has chronic diarrhea and examination reveals pigmented regions on hands, arms, and face and mild dementia. His tongue is smooth and shiny. This patient most likely has a deficiency of which of following vitamins? A. Folate B. Niacin C. Thiamine D. Vitamin K

(u) A. See B for explanation. (c) B. This patient's syndrome known as pellagra is due to niacin (vitamin B3) deficiency. It is often seen in people where corn is the major source of energy and is still endemic in parts of Africa. The syndrome includes glossitis, pigmented dermatitis, dementia, and diarrhea. (u) C. See B for explanation. (u) D. See B for explanation.

A 28 year-old HIV positive male presents complaining of fever, dysphagia, odynophagia, and trismus for the past 2 days. Physical examination reveals an ill-appearing male with a temperature of 101.3 degrees F, poor dentition, "woody" edema in the sublingual area and neck, tongue displaced posteriorly, and drooling. The immediate managment of this patient includes which of the following? A. CT of the neck B. intubation C. start IV penicillin D. incision and drainage of the abscess

(u) A. See B for explanation. (c) B. This person presents with the classic signs and symptoms of Ludwig's angina. He is unable to handle his secretions and the displacement of his tongue suggests impending airway obstruction. Intubation to secure his airway is the most immediate concern. CT of the neck for diagnosis, starting IV antibiotics, and incision and drainage are actions that need to be taken after securing the airway. (u) C. See B for explanation. (u) D. See B for explanation.

A 17 year-old male is accidentally struck in the right eye while playing football and is immediately transported to the hospital. In the emergency room, he complains of severe pain behind the eye as well as double vision. On examination, he has exophthalmos, cannot move his right eye upward and blood is noted in the anterior chamber. Which of the following is the most appropriate course of action at this time? A. Apply ice packs and cold compresses B. Immediately refer the patient to an ophthalmologist C. Attempt to keep the patient calm and order a skull x-ray D. Administer a dose of intramuscular broad-spectrum antibiotic

(u) A. See B for explanation. (c) B. This scenario describes a "blow-out" fracture of the orbit with hyphema and, because of the signs and symptoms presented, warrants an immediate consult by an ophthalmologist. (u) C. See B for explanation. (u) D. See B for explanation.

What class of medications has been found most beneficial in the treatment of alcoholism not associated with a concomitant psychiatric illness? A. Antipsychotics B. Benzodiazepines C. Melatonin agonists D. Selective serotonin reuptake inhibitors

(u) A. See B for explanation. (c) B. Though no pharmacologic therapies have substantial supportive data in treating alcoholism not associated with a mood or anxiety disorder short term use of benzodiazepines has the greatest benefit especially with acute cessation of alcohol. (u) C. See B for explanation. (u) D. See B for explanation.

: (21) 151. Clinical Therapeutics/Pulmonology A patient who recently emigrated from Southeast Asia presents with a 6 month history of weight loss, frequent cough with hemoptysis and fever. A sputum smear is positive for acid fast bacilli. Which of the following lab tests are most important in routine treatment of this condition? A. Amylase/lipase B. AST/ALT C. PT/PTT D. Sodium/potassium

(u) A. See B for explanation. (c) B. Tuberculosis, as indicated in this case, is treated with several hepatotoxic agents which require close monitoring of liver function to avoid complications. (u) C. See B for explanation. (u) D. See B for explanation.

Which of the following is the study of choice to diagnose upper gastrointestinal malignancy? A. Abdominal CT B. Upper endoscopy C. Barium swallow D. Abdominal ultrasound

(u) A. See B for explanation. (c) B. Upper endoscopy is the study of choice to diagnose gastroduodenal ulcers, erosive esophagitis and upper gastrointestinal malignancy. (u) C. See B for explanation. (u) D. See B for explanation.

A 36 year-old woman admits that her husband has abused her for over ten years. You should inform the woman that she is at most risk for injury or death A. just before a holiday. B. just after leaving an abusive spouse. C. when an abusive spouse arrives home after work. D. when an abusive spouse has been drinking heavily.

(u) A. See B for explanation. (c) B. Women are more likely to be assaulted or murdered when attempting to report the abuse or leave the abusive relationship; up to 75% of domestic assaults occur after separation. (u) C. See B for explanation. (u) D. See B for explanation.

: (9) 47. Clinical Intervention/Urology/Renal A 28 year-old female is seen for her fourth documented case of uncomplicated cystitis this year. You recommend prophylaxis with nitrofurantoin (Macrodantin) 100 mg at bedtime, however, she would prefer not to take a daily medication. What alternative single-dose schedule can you recommend? A. Day 1 of menstrual cycle B. Following intercourse C. After each bowel movement D. Weekly every Sunday

(u) A. See B for explanation. (c) B. Women who have more than three episodes of cystitis per year are considered candidates for prophylactic antibiotics to prevent recurrence after treatment for UTI. Single dosing at bedtime or at the time of intercourse is the recommended schedule. (u) C. See B for explanation. (u) D. See B for explanation.

: (27) 223. Health Maintenance/Hematology During the evaluation of a post motor vehicle accident patient who incurred blunt abdominal trauma, you notice a Grey-Turner's sign. He undergoes corrective surgery. Which of the following is an appropriate preventive strategy for this patient? A. Aspirin therapy B. Pancreatic enzymes C. Pneumococcal vaccine D. Tdap vaccine

(u) A. See C for explanation. (u) B. Although the pancreas may be one of the organs involved in a blunt trauma abdominal injury, replacement enzymes are not indicated. (c) C. For a patient who has functional or anatomic asplenia, pneumococcal vaccine should be administered as the spleen is specifically responsible for combating infection by streptococcus pneumoniae. (u) D. See C for explanation.

A 57 year-old man is being evaluated for shortness of breath. The following spirometric data are obtained: VC 4.90 L (predicted), 5.15 L (observed) 105% predicted FRC 3.99 L (predicted), 4.37 L (observed) 110% predicted RV 2.47 L (predicted), 3.17 L (observed) 128% predicted FEV1 3.50 L (predicted), 2.35 L (observed) 67% predicted These findings are consistent with which of the following? A. No demonstratable abnormality B. Restrictive lung disease C. Obstructive lung disease D. A ventilation/perfusion mismatch

(u) A. See C for explanation. (u) B. Restrictive lung disease would show decreased total lung capacity, vital capacity, and normal to increased FEV1. (c) C. Spirometry findings in obstructive lung disease typically show normal or increased total lung capacity, decreased vital capacity, prolonged FEV1, and increased residual volume. (u) D. A ventilation/perfusion scan would be abnormal with a pulmonary embolism.

Which of the following is considered the antibiotic of choice in the treatment of human bite wounds? A. Ampicillin B. Penicillin C. Augmentin D. Ciprofloxacin

(u) A. See C for explanation. (u) B. See C for explanation (c) C. Augmentin is considered to be the antibiotic of choice for human bites that may be contaminated with Eikenella corrodens, strep viridans, and staph aureus. (u) D. See C for explanation.

A diabetic patient returns for follow-up of non-fasting blood work done at a local health fair. The total cholesterol is 230 mg/dl. Which of the following is the appropriate next step? A. referral to a cardiologist B. reassurance C. obtain a fasting HDL and LDL lipid measurement D. start a HMG Co-A reductase inhibitor

(u) A. See C for explanation. (u) B. See C for explanation. (c) C. A patient with elevated total cholesterol needs further evaluation through a fasting total lipid profile including LDL and HDL. (u) D. See C for explanation.

What physical examination finding is most specific for acute cholecystitis? A. Psoas sign B. Rovsing's sign C. Murphy's sign D. Cullen's sign

(u) A. See C for explanation. (u) B. See C for explanation. (c) C. A sharp increase in tenderness with a sudden stop in inspiratory effort constitutes a postive Murphy's sign and is most specific for cholecystitis. (u) D. See C for explanation.

A 65 year-old with COPD receiving their first pneumococcal conjugate vaccination should be revaccinated in A. 1 year. B. 3 years. C. 5 years. D. 10 years.

(u) A. See C for explanation. (u) B. See C for explanation. (c) C. A single revaccination for a person over the age of 65 is recommended if it has been more than 5 years since they received their first vaccination. (u) D. See C for explanation.

: (15) 86 221. Clinical Therapeutics/Pulmonology A 30 year-old patient presents with a three year history of episodic of wheezing, dyspnea and dry to minimally productive cough. These episodes are typically worse at night and during the fall and spring seasons. The patient admits to a long history of allergies and tobacco use. Exam reveals numerous nasal polyps, xerotic skin and expiratory wheezes in bilateral lung fields. There is no cyanosis or abnormal heart sounds noted. His pulmonary function test reveals a low FEV1/FVC ratio. Which of the following class of medications would be best to treat an acute exacerbation of this disorder? A. Antihistamines B. Beta agonists C. Leukotriene modifiers D. Methylxanthines

(u) A. See B for explanation. (c) B. All of these medications may play a role in the long-term management of asthma however, only beta agonists are indicated in the management of an acute exacerbation. (u) C. See B for explanation. (u) D. See B for explanation.

A postmenopausal woman is at greatest risk of death from which of the following? A. stroke B. heart disease C. ovarian cancer D. breast cancer

(u) A. See B for explanation. (c) B. Although women tend to be concerned about dying from breast cancer, heart disease is the number one killer of postmenopausal women. (u) C. See B for explanation. (u) D. See B for explanation.

What are the most effective agents in treating somatoform spectrum pain disorder? A. Analgesics B. Antidepressants C. Antipsychotics D. Anxiolytics

(u) A. See B for explanation. (c) B. Antidepressants, especially SSRIs and TCAs are the main pharmacologic treatments used in somatoform spectrum pain disorder as analgesics and anxiolytics provide little to no relief and generally result in dependency. As there is no associated psychosis, delusions or hallucinations antipsychotics are not helpful. (u) C. See B for explanation. (u) D. See B for explanation.

: (11) 11. Health Maintenance/Pulmonology Which of the following is most likely to cause a false negative PPD? A. Aspirin allergy B. Diagnosis of AIDS C. BCG immunization D. Pregnancy

(u) A. See B for explanation. (c) B. Any process that results in reduced immune response can cause a false negative PPD. Past immunization with BCG would typically result in a false positive while aspirin allergy and pregnancy generally have no effect in and of themselves on PPD results. (u) C. See B for explanation. (u) D. See B for explanation.

Treatment of Bell's palsy includes which of the following? A. Acyclovir B. Reassurance of the patient's recovery C. Referral to a neurosurgeon D. Electromyography

(u) A. See B for explanation. (c) B. Bell's palsy is a peripheral neuropathy of cranial nerve VII. Although it has been suggested it may be related to an activation of herpes simplex virus, there is little empiric evidence for this. Approximately 60% of cases of Bell's palsy recover without treatment and patient reassurance of this is advised. Electromyography may provide aid in the prognosis, but not as a treatment option. A neurosurgeon has no role in the management of Bell's palsy. (u) C. See B for explanation. (u) D. See B for explanation.

According to the Joint National Commission VII Guidelines, blood pressure targets are lower in patients with diabetes mellitus and what other condition? A. Liver disease B. Renal disease C. Thyroid disease D. Peripheral vascular disease

(u) A. See B for explanation. (c) B. Blood pressure targets for hypertensive patients at the greatest risk for cardiovascular events, particularly those with diabetes and chronic kidney disease, are lower (less than 130/80) than for those individuals with lower cardiovascular risk (goal is less than 140/90). (u) C. See B for explanation. (u) D. See B for explanation.

A 52 year-old female is diagnosed with chronic myeloid leukemia (CML). Which of the following would you expect to find on physical examination of this patient? A. Hepatomegaly B. Splenomegaly C. Posterior cervical lymphadenopathy D. Inguinal lymphadenopathy.

(u) A. See B for explanation. (c) B. CML patients present with an enlarged spleen. (u) C. CML patients typically present with enlarged spleen and fatigue, but there are not enlarged lymph nodes. (u) D. Patients with CML do not present with inguinal lymphadenopathy.

: (9) 50. Diagnosis/Obstetrics/Gynecology A 35 year-old primigravida presents to your office at 16 weeks gestation. BP measurements in the first trimester were within normal limits. On physical examination her blood pressure after 5 minutes of rest is 160/100 mmHg. A repeat measurement 1 week later is 154/98 mmHg. Fundoscopic exam is normal, and no other abnormalities are noted. Which of the following is the most likely diagnosis in this patient? A. Preeclampsia B. Chronic hypertension C. Gestational hypertension D. Molar pregnancy

(u) A. See B for explanation. (c) B. Chronic hypertension in a pregnant patient is defined as hypertension present before the 20th week of pregnancy or hypertension present before pregnancy. (u) C. See B for explanation. (u) D. See B for explanation.

: (15) 199. Diagnosis/Endocrinology Which of the following symptoms is more commonly seen in the initial presentation of diabetes mellitus type 2 over type 1? A. Polyphagia B. Chronic skin infections C. Nocturnal enuresis D. Weight loss

(u) A. See B for explanation. (c) B. Chronic skin infections, including candidal vaginitis in women, are a common presenting symptom in patients with diabetes mellitus type 2. Patients are often overweight or obese. Polyphagia, nocturnal enuresis and weight loss are more consistent with the presentation of diabetes mellitus type 1. (u) C. See B for explanation. (u) D. See B for explanation.

: (9) 43. Scientific Concepts/Pulmonology Which of the following is the most common cause of a transudative pleural effusion? A. Bacterial pneumonia B. Congestive heart failure C. Lung cancer D. Pulmonary embolism

(u) A. See B for explanation. (c) B. Congestive heart failure is the most common cause of a transudative pleural effusion. Lung cancer and pneumonia only cause an exudative effusion. Pulmonary embolism can cause either type of effusion but most commonly it is exudative. (u) C. See B for explanation. (u) D. See B for explanation.

Which of the following is an absolute contraindication for the performance of exercise stress testing for patients who wish to start an exercise program? A. Second degree heart block type 1 B. Severe aortic stenosis C. Atrial fibrillation with controlled ventricular response D. Recent diagnosis of lung cancer

(u) A. See B for explanation. (c) B. Contraindications to stress testing include rest angina within the last 48 hours, unstable cardiac rhythm, hemodynamically unstable patient, severe aortic stenosis, acute myocarditis, uncontrolled heart failure, and active infective endocarditis. (u) C. See B for explanation. (u) D. See B for explanation.

Perfusion of the coronary arteries occurs primarily during which of the following? A. Systole B. Diastole C. Afterload D. Preload

(u) A. See B for explanation. (c) B. Coronary artery perfusion occurs primarily during diastole. (u) C. See B for explanation. (u) D. See B for explanation.

Which of the following is the minimum criteria in millimeter diameter of induration for a positive tuberculosis skin-test screening result in an HIV-positive patient? A. 0 B. 5 C. 10 D. 15

(u) A. See B for explanation. (c) B. Current minimum criteria for a positive skin test is 5 mm in diameter for individuals at very high risk, including those who are HIV infected and recent contacts. (u) C. See B for explanation. (u) D. See B for explanation.

: (8) 17. Health Maintenance/Cardiology Which subset of the asymptomatic general population should undergo one-time screening for abdominal aortic aneurysm? A. 65 year-old female with chronic kidney disease B. 65 year-old healthy male who has smoked since age 20 C. 74 year-old male who has diabetes mellitus D. 74 year-old female who has smoked since age 15

(u) A. See B for explanation. (c) B. Data support the use of abdominal ultrasound to screen 65- to 74-year old men, but not women, who have a history of smoking. Repeated screening does not appear to be needed if the aorta shows no enlargement. (u) C. See B for explanation. (u) D. See B for explanation.

Which electrolyte abnormality is associated with an increase in the risk for digoxin toxicity?] A. hypercalcemia B. hypokalemia C. hypermagnesemia D. hyponatremia

(u) A. See B for explanation. (c) B. Decreased concentration of potassium results in the increased activity of cardiac glycosides by increasing tissue binding and decreasing renal excretion of digoxin. Potassium loss is the only significant electrolyte abnormality that significantly affects digoxin metabolism. (u) C. See B for explanation. (u) D. See B for explanation.

Which of the following is the most common cause of blindness in adults under the age of 70 years in the USA? A. hypertension B. diabetes mellitus C. macular degeneration D. retinal artery occlusion

(u) A. See B for explanation. (c) B. Diabetic retinopathy is the leading cause of blindness in adults under the age of 70 in the US. (u) C. See B for explanation. (u) D. See B for explanation.

A 60 year-old female with a 30 pack year smoking history complains of new onset shortness of breath. On physical examination, dullness is noted on percussion with dimished breath sounds over her left base. Chest x-ray shows a new left pleural effusion. Which of the following is the next step in the management of this patient? A. Repeat chest x-ray in two months B. Perform diagnostic thoracentesis C. Order MRI of the chest D. Treat with antibiotic

(u) A. See B for explanation. (c) B. Diagnostic thoracentesis should be performed whenever there is a new pleural effusion and no clinically apparent cause. (u) C. See B for explanation. (u) D. See B for explanation.

: (13) 222. Diagnosis/Orthopedics/Rheumatology A fracture of the distal 5th metacarpal is most likely to occur after which of the following events? A. Falling on an outstretched hand B. Punching a wall C. Closing the finger in a car door D. Twisting the finger playing basketball

(u) A. See B for explanation. (c) B. Distal 5th metacarpal fractures result from a direct blow delivered to the hand or by the hand striking a solid object. (u) C. Closing a finger in a car door typically results in a distal phalanx fracture of the middle finger. (u) D. See B for explanation.

: (12) 146. Clinical Therapeutics/Orthopedics/Rheumatology Which of the following is the indicated treatment for a patient who presents with headache, fever, and faint macular eruptions on the ankles and wrists several days after returning from a vacation in North Carolina? A. Trimethoprim sulfamethoxazole (Bactrim) B. Doxycycline (Doryx) C. Clindamycin (Cleocin) D. Methotrexate (Rheumatrex)

(u) A. See B for explanation. (c) B. Doxycycline is given orally or parenterally for 7 days for the treatment of Rocky Mountain spotted fever. (u) C. See B for explanation. (u) D. See B for explanation.

: (3) 67. Scientific Concepts/Gastrointestinal/Nutritional A 42 year-old male with chronic complaints of heartburn and regurgitation presents for follow-up after undergoing endoscopic evaluation which shows evidence of Barrett's esophagus. Which of the following is the most serious potential complication in this patient? A. Peptic stricture B. Esophageal adenocarcinoma C. Esophageal web formation D. Mallory-Weiss tear

(u) A. See B for explanation. (c) B. Esophageal carcinoma is the most serious complication of Barrett's esophagus. (u) C. See B for explanation. (u) D. See B for explanation.

Which of the following fractures is associated with the greatest risk of avascular necrosis of the femoral head? A. intertrochanteric B. femoral neck C. subtrochanteric D. pelvic rim

(u) A. See B for explanation. (c) B. Femoral neck fractures lead to the greatest disruption of arterial blood supply to the femoral head. (u) C. See B for explanation. (u) D. See B for explanation.

: (28) 53 130. Health Maintenance/Cardiology A new patient with a history of hypertension on verapamil (Calan) presents for routine examination. Electrocardiogram reveals irregular R-R intervals, with narrow QRS complexes. There are no definable P-waves. Which of the following is most appropriate for this patient? A. Annual echocardiogram B. Warfarin (Coumadin) therapy C. Internal defibrillator D. Infective endocarditis prophylaxis

(u) A. See B for explanation. (c) B. For patients with atrial fibrillation, even when it is paroxysmal or occurs rarely, anticoagulation with warfarin to an INR target of 2.0-3.0 should be established and maintained indefinitely. (u) C. See B for explanation. (u) D. See B for explanation.

Patients who undergo percutaneous angioplasty or who have coronary artery revascularization often are treated with glycoprotein IIb/IIIa inhibitors. What is the major side effect associated with these agents? A. Hypotension B. Bleeding C. Coronary vasospasm D. Acute renal failure

(u) A. See B for explanation. (c) B. Glycoprotein IIb/IIa inhibitors have their activity in the final stages of platelet bridging and are associated with bleeding when used in the management of acute myocardial infarction. Since they are effective at treating and preventing new clot formation, bleeding is the main concern and complication with the use of these agents. (u) C. See B for explanation. (u) D. See B for explanation.

: (30) 84 215. Diagnosis/Obstetrics/Gynecology A 28 year-old G0 P0 female presents with a 2 week history of pelvic pain and scant vaginal bleeding. She reports a history of regular menses every 28 days, however, her LMP was 37 days ago. On examination, her uterus is normal in size. There is a small amount of blood in the vagina. Her left adnexa is somewhat tender. Which of the following is the most likely diagnosis? A. Ruptured corpus luteum cyst B. Pelvic inflammatory disease C. Ectopic pregancy D. Threatened abortion

(u) A. See C for explanation. (u) B. See C for explanation. (c) C. An ectopic pregancy is one in which the blastocyst implants anywhere other than the endometrial lining of the uterine cavity. The classic symptoms of an ectopic pregnancy are amenorrhea, followed by vaginal bleeding, and abdominal pain on the affected side. (u) D. See C for explanation.

A newborn weighs 8 pounds at birth. On average, what should the infant weigh at 1 year of age? A. 16 pounds B. 20 pounds C. 24 pounds D. 28 pounds

(u) A. See C for explanation. (u) B. See C for explanation. (c) C. An infant will triple birth weight within the first year of life. A newborn that weighs 8 pounds at birth will weigh approximately 24 pounds at 1 year of age. (u) D. See C for explanation.

At birth an infant weighs 8 pounds. The optimal weight for this infant at 1 year would be A. 16 pounds. B. 20 pounds. C. 24 pounds. D. 28 pounds.

(u) A. See C for explanation. (u) B. See C for explanation. (c) C. An infants birth weight should double by 4-5 months of age and triple by 1 year of age. (u) D. See C for explanation

1. Health Maintenance/Obstetrics/Gynecology Which of the following portends the greatest risk for the development of breast cancer? A. Nulliparity B. Early menarche C. BRCA 1 positivity D. Maternal family history of breast cancer

(u) A. See C for explanation. (u) B. See C for explanation. (c) C. BRCA1 positivity is associated with half of the early onset breast cancers and 90% of the hereditary ovarian cancers. (u) D. See C for explanation.

: (6) 205. Diagnosis/Obstetrics/Gynecology A 25 year-old woman has noted an enlarging painful swelling of the right labia for three days. Which of the following is the most likely diagnosis? A. Condyloma acuminata B. Gartner duct cyst C. Bartholin gland abscess D. Squamous cell carcinoma

(u) A. See C for explanation. (u) B. See C for explanation. (c) C. Bartholin glands, located on the inferior borders of the vagina, are a common location for N. gonorrhoeae or C. trachomatis infection. When the gland becomes full or painful, incision and drainage is appropriate. (u) D. See C for explanation.

: (8) 21. History & Physical/Obstetrics/Gynecology A patient presents to the office with a last menstrual period of May 4th. What is the estimated date of confinement? A. January 4 B. January 11 C. February 11 D. February 12

(u) A. See C for explanation. (u) B. See C for explanation. (c) C. Calculation is based on Naegele's rule by adding 7 days to the first day of the last menstrual period and then subtracting 3 months. (u) D. See C for explanation.

A 67 year-old African American male presents for a new patient evaluation. History reveals an aphasic CVA which limits his history. Funduscopic examination reveals an abnormal vessel light reflex described as a silver or copper-wire appearance. Where the vessels intersect, there appears to be some nicking. He has no carotid bruits, and his cardiac exam is normal. What is the most likely cause of his ocular findings? A. Cytomegalovirus retinitis B. Diabetic retinopathy C. Hypertensive retinopathy D. Sickle cell retinopathy

(u) A. See C for explanation. (u) B. See C for explanation. (c) C. Chronic hypertension accelerates the development of atherosclerosis. The retinal arterioles become more tortuous and narrow and develop abnormal light reflexes (silver-wiring and copper-wiring). There is increased venous compression at the retinal arteriovenous crossings (arterio-venous nicking), an important factor predisposing to branch retinal vein occlusion. (u) D. See C for explanation.

Which of the following will result in decreased serum theophylline levels in a patient with COPD? A. Cimetidine B. Congestive heart failure C. Cigarette smoking D. Ciprofloxacin

(u) A. See C for explanation. (u) B. See C for explanation. (c) C. Cigarette smoking will increase the hepatic clearance of theophylline, resulting in decreased levels in the system. The use of cimetidine or ciprofloxacin or the presence of congestive heart failure will reduce hepatic clearance and causing an increase in theophylline serum levels. (u) D. See C for explanation.

: (11) 169. Clinical Intervention/Infectious Diseases A 17 year-old college student is brought to the emergency department with a 10 hour history of fever, headache and increasing listlessness. Examination is significant for a temperature of 103 F, nuchal rigidity and a petechial rash on the trunk and extremities. Which of the following is the most appropriate initial treatment? A. Blood culture and lumbar puncture B. Gram stain of the petechiae C. Blood culture and administration of IV fluids and empiric antibiotics D. Administration of corticosteroids

(u) A. See C for explanation. (u) B. See C for explanation. (c) C. Clinical presentation is meningococcal meningitis. Blood cultures must be obtained and IV antimicrobial therapy initiated immediately. This may be done prior to LP. (u) D. See C for explanation.

: (8) 170. Clinical Therapeutics/Psychiatry/Behavioral Medicine A 32 year-old male with a history of schizoaffective disorder was started on a new antipsychotic 12 weeks ago. He presents with a fever of 102 F, shaking chills, and rigors. Lab testing is significant for a WBC count of 200. Which of the following antipsychotic agents is the patient most likely taking? A. Risperidone (Risperdal) B. Onlazapine (Zyprexa) C. Clozapine (Clozaril) D. Quetiapine (Seroquel XR)

(u) A. See C for explanation. (u) B. See C for explanation. (c) C. Clozapine has a 1-2% incidence of agranulocytocis. Patients should be instructed to promptly report the onset of fever, sore throat, weakness or other signs of infection. The drug must be discontinued if the WBC drops below 3,000 or 50% of the patient's normal count. (u) D. See C for explanation.

A patient undergoes biopsy for suspected myocarditis. Which of the following is the most likely etiologic agent? A. West Nile virus B. Rhinovirus C. Coxsackie B virus D. Cytomegalovirus

(u) A. See C for explanation. (u) B. See C for explanation. (c) C. Coxsackie B virus, Hepatitis C, adenovirus, and HIV are the predominant agents in clinically significant acute viral myocarditis in the US. (u) D. See C for explanation.

: (27) 122. Clinical Intervention/Pulmonology Which of the following is considered the definitive treatment of severe obstructive sleep apnea? A. Continuous positive airway pressure (CPAP) B. Nasal septoplasty C. Tracheostomy D. Uvulopalatopharyngoplasty (UPPP)

(u) A. See C for explanation. (u) B. See C for explanation. (c) C. Despite the numerous drawbacks, including granuloma formation and speech difficulties, tracheostomy is the only treatment that relieves upper airway obstruction and the physiologic consequences of severe obstructive sleep apnea. CPAP, UPPP and nasal septoplasty carry varying degrees of success based on patient compliance and/or anatomical variances. (u) D. See C for explanation.

A 6 year-old male, a recent immigrant from Latin America, is brought to the emergency department with difficulty swallowing and breathing. On physical examination you note a gray-tan pseudomembrane in the back of the pharynx. There is also tender cervical lymphadenopathy. What physical examination finding is most specific for diagnosis in this patient? A. Difficulty swallowing B. Respiratory compromise C. Pseudo-membrane in pharynx D. Cervical adenopathy

(u) A. See C for explanation. (u) B. See C for explanation. (c) C. Diphtheria most commonly presents with sore throat, adherent tonsillar, pharyngeal, or nasal pseudomembrane and low grade fever. The pseudo-membrane is most specific for diphtheria. (u) D. See C for explanation.

A 27 year-old female presents with 5-6 headaches monthly for the past year that are described as severe, throbbing and initially unilateral. They is associated with nausea, photophobia and phonophobia. There have been no concomitant sensory or motor deficits. Her physical examination and brain MRI are normal. What treatment would be best to reduce the frequency of the headaches? A. Sumatriptan (Imitrex) B. Isometheptene (Midrin) C. Propranolol (Inderal) D. Hydrocodone (Vicodin)

(u) A. See C for explanation. (u) B. See C for explanation. (c) C. Each medication listed can be used in the management of migraine headaches, however only propranolol is indicated for reducing the frequency of migraine headaches. (u) D. See C for explanation.

An 18 year-old male presents with pain in his wrist after he fell off of a moving motor cycle. Physical examination reveals tenderness in the anatomic snuffbox. No fracture is noted on plain radiography of the wrist. Which of the following is the recommended treatment for this patient? A. Ace wrap of the wrist B. Closed reduction of the fracture site C. Thumb spica cast application D. Open reduction of the fracture site

(u) A. See C for explanation. (u) B. See C for explanation. (c) C. Even with normal initial radiographs, patients with a consistent history and tenderness in the anatomical snuffbox are treated as a stable fracture with immobilization in a thumb spica cast. Casting is recommended for all presumed nondisplaced scaphoid fractures. (u) D. See C for explanation.

Which of the following Rh genotypes in a mother and father would represent a risk for hemolytic disease of the newborn? A. Mother Rh-positive, father Rh-negative B. Mother Rh-positive, father Rh-positive C. Mother Rh-negative, father Rh-positive D. Mother Rh-negative, father Rh-negative

(u) A. See C for explanation. (u) B. See C for explanation. (c) C. If an Rh-negative woman carries an Rh-positive fetus, she may develop antibodies against Rh when fetal blood cells enter her circulation. (u) D. See C for explanation.

: (27) 98. Health Maintenance/Gastrointestinal/Nutritional A 43 year-old female calls the office stating that her husband was recently diagnosed with Hepatitis A. She denies a past history of or immunization for this disease and specifically denies recent jaundice, fever, nausea, or abdominal pain. Which of the following is the most appropriate next step in the care of this patient? A. Prednisone B. Ribavirin (Virazole) C. Immune globulin D. Lamivudine (Epivir)

(u) A. See C for explanation. (u) B. See C for explanation. (c) C. Immune globulin is indicated in close contacts of patients with Hepatitis A who have not previously had the disease or been immunized against it. (u) D. See C for explanation.

: (28) 65. Diagnosis/Cardiology A 1 month-old infant is brought to your office by his mother for routine evaluation and immunizations. Examination reveals a continuous, rough machinery pansystolic murmur that is best heard in the first and second interspaces of the left sternal border associated with a thrill. Which of the following is the most likely diagnosis in this patient? A. Atrial septal defect B. Coarctation of the aorta C. Patent ductus arteriosus D. Ventricular septal defect 30

(u) A. See C for explanation. (u) B. See C for explanation. (c) C. In an infant with patent ductus arteriosus, there are no symptoms unless left ventricular failure or pulmonary hypertension develops. The pulse pressure is wide, and diastolic pressure is low. A continuous, rough, machinery murmur, accentuated in late systole at the time of S2 is heard best in the left first and second interspaces at the left sternal border. Thrills are common. (u) D. See C for explanation.

What spinal nerve root is most likely affected in a patient with weak wrist extension, thumb and index finger paresthesias and diminished triceps reflex? A. Cervical 4 B. Cervical 5 C. Cervical 6 D. Cervical 7

(u) A. See C for explanation. (u) B. See C for explanation. (c) C. In contrast, cervical 5 would be associated with deltoid and biceps weakness and diminished biceps reflex while cervical 7 would result in triceps weakness and paresthesias in the middle finger and diminished brachioradialis reflex. (u) D. See C for explanation.

A 16 year-old male with a history of tetralogy of Fallot presents to clinic for a follow-up visit status post replacement of his right ventricle to pulmonary artery conduit. He has complaints of chest pain with inspiration, fever and general malaise. Cardiac examination reveals a rub with muffled heart sounds. Labs show an elevated erythrocyte sedimentation rate (ESR) and leukocytosis. Which of the following is the most effective treatment? A. Acetaminophen/oxycodone B. Azithromycin C. Indomethacin D. Furosemide

(u) A. See C for explanation. (u) B. See C for explanation. (c) C. Indomethacin is suitable for controlling pain in Dressler's syndrome. ASA is preferred. Narcotics, diuretics or antibiotics are not recommended. (u) D. See C for explanation.

A 3 year-old male with cystic fibrosis develops pneumonia. Which of the following is the most likely etiology of the pneumonia? A. Escherichia coli B. Staphylococcus epidermidis C. Pseudomonas aueroginosa D. Streptococcus pneumoniae

(u) A. See C for explanation. (u) B. See C for explanation. (c) C. Initially in the first few months of life, respiratory infection is common with Staphylococcus aureus and Haemophilus influenzae, but after that Pseudomonas aueroginosa becomes the major causative organism for infections. (u) D. See C for explanation.

In infants, the eyes should move in parallel without deviation by the age of A. 2 weeks. B. 3 months. C. 6 months. D. 1 year.

(u) A. See C for explanation. (u) B. See C for explanation. (c) C. Intermittent alternating convergent strabismus is frequently noted for the first 6 months of life, but referral is indicated if it persists beyond 6 months. (u) D. See C for explanation

: (28) 118. Clinical Therapeutics/ENT/Ophthalmology A 23 year-old female presents with rhinorrhea, nasal pruritis and paroxysms of sneezing. She reports no facial pressure or fullness. Pale boggy nasal mucosa is noted on physical examination. Which of the following is the initial treatment of choice for this condition? A. Oral prednisolone (Prelone) B. Oral pseudoephedrine (Sudafed) C. Intranasal fluticasone (Flonase) D. Intranasal cromolyn (Nasalcrom)

(u) A. See C for explanation. (u) B. See C for explanation. (c) C. Intranasal steroid treatment is the most effective, and is considered the treatment of choice in seasonal allergic rhinitis or perennial allergic rhinitis. (u) D. See C for explanation.

: (9) 44. Clinical Therapeutics/Neurology Which of the following is the initial treatment of choice for status epilepticus? A. Phenobarbital B. Valproic Acid (Depakene) C. Lorazepam (Ativan) D. Propofol (Diprivan)

(u) A. See C for explanation. (u) B. See C for explanation. (c) C. Lorazepam is the most appropriate initial medication in the management of status epilepticus. (u) D. See C for explanation.

Elevated levels of LP(a) (Lipoprotein a) are considered to be a risk factor for coronary artery disease through which of the following proposed mechanisms? A. Direct inhibition of HDL B. Increasing the formation of VLDL cholesterol C. Competes for binding to the plasminogen receptor D. Enhancement of naturally circulating triglycerides

(u) A. See C for explanation. (u) B. See C for explanation. (c) C. Lp(a) lipoproteins are secreted by the liver, constitute 10% or less of the total plasma lipoprotein mass, possess kringle domains homologous to plasminogen, and are associated with vascular disease risk. Having domains homologous to plasminogen, Lp(a) will compete with actual plasminogen for its receptor sites. Plasminogen activates plasmin, which facilitates degradation of fibrin and matrix components. The main component of LP (a) is LDL, a known risk factor for atherosclerosis. (u) D. See C for explanation.

: (8) 38. Clinical Intervention/Gastrointestinal/Nutritional A 34 year-old female 3 days status-post appendectomy complains of mild diffuse abdominal pain with associated nausea and vomiting. On examination you note diminished bowel sounds. There is generalized abdominal distension with mild tenderness on palpation. There are no signs of peritoneal inflammation. Abdominal radiographs show distended gas-filled loops of the small and large intestine. Abdominal CT scan shows no mechanical obstruction. Which of the following is the most appropriate management for this patient? A. Docusate sodium (Colace) B. Colonoscopic decompression C. Restrict oral intake D. Exploratory laparoscopy

(u) A. See C for explanation. (u) B. See C for explanation. (c) C. Most adynamic ileuses respond to restriction of oral intake and gradual advancement of diet as bowel function returns. (u) D. See C for explanation.

165. Clinical Therapeutics/Gastrointestinal/Nutritional A 27 year-old female is brought to the hospital after being found unconscious with a half-empty bottle of Hydrocodone/Acetominophen (Vicodin) in her hand. Which of the following is the most appropriate treatment for this patient? A. Flumazenil (Romazicon) B. Disulfiram (Antabuse) C. Naloxone (Narcan) D. Activated charcoal

(u) A. See C for explanation. (u) B. See C for explanation. (c) C. Nalxone is the antidote for opioid ingestion. (u) D. See C for explanation.

A 29 year-old female G1P1 presents to the office with a one-month history of amenorrhea and a positive home pregnancy test. The first day of her last menstrual period (LMP) was April 4. Using Nägele's rule what is her EDC? A. January 1 B. January 7 C. January 11 D. January 18

(u) A. See C for explanation. (u) B. See C for explanation. (c) C. Nägele's rule is LMP minus 3 months plus 7 days. April 4 minus 3 months equals January 4 plus 7 days equals January 11. (u) D. See C for explanation.

A patient receiving heparin therapy for 6 days for deep vein thrombosis develops thrombosis at the IV site. The INR is 1.1. The aPTT is 66 seconds and the platelet count is 47,000 down from 148,000 on admission. Which of the following is the most appropriate treatment? A. Protamine sulfate administration B. Platelet administration C. Discontinue the heparin D. Schedule the patient for plasmapheresis

(u) A. See C for explanation. (u) B. See C for explanation. (c) C. Once the diagnosis of heparin induced thrombocytopenia is suspected treatment should include immediate discontinuation of all forms of heparin and treatment with a direct thrombin inhibitor should begin. Platelet administration is generally not indicated. Protamine sulfate is used for heparin overdosage and plasmapheresis would not be indicated as the platelet count will resume when the heparin is discontinued. (u) D. See C for explanation.

175. Clinical Therapeutics/Obstetrics/Gynecology A 22 year-old nulliparous female undergoes laparoscopy for progressive dysmenorrhea and chronic pelvic pain. She is found to have minimal chocolate-colored cysts on the left ovary, which are cauterized at time of surgery. She is not currently sexually active, uses no contraception, but does plan to marry and have children soon. What is the recommended therapy for this patient at this time? A. Naproxen sodium (Anaprox) B. Leuprolide acetate depot (Lupron) C. Levonorgestrel and ethinyl estradiol (Seasonique) D. Danocrine (Danazol)

(u) A. See C for explanation. (u) B. See C for explanation. (c) C. Oral contraceptives are considered first line therapy for the treatment of endometriosis, often with the use of NSAIDs in combination. Continuous therapy, is often used to prevent secondary dysmenorrhea. Therapy causes a decidual reaction in the functioning endometriotic tissue decreasing symptoms. (u) D. See C for explanation.

What is the optimal INR for a patient with a mechanical mitral valve prosthesis on warfarin (Coumadin)? A. 1.5-2.5 B. 2.0-3.0 C. 2.5-3.5 D. 3.5-4.5

(u) A. See C for explanation. (u) B. See C for explanation. (c) C. Patients with mechanical MV prostheses should maintain an INR between 2.5-3.5. Aortic mechanical valves can be maintained at an INR of 2.0-3.0. (u) D. See C for explanation.

: (8) 104. Scientific Concepts/Cardiology What is the most common cause of secondary hypertension? A. Abnormal development of aortic elasticity B. Defect in natriuresis C. Renal parenchymal disease D. Sympathetic nervous system hyperactivity

(u) A. See C for explanation. (u) B. See C for explanation. (c) C. Renal parenchymal disease is the most common cause of secondary hypertension. HTN may result from diabetic and inflammatory glomerular diseases, tubular interstitial disease, and polycystic kidneys. Most cases are related to increased intravascular volume or increased activity of the rennin-angiotensin-aldosterone system. (u) D. See C for explanation.

Which of the following is the most common cause of infective endocarditis in an IV drug abuser? A. Haemophilus parainfluenza B. Enterococci C. Staphylococcus aureus D. Viridans streptococci

(u) A. See C for explanation. (u) B. See C for explanation. (c) C. S. aureus accounts for more than 60% of all cases of endocarditis in IV drug abusers. (u) D. See C for explanation.

Seizures that first manifest in early to middle adult life should be considered suspicious of which of the following causes? A. Cerebrovascular disease B. Encephalitis C. Tumor D. Idiopathic epilepsy

(u) A. See C for explanation. (u) B. See C for explanation. (c) C. Seizures that develop during adolescence and adult life are predominantly due to tumor, trauma, drug use, or alcohol withdrawal. (u) D. See C for explanation.

Which of the following laboratory values would be noted in a patient with primary hyperthyroidism? A. Increased TSH, increased free T4 B. Increased TSH, decreased free T4 C. Decreased TSH, increased free T4 D. Decreased TSH, decreased free T4

(u) A. See C for explanation. (u) B. See C for explanation. (c) C. Serum TSH is depressed and serum T3, T4 and uptake are usually all increased in patients with primary hyperthyroidism. (u) D. See C for explanation

A 30 year-old female presents with left wrist pain after slipping on the ice while walking to her car. On examination, pain is noted on palpation over the anatomical snuff box. X-ray of her wrist shows no identifiable fracture. Which of the following is the most appropriate treatment in this patient? A. No treatment necessary B. ACE wrap application C. Splint application D. Immediate orthopedic referral

(u) A. See C for explanation. (u) B. See C for explanation. (c) C. Tenderness in the snuff box should be treated as a suspected scaphoid fracture. The patient should be treated as if it is fractured and placed in a splint with a referral to an orthopedic specialist for further evaluation and repeat imaging. (u) D. See C for explanation.

A 65 year-old male presents with back pain two days after he was shoveling snow. The patient complains of pain in his low back that radiates into his buttocks, posterior thigh and calf, and the bottom of his foot. There is associated numbness of his lateral and plantar surface of his foot. Which of the following disc herniations is most likely to be affected? A. L3-L4 B. L4-L5 C. L5-S1 D. S1-S2

(u) A. See C for explanation. (u) B. See C for explanation. (c) C. The S1 nerve root impingement is most likely to occur from the herniation of the L5-S1 disc space. The S1 disc affects Achilles' reflex, the gastrocnemius and soleus muscles, and the abductor hallucis and gluteus maximus muscles. (u) D. See C for explanation.

Which of the following is typically noted on physical examination in a patient with diphtheria? A. papular rash on trunk B. supraclavicular adenopathy C. pharyngeal pseudomembranes D. splenomegaly

(u) A. See C for explanation. (u) B. See C for explanation. (c) C. The classic exam finding noted in diphtheria is a gray pharyngeal pseudomembrane. Rash, splenomegaly, and supraclavicular adenopathy are not noted in diphtheria. (u) D. See C for explanation.

Which of the following views on plain films is preferred to identify spondylolysis? A. Anterior B. Posterior C. Oblique D. Lateral

(u) A. See C for explanation. (u) B. See C for explanation. (c) C. The defect in the pars articularis (usually bilateral) is best visualized on the oblique projections on plain films. (u) D. See C for explanation.

A patient complains of loss of sensation at the level of the umbilicus. Which of the following dermatomes is affected? A. T6 B. T8 C. T10 D. T12

(u) A. See C for explanation. (u) B. See C for explanation. (c) C. The dermatome T10 is at the level of the umbilicus. (u) D. See C for explanation.

A patient is evaluated for an overblown sense of self-importance and sense of entitlement. He is described as being arrogant, envious, exploitative and lacking in empathy. He is prone to mood swings and though he has a productive career in business he has few meaningful personal relationships. Which of the following is the most appropriate management for this patient? A. Alprazolam (Xanax) B. Clozaril (Clozapine) C. Lithium (Lithobid) D. Olanzapine (Zyprexa)

(u) A. See C for explanation. (u) B. See C for explanation. (c) C. The diagnosis of narcissistic personality disorder is indicated in this scenario. His success in his career helps to lessen the probability of schizophrenia and/or delusions of grandeur. Antipsychotics are therefore not indicated and anxiolytics would not be helpful. Lithium can be used with this diagnosis if mood swings are prominent along with antidepressants to address the frequent co-morbid depression. (u) D. See C for explanation.

As a rule, solid foods such as cereal and fruits are best introduced into an infant's diet at approximately A. 1 to 3 weeks. B. 4 to 8 weeks. C. 4 to 6 months. D. 10 to 14 months.

(u) A. See C for explanation. (u) B. See C for explanation. (c) C. The inclusion of solid foods in the diet is best done at approximately 4-6 months. Before 4 to 6 months of age does not contribute significantly to the infant's health and it increases their risk of allergies and atopy. (u) D. After 6 months, meat, eggs, and starchy foods can be introduced.

A patient describes a history of intermittent and uncontrollable twitching of his right hand that spreads to involve the entire arm after a few minutes. Afterward, the arm is extremely weak. There are no other areas of involvement, sensory deficit or altered consciousness. What is the most likely seizure diagnosis? A. Absence B. Complex-partial C. Simple-partial D. Myoclonic

(u) A. See C for explanation. (u) B. See C for explanation. (c) C. The lack of altered consciousness and focal motor symptoms are strongly indicative of this type of seizure. (u) D. See C for explanation.

Hairy leukoplakia has the greatest prevalence of distribution on the A. palate. B. floor of the mouth. C. lateral tongue. D. gingiva.

(u) A. See C for explanation. (u) B. See C for explanation. (c) C. The lateral border of the tongue is where hairy leukoplakia is commonly seen. (u) D. See C for explanation.

What is the mechanism of action of salmeterol (Serevent) in the treatment of asthma? A. Anti-inflammatory B. Immunotherapy for specific allergens C. Relaxing of bronchial smooth muscle D. Reduction of leukotriene production

(u) A. See C for explanation. (u) B. See C for explanation. (c) C. The mechanism of action for salmeterol is the relaxation of bronchial smooth muscle. (u) D. See C for explanation.

Mastitis associated with breastfeeding is most commonly caused by what bacteria? A. Listeria monocytogenes B. Escherichia coli C. Staphylococcus aureus D. Streptococcus pyogenes

(u) A. See C for explanation. (u) B. See C for explanation. (c) C. The most common pathogen associated with postpartum mastitis is Staphylococcus aureus which arises from the nursing infant's throat and nose. (u) D. See C for explanation.

: (9) 114. History & Physical/Neurology A 24 year-old pregnant female presents with an abrupt onset of facial weakness and mild disturbance of taste this morning upon awaking. She denies drug use and other neurological complaints. Examination findings include a loss of forehead markings on right. Which of the following additional exam findings would likely be present? A. Agitation and word searching B. Contralateral unequal and dilated pupil C. Ipsilateral restriction of eye closure D. Deviated tongue upon protrusion

(u) A. See C for explanation. (u) B. See C for explanation. (c) C. The patient has signs and symptoms of Bell's palsy. With Bell's palsy, it is common to have cranial nerve VII paralysis which will not allow the patient to completely blink the eye on the affected side. (u) D. See C for explanation.

A 21 year-old female presents with dysuria. On examination of the urine, many squamous epithelial cells are noted. Which of the following is the next best step in the evaluation or treatment of this patient? A. Order urine culture and sensitivity B. Obtain renal ultrasound C. Repeat urinalysis with a clean catch sample D. Refer to a nephrologist

(u) A. See C for explanation. (u) B. See C for explanation. (c) C. The presence of many squamous epithelial cells indicates contamination with vaginal flora. The test should be repeated with a clean catch specimen. (u) D. See C for explanation.

: (27) 64. Scientific Concepts/Dermatology Which of the following is the single most important prognostic factor in regard to melanoma? A. Histopathology of the lesion B. Diameter of the lesion C. Thickness of the lesion D. Ulceration of the lesion

(u) A. See C for explanation. (u) B. See C for explanation. (c) C. The single most significant prognostic factor in melanoma is tumor thickness. Ten year survival rate in years according to thickness : < 1mm, 95%; 1-2mm, 80%; 2-4 mm, 55%; >4mm, 30%. (u) D. See C for explanation.

A 50 year-old female has a history of severe, stabbing pains, lasting only seconds, over the cheek and chin areas. She also experiences intense pain in these areas with chewing, washing her face and even with smiling. Examination reveals no sensory or motor deficits. Head computer tomography (CT) and erythrocyte sedimentation rate (ESR) are normal. She has been unresponsive to maximum doses of carbamazepine. What is the next best option for this patient? A. Combination simvastatin (Zocor) and sertraline (Zoloft) B. Bilateral deep brain stimulation C. Stereotactic (Gamma knife) radiosurgery D. High dose corticosteroid therapy

(u) A. See C for explanation. (u) B. See C for explanation. (c) C. Trigeminal neuralgia is generally responsive to Carbamazepine however, for those who fail to respond to medicinal therapy surgery is the next valid option. Though there are various procedures, Stereotactic (Gamma knife) radiosurgery has provided the best results. (u) D. See C for explanation.

A 17 year-old female is seen who has a history of eating large amounts of food at night 3 to 4 times weekly. These episodes are always followed by extreme guilt and either induced vomiting or hours of strenuous exercise. She excels at work and school and maintains many active relationships. Her physical examination reveals a normal BMI and an otherwise normal exam. What is the best initial intervention for this patient? A. Begin an anxiolytic B. Immediate hospitalization C. Start an antidepressant D. Test for substance abuse

(u) A. See C for explanation. (u) B. See C for explanation. (c) C. Uncomplicated bulimia generally does not require hospitalization as the physical manifestations encountered in bulimia are generally mild if present at all. Some patients with bulimia have concomitant substance abuse issues but her successful relationships and work activities lessen the probability. Antidepressants, not anxiolytics, have been found helpful in lessening the binge/purge cycles and improving overall well-being even outside of comorbid mood disorders. (u) D. See C for explanation.

: (27) 72. Diagnostic Studies/Endocrinology A supervised vasopressin challenge test is a significant part of the diagnostic workup for which of the following endocrine disorders? A. Addison disease B. Cushing disease C. Diabetes insipidus D. Diabetes mellitus

(u) A. See C for explanation. (u) B. See C for explanation. (c) C. Vasopressin is given in an initial dose of 5-10 mcg intranasally (or 1 mg subcutaneously or intravenously) with measurement of urine volume for 12 hours before and 12 hours after administration. Patients with central diabetes insipidus notice a distinct reduction in thirst and polyuria. Serum sodium should remain normal. (u) D. See C for explanation.

In a patient with dyshidrotic eczema on which area of the body would associated vesicles be found? A. Axillae B. Intertriginous areas C. Palms D. Upper back

(u) A. See C for explanation. (u) B. See C for explanation. (c) C. Vesicles associated with dyshidrotic eczema are most commonly found on the palms with the dorsum of the hands spared. The soles may also be affected in a similar fashion. (u) D. See C for explanation.

When is the recommendation for the next Pap smear in a 36 year-old patient with a history of 3 consecutive negative annual Pap smears and no history of cervical dysplasia? A. Yearly Pap smears B. Pap smear in 4 years C. Pap smear in 3 years D. Patient does not need any further Pap smears

(u) A. See C for explanation. (u) B. See C for explanation. (c) C. Women who have had 3 consecutive negative annual Pap smears results may be screened every 2 or 3 years if they are 30 or older with no history of CIN 2 or 3, immunosuppression, HIV infection, or diethylstilbestrol (DES) exposure in utero. (u) D. See C for explanation.

167. Diagnosis/Pulmonology A 46 year-old patient presents with a 4 year history of worsening cough, shortness of breath and dyspnea. He admits to bouts of bronchitis 5-6 times yearly but otherwise denies fevers, weight loss or hemoptysis. He has worked the past 30 years as a sandblaster. His chest radiograph reveals "egg-shell" calcification of the hilar lymph nodes and numerous ground-glass densities throughout both lung fields. Which of the following is the most likely diagnosis? A. Asthma B. Chronic bronchitis C. Silicosis D. Emphysema

(u) A. See C for explanation. (u) B. The criteria for chronic bronchitis are a daily productive cough for three consecutive months for at least two consecutive years. (c) C. The prolonged occupational exposure and chest radiograph findings in a symptomatic patient make the diagnosis. Both a restrictive and obstructive lung pattern can occur with silicosis. The deposition of silica along with its cytotoxic affects are what result in the mixed pulmonary dysfunction. (u) D. See C for explanation.

: (8) 160. History & Physical/Gastrointestinal/Nutritional A 41 year-old female presents with RUQ pain. She is febrile and slightly tachycardic. Examination demonstrates abdominal tenderness and abrupt cessation of inspiration with palpation under the right costal margin during deep inspiration. Ingestion of which of the following is most likely to have proceeded the onset of this condition? A. Alcoholic beverage B. Dairy product C. High-fiber meal D. Fatty meal

(u) A. See D for explanation. (u) B. See D for explanation. (u) C. See D for explanation. (c) D. Cholecystitis presents with complaint of RUQ pain and on exam there may be a positive Murphy's sign. It is associated with meals high in fat. Alcohol, dairy products and high fiber meals are not associated with cholecystitis and would not present as fever, tenderness and positive Murphy's sign.

An 18 month-old male presents with his parents who report symptoms of a barking cough and intermittent stridor that has worsened over the past 12 hours. They note improvement in symptoms when he was taken outdoors to the cool night air. Which of the following is the most likely organism causing this patient's symptoms? A. Rubeola virus B. Adenovirus C. Influenza virus D. Para-influenza virus

(u) A. See D for explanation. (u) B. See D for explanation. (u) C. See D for explanation. (c) D. Croup is most often caused by parainfluenza virus.

: (8) 110. History & Physical/Obstetrics/Gynecology What physical exam finding indicates the delivery of the placenta is imminent in the third stage of labor? A. Decrease in vaginal bleeding B. Increase in uterine contractions C. Shortening of the umbilical cord length D. Uterus is higher and more globular in abdomen

(u) A. See D for explanation. (u) B. See D for explanation. (u) C. See D for explanation. (c) D. Delivery of the placenta occurs in the third stage of labor and is imminent when the uterus rises in the abdomen, becoming more globular in configuration, indicating that the placenta has separated and has entered the lower uterine segment. A gush of blood and/or lengthening of the umbilical cord, as well as a rising uterus 46 are the three classic signs of placental separation.

Which of the following groups is most likely to present with Duchenne's muscular dystrophy? A. Adolescent females B. Middle-aged males C. Infant females D. Toddler-aged males

(u) A. See D for explanation. (u) B. See D for explanation. (u) C. See D for explanation. (c) D. Duchenne's muscular dystrophy, a genetic defect on the short arm of the X chromosome, affects toddler-aged males.

: (8) 22. Scientific Concepts/Cardiology Empiric treatment for infective endocarditis should target which organism? A. Haemophilus influenzae B. Moraxella catarrhalis C. Mycoplasma pneumoniae D. Staphylococcos aureus

(u) A. See D for explanation. (u) B. See D for explanation. (u) C. See D for explanation. (c) D. Empiric regimens for endocarditis while culture results are pending should include agents active against staphylococci, streptococci, and enterococci.

A 57 year-old male presents with episodic diplopia over the past two months. Symptoms progressed over the last two days with the onset of bilateral facial weakness made worse with repetitive use. Weakness improves somewhat with rest. He denies fever, headache or areas of pain. Exam reveals a nasal voice, drooping eyelids and a normal sensory exam. Which of the following is the most likely diagnosis? A. Multiple sclerosis B. Guillain-Barre syndrome C. Lambert-Eaton syndrome D. Myasthenia gravis

(u) A. See D for explanation. (u) B. See D for explanation. (u) C. See D for explanation. (c) D. Episodic diplopia progressing to proximal weakness worsened by repetition and lacking sensory deficits is typical of myasthenia gravis. In Lambert-Eaton syndrome muscular responses actually improve with repetition and the weakness in Guillain-Barre syndrome always begins peripherally.

: (8) 182. Diagnostic Studies/Obstetrics/Gynecology Which of the following lab findings is suggestive of menopause? A. Decrease in FSH, increase in LH B. Decrease in FSH, decrease in LH C. Increase in FSH, increase in estradiol D. Increase in FSH, decrease in estradiol

(u) A. See D for explanation. (u) B. See D for explanation. (u) C. See D for explanation. (c) D. FSH greater than 30 mIU/mL is diagnostic of menopause. LH is also elevated in menopause. Estradiol starts to decrease around menopause to levels of 10-15 pg/mL.

: (8) 40. Clinical Intervention/Obstetrics/Gynecology A 28 year-old G1 P0 female at 36 weeks gestation, presents to labor and delivery after she experienced a sudden gush of fluid that soaked her underwear. She admits to urinary frequency, but denies any contractions, vaginal bleeding or dysuria. Which of the following is the most appropriate next step in the management of this patient? A. Induction of labor with oxytocin (Pitocin) B. Digital examination for the presence of cervical dilation C. Urinalysis for urinary tract infection D. Sterile speculum examination for vaginal pooling and ferning

(u) A. See D for explanation. (u) B. See D for explanation. (u) C. See D for explanation. (c) D. Fluid passing through the vagina is presumed to be amniotic fluid until proven otherwise. Urinary leakage is common during pregnancy, especially near term and can be difficult for the patient to decifer between urinary incontinence and premature rupture of membranes.

A 9 year-old patient presents with conjunctivitis after swimming at the local pool. On examination, there is visible lid edema with redness of the palpebral conjunctiva, copious watery discharge, and scanty exudate. The sanitation system of the public pool is through the use of a salt water system; therefore, the possibility of a chemical induced conjunctivitis is almost non-existent. Which of the following should be instituted to prevent the sequalae of the condition? A. Ketorolac tromethamine (Acular) B. Dexamethasone opthalmic C. Naphazoline HCL (Naphcon A) D. Sulfacetamide opthalmic

(u) A. See D for explanation. (u) B. See D for explanation. (u) C. See D for explanation. (c) D. One of the most common causes of viral conjunctivitis is adenovirus type 3. Contaminated swimming pools can be source of infection. Topical sulfonamides prevent secondary bacterial infection.

What is the recommended target LDL to reduce the risk of coronary artery disease in a diabetic patient? A. 200 mg/dL B. 160 mg/dL C. 130 mg/dL D. 100 mg/dL

(u) A. See D for explanation. (u) B. See D for explanation. (u) C. See D for explanation. (c) D. The National Cholesterol Education Program clinical practice guidelines have designated diabetes as a coronary risk equivalent and have recommended that patients with diabetes should have an LDL cholesterol goal of 100 mg/dL.

Which of the following is the most likely organism in a 2 year-old child with acute otitis media? A. Staphylococcus aureus B. Moraxella catarrhalis C. Pseudomonas aeruginosa D. Streptococcus pneumoniae

(u) A. See D for explanation. (u) B. See D for explanation. (u) C. See D for explanation. (c) D. The most common pathogens in children with acute otitis media are Streptococcus pneumoniae, Haemophilus influenzae, and Streptococcus pyogenes.

: (12) 202. Diagnosis/Pulmonology On a frontal chest radiograph view, you notice a visceral pleural line with a radiolucent area devoid of vascular and pulmonary markings on the right side only. Which of the following is the most likely diagnosis? A. Asthma B. Emphysema C. Pneumonia D. Pneumothorax

(u) A. See D for explanation. (u) B. See D for explanation. (u) C. See D for explanation. (c) D. Unilateral findings of a visceral pleural line with no vascular pulmonary markings in between this line in the chest wall are almost always indicative of a pneumothorax. Certain conditions like emphysema can result in a pneumothorax but these findings are not diagnostic of emphysema.

Adult patients with a compromised immune system should not receive which of the following immunizations? A. Hepatitis A B. Pneumococcal C. Influenza D. Varicella

(u) A. See D for explanation. (u) B. See D for explanation. (u) C. See D for explanation. (c) D. Varicella vaccine is a live vaccine and is contraindicated in those individuals with a compromised immune system. Hepatitis A, pneumococcal and influenza are recommended for patients with a compromised immune system.

144. Scientific Concepts/Gastrointestinal/Nutritional A 10 year-old African immigrant presents 2 weeks after arriving in the US describing dry eyes and difficulty with night vision. Examination reveals numerous, small white conjunctival patches bilaterally. This patient's symptoms are most likely due to which of the following deficiencies? A. Vitamin B6 B. Vitamin C C. Vitamin D D. Vitamin A

(u) A. See D for explanation. (u) B. See D for explanation. (u) C. See D for explanation. (c) D. Vitamin A deficiency is one of the most common causes of blindness in developing countries. It is also seen secondary to malabsorption disorders and frequent mineral oil laxative use. Night blindness is one of the earliest symptoms.

Prolonged use of a proton pump inhibitor can lead to low levels of which of the following nutrients? A. Folic acid B. Magnesium C. Vitamin B6 D. Vitamin B12

(u) A. See D for explanation. (u) B. See D for explanation. (u) C. See D for explanation. (c) D. Vitamin B12 requires gastric acid for absorption in the stomach. Prolonged use of a proton pump inhibitor suppresses gastric acid production.

: (8) 8. Health Maintenance/Psychiatry/Behavioral Medicine Which of the following is the drug of choice for the prevention of Wernicke's encephalopathy? A. Folic acid B. Calcium gluconate C. Vitamin B 12 D. Thiamine

(u) A. See D for explanation. (u) B. See D for explanation. (u) C. See D for explanation. (c) D. Wernicke's encephalopathy, also called alcoholic encephalopathy, is an acute neurological disorder of thiamine deficiency. Patients are given 100mg po BID to TID for 1 to 2 weeks or parenterally 100mg in each liter of glucose.

Which of the following is the most appropriate therapeutic agent for acute influenza? A. azithromycin (Zithromax) B. acyclovir (Zovirax) C. tetracycline (Sumycin) D. zanamivir (Relenza)

(u) A. See D for explanation. (u) B. See D for explanation. (u) C. See D for explanation. (c) D. Zanamivir is an anti-viral agent that is active against the influenza virus.

An O2 saturation of 90% corresponds to what PO2 value? A. 90 mmHg B. 80 mmHg C. 70 mmHg D. 60 mmHg

(u) A. See D for explanation. (u) B. See D for explanation. (u) C. See D for explanation. Copyright © 2010. Physician Assistant Education Association 84 (c) D. O2 sat values above 90% correspond with a PO2 >70 mmHg and values less than 94% represent hypoxemia. Less than 90% O2 sat warrants measurement of arterial blood gasses.

Which of the following forms of lung cancer is associated with the poorest prognosis? A. squamous cell B. adenocarcinoma C. large cell D. small cell

(u) A. See D for explanation. (u) B. See D for explanation. (u) C. See d for explanation. (c) D. Small cell lung cancer is the most common type of lung cancer that is metastatic at the time of discovery, and therefore has the poorest prognosis.

Which of the following can be used to treat chronic bacterial prostatitis? A. Penicillin B. Cephalexin (Keflex) C. Nitrofurantoin (Macrobid) D. Levofloxacin (Levaquin)

(u) A. See D for explanation. (u) B. See D for explanation. (u)\C. See D for explanation. (c) D. Chronic bacterial prostatitis (Type II prostatitis) can be difficult to treat and requires the use of fluoroquinolones or trimethoprim-sulfamethoxazole, both of which penetrate the prostate.

: (8) 157. Clinical Therapeutics/Hematology A 13 year-old male presents with a two day history of fatigue and fever. He also complains of recent epistaxis and bleeding gums. Examination demonstrates pale skin with palpable purpura. Laboratory evaluation demonstrates pancytopenia with 42% blasts and a positive D-dimer. Which of the following treatment regimens is most likely to bring remission? A. Single agent chemotherapy B. Combination chemotherapy C. Radiation therapy D. Chemotherapy and radiation therapy

(u) A. Single agent chemotherapy is not sufficient for treatment of ALL. (c) B. Combination chemotherapy is key in the treatment of acute lymphocytic leukemia and produces remission in 90% of patients. (u) C. Radiation therapy is not recommended for ALL. (u) D. Radiation and chemotherapy is not appropriate treatment for ALL.

A 34 year-old female construction worker presents with episodic blanching of her fingers when exposed to cold weather. The physical examination of her extremities and digits is normal at this time. Which of the following is the most likely diagnosis? A. Sjogren syndrome B. Scleroderma C. Raynaud phenomenon D. Systemic lupus erythematosus

(u) A. Sjogren syndrome is a secondary cause of Raynauds, but it is not the disorder causing the symptoms. (u) B. Scleroderma is a secondary cause of Raynauds, but it is not the disorder causing the symptoms. (c) C. Raynauds phenomenon is caused by vascular spasm when exposed to cold or stressful situations. (u) D. Systemic Lupus Erythematosus is a secondary cause of Raynauds, but it is not the disorder causing the symptoms.

A 37 year-old right-handed landscaper comes to the office for evaluation of recurrent itching and stinging of the skin on his right hand. Physical examination reveals confluent papules, vesicles, erosions and crusts on the dorsum of his right hand. Which of the following is the most appropriate initial diagnostic study in this patient? A. Skin scraping and microscopy B. Patch testing C. RAST testing D. Skin biopsy

(u) A. Skin scraping and microscopy are used to identify the organism, ova or feces in the laboratory diagnosis of scabies. (c) B. In patch testing, substances are applied to the skin in shallow cups and left in place for 24-48 hours. Sensitivity to an allergen is confirmed by a papular vesicular reaction that occurs within 48-72 hours of the laboratory testing. (u) C. RAST testing is more specific and would not be done as the initial diagnostic study in this patient. (u) D. A skin biopsy is indicated for all skin lesions that are suspected to be neoplasms.

A 12 year-old female presents for a routine sports physical. The physical exam reveals asymmetry of the posterior chest wall on forward bending. This is most consistent with which of the following? A. spondylolysis B. spondylolisthesis C. scoliosis D. herniated disc

(u) A. Spondylolysis presents with limitation of lumbar flexibility and tight hamstring muscles. (u) B. Spondylolisthesis presents with reduced lumbar lordosis and sacral kyphosis. (c) C. Asymmetry of the posterior chest wall on forward bending is the most striking and consistent abnormality in patients with idiopathic scoliosis. (u) D. Herniated disc presents with lumbar muscle spasm and a positive straight leg test.

A recent Haitian immigrant presents to the clinic for an employment physical examination before starting work at a local hospital. The patient has a history of receiving bacilli Calmette-Guerin (BCG) vaccination. Screening for tuberculosis for this employee should include which of the following tests? A. Sputum induction B. PPD skin test C. Chest x-ray D. No screening needed

(u) A. Sputum induction should not be used as a screening test for tuberculosis. (u) B. False-positive tuberculin skin test reactions can occur in persons previously vaccinated against M. tuberculosis with BCG. PPD should be avoided as a screening test in these patients. (c) C. Chest x-ray is the test of choice in patients where the PPD test is not indicated or in high-risk individuals. (u) D. This patient has recently emigrated from a possible endemic region and should be screened for tuberculosis.

A patient diagnosed with Barrett's esophagus is at an increased risk for the development of what type of cancer? A. Squamous cell B. Transitional cell C. Adenocarcinoma D. Atypical carcinoid

(u) A. Squamous cell is not typical for esophageal cancer. (u) B. Transitional cell is a cancer of the bladder. (c) C. The most serious complication of Barrett's esophagus is esophageal adenocarcinoma. (u) D. Atypical carcinoid is more typical of lung cancer.

What type of chest pain is most commonly associated with a dissecting aortic aneurysm? A. Squeezing B. Dull, aching C. Ripping, tearing D. Burning

(u) A. Squeezing pain is more characteristic of angina or esophageal pain. (u) B. Dull, aching pain is more characteristic of chest wall pain, possibly angina, or anxiety. (c) C. A dissecting aortic aneurysm often presents with a very severe ripping, tearing-like pain. (u) D. Burning pain is more characteristic of esophageal reflux, esophagitis, or tracheobronchitis.

151. Clinical Intervention/Orthopedics/Rheumatology Which of the following interventions is initially indicated for helping to relieve the symptoms of plantar fasciitis? A. Steroid injections B. Short leg walking cast C. Arch supports D. Surgical release

(u) A. Steroid injections can be used to relieve symptoms but may be harmful by causing rupture if given in the plantar tendon greater than 3 or 4 times. (u) B. Short leg walking cast may be used for severe cases refractory to initial treatment. (c) C. Arch supports, NSAIDs and stretching exercises are the initial interventions to help relieve symptoms. (u) D. Surgical release of the plantar tendon is usually reserved for patients who fail all therapies.

A 32 year-old presents with a 3-day history of diarrhea. The patient denies blood, mucus, or night awakening with diarrhea. He recently returned from a business trip to Canada. On physical examination, the patient is afebrile and vital signs reveal BP 115/80, pulse is 76, and respirations are 14. The abdominal examination reveals hyperactive bowel sounds, but is otherwise unremarkable. Which of the following is the most appropriate initial intervention? A. Stool for culture, ova, and parasites B. Proctosigmoidoscopy C. Metronidazole (Flagyl) D. Supportive treatment

(u) A. Stool culture and examination for ova and parasites are indicated when the diarrhea has persisted longer than 3 weeks or is associated with abdominal pain, fever, and/or bloody stools. (u) B. Proctosigmoidoscopy is indicated when inflammatory bowel disease is suspected on the basis of fever, bloody diarrhea, or abdominal pain. (u) C. Metronidazole is indicated with a confirmed diagnosis of Giardia lamblia or amebic disease. (c) D. Symptomatic treatment, including dietary management and over-the-counter antidiarrheals, is indicated for afebrile patients with watery diarrhea of less than 5 days duration.

The most distinctive sign of pertussis is A. stridor without cough. B. productive cough with basilar rales. C. loose cough with coarse rhonchi. D. paroxysmal cough with crowing inspiration.

(u) A. Stridor without cough suggests foreign body aspiration. (u) B. A productive cough and rales suggest pneumonia. (u) C. Coarse rhonchi and a loose cough suggest bronchitis. (c) D. A paroxysmal cough with a loud inspiration (the whoop) is noted in pertussis.

Which of the following medications used to treat rheumatoid arthritis is contraindicated in patients with chronic hepatitis? A. Sulfasalazine B. Methotrexate C. Minocycline D. Infliximab

(u) A. Sulfasalazine is a second line medication that can cause neutropenia and thrombocytopenia. (c) B. Methotrexate is contraindicated in patients with chronic hepatitis. (u) C. Minocycline is used for early rheumatoid arthritis with minimal adverse effects. (u) D. Infliximab is a tumor necrosing factor inhibitor and should be used cautiously in patients with heart failure.

In addition to tobacco products, which of the following is also considered a major risk factor in the development of oral cancer? A. Sun exposure B. Alcohol abuse C. Occupational exposure D. History of oral candidiasis

(u) A. Sun exposure is a risk factor for cancer of the lip, but is not considered a major risk factor for oral cancer. (c) B. Major risk factors for development of oral cancer are use of tobacco products and alcohol abuse. (u) C. While occupational exposures and presence of premalignant lesions, such as leukoplakia, are risk factors for development of oral cancer, they are not considered major risk factors. (u) D. History of oral candidiasis has no correlation to development of oral cancer.

: (8) 193. Clinical Intervention/Pulmonology A 75 year-old man with a long history of COPD presents with acute onset of worsening dyspnea, increased productive cough, and marked agitation. While in the emergency department he becomes lethargic and obtunded. His ABG reveals a PaO2 40 mmHg, PaCO2 65 mmHg, and arterial pH 7.25. Which of the following is the most appropriate management at this point? A. Oxygen supplementation with a 100% non-rebreather mask B. Noninvasive positive pressure ventilation (NIPPV) C. Endotracheal intubation and mechanical ventilation D. Emergency tracheostomy

(u) A. Supplemental oxygen and positive pressure ventilation are inadequate for patients with overt respiratory failure. (u) B. See A for explanation. (c) C. This patient is in severe respiratory arrest with markedly impaired mental status; conventional mechanical ventilation is required. (h) D. Tracheostomy is indicated for an obstructed airway.

A 75 year-old man with a long history of COPD presents with acute onset of worsening dyspnea, increased productive cough, and marked agitation. While in the emergency department he becomes lethargic and obtunded. His ABG's reveal a PaO2 40 mmHg, PaCO2 65 mmHg, and arterial pH 7.25. Which of the following is the most appropriate management at this point? A. oxygen supplementation with a 100% non-rebreather mask B. noninvasive positive pressure ventilation (NIPPV) C. endotracheal intubation and mechanical ventilation D. emergency tracheostomy

(u) A. Supplemental oxygen and positive pressure ventilation are inadequate for patients with overt respiratory failure. (u) B. See A for explanation. (c) C. This patient is in severe respiratory arrest with markedly impaired mental status; conventional mechanical ventilation is required. (h) D. Tracheostomy is indicated for an obstructed airway.

A 60 year-old female injured her right wrist when she slipped and fell onto her outstretched hand. Radiographs show a fracture through the metaphysis of the distal radius with dorsal displacement and angulation. Which of the following splints is the best method of temporary immobilization? A. Dorsal forearm B. Ulnar gutter C. Volar forearm D. Volar with thumb spica

(u) A. The dorsal forearm splint is best used as an alternative to the ulnar or radial gutter splint for protection of fractures of metacarpals two through five. (u) B. Ulnar gutter splints are best for treatment of stable fractures and dislocations of the ulnar wrist and metacarpals. (c) C. The volar forearm splint is best for temporary immobilization of forearm, wrist and hand fractures and is the splint of choice for Colles' fracture. (u) D. A volar splint with thumb spica is used to immobilize the first metacarpophalangeal joint and is useful for scaphoid fractures.

A person with type 2 diabetes who requires insulin brings in a home monitoring record for review. The current insulin regimen is 15 units of NPH with 5 units of regular in the morning, and 10 units of NPH with 5 units of regular in the evening. A trend of elevated blood sugar readings occurring at noon should prompt an insulin increase of which of the following? A. Morning NPH dose B. Morning regular dose C. Evening NPH dose D. Evening regular dose

(u) A. The effectiveness of the morning NPH dose would be reflected by the 5 PM sugar reading. (c) B. The elevated noon blood sugar reading indicates a need to increase the morning regular dose. (u) C. The morning glucose reading would reflect the effectiveness of the evening NPH. (u) D. The evening (10 PM) reading would reflect the effect of the evening regular dose.

A 29 year-old female with history of IV drug abuse presents with ongoing fevers for three weeks. She complains of fatigue, worsening dyspnea on exertion and arthralgias. Physical examination reveals a BP of 130/60 mmHg, HR 90 bpm, regular, RR 18, unlabored. Petechiae are noted beneath her fingernails. Fundoscopic examination reveals exudative lesions in the retina. Heart examination shows regular rate and rhythm, there is a grade II-III/VI systolic murmur noted, with no S3 or S4. Lungs are clear to auscultation bilaterally, and the extremities are without edema. Which of the following is the diagnostic study of choice in this patient? A. Electrocardiogram B. CT angiogram of the chest C. Cardiac catheterization D. Transesophageal echocardiogram

(u) A. See D for explanation. (u) B. See D for explanation. (u) C. See D for explanation. (c) D. This patient's signs and symptoms are consistent with infective endocarditis. The diagnostic study of choice would be a transesophageal echocardiogram.

In addition to tremor, which of the following are cardinal symptoms of Parkinson disease? A. Cognitive decline and rigidity B. Personality change and bradycardia C. Eye movement abnormalities and hyperkinesias D. Rigidity and bradykinesia

(u) A. See D for explanation. (u) B. See D for explanation. (u) C. See D for explanation. (c) D. Tremor, rigidity, bradykinesia and postural instability are the cardinal features of Parkinsonism and may be present in any combination.

A stuporous patient is brought to the emergency room with a five day history of progressive lethargy and confusion along with polyuria and polydipsia. On examination the patient is dehydrated and is without Kussmaul respirations. Serum glucose is 1200 mg/dL (75-110 mg/dL), serum sodium 150 mEq/L (136-146 mEq/L) serum pH is 7.5 and serum osmolality is 320 mosm/kg (280-300 mosm/kg). Urinalysis reveals no ketones. What is the most likely diagnosis? A. Primary hyperaldosteronism B. Diabetic ketoacidosis C. Lactic acidosis D. Hyperglycemic hyperosmolar state

(u) A. The extreme hyperglycemia eliminates primary hyperaldosteronism as the primary diagnosis. (u) B. The lack of acidemia, ketones and Kussmaul respiratory pattern help to eliminate this as the primary diagnosis. (u) C. The lack of a Kussmaul respiratory pattern greatly lessens the likelihood of lactic acidosis. (c) D. Extreme hyperglycemia with normal pH and negative ketones are hallmark for this clinical picture.

What phase of the female menstrual cycle occurs at the time of elevated estrogen and LH/FSH surge? A. Follicular phase B. Proliferative phase C. Ovulation D. Secretory phase

(u) A. The follicular phase begins with the onset of menses (day 1 of the menstrual cycle) and ends on the day of the LH surge. (u) B. Progesterone causes differentiation of the endometrial components and converts proliferative endometrium into a secretory endometrium. (c) C. Ovulation occurs within 30-36 hours of the LH surge and at the time of elevated estrogen. (u) D. Secretory phase occurs when estrogen is elevated.

A 32 year-old female presents with an 8-week history of a burning sensation deep in the chest, which starts about 30 minutes after she eats and worsens when she lays down at night. She has had some relief with an antacid and denies dysphagia. Which of the following interventions would be beneficial for this patient? A. Raising the foot of the bed B. Taking Cimetidine (Tagamet) 30 minutes after a meal C. Eating 3 large meals per day D. Avoiding mint

(u) A. The head of the bed should be raised 6 inches to allow gravity to decrease reflux. (u) B. H2 blockers have a 30-minute onset of action and so are best taken 30 minutes before a meal known to cause reflux. (u) C. Patients usually do better with smaller, more frequent meals, along with the recommendation to avoid eating 3 hours before bedtime. (c) D. Both peppermint and chocolate are substances known to aggravate reflux.

Which of the following is the mechanism of action of infliximab (Remicade)? A. block lymphocyte proliferation B. direct effect on smooth muscle C. monoclonal antibody that binds to tumor necrosis factor D. 5-HT antagonist

(u) A. The immunomodulators (6-MP) work by blocking lymphocyte proliferation. (u) B. The mechanism of action of mebeverine is direct effect on smooth muscle in irritable bowel syndrome. (c) C. Infliximab is a monoclonal antibody that binds to tumor necrosis factor. (u) D. The anti-nausea and vomiting medications like ondansetron work by blocking 5-HT receptors

Which of the following x-ray views will show the presence of a "Scotty dog" deformity seen with spondylolysis? A. Lateral B. Oblique C. Anteroposterior D. Open-mouth odontoid

(u) A. The lateral view is the most appropriate for evaluation of the possible presence of spondylolisthesis, not spondylolysis. (c) B. Spondylolysis results from a defect through the pars interarticularis, which is seen as a defect in the neck of the "Scotty dog" on the oblique view. (u) C. Anteroposterior views will show the alignment of the spinous processes, but not a defect in the pars interarticularis. (u) D. The open-mouth odontoid view is used to visualize the odontoid process and the relationship between the C1 and C2 vertebrae.

An immunocompromised patient presents with signs and symptoms consistent with Legionella pneumophila who has not responded to initial antibiotic therapy with a macrolide. Which of the following should be added? A. Clarithromycin (Biaxin) B. Rifampin (Rifadin) C. Levofloxacin (Levaquin) D. Amoxicillin-clavulanate (Augmentin)

(u) A. The macrolides (Clarithromycin) and fluoroquinolones (Levofloxacin) should be used for initial treatment, but not for adding to failed treatments when a macrolide was already used. (c) B. Rifampin should be used as an adjunct in patients with either a macrolide or quinolone antibiotic, who have failed therapy, are immunocompromised or have severe illness. (u) C. See A for explanation. (u) D. Legionella pneumophila does not respond to Beta-lactam antibiotics.

A 25 year-old G2P2 married female presents to the clinic for birth control counseling. Her past history includes deep vein thrombosis with her last pregnancy. She does not want another pregnancy for at least 4 years. The birth control method that would be best for this patient is which of the following? A. Vaginal ring B. Transdermal patch C. Combined oral contraceptive pill D. Levonorgestrel releasing IUD

(u) A. The risk of venous thromboembolism with combined hormone in ring is similar to the oral contraceptive pill (OCP). (u) B. The risk of venous thromboembolism with combined hormone in patch is similar to OCP. (u) C. The use of oral OCPs triples a user's risk of venous thromboembolism. (c) D. IUD releases potent progestin only; is not a combination contraceptive.

An 18 year-old male is involved in a motor vehicle accident with a question of cervical spine fracture. What is the imaging test of choice to initially evaluate this patient and clear his cervical c-spine? A. Positron emission tomography B. Magnetic resonance imaging C. Computed tomography D. Lateral radiograph

(u) A. There is no role for positron emission tomography in suspected cervical spine injury. (u) B. MRI and CT of the spine may be performed in the setting of acute cervical spine injury when a major fracture or dislocation is identified. (u) C. See B for explanation. (c) D. Cervical spine x-rays are most commonly used as the initial screen for cervical spine injury. A cervical spine series consists of a lateral view, anteroposterior (AP) view, and an odontoid view. The lateral view detects up to 80% of traumatic spine injuries.

Which of the following can be a very serious consequence of using antidiarrheals in a patient with inflammatory bowel disease? A. Lymphoma B. Toxic megacolon C. Bone marrow suppression D. Delayed serum sickness-like reaction

(u) A. There is suggestion that lymphoma may result from the use of infliximab, an anti-TNF agent, utilized in refractory cases of Crohn's disease; however a clear link between the development of lymphoma and the use of infliximab has not been established. (c) B. Antidiarrheals may cause the development of toxic megacolon when used by patients with active severe inflammatory bowel disease. (u) C. Bone marrow suppression is a side effect of methotrexate, which is being increasingly used in the treatment of Crohn's disease. (u) D. Delayed serum sickness-like reaction is another potential side effect of infliximab.

A 26 year-old man presents to the emergency room complaining of shortness of breath, palpitations, and tingling of the lips and fingers. He appears anxious and describes a sensation of impending doom. His ECG and plain chest radiograph are normal. Which of the following arterial blood gas findings would you expect in this patient? A. pH 7.32 pCO2 49 mm Hg bicarbonate 24 mEq/L B. pH 7.40 pCO2 40 mm Hg bicarbonate 25 mEq/L C. pH 7.50 pCO2 23 mm Hg bicarbonate 21 mEq/L D. pH 7.52 pCO2 40 mm Hg bicarbonate 44 mEq/L

(u) A. These values are consistent with a respiratory acidosis. (u) B. These values fall within the normal range. (c) C. This patient is suffering an acute anxiety attack and resultant hyperventilation and respiratory alkalosis. Labs show alkaline pH, depressed pCO2 and low to normal bicarbonate. (u) D. These values suggest metabolic alkalosis.

A 30 year-old female presents to the emergency department with a syncopal episode. She has a history of irregular menstrual cycles and infertility. She has scanty, persistent vaginal bleeding and sharp pelvic pain. A left adnexal mass is palpated. The most likely diagnosis is A. placenta abruptio. B. ectopic pregnancy. C. pelvic inflammatory disease. D. ruptured ovarian cyst.

(u) A. This is primarily a third trimester cause of vaginal bleeding. (c) B. Infertility increases the risk of developing ectopic pregnancy. The onset of vaginal bleeding, pelvic pain, and formation of an adnexal mass makes this the most likely diagnosis. (u) C. Pelvic inflammatory disease typically presents with fever, abdominal pain, purulent vaginal discharge, and cervical motion tenderness. (u) D. Acute pelvic pain may occur secondary to bleeding from the rupture of a hemorrhagic ovarian cyst, but no adnexal mass would be palpable on pelvic examination

A patient is 12 hours post-closed reduction of a tibial fracture and is in a long leg cast. Despite pain medication the patient complains of unrelieved pain and cannot move his toes. Which of the following is the most appropriate clinical intervention in this patient? A. Elevate the leg B. Bivalve the cast C. Encourage ambulation D. Place on PCA pump

(u) A. This measure may be used while the necessary equipment is obtained to bivalve the cast, but it will not treat the underlying condition. (c) B. The cast for this patient is too tight and the pressure needs to be released. Bivalving the cast is the best option for this patient. (u) C. The cast for this patient is too tight and the pressure needs to be released. Ambulation is not appropriate in this patient. (u) D. Increasing the patient's pain medication will not treat the underlying condition.

A 43 year-old male with a history of a right medial meniscectomy and a strong family history of osteoarthritis presents to the clinic for a routine physical exam. He states he is very active and runs 20-25 miles a week and competes routinely in 5 km races. He is 5' 10" and 160 lbs, BP is 128/76 and P 72. His physical examination is unremarkable. Which of the following would you recommend to this patient to delay the onset of osteoarthritis? A. weight loss B. corticosteroid injections C. 1500 mg of calcium daily D. consider swimming or biking instead of running

(u) A. The patient's BMI is 23. Obesity is not a contributing factor for this patient. (u) B. Corticosteroid injections may be used for symptomatic treatment but are not used for prevention. (u) C. Calcium supplementation is indicated for osteoporosis prevention. (c) D. Swimming and biking would promote excellent joint motion and muscle strength and void the high-impact of competitive short-distance running.

Which of the following findings confirm the adequacy of a sputum specimen for Gram stain and culture? A. Decreased red blood cells B. Decreased bronchial epithelial cells C. Increased Gram positive cocci D. Increased polymorphonuclear leukocytes

(u) A. The presence or absence of red blood cells is not a criterion for determining the adequacy of a sputum sample. (u) B. The presence of bronchial epithelial cells confirms the sample came from the lower respiratory tract. (u) C. The presence of Gram positive cocci has no bearing on the adequacy of the sputum sample. (c) D. The presence of increased polymorphonuclear leukocytes and the absence of squamous epithelial cells are the criteria utilized to evaluate the adequacy of a sputum sample.

A 30 year-old female presents to the clinic for her 6 week post partum examination. She was diagnosed with gestational diabetes mellitus during her pregnancy and was successfully treated with diet alone. Which of the following patient education statements is most appropriate for this patient? A. The risk of gestational diabetes occurring in future pregnancies is very low. B. The risk of developing diabetes in 10-15 years is relatively low. C. Lifestyle modification, diet & exercise will not prevent her from developing diabetes D. Glucose testing every 1-3 years is recommended

(u) A. The risk of recurrence in future pregnancies is 60%. (u) B. She has a 50-60% risk of developing diabetes within 10-15 years. (u) C. Lifestyle modification may delay or prevent developing diabetes. (c) D. Those with normal glucose should be reassessed every 3 years; those with prediabetes should be assessed annually.

Which of the following is the earliest and most reliable clinical manifestation of pre-eclampsia? A. onset of proteinuria B. elevation of blood pressure C. excessive weight gain and edema D. headache and visual distrubances

(u) A. While proteinuria is a defining diagnostic criterion of pre-eclampsia, it is a late finding and may not be present even with eclampsia. (c) B. Elevated blood pressure is the most important and reliable diagnostic criterion. It is seen early in the course of pre-eclampsia and may occur suddenly. (u) C. Since weight gain and edema are common occurrences during pregnancy, they are less reliable indicators of pre-eclampsia. Most current sources no longer list them as diagnostic criterion for pre-eclampsia. (u) D. Headache and visual disturbances indicate severe pre-eclampsia and are late findings that indicate a need to deliver the fetus.

: (28) 111. Diagnostic Studies/Hematology A 33 year-old female 2 years status-post gastric bypass surgery presents to the office with complaints of generalized fatigue and paresthesias in her hands. Examination reveals pallor, mild glossitis and decreased vibratory sense. Which of the following laboratory tests is most diagnostic for her condition? A. Serum ferritin B. Serum vitamin B12 C. RBC with indices D. Reticulocyte count

(u) A. While iron studies should be part of a work-up for this anemia, the iron studies will not demonstrate the diagnosis of B12 deficiency. (c) B. Vitamin B12 deficiency leads to neurologic symptoms of paresthesias, glossitis, and pallor. Gastric bypass surgery aids in malabsorption. (u) C. RBC with indices can assist in pointing toward the diagnosis and classification of the anemia. It will not necessarily confirm the B12 deficiency diagnosis. (u) D. See B for explanation.

Which of the following is the most likely presentation of an acute pulmonary embolism (PE) in a patient without preexisting cardiac or pulmonary disease? A. Anginal chest pain B. Cough C. Tachypnea D. Palpitations

(u) A. While it is quite common for PE to present with pleuritic chest pain, angina-like pain is only rarely reported. (u) B. Cough is reported in roughly one-third of patients with PE. But is not the most common presentation. (c) C. Tachypnea is the most common symptom in acute PE. (u) D. Palpitations are uncommonly reported in acute PE.

Which of the following is considered a risk factor for retinopathy of prematurity? A. Maternal rubella infection B. Maternal alcohol abuse C. Low birth weight D. Family history of retinal detachment

(u) A. While maternal rubella infection is a risk factor for ocular disease in the newborn, it is not a specific risk factor for retinopathy of prematurity. (u) B. Maternal alcohol use is associated with the development of fetal alcohol syndrome which includes craniofacial abnormalities, but does not include increased risk of retinopathy of prematurity. (c) C. Risk factors for retinopathy of prematurity include low birth weight, perinatal oxygen therapy, prematurity and sepsis. (u) D. If retinopathy of prematurity is not treated, retinal detachment causing blindness may result, but a family history of retinal detachment is not considered a risk factor for the development of retinopathy of prematurity.

Which of the following is considered a risk factor for retinopathy of prematurity? A. maternal rubella infection B. maternal alcohol abuse C. perinatal oxygen therapy D. family history of retinal detachment

(u) A. While maternal rubella infection is a risk factor for ocular disease in the newborn, it is not a specific risk for retinopathy of prematurity. (u) B. Maternal alcohol abuse is associated with the development of fetal alcohol syndrome, which includes craniofacial abnormalities, but does not include increased risk for retinopathy of prematurity. (c) C. Risk factors for retinopathy of prematurity include perinatal oxygen therapy, low birth weight, prematurity, and sepsis. (u) D. If retinopathy of prematurity is not treated, retinal detachment may occur causing blindness. A family history of retinal detachment is not considered a risk factor for the development of retinopathy of prematurity

Which of the following has been found to be an important reservoir in the transmission of pertussis? A. mosquitoes B. adult humans C. domestic pets D. white-tailed deer

(u) A. While mosquitoes have been implemented in the spread of several infectious diseases, pertussis is not one of them. (c) B. Since neither the vaccine nor the disease itself provides lasting immunity against pertussis, adult humans are an important reservoir in the transmission of pertussis. (u) C. Contact with domestic pets has led to the development of upper and lower respiratory infections that are caused by Bordetella bronchiseptica, but not pertussis. (u) D. White-tailed deer are part of the transmission cycle for Lyme disease, not pertussis.

A 26 year-old male presents with headache, sinus pressure, and sinus congestion for over a month. He has a thick nasal discharge in the mornings, but this improves as the day goes on. He is afebrile. On exam, there is tenderness over the face. TMs have normal light reflex. Nasal mucosa reveals thick yellowish discharge. Neck is supple, without lymphadenopathy. Which of the following is the diagnostic study of choice? A. transillumination of sinuses B. routine sinus films C. CT scan of sinuses D. nasal culture

(u) A. Transillumination is used in the initial evaluation of chronic or acute sinusitis, but is not sensitive or specific. (u) B. See C for explanation. (c) C. CT scan is more sensitive than plain films for the diagnosis and management of chronic sinusitis, and is considered the gold standard for sinus imaging. (u) D. Nasal culture is not indicated in the evaluation of chronic sinus infections.

Which of the following is associated with meconium-stained amniotic fluid during labor? A. transition B. prematurity C. fast labor D. fetal distress

(u) A. Transition is the last phase of the first stage of labor. Complete cervical dilation marks the end of this stage of labor. (u) B. Meconium passage occurs most commonly in post-term deliveries, not pre-term deliveries. Passage of meconium is related to mature development of the gastrointestinal tract and is rarely seen before 36 weeks gestation. (u) C. Prolonged labor, not fast labor, is associated with potential passage of meconium into the amniotic fluid. (c) D. Passage of meconium is associated with fetal distress usually due to asphyxia.

A patient is hospitalized with a change in mental status. Examination reveals that he is unable to maintain dorsiflexion of the wrists after pronating his arms in front of his body. Which of the following is the most likely diagnosis? A. cocaine overdose B. hyperthyroidism C. hepatic encephalopathy D. Parkinson's disease

(u) A. Tremor and agitation are part of acute cocaine intoxication, not asterixis. (u) B. Hyperthyroidism causes a fine resting tremor, not asterixis. (c) C. This is the description for asterixis that is seen with hepatic encephalopathy, uremia, and carbon dioxide narcosis. (u) D. Parkinson's disease has resting tremor, rigidity, akinesia, and postural hypotension, not asterixis.

A pediatric patient presents with a history of multiple recurrent respiratory infections associated with failure to thrive. A sweat chloride test is elevated. Which of the following is a common cause of death in patients with this condition? A. Diabetic ketoacidosis B. Pulmonary infection C. Intestinal obstruction D. Acute respiratory failure

(u) A. While patients with cystic fibrosis most likely will eventually develop insulin-dependent diabetes mellitus, diabetic ketoacidosis is not a common cause of death. (c) B. This patient has cystic fibrosis. The most common causes of death include pulmonary complications, such as infections, and terminal chronic respiratory failure associated with cor pulmonale. (u) C. While intestinal obstruction may occur in patients with cystic fibrosis, it is not a common cause of death. (u) D. See B for explanation.

: (27) 60. Diagnostic Studies/Obstetrics/Gynecology A 24 year-old G2 P1 at 32 weeks of gestation presents to labor and delivery with contractions. Her previous pregnancy resulted in an uncomplicated term delivery at 37 weeks. Which of the following will be most helpful in predicting the likelihood of preterm delivery in this patient? A. Fetal fibronectin B. Screening for candidiasis C. Weekly digital cervical examinations D. Ultrasound of lower uterine segment

(c) A. An increase in the concentration of fetal fibronectin in cervicovaginal secretions is found with preterm labor. A preterm rise in the concentration may be associated with an increase likelihood of birth between 22 and 34 weeks of gestation and birth within 7-14 days of the test. (u) B. See A for explanation. (h) C. Digital examinations increase the risk for infection and preterm labor and delivery and are therefore contraindicated. (u) D. See A for explanation.

A 20 year-old male presents with a mass in the groin. On examination with the patient standing, a mass is noted that extends into the scrotum. The patient denies any trauma. The most likely diagnosis is A. an indirect inguinal hernia. B. a direct inguinal hernia. C. an obturator hernia. D. a femoral hernia.

(c) A. An indirect inguinal hernia is caused by a patent processus vaginalis and the hernial contents may be felt in the ipsilateral scrotum. (u) B. A direct inguinal hernia is symmetrical, round and disappears easily with the patient lying down. It is the result of a weakness in the inguinal 3external ring. Hernial contents may radiate anteriorly rather than into the scrotum. (u) C. Obturator hernia's are more commonly seen in elderly women and are rarely palpable in the groin. (u) D. Femoral hernias are rare in males and do not typically reduce with lying down.

A foreign body lodged in the trachea that is causing partial obstruction will most likely produce what physical examination finding? A. stridor B. aphonia C. inability to cough D. progressive cyanosis

(c) A. An inspiratory wheeze is called stridor, which indicates a partial obstruction of the trachea or larynx. (u) B. Aphonia, inability to cough and progressive cyanosis are seen with complete obstruction of the trachea, not partial obstruction. (u) C. See B for explanation. (u) D. See B for explanation.

A patient presents uncertain as to whether he has completed the Hepatitis B vaccination series. What laboratory test would be most helpful in determining his immunization status? A. Anti-HBs B. HBeAg C. HBsAg D. Anti-HBc

(c) A. Anti-HBs is the circulating antibody that develops to surface antigen in response to either past Hepatitis B infection or to Hepatitis B immunization. It represents immunity to the Hepatitis B virus in otherwise immunocompetent patients. (u) B. HBeAg is seen with active disease and not from immunization. (u) C. The surface antigen to Hepatitis B (HBsAg) is most often used to show acute infection and not immunity. (u) D. Anti-HBc is seen with active disease and not from immunization.

What is the mechanism of action for aspirin? A. Inhibits platelet aggregation by blocking cyclooxygenase-1 activity B. Exerts antiplatelet effects by blocking the platelet membrane adenosine diphosphate receptors C. Inhibits the platelet membrane glycoprotein IIb/IIIa receptor D. Converts plasminogen to the natural fibrinolytic agent plasmin

(c) A. Aspirin inhibits platelet aggregation by blocking cyclooxygenase-1 activity. (u) B. This the mechanism of action for ADP antagonists such as clopidogrel or ticlopidine. (u) C. This is the mechanism of action for glycoprotein IIb/IIIa receptors such as (u) D. This is the mechanism of action for tissue plasminogen activators (tPA).

A patient presents with a nontender, painless, nodule involving a meibomian gland. Which of the following is the most likely diagnosis? A. Chalazion B. Dacryocystitis C. Entropion D. Hordeolum

(c) A. Chalazion is characterized by a hard, nontender swelling on the upper or lower lid with redness and swelling of the adjacent conjunctiva and is due to granulomatous inflammation of a meibomian gland. (u) B. Dacryocystitis is an infection of the lacrimal sac due to obstruction of the nasolacrimal system. (u) C. Entropion is an outward turning of the lower lid. (u) D. A hordeolum is a bacterial inflammation of the base of the eyelash.

A construction worker presents with severe eye pain after splashing concrete in his eye. Examination is complicated by the patient's difficulty opening his eye. What is the most appropriate therapy - remembering that cement is an alkali? A. Irrigation with saline B. Instill corticosteroids drops C. Neutralize with acidic solution D. Prescribe local anesthetic drops

(c) A. Chemical burns are treated by copious irrigation of the eyes with saline solution, plain water, or buffering solution if available as soon as possible after exposure. Neutralization of an acid with an alkali or vice versa generates heat and may cause further damage. Alkali injuries are more serious and require prolonged irrigation, since alkalis are not precipitated by the proteins of the eye as are acids. (u) B. See A for explanation. (u) C. See A for explanation. (u) D. See A for explanation.

A patient with obsessive compulsive disorder would most likely have which of the following findings? A. Raw, red hands B. Priapism C. Memory impairment D. Abdominal pain

(c) A. Common manifestations of obsessive compulsive disorder include phobias of germ and contaminants which results in frequent handwashing leading to chafed and reddened hands. The other answers are inconsistent with obsessive compulsive disorder. (u) B. See A for explanation. (u) C. See A for explanation. (u) D. See A for explanation.

: (27) 159. Diagnosis/Cardiology A 67 year-old patient with a 15 year history of chronic obstructive pulmonary disease (COPD) presents with worsening respiratory symptoms and abdominal pain. On examination, you observe peripheral edema, jugular venous distention, epigastric pulsations, and tender hepatomegaly. You auscultate a systolic ejection murmur. Echocardiogram reveals a dilated right ventricle with decreased ejection fraction and a normal appearing left ventricle. Which of the following is the most appropriate diagnosis for this patient? A. Cor pulmonale B. Hepatitis C. Portal hypertension D. Pulmonic stenosis

(c) A. Cor pulmonale denotes RV hypertrophy and eventual failure resulting from pulmonary disease. It is most commonly caused by COPD. The signs and symptoms presented are classic for the diagnosis. (u) B. See A for explanation. (u) C. See A for explanation. (u) D. See A for explanation.

: (8) 5. Diagnostic Studies/ENT/Ophthalmology Which of the following imaging modalities is most appropriate in the evaluation of chronic sinusitis? A. Coronal CT B. Sinus MRI C. Panorex D. Sinus radiographs

(c) A. Coronal CT is the most effective imaging to determine the anatomy of the sinuses as well as the presence and extent of sinusitis. (u) B. MRI is of limited value due to less than optimal visualization of bony structures. (u) C. Panorex imaging provides limited viusalization of the sinus structures. (u) D. Sinus radiographs are rarely used due to limited visualization of soft tissues.

A 17 year-old patient presents to the emergency department with agitation and hallucinations, and has one seizure. He admits to using "some drugs" but does not know what they were. On physical examination, temperature is 103 degrees F, BP 140/90, pulse 120, respirations 20. Remainder of the examination is unremarkable. Which of the following diagnostic studies will be of most help in managing this patient? A. Drug screen B. Urine dipstick C. Complete blood count D. Serum creatinine kinase

(a) A. Although a drug screen may identify specific drugs, the results will not alter the care of this patient. (u) B. Urine dipstick is not sensitive for myoglobinuria. (u) C. This patient is at risk for myoglobinuria, and a complete blood count will not alter the treatment. (c) D. Serum creatinine kinase is the most sensitive test to detect rhabdomyolysis, a serious complication of seizures and hyperthermia related to drug abuse.

A patient presents with moderate mitral stenosis. Which of the following complications is associated with an increased risk of systemic embolization in this patient? A. atrial fibrillation B. pulmonary hypertension C. increased left atrial pressure D. left ventricular dilatation

(c) A. 50-80% of patients with mitral stenosis will develop paroxysmal or chronic atrial fibrillation; 20-30% of patients with atrial fibrillation will have systemic embolization. (u) B. Pulmonary hypertension can occur in patients with severe mitral stenosis with symptoms of low cardiac output and right sided heart failure. Pulmonary hypertension does not cause systemic embolization. (u) C. Patients with mitral stenosis can have increased left atrial pressures relative to the left ventricular diastolic pressures; this does not usually cause systemic embolization. (u) D. Left ventricular dilatation is more common in aortic valve disease than mitral valve disease.

Which of the following is a milestone usually achieved by a 15-month old infant? A. walks alone B. puts three words together C. feeds self well with spoon D. builds tower of seven cubes

(c) A. A 15-month-old infant should be able to walk alone. (u) B. The ability to put three words together, feed oneself well with a spoon and build a tower of seven cubes does not occur until 24 months. (u) C. See B for explanation. (u) D. See B for explanation.

For patients undergoing highly emetogenic chemotherapy regimens, which of the following would be the first-line antiemetic medication? A. Ondansetron (Zofran) B. Lorazepam (Ativan) C. Dronabinol (Marinol) D. Prochlorperazine (Compazine)

(c) A. A 5HT3 receptor blocker such as ondansetron is the agent of choice, given its high efficacy and low risk of side effects (headache being the major, and usually easily manageable, one). (u) B. Benzodiazepines (lorazepam) are associated with significant psychologic side effects including sedation and amnesia, although its anxiolytic properties make it a good choice for patients suffering from anticipatory nausea. (u) C. Cannabinoids (dronabinol) likewise have significant psychogenic side effects. (u) D. The side effect profile of phenothiazides (prochlorperazine) includes hypotension, restlessness, sedation, and extrapyramidal symptoms.

: (11) 207. History & Physical/Urology/Renal Which of the following is often described as feeling like a "bag of worms" on testicular exam? A. Varicocele B. Testicular torsion C. Epididymal cyst D. Hydrocele

(c) A. A collection of varicosed veins of the spermatic cord is known as a varicocele. The varicosities feel like a bag of worms when palpated on physical examination. (u) B. Testicular torsion is a surgical emergency which occurs when the testis twists within the scrotum which in turn twists the spermatic cord. This impinges on the blood supply to the testis leading to infarction and significant pain. (u) C. An epididymal cyst is a painless, benign, sperm-filled lesion located on the epididymis. (u) D. A hydrocele is a painless collection of serous fluid within the tunica vaginalis of the scrotum.

Which of the following rotator cuff tendons is most likely to sustain injury because of its repeated impingement (impingement syndrome) between the humeral head and the undersurface of the anterior third of the acromion and coracoacromial ligament? A. Supraspinatus B. Infraspinatus C. Teres minor D. Subscapularis

(c) A. A critical zone exists for the supraspinatus tendon due to its superior insertion site. It is susceptible for injury because it has a reduction in its blood supply that occurs with abduction of the arm. Impingement of the shoulder is most commonly seen with the supraspinatus tendon, the long head of the biceps tendon and/or the subacromial bursa. (u) B. See A for explanation. (u) C. See A for explanation. (u) D. See A for explanation.

Which of the following is the most significant factor in the production of menopausal symptoms? A. decrease in estrogen B. decrease in testosterone C. increase in progesterone D. increase in androstenedione

(c) A. A decrease in estrogen is responsible for the majority of menopausal symptoms including, but not limited to, hot flashes, sleep disturbances, genital tract atrophy and mood changes. (u) B. While there is a slight decrease in testosterone levels during menopause, this has not be shown to be responsible for the majority of menopausal symptoms. (u) C. Progesterone and androstenedione levels both decrease, not increase, in the menopausal female patient and therfore can not account for any menopausal symptoms. (u) D. See C for explanation.

: (14) 82. Diagnostic Studies/Pulmonology A 70 year-old patient presents with complaints of chronic, worsening shortness of breath for the past few years. He admits to a 60 pack-year history of tobacco use but no industrial exposures. Examination reveals a barrel chest, hyperresonant thorax and faint breath and heart sounds. Which of the following would be the most likely pulmonary function abnormality in this patient? A. Decreased FEV1, decreased FEV1/FVC ratio B. Decreased FEV1, increased FEV1/FVC ratio C. Increased FEV1, decreased FEV1/FVC ratio D. Increased FEV1, increased FEV1/FVC ratio

(c) A. A decreased FEV1 is the most dominant pulmonary function test finding in COPD. While the forced vital capacity is also reduced, it is generally not in the same proportion as FEV1. A reduction in both of these would result in a decreased FEV1/FVC ratio. (u) B. See A for explanation. (u) C. See A for explanation. (u) D. See A for explanation.

A defect in which of the following physiologic processes can lead to the development of a microcytic, hypochromic anemia? A. Hemoglobin production B. IgG antibody formation C. Red blood cell nuclear maturation D. Red blood cell membrane defects

(c) A. A deficiency in substances required for hemoglobin synthesis result in RBCs with a deficient mean corpuscular hemoglobin concentration and usually microcytosis. (u) B. IgG antibody formation would led to the development of a hemolytic anemia. (u) C. Alterations in red blood cell nuclear maturation led to the development of macrocytic red blood cells. (u) D. Red blood cell membrane defects led to the development of hemolytic anemia.

An x-ray reveals a break in the cortex of one side of the ulna shaft without a separation or break of the opposite cortex describes what type of fracture? A. greenstick B. transverse C. torus (buckle) D. epiphyseal

(c) A. A greenstick fracture is a break in the cortex of one side of bone shaft without a break in the opposite cortex. (u) B. A transverse fracture is a complete fracture of both cortices. (u) C. A torus fracture is a bowing, bending, or buckling without a break in the cortex. (u) D. Epiphyseal fracture occurs at the growth plate.

A patient involved in a minor motor vehicle crash is brought to the emergency department by a family member who was riding in the car, but was unhurt. The family member states that the patient was unconscious for about 2 minutes, but seems "okay" now. What diagnostic procedure would be most helpful in assessing this patient? A. head CT scan B. lumbar puncture C. skull radiographs D. electroencephalogram

(c) A. A head CT scan would provide evidence of fractures and demonstrate intracranial hemorrhage and cerebral edema if present. (u) B. A lumbar puncture would not be immediately indicated, but if the patient worsens it might be a consideration. (u) C. Skull radiographs would only reveal skull fracture and not any potentially lethal intracranial injuries. (u) D. An electroencephalogram is not needed because there is no history of seizures.

A mother brings a 3 month-old infant to the office because she is concerned about a red, vascular, nodular growth on the child's back. It appears to be enlarging slightly and the vessels are slightly dilated. It seems to cause the child no discomfort. The most likely diagnosis is A. a hemangioma. B. a pigmented nevus. C. a salmon patch (stork bite). D. a malignant melanoma.

(c) A. A hemangioma is a bright red to deep purple vascular nodule or plaque that often develops at birth, may enlarge, and may regress and disappear with aging. (u) B. A pigmented nevus is not a vascular lesion. (u) C. A salmon patch is a light red macule over the nape of the neck or the glabella. (u) D. A malignant melanoma presents as a hyperpigmented, asymmetric lesion with irregular borders and is typically seen in the adult population.

A 68 year-old male presents with jaundice, weight loss, and boring abdominal pain which radiates to the back. The gallbladder is palpable on physical examination. This finding is most consistent with which of the following? A. Pancreatic tumor B. Hemorrhagic pancreatitis C. Cholecystitis D. Cholelithiasis

(c) A. A large palpable gallbladder resulting from pressure from a tumor in the pancreatic head is known as Courvoisier's sign. (u) B. Ecchymosis of the flank associated with hemorrhagic pancreatitis is known as Grey Turner's sign (u) C. A sharp increase in tenderness with a sudden stop in inspiratory effort indicative of acute cholecystitis is known as Murphy's sign. (u) D. Pain in the right lower quadrant during left-sided pressure on the abdomen suggesting acute appendicitis is known as Rovsing's sign.

Which of the following is the most common complication that occurs in the setting of acute pericarditis? A. Pericardial effusion B. Left ventricular failure C. Superior vena cava syndrome D. Subclavian steal syndrome

(c) A. Accumulation of transudate, exudate or blood in the pericardial sac can occur due to pericardial inflammation. (u) B. Patients with acute pericarditis may have problems with filling which affects the right ventricle more than the left ventricle. (u) C. Patients with lung malignancy may develop superior vena cava syndrome as a result of tumor invasion into the superior vena cava. (u) D. Patients with subclavian steal syndrome typically present with arm ischemia and syncope and is not related to pericarditis.

A patient is concerned about being exposed to condyloma acuminata. Which of the following tests is most appropriate to perform in order to better identify these lesions? A. Acetowhitening B. Tzanck smear C. Potassium hydroxide test D. Wood's light fluoroscopy

(c) A. Acetowhitening is used to facilitate the diagnosis of condyloma acuminata lesions. A 3 to 5% acetic acid solution is applied to these suspected genital warts for five to ten minutes. Condyloma lesions will whiten and appear as circumscribed macular or papular lesions with a granular surface. (u) B. Tzanck smears are used in making the diagnosis of herpetic lesions, such as herpes simplex, varicella, and herpes zoster infection. (u) C. Potassium hydroxide tests are used to rule out the presence of dermatophyte (fungal and yeast) infections. (u) D. Wood's light fluoroscopy is used in the assessment of dermatophyte infection and a bacterial infection known as erythrasma.

A 45 year-old male nonsmoker presents with a 3-month history of difficulty swallowing. He describes the gradual onset of dysphagia for solids and liquids; he now complains of substantial discomfort after eating. What is his most likely diagnosis? A. Achalasia B. Schatzki ring C. Esophageal carcinoma D. Zenkers diverticulum

(c) A. Achalasia is by far the most common cause of dysmotility. This is particularly true in a non-smoker (less likely to be esophageal carcinoma or a lung cancer causing a paraneoplastic syndrome) and someone who has not been to an area endemic for Chagas disease. (u) B. See A for explanation. (u) C. See A for explanation. (u) D. See A for explanation.

You suspect that a 13 year-old patient has bacterial conjunctivitis. What physical examination finding would you expect? A. Crusting and matting of the eyelids with discharge and diffuse injection of the conjunctival surface B. Clear watery stringy discharge with conjunctival hyeremia and edema C. Circumcorneal injection of the conjunctiva D. Painless injection of the subconjunctival sclera bilaterally

(c) A. Crusting and matting of the eyelids with discharge and diffuse injection of the conjunctival surface is a common presentation of a bacterial conjunctivitis. (u) B. Clear watery stringy discharge with conjunctival hyeremia and edema is a common presentation of an allergic conjunctivitis. (u) C. Circumcorneal injection of the conjunctiva is a common presentation of iridocyclitis. (u) D. Painless injection of the subconjunctival sclera bilaterally is a common presentation of episcleritis.

A female patient presents with a few common warts on her hands. She has tried over-the-counter preparations with little success and desires something that might work quicker, but has the least chance of scarring. Which of the following is the most appropriate intervention to accomplish this goal? A. cryosurgery B. dermabrasion C. electrosurgery D. topical acyclovir

(c) A. Cryosurgery with liquid nitrogen is effective treatment for common warts and has less of a chance of scarring than electrosurgery. (u) B. Dermabrasion is a technique primarily utilized in the treatment of pitted acne scars and does not play a role in treatment of common warts. (u) C. Electrosurgery is more effective in treatment of common warts, but has a greater chance of scarring than cryosurgery. (u) D. Topical acyclovir may be indicated in treatment of herpes simplex viral infections, but is not used to treat common warts caused by human papillomaviruses.

What is the term for blue discoloration about the umbilicus? A. Cullen's sign B. Murphy's sign C. Rovsing's sign D. Turner sign

(c) A. Cullen's sign is a blue discoloration about the umbilicus and can occur in hemorrhagic pancreatitis and results from hemoperitoneum. (u) B. Murphy's sign is seen in liver and gallbladder disease by which the patient abruptly halts deep inspiration due to pain in the right upper quadrant while the examiner's hand is keeping stable pressure to the area. (u) C. Rovsing's sign is positive when the patient experiences right sided abdominal pain with palpation to the left side. This is generally seen in those with appendicitis. (u) D. Turner sign is also seen in severe, acute pancreatitis but is represented by a green-brown discoloration of bilateral flanks.

A 21 year-old male with a diagnosis of type 1 von Willebrand disease undergoes dental extraction of his wisdom teeth. The patient comes to the clinic with continued oozing of the dental sockets despite packing. Treatment should begin with which of the following? A. DDAVP B. Factor VIII C. vWF concentrate D. FFP

(c) A. DDAVP causes the release of vWF and factor VIII from storage sites significantly which is needed to complete hemostasis. Factor VIII is indicated for patients with Hemophilia A. Fresh frozen plasma is indicated in Coumadin overdosage and vWF concentrate is indicated in type 2 and 3 patients with von Willebrand disease. (u) B. See A for explanation. (u) C. See A for explanation. (u) D. See A for explanation.

Which of the following neurotransmitters is decreased in early Parkinson's disease? A. Dopamine B. Serotonin C. Norepinephrine D. Acetylcholine

(c) A. Decreased dopamine levels are typical of Parkinson's disease even early in the course of the disease. (u) B. See A for explanation. (u) C. See A for explanation. (u) D. See A for explanation

: (5) 135. Clinical Therapeutics/Urology/Renal A 6 year-old boy is evaluated for nocturnal enuresis. You offer behavioral education and write a prescription for the treatment of choice. Because of that drug's potential side effect, you remind the parents that the boy should eat plenty of foods containing which of the following? A. Salt B. Potassium C. Sugar D. Magnesium

(c) A. Desmopressin nasal spray has become the pharmacologic treatment of choice for nocturnal enuresis. Even in nasal spray formulation the drug has the potential to cause hyponatremia. 55 (u) B. See A for explanation. (u) C. See A for explanation. (u) D. See A for explanation.

A 7 year-old boy wets the bed nearly every night. Which of the following is the best pharmaceutical agent to use in treating this patient? A. desmopressin (DDAVP) B. paroxetine (Paxil) C. lorazepam (Ativan) D. hyoscyamine (Levsin)

(c) A. Desmopressin, while not curative, will relieve symptoms. (u) B. Paroxetine is not indicated for enuresis. (u) C. Lorazepam is used for treatment of sleep terrors. (u) D. Hyoscyamine is used in the treatment of pediatric overactive bladder.

: (8) 7. Diagnostic Studies/Infectious Diseases Which of the following is the most sensitive laboratory test to diagnose herpes simplex virus? A. Polymerase chain reaction (PCR) B. Enzyme-linked immunosorbent assay (ELISA) C. Tzanck smear D. HSV antibody titer

(c) A. Diagnosis is usually made clinically however, PCR is more sensitive than culture. (u) B. See A for explanation. (u) C. See A for explanation. (u) D. See A for explanation.

Which of the following is the most characteristic physical examination finding with Grave's disease? A. Diffuse thyroid gland enlargement B. Single thyroid gland nodule C. Unilateral thyroid lobe enlargement D. Multiple thyroid gland nodules

(c) A. Diffuse thyroid gland enlargement is the typical presentation for Grave's disease. (u) B. Single thyroid gland nodule would be more worrisome for thyroid malignancy. (u) C. Unilateral thyroid lobe enlargement is an atypical presentation for any thyroid malady. (u) D. Multiple thyroid gland nodules are more suggestive of a metabolic disorder.

A 70 year-old female with history of hypertension, diabetes, and hypothyroidism presents with complaint of sudden onset of left lower extremity pain. Examination reveals a cool left lower extremity with a mottled appearance. Dorsalis pedis and posterior tibialis pulses are absent. Which of the following is the most likely diagnosis? A. Acute arterial occlusion B. Thromboangiitis obliterans C. Deep vein thrombosis D. Peripheral neuropathy

(c) A. Acute arterial occlusion presents with sudden onset of extremity pain, with absent or diminished pulses. The extremity will be cool to the touch and have a mottled appearance. (u) B. Thromboangitis obliterans occurs in younger patients and primarily effects the distal extremities, especially the toes. It is typically secondary to smoking. (u) C. Deep vein thrombosis presents with lower extremity pain and edema. Pulses would be intact. (u) D. Patients with peripheral neuropathy would have diminished sensation. They would not have a mottled appearance and pulses would be intact.

: (14) 89. Health Maintenance/Pulmonology Which of the following should be screened for in a 35 year-old, non-smoker diagnosed with emphysema? A. Alpha-1 antitrypsin deficiency B. Cystic fibrosis C. Celiac sprue D. Surfactant deficiency syndrome

(c) A. Alpha-1 antitrypsin deficiency should be screened for as a potentially causative agent in patients diagnosed with emphysema prior to the age of 40. This is especially true for non-smokers. (u) B. See A for explanation. (u) C. See A for explanation. (u) D. See A for explanation.

While awaiting operative removal of pheochromocytoma, which of the following classes of medications are used for control of hypertension? A. alpha-adrenergic blocker B. beta-adrenergic blocker C. ACE inhibitor D. diuretic

(c) A. Alpha-adrenergic blockers are used preoperatively to control hypertension in a patient with pheochromocytoma that occurs from unopposed alpha stimulation when the tumor is manipulated. (u) B. Beta-adrenergic blockers are used to control tachycardia and arrhythmias, if present, after the hypertension has been controlled. (u) C. ACE inhibitors and diuretics have no role in the treatment of hypertension from pheochromocytoma. (u) D. See C for explanation.

: (7) 33. Clinical Therapeutics/Urology/Renal A 58 year-old male with hypertension, hyperlipidemia, occasional gastritis, and mild osteoarthritis needs to be started on tamsulosin (Flomax) for symptomatic prostatic hypertrophy. Which of the following medications may result in an adverse reaction in this patient? A. Amlodipine (Norvasc) B. Simvastatin (Zocor) C. Famotadine (Pepcid) D. Acetaminophen (Tylenol)

(c) A. Alpha-blockers, such as tamsulosin, are treatment for prostatic hyperplasia, but also used for hypertension. You would in essence be increasing his antihypertensive therapy, and since he is currently well controlled, he may develop orthostatic hypotension as the result of your actions. (u) B. Simvastatin treats hyperlipidemia and is not known to adversely interact with alpha-blockers. (u) C. Pepcid treats his heartburn and is not known to adversely interact with alpha-blockers. (u) D. Acetaminophen treats mild arthritis and is not known to adversely interact with alpha-blockers.

: (4) 68. History & Physical/Pulmonology Which of the following is the most common extra-pulmonary finding in a patient with sarcoidosis? A. Skin lesions B. Splenomegaly C. Paresthesias D. Joint inflammation 31

(c) A. Although sarcoidosis can affect nearly all organ systems dermatologic and ocular signs and symptoms dominate after pulmonary issues. (u) B. See A for explanation. (u) C. See A for explanation. (u) D. See A for explanation.

A 52 year-old female complains of bouts of anxiety and depression, the latter very deep but short lived (<24 hours). She states "I often feel like I am going to jump out of my skin". Her periods have become less frequent over the past 6 months. She denies suicidal ideations. She is not sleeping through the night. Which of the following the most appropriate next step in this patient? A. Check a follicle-stimulating hormone level B. Start a tricyclic antidepressant C. Refer her to a psychiatrist D. Endometrial biopsy

(c) A. An FSH level should be assessed in this patient to evaluate for menopause as a cause for her psychiatric symptoms. (u) B. See A for explanation. (u) C. See A for explanation. (u) D. Endometrial biopsy is indicated in a perimenopausal female with menorrhagia.

Which of the following conditions is most suggestive of an abdominal aortic aneurysm? A. Abdominal mass B. Hypertension C. Chest pain D. Syncope

(c) A. An abdominal aortic aneurysm presents with a pulsatile upper abdominal mass. (u) B. Hypertension is not suggestive of an abdominal aortic aneurysm. (u) C. Abdominal aortic aneurysm presents with midabdominal or lower back pain. (u) D. Syncope is not common in abdominal aortic aneurysm, unless it ruptures.

A 65 year-old patient with known history of alcohol and tobacco abuse presents with solid food dysphagia. The patient also has a 24 lb weight loss over the past 6 months. Which of the following is the most appropriate intervention? A. endoscopy with biopsy B. chest x-ray C. barium esophagogram D. CT scan of the thorax

(c) A. Endoscopy with biopsy establishes the diagnosis of esophageal cancer with a high degree of reliability. (u) B. Chest x-ray may show adenopathy, a pulmonary or bony metastases or sign of tracheoesophageal fistula. Barium esophagogram is obtained as the first study to evaluate the dysphagia. CT scan should be obtained once the diagnosis of carcinoma has been made to evaluate for pulmonary or hepatic metastases, lymphadenopathy, and local tumor extension. None of these tests will reveal the diagnosis of carcinoma. (u) C. See B for explanation. (u) D. See B for explanation.

Which of the following skin rashes is pathognomonic of rheumatic fever? A. erythema marginatum B. erythema nodosum C. diffuse petechiae D. purulent bullae

(c) A. Erythema marginatum is one of the major Jones criteria for the diagnosis of acute rheumatic fever. (u) B. Erythema nodosum lesions are associated with many diverse causes including various infections, malignancies, drug sensitivities and inflammatory/granulomatous diseases. (u) C. Diffuse petechiae are most commonly seen in conditions that cause thrombocytopenia. (u) D. Purulent bullae are noted in patients with scalded-skin syndrome.

Erythema nodosum is characterized by A. subcutaneous red tender nodules. B. brown pigmentation on the lower extremities. C. tender lymph nodes in the groin. D. scaling red macules.

(c) A. Erythema nodosum produces erythematous red tender nodules, especially on the shins. (u) B. Brown pigmentation on the lower extremities is a feature of chronic venous insufficiency. (u) C. Lymphovenereum granuloma is likely to produce tender lymph nodes in the groin as can acute infection in the lower extremities. (u) D. Scaling red macules are a feature of tinea corporis.

A patient should be tested for tuberculosis prior to being treated with A. etanercept (Enbrel). B. cyclosporine (Neoral). C. methotrexate (Rheumatrex). D. prednisone (Deltasone).

(c) A. Etanercept is an anti-cytokine agent used in the treatment of rheumatoid arthritis and has as a side effect the potential for serious infections. One of these side effects includes reactivation of dormant tuberculosis. (u) B. Cyclosporine, methotrexate, and prednisone do not have the requirement to check for tuberculosis prior to initiating treatment. (u) C. See B for explanation. (u) D. See B for explanation.

: (8) 16. Diagnostic Studies/Pulmonology In a patient with a low probability of pulmonary embolism (PE) based on the Wells criteria, which of the following lab studies is most helpful in ruling out a PE? A. D dimer assay B. PT with INR C. Factor V assay D. Clotting time

(c) A. Even though useful in ruling in a pulmonary embolism, the D dimer is negative in over 95% of patients who do not have a pulmonary embolism. (u) B. See A for explanation. (u) C. See A for explanation. (u) D. See A for explanation.

Which of the following lifestyle recommendations would most benefit the heart failure patient's quality of life? A. Begin a regular exercise program B. Total salt restriction C. Home monitoring of blood pressure D. Increase home oxygen requirements

(c) A. Exercise training improves activity tolerance and deconditioning offering some recompensation. (u) B. Although salt restriction is a recommendation total salt restriction would be unrealistic to achieve. (u) C. Home monitoring of blood pressure will not improve a heart failure patient's quality of life. (u) D. Increasing the requirements for home oxygen could signal that they are worsening and is palliative for their quality of life.

A 2 month-old infant has had a single, generalized tonic-clonic convulsion lasting 4 to 5 minutes. There is no history of trauma and the infant had been well previously. Physical findings include a temperature of 39.6 degrees C (103.2 degrees F), a bulging tympanic membrane on the right, and an inflamed pharynx. The next most appropriate step is to A. perform a lumbar puncture. B. order x-ray studies of the skull. C. obtain an electroencephalogram. D. send home with antibiotics and an anticonvulsant

(c) A. Febrile convulsions are uncommon under the age of 3 months. The physical findings suggest the possibility of meningitis, so lumbar puncture with CSF analysis is indicated. (u) B. See A for explanation. (u) C. See A for explanation. (h) D. See A for explanation. In addition, anticonvulsant therapy is not indicated for febrile seizures.

: (27) 95. Diagnostic Studies/Obstetrics/Gynecology A 25 year-old G3 P2 patient presents for a routine obstetric appointment at 28 weeks gestation. Her EDC is based on her LMP. The fundal height is measured at 34 cm for this singleton pregnancy. Her weight gain has been normal thus far during this pregnancy. Which of the following laboratory or diagnostic studies is recommended to further evaluate this patient? A. Ultrasonography B. Nonstress test C. Quadruple screen D. Glucose tolerance test

(c) A. Fetal growth can be assessed by fundal height measurement and correlates by centimeters to weeks of gestation in a singleton pregnancy. Deviation in fundal height measurement should be evaluated by ultrasound. (u) B. See A for explanation. (u) C. See A for explanation. (u) D. See A for explanation.

What is the mechanism for the relapsing fevers associated with malaria? A. Release of malarial merozoites into the bloodstream B. Release of toxins from Plasmodium species C. Attachment of Plasmodium species to receptor sites on the red blood cells D. Invasion of hepatocytes by Plasmodium species

(c) A. Fever, chills, and sweats coincide with the release of merozoites from red blood cells that have been infected with Plasmodium species. Each merozoite may infect a new red blood cell, leading to a cycle of invasion growth, and release. (u) B. See A for explanation. (u) C. See A for explanation. (u) D. See A for explanation.

A patient presents with eye pain and blurred vision. Snellen testing reveals vision of 20/200 in the affected eye and 20/20 in the unaffected eye. Fluorescein staining reveals the presence of a dendritic ulcer. Which of the following is the most likely diagnosis? A. Viral keratitis B. Fungal corneal ulcer C. Acanthamoeba keratitis D. Bacterial corneal ulcer

(c) A. Herpes Simplex virus is a common cause of dendritic ulceration noted on fluorescein staining. (u) B. Fungal corneal ulcers have an indolent course with intraocular infection being common but fluorescein staining is negative for a dendritic pattern. (u) C. Acanthamoeba keratitis has a waxing and waning course over several months and has no fluorescein staining in a dendritic pattern. (u) D. Bacterial corneal ulcers can progress aggressively resulting in corneal perforation. Fluorescein staining does not occur in a dendritic pattern.

To further assess ascites in a patient, the physician assistant instructs the patient to turn onto one side while performing percussion. Which of the following is the reason for this maneuver? A. Testing for shifting of dullness on percussion B. Shifting of internal organs making percussion easier C. Trying to elicit any pain while moving D. Trying to produce a caput medusa

(c) A. In ascites, dullness shifts to the more dependent side as the fluid relocates into dependent space, while tympany shifts to the top as the gas-filled organs float to the top of the ascitic fluid. (u) B. See A for explanation. (u) C. Pain with movement is associated with peritonitis and not ascites. (u) D. Caput medusa is the dilation of the superficial abdominal veins due to increased intraabdominal fluid accumulation. It is visible with the patient standing and does not need a special maneuver for identification.

Which of the following is the most common cause of acute epididymitis in men under the age of 40? A. Chlamydia trachomatis B. Ureaplasma urealyticum C. Pseudomonas aeruginosa D. Escherichia coli

(c) A. In men under the age of 40, acute epididymitis is typically caused by Chlamydia trachomatis and Neisseria gonorrhoeae. (u) B. See A for explanation. (u) C. See A for explanation. (u) D. See A for explanation.

Which of the following physical findings is suggestive of atrial septal defect? A. Fixed split S2 B. Increased pulse pressure C. Continuous mechanical murmur D. Difference in blood pressure between the left and right arm

(c) A. An atrial septal defect will cause a shunt of blood from the left to the right atrium. This will result in an equalization in the amount of blood entering both the left and right ventricles which effectively eliminates the normally wide splitting that inspiration typically causes in hearts without an atrial septal defect. (u) B. Pulse pressures reflect the difference in aortic and left ventricular volumes that occur during ventricular systole Increased pulse pressures are seen in aortic regurgitation which is a different entity than atrial septal defect. (u) C. Continuous mechanical murmurs are noted in patients with patent ductus arteriosus. (u) D. Differences in blood pressure between the left and right arms are seen in conditions such as coarctation of the aorta.

A patient has been followed for 3 years with a continual decline in glomerular filtration rate (GFR). Currently the GFR is 10 ml/min and examination of the patient reveals a pericardial friction rub. Which of the following is the most appropriate intervention at this time? A. Hemodialysis B. Continue to observe C. Administration of high dose steroids D. Bilateral nephrectomy

(c) A. The patient has end stage renal disease and with the pericardial friction rub is in need of immediate hemodialysis. (u) B. The patient already has significant renal compromise, further observation will continue to lead to further complications. (u) C. High dose steroids have no benefit in end stage renal disease. (u) D. Surgical removal of the kidneys will not change the progression of the disease.

: (6) 133. Diagnostic Studies/Neurology A 58 year-old male presents with a 3 month history of spastic paresis in his legs. Today, he has experienced diplopia on lateral gaze and scotoma in his right eye. Examination reveals decreased visual acuity of the right eye. Which of the following MRI findings is most likely in this patient? A. Multifocal white matter disease B. Chiari malformation C. Cerebral atrophy D. Intracranial hemorrhage

(c) A. The patient is experiencing symptoms of multiple sclerosis (MS). MRI findings demonstrate multifocal white matter disease. (u) B. See A for explanation. (u) C. See A for explanation. (u) D. See A for explanation.

A 27 year-old female presents to your office for evaluation of weakness, visual loss, and sensory loss over the right great toe. These symptoms have occurred during three episodes approximately three months apart with each episode lasting about three days. Which of the following tests would be most useful in further evaluating this patient? A. MRI of the brain B. Electromyograph C. Glucose tolerance test D. Electroencephalograph

(c) A. Multiple sclerosis typically presents with relapsing weakness of the limbs, sensory loss, paresthesias, and visual changes. Diagnosis is based on history and either abnormal brain or spinal cord MRI, or visual, auditory, or somatosensory evoked electrical response. (u) B. See A for explanation. (u) C. Glucose tolerance test is used in the evaluation of diabetes mellitus. (u) D. Electroencephalograph is used to evaluate patients for possible seizure disorder.

: (8) 32. Clinical Therapeutics/Pulmonology Which of the following is the most effective outpatient treatment for Mycoplasma pneumonia? A. Doxycycline (Doryx) B. Amoxicillin (Amoxil) C. Trimethoprim (Septra) D. Cephalexin (Keflex)

(c) A. Mycoplasma is an intracellular parasite which would require an antibiotic, like doxycyline, to penetrate the cell membrane and eradicate the infection. The other antibiotic selections do not achieve this action. (u) B. See A for explanation. (u) C. See A for explanation. (u) D. See A for explanation.

A 13 year-old male with sickle cell trait is interested in playing football for his school. Which of the following considerations should be reviewed with the patient and his parents? A. avoid dehydration B. avoid all contact sports C. begin daily use of aspirin D. begin hydroxyurea supplements

(c) A. No restriction of sports activity is required of patients with sickle cell trait. Hydration is required in all sports activities. (u) B. See A for explanation. (u) C. There is no increased risk of thrombosis so daily use of aspirin is not needed. (u) D. Hydroxyurea is used in the treatment of sickle cell disease; sickle cell trait requires no treatment.

What is the recommendation for primary prevention of stroke in a patient under sixty years of age with atrial fibrillation? A. No therapy is needed B. Aspirin C. Warfarin D. Maze procedure

(c) A. No therapy is recommended for primary stroke prevention in this patient. (u) B. See A for explanation. (u) C. See A for explanation. (u) D. See A for explanation.

In patents with diabetic retinopathy, what clinical intervention is most successful in preserving vision? A. Panretinal laser photocoagulation B. Iridectomy C. Radial keratotomy D. Vitrectomy

(c) A. Panretinal laser photocoagulation is indicated for preservation of vision in patients with diabetic retinopathy. (h) B. Iridectomy is of no value in preserving the retina and iridectomy is harmful in this situation due to the trauma it causes to the eye. (h) C. Radial keratotomy is indicated to correct myopia. This surgery destroys normal eye architecture and has no benefit in diabetic retinopathy. (h) D. Vitrectomy is indicated for treatment of retinal tears and not to preserve an intact retina.

Absorption of vitamin B12 requires intrinsic factor that is secreted by which of the following? A. parietal cells B. chief cells C. mucous cells D. enteroendocrine cells

(c) A. Parietal cells are responsible for acid secretion and intrinsic factor. (u) B. Chief cells are responsible for the secretion of the proteolytic proenzymes pepsinogen I and II. (u) C. Mucous cells secrete mucous and pepsinogen II. (u) D. Enteroendocrine cell secrete gastrin.

A 48 year-old male presents with complaints of heartburn that occurs approximately 45 minutes after eating about three times a week that is relieved by antacids. He claims to have followed advice about elevating the head of the bed, avoiding spicy foods, and losing weight, but continues to have heartburn. Which of the following is the most appropriate next step? A. Ranitidine (Zantac) B. Sucralfate (Carafate) C. Metoclopramide (Reglan) D. Misoprostol (Cytotec)

(c) A. Ranitidine, an H2 receptor blocker, is indicated for the treatment of mild, intermittent symptoms of gastroesophageal reflux disease. (u) B. Sucralfate is used in the treatment of duodenal ulcers. (u) C. Metoclopramide is indicated for the treatment of gastroparesis as a first-line agent and as a second-line agent in the treatment of refractory gastroesophageal reflux. (u) D. Misoprostol is indicated for the prevention of NSAID-induced gastritis.

Which of the following is the treatment of choice for patients with celiac disease? A. Gluten-free diet B. Small bowel resection C. Clindamycin D. Whipple procedure

(c) A. Removal of all gluten from the diet is essential to therapy in celiac disease. (u) B. Celiac disease is a dietary disorder that resolves with gluten free diet. Small bowel resection is not indicated. (u) C. Antibiotics are not therapeutic for celiac disease. (u) D. Whipple procedure is used in the treatment of pancreatic carcinoma and is not indicated in the management of celiac disease.

Which of the following is the most common cause of secondary hypertension? A. Renal parenchymal disease B. Primary aldosteronism C. Oral contraceptive use D. Cushing's syndrome

(c) A. Renal parenchymal disease is the most common cause of secondary hypertension. (u) B. Primary aldosteronism can cause secondary hypertension, but it is not the most common cause. (u) C. Oral contraceptives can cause small increases in blood pressure but considerable increases are much less common. (u) D. Cushing's disease is a less common cause of secondary hypertension.

A newborn is seen for an initial two week visit. Physical examination reveals a thrill and a continuous machinery murmur in the left second intercostal space. Which of the following is the most likely diagnosis? A. patent ductus arteriosus B. ventricular septal defect C. tetralogy of Fallot D. coarctation of the aorta

(c) A. Patent ductus arteriosus is characterized by a classic harsh, machinery-like murmur that is continuous through systole and diastole. This is heard best at the left second interspace and is commonly associated with a thrill. (u) B. Ventricular septal defect is characterized by a holosystolic murmur at the lower left sternal border. (u) C. Tetralogy of Fallot is characterized by a systolic thrill at the left sternal border with a systolic ejection murmur that may or may not have an associated systolic click. (u) D. Coarctation of the aorta is associated with a systolic ejection click or a short systolic murmur at the left sternal border.

A 25 year-old female presents with signs and symptoms of depression. She does not have any other known medical problems. What diagnostic study is indicated in the initial evaluation of this patient? A. Thyroid stimulating hormone (TSH) B. Prolactin C. Growth hormone (GH) D. Cortisol

(c) A. Patients who are presenting with symptoms of depression should be evaluated with a TSH because 10 percent of patients evaluated for depression have previously undetected thyroid dysfunction. (u) B. See A for explanation. (u) C. See A for explanation. (u) D. See A for explanation

Which of the following is first-line treatment for a symptomatic bradyarrhythmia due to sick sinus syndrome? A. Permanent pacemaker B. Radiofrequency ablation C. Antiarrhythmic therapy D. Anticoagulation therapy

(c) A. Permanent pacemakers are the therapy of choice in patients with symptomatic bradyarrhythmias in sick sinus syndrome. (u) B. Radiofrequency ablation is used for the treatment of accessory pathways in the heart. (u) C. See A for explanation. (u) D. See A for explanation.

Which of the following is first-line treatment for symptomatic bradyarrhythmias due to sick sinus syndrome (SSS)? A. Permanent pacemaker B. Radiofrequency ablation C. Antiarrhythmics D. Anticoagulation therapy

(c) A. Permanent pacemakers are the therapy of choice in patients with symptomatic bradyarrhythmias in sick sinus syndrome. (u) B. Radiofrequency ablation is used for the treatment of accessory pathways in the heart. (u) C. See A for explanation. (u) D. See A for explanation.

Treatment of the patient with Pediculosis pubis consists of which of the following? A. Permethrin (Nix) cream B. Clotrimazole (Gyne-Lotrimin) C. Podofilox (Condylox) solution D. Selenium sulfide (Selsun) suspension

(c) A. Permethrin 1% cream/shampoo is used to kill the louse and remove the eggs from the hair shafts. (u) B. Clotrimazole is an antifungal agent and is not used to treat parasitic infestation. (u) C. Podofilox 0.5% solution is used to treat Condyloma accuminata. (u) D. Selenium sulfide suspension is used to treat Tinea versicolor fungal infection.

A patient presents with 3 weeks of worsening pruritic rash located on the upper extremities and interdigital spaces. He is a migrant farm worker with no history of skin disorders. Examination reveals excoriated, erythematous papules with numerous 3-4 mm long, narrow tracts spreading from the papules. What is the best treatment for this patient? A. Permethrin (Nix) B. Prednisone (Deltasone) C. Montelukast (Singulair) D. Pimecrolimus (Elidel)

(c) A. Permethrin is used to kill the scabies mite. (u) B. Corticosteroids have no affect on mites. (u) C. Montelukast would possibly provide symptomatic relief but would not be curative for scabies. (u) D. Pimecrolimus has no affect on destruction of mites.

What behavior would be most typical for a patient diagnosed with schizoid personality disorder? A. Chooses solitary activities B. Odd thinking and speech C. Reckless disregard for safety of others D. Uses physical appearance to draw attention

(c) A. Persons with schizoid personalities are very withdrawn and do not seek or enjoy relationships and are indifferent to praise or criticism. They generally appear cold and unfeeling to others. (u) B. Schizotypal personalities have this along with magical thinking and ideas of reference. They may become withdrawn due to the lack of acceptance. (u) C. Reckless disregard for safety of others is typical of antisocial personality disorders. (u) D. Using physical appearance to draw attention is more typical of histrionic disorder.

Which of the following is the most likely to develop into a persistent cough in the adult patient? A. Pertussis B. Allergic rhinitis C. Pharyngitis D. Heart failure

(c) A. Pertussis is suspected in patients with persistent cough that lasts longer than 2-3 weeks. Allergic rhinitis, pharyngitis and heart failure are all potential causes of acute cough. (u) B. See A for explanation. (u) C. See A for explanation. (u) D. See A for explanation.

Which of the following agents can be used as a urinary analgesic? A. Phenazopyridine (Pyridium) B. Oxybutynin (Ditropan) C. Finasteride (Proscar) D. Imipramine (Tofranil)

(c) A. Phenazopyridine is a urinary tract analgesic used in the treatment of urinary tract discomfort. (u) B. Oxybutynin is an antispasmodic and anticholinergic used in the treatment of overactive bladder. (u) C. Finasteride is an alpha-blocker used in the treatment of benign prostatic hypertrophy. (u) D. Imipramine is an anticholinergic used to treat childhood enuresis.

A 32 year-old male with a history of pheochromocytoma is seen in the office. The patient is scheduled for adrenalectomy, however has developed a throbbing headache and racing heart. Vital signs reveal pulse 126 bpm, blood pressure 160/115 mmHg, and respiratory rate 20. The patient appears diaphoretic and anxious. Which of the following is the most appropriate acute management in this patient? A. Oral Phenoxybenzamine (Dibenzyline) B. Hydrochlorothiazide (Diuril) C. Lisinopril (Prinivil) D. Bumetanide (Bumex)

(c) A. Phenoxybenzamine is an alpha-blocker utilized to control hypertension in patient with a pheochromocytoma. (u) B. Hydrochlorothiazide is not appropriate in the management of hypertension in a patient with a pheochromocytoma. (u) C. ACE inhibitors are not indicated in the management of hypertension in a patient with pheochromocytoma. They may be used if blood pressure is difficult to control with alpha-blockers alone. (u) D. Bumetanide is not appropriate in the management of hypertension in a patient with a pheochromocytoma.

: (29) 106. Health Maintenance/Pulmonology In which of the following populations is the pneumococcal conjugate vaccine (Pneumovax) indicated for a patient younger than 65 years of age? A. Chronic alcohol abusers B. Pregnant women C. Healthcare workers D. Children receiving aspirin therapy

(c) A. Pneumococcal vaccination is not indicated in pregnant women, healthcare workers or children receiving aspirin therapy. Vaccination is indicated in chronic alcohol abusers. (u) B. See A for explanation. (u) C. See A for explanation. (u) D. See A for explanation.

What is the most common cause of secondary amenorrhea? A. Pregnancy B. Ovarian failure C. Imperforate hymen D. Hypothalamic amenorrhea

(c) A. Pregnancy is the most common cause of amenorrhea and is essential to exclude by a serum or urine pregnancy test. (u) B. With ovarian failure, the ovarian follicles are resistant to stimulation. Ovarian failure can be caused by chromosomal abnormalities, premature menopause, or a complication of chemotherapy. (u) C. An imperforate hymen would prevent menstrual bleeding, this is a cause of primary amenorrhea. (u) D. The pulsatile release of GnRH is disrupted and the anterior pituitary gland is not stimulated to release FSH and LH. This can be caused by different etiologies including: weight loss, weight gain, excessive exercise, drug induced, tumors, anorexia, and other chronic medical illnesses.

A 22 year-old male presents to the clinic complaining of scrotal pain that radiates into the groin. Patient admits to being a weightlifter and was lifting 24 hours prior to this pain developing into the scrotum. The patient admits to being sexually active with only his male partner. Examination reveals a reddened scrotum and it is difficult to distinguish the epididymis from the testes on the right side. Elevation of the right testicle brings relief of the pain. This is known as a positive A. Prehn's sign. B. Cullen's sign. C. Rovsing's sign. D. Murphy's sign.

(c) A. Prehn's sign is seen in epididymitis when elevation of the scrotum with the affected epididymis to the level of the symphysis pubis brings relief from the pain. (u) B. Cullen's sign is a bluish discoloration of the umbilicus resulting from hematoperitoneum. (u) C. Rovsing's sign is seen in appendicitis when pressure on the left quadrant produces pain in the right lower quadrant. (u) D. Murphy's sign is seen in acute cholecystitis with a sharp increase in tenderness when the gall bladder touches the examining hand causing a stop of the inspiratory effort.

: (9) 54. Diagnosis/Urology/Renal A 74 year-old female presents to the ED with nausea, vomiting, malaise, and confusion. The patient lives alone, but was brought in by her daughter who stopped to visit and noticed the changes in her mother since she had last seen her 48 hours before. Her urinalysis is negative for infection and her BUN:Creatinine ratio is greater than 20:1. The patient is most likely suffering from which type of acute kidney failure? A. Prerenal azotemia B. Postrenal azotemia C. Intrinsic renal failure D. Chronic renal failure

(c) A. Prerenal azotemia can present as nonspecific symptoms including nausea, vomiting, malaise, and confusion. Prerenal causes are most common and include hypovolemia, which is the likely etiology in this case as her BUN:Creatinine ratio is consistent with dehydration. (u) B. Postrenal azotemia is relatively uncommon and can occur as the result of obstruction of urine flow from both kidneys. Causes include ureteral obstruction and bladder obstruction. Kidney stones (or in men, BPH) are a common cause. (u) C. Intrinsic renal failure occurs as a direct result of insult to the kidney. It should be considered if pre- or postrenal azotemia is not the appropriate diagnosis. (u) D. These are acute changes and we have no reason to suspect that the patient has longstanding kidney insufficiency.

Which of the following is a cause of acute kidney failure due to prerenal azotemia? A. Excessive diuresis B. Urinary tract obstruction C. Radiologic contract media D. Aminoglycosides

(c) A. Prerenal azotemia is due to renal hypoperfusion which can occur with intravascular volume depletion such as excessive diuresis, hemorrhage, and gastrointestinal losses. (u) B. Postrenal azotemia is due to obstruction of urinary flow from both of the kidneys. (u) C. Radiologic contrast media can be directly nephrotoxic causing acute tubular necrosis, which is an intrinsic renal disease. (u) D. Exogenous nephrotoxins, such as aminoglycosides, cause acute tubular necrosis, which is an intrinsic renal disease.

: (7) 172. Health Maintenance/ENT/Ophthalmology Which of the following is the most common cause of sensorineural hearing loss in adults? A. Presbycusis B. Acoustic neuroma C. Ménière's disease D. Otosclerosis

(c) A. Presbycusis is the most common form of sensorineural hearing loss. (u) B. Acoustic neuroma results in sensorineural hearing loss; however, this is less common than age related hearing loss (presbycusis). (u) C. Sensorineural hearing loss is part of the complex of Ménière's disease; however this is less common than presbycusis. (u) D. Otosclerosis is a common cause of conductive hearing loss.

What is the most effective prophylaxis against respiratory syncytial virus (RSV) infection in the general pediatric population? A. Proper hand-washing techniques B. A monoclonal antibody C. H. Influenzae B vaccine D. Oseltamivir (Tamiflu)

(c) A. Proper hand washing and reduction in exposure is most effective in general population to prevent RSV. (u) B. Prophylaxis with a monoclonal antibody has proven effective in high risk infants but is not indicated in the general pediatric population. (u) C. Prophylaxis with H. Influenzae B vaccine has reduced incidence of epiglottis, not RSV. (u) D. Oseltamivir is not indicated in the prophylaxis of RSV.

Radioactive iodine (I131) is most successful in treating hyperthyroidism that results from A. Grave's disease. B. subacute thyroiditis. C. Hashimoto's thyroiditis. D. papillary thyroid carcinoma.

(c) A. Radioactive iodine (I131) is an excellent method to destroy overactive thyroid tissue of Grave's disease. (u) B. Radioactive iodine is ineffective in subacute thyroiditis due to the thyroid's low uptake of iodine. (u) C. Radioiodine uptake is low in Hashimoto's thyroiditis making radioactive therapy ineffective. (u) D. Papillary thyroid carcinoma is a common thyroid malignancy and should be treated by a thyroidectomy.

Radioactive iodine is most successful in treating hyperthyroidism that results from A. Grave's disease. B. subacute thyroiditis. C. Hashimoto's thyroiditis. D. papillary thyroid carcinoma.

(c) A. Radioactive iodine is an excellent method to destroy overactive thyroid tissue of Grave's disease. (u) B. Radioactive iodine is ineffective in subacute thyroiditis due to the thyroid's low uptake of iodine. (u) C. Radioiodine uptake is low in Hashimoto's thyroiditis and is often transient. (u) D. Papillary thyroid carcinoma is a common thyroid malignancy and must be treated by a thyroidectomy.

Which of the following describes the pathophysiological changes of pulmonary sarcoidosis? A. granulomas and inflammation of alveoli, small bronchi and small blood vessels B. inflammation and destruction of the structural components of the bronchial wall C. increased permeability of the alveolar-capillary membrane and diffuse alveolar damage D. edema of the airways with eosinophils, neutrophils, and lymphocytes

(c) A. Sarcoidosis is characterized by granulomas and inflammation of alveoli, small bronchi and small blood vessels. (u) B. Bronchiectasis is characterized by inflammation and destruction of the structural components of the bronchial wall. (u) C. Acute respiratory distress syndrome is characterized by increased permeability of the alveolar-capillary membrane and diffuse alveolar damage. (u) D. Asthma is characterized by edema of the airways with eosinophils, neutrophils, and lymphocytes

Which of the following is the laboratory test that marks recovery from Hepatitis B infection and non-infectivity? A. Hepatitis B surface antibody(anti-HBs) B. Hepatitis B surface antigen (HBsAg) C. Hepatitis B core antigen (HBcAg) D. Hepatitis A antibody (anti-HAV)

(c) A. Specific antibody to HBsAg appears in most individuals after clearance of HBsAg which indicates recovery from hepatitis B infection, non-infectivity, and immunity. (u) B. This test establishes infection with HBV and implies infectivity. (u) C. Presence of the hepatitis B core antigen reflects active infection. (u) D. Patients who have immunity to hepatitis A do not have immunity to hepatitis B.

Which of the following histories best describes spinal stenosis? A. Gradual onset of back and thigh pain exacerbated by walking and alleviated by sitting B. Acute onset of low back pain with radiation of pain to right foot, aggravated by sitting C. Aching in bilateral buttocks with associated pain felt in shoulder or neck D. Thigh pain aggravated by walking and absence of pedal pulses

(c) A. Spinal stenosis typically presents as back pain and thigh pain aggravated by ambulation and relieved by sitting. (u) B. Spinal stenosis is gradual in onset and is improved by sitting. (u) C. Spinal stenosis, unlike polymyalgia rheumatic, does not have associated shoulder or neck pain. (u) D. True claudication will have thigh pain similar to spinal stenosis but can be differentiated by absence of pulses. Spinal stenosis preserves distal pulses.

A 47 year-old HIV positive female presents with a complaint of a nonproductive cough. She is febrile, tachypneic and tachycardic. Lung exam reveals bilateral rales. Chest x-ray shows diffuse interstitial infiltrates. What is the recommended treatment in this patient? A. trimethoprim-Sulfamethoxazole (Bactrim) B. tetracycline (Sumycin) C. amantadine (Symmetrel) D. ticarcillin (Ticar)

(c) A. TMP-SMX is the drug of choice for all forms of pneumocystis. (u) B. Tetracycline, amantadine and ticarcillin are not effective against PCP. (u) C. See B for explanation. (u) D. See B for explanation.

A 78 year-old patient who is in acute distress with near-syncope and lightheadedness is being examined. The patient's pulse is 40/min and blood pressure is 90/56 mm Hg. Examination of the patient at 45 degrees of elevation reveals cannon "a" waves. Which of the following is the most likely explanation for these abnormal waves? A. atrioventricular dissociation B. aortic stenosis C. systolic hypertension D. left ventricular hypertrophy

(c) A. The patient is in a third-degree heart block with the atria contracting against a closed atrioventricular valve, which would be the scenario in a patient who has an escape rate of 40. Elderly patients are at risk for heart conduction problems such as complete heart block. (u) B. Left ventricular hypertrophy, systolic hypertension, and aortic stenosis are not causes of "a" waves. (u) C. See B for explanation. (u) D. See B for explanation

A 52 year-old male, who underwent transphenoidal surgery and radiation for acromegaly 15 years ago, presents with the following exam findings: weight gain of 20 lbs, puffy eyes, coarse dry skin and normal visual fields. Laboratory findings reveal the following: TSH 0.3 μu/ml (normal 0.5-5.0), free T4 0.3 ng/ml (normal 0.7-2.7). Which of the following drugs would be indicated for this patient? A. levothyroxine (Synthroid) B. growth hormone C. PTU (Propylthiouracil) D. bromocriptine (Parlodel)

(c) A. The patient is suffering from hypothyroidism as indicated by the low free T4. TSH is not elevated in this case secondary to the previous history of transphenoidal surgery and radiation for his acromegaly. (u) B. See A for explanation. (u) C. PTU is utilized for hyperthyroidism. (u) D. Bromocriptine is utilized for treatment of prolactinoma.

A 65 year-old patient has a long history of schizophrenia that is treated with phenothiazines. On an unrelated clinic visit, the patient has difficulty sticking out her tongue, facial tics, increased blink frequency, and lip-smacking behavior. These involuntary movements are most suggestive of A. tardive dyskinesia. B. Parkinson's disease. C. Huntington's disease. D. Tourette's syndrome.

(c) A. Tardive dyskinesia is characterized by abnormal involuntary movements of the face, mouth, tongue, trunk, and limbs and may develop after months or years of treatment with neuroleptic drugs. (u) B. Infrequent blinking, tremor, rigidity, and bradykinesia are characteristic of Parkinsonism. (u) C. Although part of the differential for involuntary movements, this disease has a positive family history and usually appears by age 50. (u) D. Facial motor tics are the most common manifestation of this disorder, but symptoms begin before age 21.

: 3. Diagnosis/Pulmonology A 58 year-old with a 20 pack-year history of tobacco use presents to the emergency department with sudden onset of severe, progressive right-sided chest pain and shortness of breath. She denies fever, nausea and productive cough but reports returning from a mission trip to South Africa yesterday. Her exam reveals tachycardia, tachypnea, clear lung fields and 3+ edema and erythema of the left leg. Her chest radiograph is normal. Which of the following is the most likely diagnosis? A. Pulmonary embolism B. Pneumothorax C. Pneumonia D. Pleurisy

(c) A. The patient's risk factors, acutely progressive presentation and normal chest radiograph make a pulmonary embolism the most likely and most worrisome diagnosis. (u) B. See A for explanation. (u) C. See A for explanation. (u) D. See A for explanation.

Which of the following is a non-invasive quick method of evaluating a patient with suspected lower extremity arterial insufficiency? A. Ankle-Brachial Index B. Striker Tonometry C. CT Angiography D. Lower extremity arteriography

(c) A. The single most useful index is the ankle pressure. This can be obtained with an Ankle-Brachial Index (ABI) which the severity of signs and symptoms of arterial insufficiency are correlated to the findings on the ABI. It is a non-invasive study that can be performed in an office setting. (u) B. Striker Tonometry is used to evaluate compartment pressures. (u) C. CT angiography is an invasive test involving radiation and contrast that cannot be done in an office setting. (u) D. Lower extremity arteriography is an invasive test involving radiation and contrast that cannot be done in an office setting.

A 28 year-old male presents with burns sustained from hot grease splashed on his left hand earlier this afternoon. The burn extends from his palm to the volar aspect of his wrist and has an erythematous base, covered by an intact blister. There are a few small scattered blisters over the dorsum of the left hand. Which of the following is the initial intervention of choice? A. Tetanus prophylaxis B. Admission to a burn unit C. Intravenous fluid administration D. Debridement of blisters

(c) A. Tetanus prophylaxis should be initially considered in all burn patients. (u) B. Admission to a burn unit is not indicated for adult patients with uncomplicated partial thickness burns covering less than 15 to 20% of total body surface area (TBSA). (u) C. IV fluids are indicated for severe partial thickness burns covering more than 10% TBSA or in burns with complications. (u) D. Debridement of blisters is controversial, however blisters on the palms and soles should remain intact.

A 34 year-old male presents with symptoms of painful urethral discharge. History reveals recent (7 days ago) intercourse with a new partner. A Gram stain is negative for intracellular diplococci. Assuming no allergies, which of the following is the antibiotic of choice? A. doxycycline B. penicillin G C. ciprofloxacin (Cipro) D. cephalexin (Keflex)

(c) A. Tetracyclines, such as doxycycline, are drugs of first choice for Chlamydia. (u) B. Penicillin G is not used in treating chlamydial infections. (u) C. Cephalexin and ciprofloxacin are not effective in treating Chlamydial infections. (u) D. See C for explanation.

Which of the following would be expected on physical examination of a newborn diagnosed with Tetrology of Fallot? A. Palpable right ventricular lift B. Pulse discrepancy between arms and legs C. Mid-diastolic murmur with opening snap D. Polymorphous exanthema

(c) A. Tetralogy of Fallot is commonly associated with a palpable right ventricular lift. (u) B. Coarctation of the aorta is associated with a pulse discrepancy between the upper and lower extremities. (u) C. A mid-diastolic murmur with an opening snap is heard in a patient with mitral stenosis, not Tetralogy of Fallot. (u) D. Polymorphous exanthema is seen in patients with Kawasaki disease.

Screening for hypothyroidism is indicated in which of the following patients?? A. Newborn B. Young female with anxiety C. Patient with new onset of atrial fibrillation D. Patient with heat intolerance

(c) A. The American Academy of Pediatrics recommends screening of hypothyroid disease between 2-4 days of birth. (u) B. Hypothyroidism is not a cause of anxiety in young females. Anxiety is more common with hyperthyroidism. (u) C. Atrial fibrillation is more likely to occur with thyrotoxicoxis. (u) D. Hypothyroidism presents most commonly with cold intolerance.

: (28) 87 224. History & Physical/Cardiology On cardiac auscultation, you hear a mid-diastolic low-pitched mitral murmur heard best at the cardiac apex. The presence of this sound is most characteristic of which valvular abnormality? A. Aortic regurgitation B. Mitral regurgitation C. Pulmonic stenosis D. Tricuspid stenosis

(c) A. The Austin Flint murmur is a mid or late diastolic low-pitched mitral murmur heard in advanced aortic regurgitation, owing to partial obstruction of mitral inflow produced by partial closure of the mitral valve by the regurgitant jet and the rapidly rising left ventricular diastolic pressure. (u) B. Mitral regurgitation murmur would be a pansystolic murmur. (u) C. Pulmonic stenosis results in a systolic murmur. (u) D. Tricuspid stenosis may produce a soft opening snap and a mid-diastolic rumble with presystolic accentuation.

When evaluating jugular venous pulsations a prominent a wave represents which of the following? A. Atrial contraction against a closed tricuspid valve B. Rapid filling of the right atrium C. Tricuspid regurgitation D. Poor left ventricle compliance

(c) A. The a wave corresponds to right atrial contraction. (u) B. See A for explanation. (u) C. See A for explanation. (u) D. See A for explanation.

The pathologic process responsible for the renal damage in post-streptococcal glomerulonephritis is which of the following? A. Immunologic B. Vascular C. Hormonal D. Genetic

(c) A. The antigen-antibody complex that occurs as a result of streptococcal infection is the result of an abnormal immunologic response. (u) B. See A for explanation. (u) C. See A for explanation. (u) D. See A for explanation.

: (10) 28. Clinical Therapeutics/ENT/Ophthalmology A 74 year-old male presents with sudden onset of unilateral eye pain. He reports severe bulbar pain, decreased vision of the left eye and nausea. Examination is significant for decreased visual acuity, marked ciliary injection and mid-dilated pupil. Which of the following is the most appropriate initial treatment for this disorder? A. Topical pilocarpine B. Trabeculectomy C. Topical atropine D. Laser iridectomy

(c) A. The initial treatment in acute angle closure glaucoma (AACG) is aimed at induction of pupilary constriction. (u) B. Trabeculectomy or laser iridectomy are used in the management of AACG; however, this is typically not part of the initial treatment of acute disease. (h) C. Pupil dilation is contraindicated in the presence of acute angle closure glaucoma. (u) D. See B for explanation.

A 22 year-old female presents with onset of right eye vision loss 24 hours ago which has partially improved. She has experienced various widespread paresthesias, regional weakness and fatigue over the past six months. Episodes vary in location, severity and duration but invariably lead to recovery. Her exam is completely normal but MRI reveals numerous areas of periventricular gadolinium uptake. For this condition, what treatment has shown the greatest benefit with long term use? A. Interferon beta 1a (Avonex) B. Ethosuximide (Zarontin) C. Methylprednisolone (Solumedrol) D. Pyridostigmine (Mestinon)

(c) A. The interferon agents have been the longest used and best studied medications that provide the best long term benefits in multiple sclerosis. Steroids do play a role in acute exacerbations but not as long term agents. (u) B. See A for explanation. (u) C. See A for explanation. (u) D. See A for explanation.

A patient presents complaining of gradual hearing loss over the past 3 months. He admits to use of Q-tips and otherwise does not wear ear plugs or place other foreign objects in his ear. On examination external auditory canals are obstructed with cerumen. After removal of cerumen, hearing is equal on both sides. Appropriate counseling of this patient includes which of the following? A. Advise him to discontinue use of cotton swabs B. Encourage jet irrigator (i.e. WaterPik) to clean ears C. Instruct in ear irrigation with cold water D. Refer to dermatologist

(c) A. In most people, the ear canal is self-cleansing. In most cases, cerumen impaction is self-induced through ill-advised attempts at cleaning the ear. (u) B. Use of jet irrigators designed for cleaning teeth (i.e. waterPik) for wax removal should be avoided since they may result in tympanic membrane perforations. (u) C. Irrigation is performed with water at body temperature to avoid a vestibular caloric response. (u) D. See A for explanation.

A solitary pulmonary nodule is found on a pre-employment screening chest x-ray in a 34 year-old nonsmoking male. There are no old chest x-rays to compare. Which of the following is the most appropriate next step in the evaluation? A. CT scan of the chest B. Needle biopsy of the lesion C. Positron emission tomography of the chest D. Fiberoptic bronchoscopy

(c) A. In the absence of old x-rays in a nonsmoking individual less than 35 years old, CT scan of the chest is the next step in the evaluation of a solitary pulmonary nodule. (u) B. A needle biopsy would be indicated for a person greater than 35 years old and/or with a history of smoking to evaluate a solitary pulmonary nodule. (u) C. Positron emission tomography (PET scan) would be indicated if the CT scan was nonconclusive. (u) D. Fiberoptic bronchoscopy would be indicated only in the presence of a history of tobacco use or if the lesion was suggestive of malignancy.

A mean corpuscular volume (MCV) of less than 80 cubic microns is a manifestation of which of the following diagnoses? A. Iron deficiency B. Vitamin B12 deficiency C. Folate deficiency D. G6PD deficiency

(c) A. Iron deficiency is associated with microcytic anemia. (u) B. Vitamin B12 deficiency is associated with macrocytic anemia. (u) C. Folate deficiency is associated with macrocytic anemia. (u) D. G6PD deficiency is not associated with a low MCV.

The most effective preventive strategy to prevent recurrence of renal lithiasis is which of the following? A. increase in hydration B. early treatment of urinary tract infection C. limitation of calcium intake D. use of probenecid

(c) A. Keeping the urine dilute is the most effective strategy to prevent crystal accumulation in the urine and the development of urinary stones. (u) B. Treating UTI only affects the formation of calcium pyrophosphate or struvite stones. (u) C. Limitation of calcium in the diet reduces the ability of calcium to bind oxalate leading to the production of calcium oxalate stones. (u) D. Probenecid is a uricosuric agent that promoted uric acid secretion in the urine that may promote more uric acid stones.

Kussmaul breathing is characterized by A. rapid, deep labored breathing. B. irregular and varying depth of breathing. C. frequently interspersed deeper breaths. D. periods of deep breathing alternate with periods of apnea.

(c) A. Kussmaul breathing is characterized by rapid deep labored breathing. (u) B. This characterizes ataxic breathing. (u) C. This characterizes sighing. (u) D. Periods of deep breathing alternate with periods of apnea is Cheyne-Stokes breathing.

Low molecular weight heparin (LMWH) dosage is based on which of the following? A. Weight B. International normalized ratio (INR) C. Prothrombin time (PT) D. Partial thromboplastin time (PTT)

(c) A. LMWH is based on a patient's weight in kilograms. (u) B. INR is used to monitor anticoagulant medications. (u) C. PT is used to monitor warfarin efficacy. (u) D. PTT is used to monitor heparin use but not to determine the dosage.

A 25 year-old male with history of syncope presents for evaluation. The patient admits to intermittent episodes of rapid heart beating that resolve spontaneously. 12 Lead EKG shows delta waves and a short PR interval. Which of the following is the treatment of choice in this patient? A. Radiofrequency catheter ablation B. Verapamil (Calan) C. Percutaneous coronary intervention D. Digoxin (Lanoxin)

(c) A. Radiofrequency catheter ablation is the treatment of choice on patients with accessory pathways, such as Wolff-Parkinson-White Syndrome. (h) B. Calcium channel blockers such as verapamil decrease refractoriness of the accessory pathway or increase that of the AV node leading to faster ventricular rates, therefore calcium channel blockers should be avoided in patients with WPW. (u) C. Percutaneous coronary intervention is indicated in the treatment of coronary artery disease, not preexcitation syndromes. (h) D. Digoxin decreases refractoriness of the accessory pathway and increases that of the AV node leading to faster ventricular rates. It should therefore be avoided in patients with WPW.

Which of the following is the underlying pathogen for the development of Lyme disease? A. Borrelia burgdorferi B. Bartonella henselae C. Rickettsia rickettsii D. Coxiella brunetti

(c) A. Lyme disease is a tick-borne illness caused by the spirochete B. burgdorferi. (u) B. Bartonella henselae is the spirochete that causes cat scratch fever. (u) C. Rickettsia Rickettsii is the spirochete that causes Rocky Mountain Spotted Fever. (u) D. Coxiella brunetti is the spirochete that causes Q fever.

A patient with known drug dependence mentions that he commonly sees sounds and hears colors. What is his drug of choice? A. Lysergic acid diethylamide B. Cannabis sativa C. Cocaine D. Heroin

(c) A. Lysergic acid diethylamide is LSD, the phenomenon that this patient is experiencing is called synesthesia, which is common in patients who abuse hallucinogens. (u) B. See A for explanation. (u) C. See A for explanation. (u) D. See A for explanation.

: (10) 141. Health Maintenance/Endocrinology Your patient is an 18 year-old male with a family history of multiple endocrine neoplasia I (MEN I). Which of the following screening electrolyte levels would you recommend for this patient? A. Calcium B. Potassium C. Magnesium D. Sodium 57

(c) A. MEN 1 is a familial multiglandular endocrine tumor syndrome. The initial biochemical manifestation of MEN 1 is usually hypercalcemia and can be detected as early as 14-18 years old in patients with the gene mutation, although clinical symptoms do not usually manifest until the third or fourth decade. The other electrolytes are not necessarily affected. (u) B. See A for explanation. (u) C. See A for explanation. (u) D. See A for explanation.

: (8) 180. Diagnosis/Infectious Diseases A 23 year-old Peace Corps volunteer returned from Central America 2 weeks ago. She presents with a week long history of intermittent headache and myalgias. Today she is experiencing shaking chills followed by a temperature of 103.5 F and diaphoresis. She reports she had similar symptoms 2 days ago which she thought was the flu. Which of the following is the most likely diagnosis? A. Malaria B. Toxoplasmosis C. Shigellosis D. Amebiasis

(c) A. Malaria affects patients who live or travel to endemic areas. Classic presentation includes intermittent attacks of chills, fever and sweating. Patients can also experience flu-like symptoms, headache, myalgias and/or nausea and vomiting. (u) B. See A for explanation. (u) C. See A for explanation. (u) D. See A for explanation.

A patient is traveling to Africa on a church mission trip. The area he is traveling to has a high incidence of chloroquine-resistant malaria. What drug is indicated for prophylaxis in this patient? A. Atovaquone (Malarone) B. Hydroxychloroquine sulfate (Plaquenil) C. Quinine sulfate (Qualaquin) D. Lumefantrine (Benflumetol)

(c) A. Malarone is used for prophylaxis for malaria in areas with chloroquine- or mefloquine-resistant malaria. (u) B. Plaquenil is used for primary prophylaxis only in areas with chloroquine-sensitive malaria. Copyright © 2010. Physician Assistant Education Association 63 (u) C. Quinine is an antimalarial drug used for treatment not prophylaxis. (u) D. Lumefantrine is an antimalarial drug used for treatment not prophylaxis.

A 58 year-old male presents with chest pain. Vital signs include blood pressure of 210/175, pulse 80, RR 20. Which of the following would you expect to find on physical examination? A. papilledema B. carotid bruit C. diastolic murmur D. absent peripheral pulses

(c) A. Malignant hypertension is characterized by marked blood pressure elevation with papilledema, often with encephalopathy or nephropathy. (u) B. Carotid bruits are associated with carotid artery stenosis. (u) C. Diastolic murmurs are associated with valvular heart disease such as aortic regurgitation and mitral stenosis. (u) D. Peripheral pulses are absent in acute arterial occlusion or severe peripheral arterial disease.

A 45 year-old female presents to the emergency department with generalized, hot, erythema of the skin. Physical exam reveals an oral temperature of 102 degrees Fahrenheit, purulent conjunctivitis, and mucosal erosions. Her skin is painful and separates from the dermis with touch. Which of the following is the most likely cause for this condition? A. Ampicillin B. Prednisolone C. Aspirin D. Hydrochlorothiazide (HCTZ)

(c) A. Medications are most frequently implicated in toxic epidermal necrolysis. These usually include, analgesics (NSAIDs), antibiotics (Ampicillin) and anticonvulsants (Carbamazepine). (u) B. Systemic glucocorticoids may be used early in the treatment of this condition and are not a cause. (u) C. Aspirin is not linked to toxic epidermal necrolysis. (u) D. Hydrochlorothiazide diuretics are not associated with the production of toxic epidermal necrolysis.

A 46 year-old woman describes her menstrual periods as regular (occurring every 30 days), prolonged, and with a heavy flow. You document this finding as which of the following? A. Menorrhagia B. Oligomenorrhea C. Metrorrhagia D. Hypomenorrhea

(c) A. Menorrhagia describes long and heavy flow menstrual flow. (u) B. Oligomenorrhea describes menstrual periods that occur more than 35 days apart. (u) C. Metrorrhagia is bleeding that occurs at any time between menstrual periods. (u) D. Hypomenorrhea is usually light flow, sometimes only spotting

Which of the following may precipitate acute angle-closure glaucoma? A. metoclopramide B. timolol C. glyburide D. acetazolamide

(c) A. Metoclopramide and other drugs with high anticholinergic effects may precipitate acute angle-closure glaucoma from pupillary dilation. (u) B. Timolol, a beta-antagonist, is used in the treatment of acute angle-closure glaucoma. (u) C. Glyburide has no relationship to glaucoma. (u) D. Acetazolamide, a carbonic anhydrase inhibitor, may suppress the production of aqueous humor by 40-60% and is used in the emergency treatment of glaucoma.

A 45 year-old patient with type 1 diabetes mellitus is being screened for diabetic nephropathy. Which of the following urinalysis findings is most consistent with early diabetic nephropathy? A. Microalbuminuria B. Red cell casts C. White cell casts D. Renal epithelial cells

(c) A. Microalbuminuria is most consistent with early diabetic neuropathy. (u) B. Red cell casts are more indicative of acute glomerular nephritis. (u) C. White cell casts are more consistent with acute pyelonephritis. (u) D. A few renal epithelial cells normally may be found in the urine.

The causative organism for molluscum contagiosum is which of the following? A. virus B. gram-negative bacterium C. gram-positive bacterium D. parasite

(c) A. Molluscum contagiosum is caused by a poxvirus. (u) B. See A for explanation. (u) C. See A for explanation. (u) D. See A for explanation.

An otherwise healthy 23 year-old female presents to the student health office complaining of 3 days of frequent, watery, non-bloody stools. She denies significant abdominal pain, vomiting, fever or dark urine. Others on campus have been seen with the same presentation this week. Initial choice of treatment includes which of the following? A. Fluid intake and bowel rest B. Ciprofloxacin C. Hospital admission with IV fluids D. Atropine

(c) A. Most mild diarrhea will not lead to dehydration with adequate fluids and comfort with rest to the bowel. (u) B. Empiric treatment with antibiotics is not indicated this early. (u) C. See A for explanation. (h) D. Atropine is contraindicated as an antidiarrheal due to the possibility of toxic megacolon.

A 2 year-old male presents with a four day history of fever and general malaise. On examination the vitals reveal an oral temperature of 102 degrees F. The child appears to have rubor on the trunk which started one day prior to this visit. Physical examination reveals a maculopapular rash with defervescence. Which of the following is the most appropriate management at this time? A. Ibuprofen (Motrin) B. Aspirin C. Amoxicillin D. Valacyclovir (Valtrex)

(c) A. Motrin is indicated for management of the fever in Roseola infantum caused by the herpesvirus. (h) B. Aspirin is contraindicated in children for management of fever. (u) C. Amoxicillin is not indicated for viral infections. (u) D. Valacyclovir is indicated for Varicella-Zoster virus infection for decreasing the incidence of varicella pneumonia only. In healthy children valacyclovir is not indicated due to its marginal therapeutic benefit.

A mother brings her 14 month-old son to your clinic. Earlier today she lifted her son by grabbing him by the wrists and pulling him up off the floor. The child is sitting in his mother's lap with his left forearm is extended and in pronation. He is refusing to move the left arm, forearm or wrist. The arm and joints appear normal with no noted deformities, edema or erythema. Distal pulses and capillary refill are normal and he can move his fingers. Which of the following is the most likely diagnosis? A. Nursemaid's elbow B. Fractured left wrist C. Osteochondritis dissecans D. Child abuse

(c) A. This clinical history is classic for radial head dislocation or nursemaid's elbow. (u) B. There is no edema, deformity or erythema to suggest a fractured wrist. (u) C. Osteochondritis dissecans is avascular necrosis of subchondral bone, most commonly seen during adolescence. (a) D. Although child abuse could be suspected the clinical history best describes nursemaid's elbow.

A newborn male infant is seen by the PA prior to discharge home. On examination it is noted that the meatal opening is on the dorsal surface of the penile shaft just behind the coronal sulcus. The testicles are descended bilaterally. This physical examination finding is called which of the following? A. Penile epispadias B. Penile hypospadias C. Glandular epispadias D. Glandular hypospadias

(c) A. This is a penile epispadias which can develop anywhere along the penile shaft to the pubic bone. It is caused by displacement of urethra/meatal opening on the dorsal surface of the penis. If the opening occurs in the glans penis then it would be a glandular epispadias. If this abnormality occurs on the ventral surface of the penis or glans penis then it is known as a hypospadias. (u) B. See A for explanation. (u) C. See A for explanation. (u) D. See A for explanation.

A 52 year-old patient with episodes of syncope has an electrocardiogram which shows a consistently prolonged PR interval with a missing QRS every two beats. Which of the following is the most effective management? A. Permanent pacing B. Beta-blocker C. ACE Inhibitor D. Defibrillation

(c) A. This is consistent with ECG findings of a Mobitz type II AV block. Since the patient is symptomatic this type of AV block requires a permanent pacing to prevent total AV disassociation. (u) B. Beta-blockers will slow conduction from the AV node and is not indicated with this type of AV block. (u) C. There is no indication for ACE Inhibitors in Mobitz Type II heart block. (u) D. Defibrillation is not indicated in a person with AV block.

A 26 year-old female presents to clinic with a left arm that is swollen and non-tender with bluish discoloration along the upper arm and forearm. She is status post pacemaker insertion in the left upper chest for third degree heart block, one week ago. Pulses are present and the arm is warm, but not red. The pacemaker incision is healing well despite a hematoma and tenderness at the site. Which of the following statements would be appropriate patient education about this condition? A. Reassurance that the discoloration is an expected finding. B. Apply cold compresses to the site of the hematoma. C. Elevation of the involved extremity will increase the swelling. D. Aspirin should be taken to help manage pain.

(c) A. This is indicative of migratory ecchymosis and expected after insertion of a pacemaker. (u) B. Warm compresses will help to decrease the hematoma. (u) C. Elevation will help to decrease the swelling. (u) D. ASA is not the pain medicine of choice in a patient with a hematoma due to its non-clotting properties.

: (27) 70. Diagnosis/Urology/Renal Hypertension, hematuria, and dependent edema are the classic symptomatic triad of what type of kidney disease? A. Nephritic syndrome B. Nephrotic syndrome C. Hemolytic uremic syndrome D. Goodpasture syndrome

(c) A. This is the classic triad for nephritic syndrome. Edema is first seen in areas of low tissue pressure such as the periorbital and scrotal areas; hypertension is due to volume overload. Hematuria may or may not also include red blood cell casts. (u) B. Heavy proteinuria, hypoalbuminemia, hyperlipidemia, and edema are more typical of nephrotic syndrome. (u) C. Hemolytic uremic syndrome is common in children following infection, usually diarrheal illness. (u) D. Goodpasture syndrome is defined by the clinical constellation of glomerulonephritis and pulmonary hemorrhage.

An 18 year-old male high school basketball player comes to clinic for a routine physical exam. His height is 193 cm (76 in.); arm span is 201 cm (79 in.). He has long fingers and toes. Blood pressure is 146/62 mmHg and pulse is 64/min. Which of the following exam findings is most consistent with the diagnosis? A. Grade 2/6 high-frequency diastolic murmur at the third right intercostal space B. Grade 2/6 systolic ejection murmur at the second left intercostal space with a fixed widely split S2 C. Grade 2/6 continuous murmur heard best at the high left sternal border D. Grade 2/6 systolic murmur at the fourth left intercostal space that decreases with squatting

(c) A. This murmur is most consistent with aortic regurgitation which can be present in patients with Marfans syndrome and a dilated aortic root. (u) B. This murmur is most consistent with an atrial septal defect. (u) C. This murmur is most consistent with a patent ductus arteriosus and unlikely in this age patient. (u) D. This murmur is most consistent with hypertrophic obstructive cardiomyopathy.

A 56 year-old male is noted to have a recent diagnosis of polycythemia vera. His current hemoglobin is 21 gms/dl. What treatment should be instituted for this patient at this time? A. Phlebotomy B. Iron chelation therapy C. Bone marrow radiation therapy D. Normal saline IV hydration

(c) A. This patient has a diagnosis of polycythemia vera treatment begins with phlebotomy instituted on a weekly basis until the hematocrit is less than 45%. Maintainance of the hematocrit at 45% is achieved with repeated phlebotomy as necessary. (u) B. See A for explanation. (h) C. Radiation therapy has no place in this treatment. (h) D. IV hydration with normal saline will place the patient into volume overload.

A 37 year-old female with history of Turner's syndrome and coarctation of the aorta repaired at the age of 3 presents for routine examination. The patient is without complaints of chest pain, dyspnea, palpitations, or syncope. On examination vitals signs reveal a BP of 130/76, HR 70, regular, RR 16. On cardiac examination you note a grade II/VI systolic ejection murmur at the left sternal border and a grade III/VI blowing diastolic murmur. Which of the following does this patient require? A. antibiotic prophylaxis B. beta blocker therapy C. chest CT D. exercise stress test

(c) A. This patient has a history of congenital heart disease and presently has a murmur consistent with aortic regurgitation. This patient requires antibiotic prophylaxis against infective endocarditis. (u) B. Beta blocker therapy may increase the amount of regurgitation because of increased diastolic time and is not indicated in this patient. (u) C. This patient should undergo serial echocardiograms, chest CT will not give information regarding any changes in the aortic regurgitation or ejection fraction. (u) D. This patient is without any complaints; exercise stress test is not indicated.

A 56 year-old female patient comes to the office for evaluation. She complains of dyspareunia and a thin vaginal discharge. On physical examination atrophic vulvar changes are noted associated with vaginal petechiae and a thin clear discharge. What medication is recommended to treat this patient's symptoms? A. Topical estrogen (Estrace) cream B. Topical hydrocortisone (Gynecort) C. Metronidazole (MetroGel) vaginal gel D. Terconazole (Terazol) vaginal suppository

(c) A. This patient has atrophic vaginitis which is very common in postmenopausal patients. The vagina will appear atrophied, will look pale, and thin and dry in appearance. Best to treat with topical or oral estrogen preparations. (u) B. Topical hydrocortisone would not be used in atrophic vaginitis. (u) C. MetroGel is the drug of choice for bacterial vaginosis. (u) D. Terazol is the treatment for non-albicans yeast infections.

A 75 year-old female with a history of long-standing hypertension presents with shortness of breath. On examination you note a diastolic murmur at the left upper sternal border. Which of the following maneuvers would accentuate this murmur? A. Sitting up and leaning forward B. Lying on left side C. Performing Valsalva maneuver D. Standing upright

(c) A. This patient has history findings consistent with aortic insufficiency which is characterized by a diastolic murmur that is accentuated when the patient sits up and leans forward. (u) B. The left lateral decubitus position accentuates the murmur of mitral stenosis. (u) C. Valsalva and standing maneuvers help to differentiate the murmurs associated with aortic stenosis and hypertrophic cardiomyopathy. (u) D. See C for explanation.

A 66 year-old female with a history of coronary artery disease presents with a new onset of dizziness and fatigue for two weeks. She recalls nearly passing out on one occasion. Examination is unremarkable except for bradycardia. Electrocardiogram (ECG) reveals a heart rate of 50 with a normal PR interval followed by a normal QRS. There are several non-conducting P waves and no lengthening of the PR interval. Which of the following interventions is the therapy of choice? A. Permanent pacemaker B. Radio-frequency ablation C. Maze procedure D. Automatic Implantable Cardioverter Defibrillator

(c) A. This patient has symptomatic second degree type II heart block and requires a pacemaker. (u) B. Radio-frequency ablation is useful for supraventricular tachyarrhythmias. (u) C. Maze procedure is a surgical procedure for decreasing the incidence of atrial fibrillation by creating cuts simulating a pathway in the atriums. (u) D. Automatic Implantable Cardioverter Defibrillator (AICD) is used in patients at risk for sudden death.

A 49 year-old female presents complaining of several episodes of chest pain recently. Initial ECG in the emergency department shows no acute changes. Two hours later, while the patient was having pain, repeat electrocardiogram revealed ST segment elevation in leads II, III, and AVF. Cardiac catheterization shows no significant obstruction of the coronary arteries. Which of the following is the treatment of choice in this patient? A. Nifedipine (Procardia) B. Metoprolol (Lopressor) C. Lisinopril (Zestril) D. Carvedilol (Coreg)

(c) A. This patient is most likely having coronary artery spasm. This can be treated prophylactically with calcium channel blockers such as nifedipine. (h) B. Beta-blockers may exacerbate the symptoms of coronary vasospasm. (u) C. ACE inhibitors are not effective in the treatment or prevention of coronary vasospasm. (u) D. Carvedilol is not effective in the treatment or prevention of coronary vasospasm.

Which of the following is the most common underlying cause of early postpartum hemorrhage? A. uterine atony B. genital tract trauma C. coagulation disorders D. retained placental tissue

(c) A. Uterine atony is the most common cause of postpartum hemorrhage. Predisposing factors include any conditions that cause excessive uterine enlargement, abnormal labor or conditions that interfere with uterine contraction. (u) B. Genital tract trauma may lead to lacerations of the lower genital tract that cause postpartum hemorrhage, however this is much less common than uterine atony. (u) C. Coagulation disorders can cause postpartum hemorrhage, but are not the most common cause. (u) D. Retained placental tissue only acccounts for 5-10% of postpartum hemorrhage and usually occurs later.

A 70 year-old male is admitted to the ICU with fever, leukocytosis and purulent sputum. Sputum culture shows methicillin-resistant gram-positive cocci in clusters. Which of the following medications should be ordered for this patient? A. Vancomycin (Vancocin) B. Clindamycin (Cleocin) C. Azithromycin (Zithromax) D. Astreonam (Azactam)

(c) A. Vancomycin is the drug of choice for methicillin-resistant S. aureus. (u) B. Clindamycin is used if the patient is at risk for Legionella infection. (u) C. Azithromycin is more commonly used for H. influenza coverage. (u) D. Astreonam covers gram positive organisms but not methicillin-resistant S. aureus.

You are following a patient in labor at term. You evaluate the fetal monitoring tracing and note the presence of variable, repetitive decelerations in the heart rates. The contractions have a sharp deceleration slope. What is the recommended treatment for these decelerations? A. Change maternal position B. Administer tocolytic therapy C. Apply vibroacustic stimulation D. Perform cesarean section

(c) A. Variable decelerations occur from umbilical cord compression and oligohydramnios. They are treated by changing maternal positioning to relieve pressure on the umbilical cord. Additionally, amnioinfusion may be used to relieve umbilical cord compression in cases of oligohydramnios. (u) B. See A for explanation. (u) C. See A for explanation. (u) D. Delivery of the baby by c-section is not indicated in variable FHR decelerations and may be harmful to the baby.

Which of the following is an extra-articular manifestation of rheumatoid arthritis? A. Vasculitis B. Malar rash C. Coronary artery aneurysms D. Periorbital xanthelasma

(c) A. Vasculitis affecting any organ system is seen in patients with severe rheumatoid arthritis. (u) B. A malar rash is frequently seen in systemic or discoid lupus. (u) C. Coronary artery aneurysms are frequently a manifestation of Kawasaki's disease. (u) D. Xanthelasma is a skin finding that occurs in hyperlipidemia.

When performing a rectal examination, prostatic massage is contraindicated in A. acute bacterial prostatitis. B. chronic bacterial prostatitis. C. nonbacterial prostatitis. D. prostatodynia.

(c) A. Vigorous manipulation of the prostate during rectal examination may result in septicemia. This is contraindicated in the presence of fever, irritative voiding symptoms, and perineal/sacral pain. (u) B. Prostate massage can be performed in the absence of fever. Expressed prostatic secretions are cultured to help identify the organism. (u) C. Nonbacterial prostatitis is similar to chronic bacterial prostatitis, but no bacteria are cultured, and the cause may be unknown. (u) D. Prostatodynia is a noninflammatory disorder involving voiding dysfunction and pelvic floor musculature dysfunction. There is no bacterial involvement.

: (27) 66. Clinical Therapeutics/Endocrinology Which of the following disorders responds most readily to treatment with vitamin D alone? A. Osteomalacia B. Osteitis deformans C. Osteoporosis D. Osteogenesis imperfecta

(c) A. Vitamin D deficiency decreases the intestinal absorption of calcium and is the most common cause of osteomalacia. Treatment is vitamin D in various natural and pharmaceutical forms. The addition of supplemental calcium is usually not necessary when treating osteomalacia. (u) B. Bisphosphonates are the treatment of choice for Paget disease of the bone (osteitis deformans). (u) C. A combination of Vitamin D plus Calcium is needed to treat osteoporosis. Bisphosphonates are another option. (u) D. Osteogenesis imperfecta is caused by a major mutation in the gene encoding for type 1 collagen, the major collagen constituent of bone. It cannot be effectively treated with calcium or vitamin D.

A 72 year-old female presents with vulvular pruritus for the last nine months, which has progressively worsened over the last two months. She states that she went through menopause at age 54 and has been on estrogen and progesterone therapy since that time. Physical examination reveals red lesions with white plaques on the vulva. What should the next course of management include? A. Refer to a gynecologist for biopsy. B. Refer to a dermatologist for antifungal therapy. C. Treat with a topical steroid. D. Treat with estrogen cream.

(c) A. Vulvular squamous cell hyperplasia causes thickening and hyperkeratosis of the vulva. The lesions are red and moist and cause intense pruritus over time the area becomes thickened and a white plaque may develop. Biopsy must be done to evaluate for intraepithelial neoplasm or invasive tumor. (u) B. See A for explanation. (u) C. See A for explanation. (u) D. See A for explanation.

A 56 year-old white post-menopausal female had a recent surveillance DEXA bone densitometry. The T-score is -2.7 for her right hip. In counseling this patient, in addition to medications, which of the following would you recommend? A. Weight bearing exercises B. Water aerobics C. Limit sun exposure D. Daily red wine consumption

(c) A. Weight bearing exercises are an appropriate adjunct to medication in a patient with osteoporosis. Water aerobics are non-weight bearing. Sun exposure adn vitamin D supplementation are useful in preventing and treating osteomalacia. (u) B. See A for explanation. (u) C. See A for explanation. (u) D. See A for explanation.

A 23 year-old sexually active female presents with a 4 day history of painless bilateral eye exudates which she describes as copious. Visual acuity is 20/20, generalized conjunctival inflammation with sparing of the cornea is noted on physical examination. Gram stain of the exudate reveals gram negative diplococci. Appropriate management of this case is A. ceftriaxone (Rocephin). B. polymyxin ophthalmic drops (Aerosporin). C. ciprofloxacin (Cipro). D. doxycycline (Doryx).

(c) A. With sparing of the cornea, as in this case, a single 1 gram IM dose of ceftriaxone is sufficient treatment for ophthalmic gonorrhea. If the cornea is involved, 5 days of IM ceftriaxone would be required. (u) B. Polymixin is ineffective against gonococcus. (u) C. Oral ciprofloxacin is not used in cases of gonococcal conjunctivitis. (u) D. Doxycycline is ineffective against gonococcus

: (8) 163. Clinical Intervention/Neurology A 58 year-old female with a history of controlled hypertension is brought to the office after becoming incoherent during lunch 30 minutes ago. Examination reveals a right facial droop and right extremity weakness. A noncontrast head CT is negative for acute hemorrhage and an electrocardiogram reveals normal sinus rhythm. Which of the following is the most appropriate next step in the management of this patient? A. rtPA bolus and IV infusion B. Heparin bolus and IV infusion C. MRI of the brain D. lumbar puncture

(c) A. This patient meets the qualifications for rtPA according to the NIH stroke scale. While Heparin is a good choice in the acute setting, the most effective treatment choice would be thrombolytics. A noncontrast CT scan already defined no bleed or obvious mass. Lumbar puncture is ideal in cases where the diagnosis is uncertain. The window of time is appropriate for acute treatment. (u) B. See A for explanation. (u) C. See A for explanation. (u) D. See A for explanation.

A 26 year-old female arrives in the emergency department with friends who say she was standing in front of her church, dressed in a white bathrobe, claiming to be the Virgin Mary and handing out $100 bills to all passers-by. Her friends noted that she had been depressed lately, but now seems completely euphoric. She had a similar episode two years ago. Which of the following is the most appropriate treatment? A. Inpatient olanzapine (Zyprexa) therapy B. Inpatient electroconvulsive therapy C. Outpatient paroxetine (Paxil) therapy D. Outpatient psychotherapy

(c) A. Treatment of the manic phase is usually done in the hospital to protect patients from behaviors associated with grandiosity (spending inordinate amounts of money, making embarrassing speeches, etc.). Lithium, valproate, and olanzapine are considered effective in the manic stage; the depressive stage is treated with antidepressants. (u) B. See A for explanation. (u) C. See A for explanation. (u) D. See A for explanation.

A 26 year-old sexually active woman has a 3-day history of dysuria, frequency, and urgency. She has a fever of 102 degrees F orally with shaking chills and right-sided costovertebral angle tenderness. Urinalysis reveals 10-20 RBCs/HPF, 30 WBCs in clumps/HPF, 3-4 WBC casts and 3+ bacteria. Which of the following is the treatment of choice for outpatient management? A. Ciprofloxacin B. Erythromycin C. Doxycycline D. Amoxicillin

(c) A. Treatment with a quinolone is first line treatment of pyelonephritis on an outpatient basis. If the patient were admitted then empiric treatment with Ampicillin and an aminoglycoside IV would be initiated until culture and sensitivity results were obtained. (u) B. See A for explanation. (u) C. See A for explanation. (u) D. See A for explanation.

Which of the following is the only disease that forms an ulcer at the site of inoculation? A. Tularemia B. Scabies C. Lyme disease D. Rocky Mountain Spotted Fever

(c) A. Tularemia is an acute infection that is transmitted by handling the flesh of infected animals, by the bites of insect vectors and by inoculation of conjunctiva. (u) B. Scabies is caused by the infestation of a mite and is associated with significant pruritus. There may be minimal cutaneous findings although a burrow and skin tracks may be seen early in the course of this disease. (u) C. Lyme disease is associated with a rash known as erythema chronicum migrans. The rash has an expanding border and central clearing. (u) D. Systemic symptoms predominate with Rocky Mountain Spotted Fever and the rash has an acral distribution pattern that may occur on the palms and soles. It does not have any ulcer associated with it.

Ulcerative colitis usually presents with which of the following? A. bloody diarrhea B. toxic megacolon C. fever and left quadrant pain D. alternating constipation and diarrhea

(c) A. Ulcerative colitis typically presents with episodic bloody diarrhea, lower abdominal cramps, and urgency to defecate. (u) B. Toxic megacolon is a complication of ulcerative colitis, but it is not a common presentation. (u) C. Left lower quadrant pain and a palpable mass accompanied by fever is classic for diverticulitis. (u) D. Irritable bowel syndrome usually presents with constipation, painless diarrhea with mucous, or alternating constipation and diarrhea.

A patient with a history of multiple trauma has received a combination of 12 units of packed red blood cells and whole blood while in the emergency department. Later that night it is noted that the patient is beginning to ooze blood from sites of injury and needle punctures. Which of the following is the most appropriate intervention? A. Continue transfusions of packed red blood cells B. Fresh frozen plasma C. Albumin D. Factor VIII concentrate

(h) A. Continued administration of PRBC without replacement of FFP will result in possible volume overload and continued bleeding. (c) B. When a patient receives more than one total blood volume (10 units) without replacement of clotting factors, the patient will continue to bleed without administration of fresh frozen plasma. (u) C. Albumin is used for protein and intravascular volume replacement. (h) D. Factor VIII concentrate is used in the treatment of hemophilia A and would not effect the bleeding in this patient.

Which of the following diagnostic studies would be most helpful in diagnosing a retropharyngeal abscess? A. CBC B. Neck CT scan C. Rapid strep screen D. Heterophile antibody

(u) A. A CBC would be an important test, but it is not definitive for peritonsillar abscess. (c) B. A neck CT scan would identify a peritonsillar abscess. (u) C. A rapid strep screen may have been performed prior to placement on antibiotics, but does not provide a definitive diagnosis for a peritonsillar abscess. (u) D. A heterophile antibody is not indicated in the diagnosis of a retropharyngeal abscess.

: (29) 26. History & Physical/Urology/Renal A 22 year-old male presents for a physical. On examination you palpate a painless mass that cannot be separated from the right testicle. Which of the following is the most likely diagnosis? A. Spermatocele B. Varicocele C. Testicular cancer D. Epididymitis

(u) A. A spermatocele is a painless, sperm-filled mass on the epididymis. (u) B. A varicocele is a painless collection of varicosed veins in the spermatic cord. (c) C. Testicular cancer is the most common neoplasm in men 20-35 years of age. It commonly presents as a painless mass which cannot be separated away from the testicle. Any nodule within the testes warrants evaluation for malignancy. (u) D. Epididymitis is an acute infection of the epididymis. The patient most commonly appears acutely ill with a fever and an exquisitely tender epipididymis.

A 22 year-old woman presents with sneezing, runny nose, postnasal drip, and nasal congestion for the last week. She says this happens every spring. She is not allergic to any medications. Which of the following is the most appropriate pharmacologic treatment for this patient? A. Azithromycin (Zithromax) B. Phenylephrine (Neo-synephrine) C. Nedocromil D. Pseudoephedrine

(u) A. Azithromycin is used to treat bacterial infections not allergic disorders. (u) B. Neo-synephrine is not indicated in the treatment of allergic rhinitis. (c) C. Nedocromil inhibits mast cell degranulation and is an effective treatment for allergic rhinitis. It may take 2-6 weeks for full therapeutic effect. (u) D. Pseudoephedrine is a decongestant that may relieve the nasal congestion, but has no effect on the allergic response.

: (27) 142. History & Physical/ENT/Ophthalmology The corneal light reflex is used to test for the presence of which of the following conditions? A. Dysfunction of the 5th cranial nerve B. Strabismus C. Amblyopia D. Afferent papillary defect

(u) A. Abnormal corneal reflex is present with dysfunction of the 5th cranial nerve. (c) B. The corneal light reflex is used to screen for strabismus. The test is performed by holding a penlight midline and directing the light directly toward both eyes. Normally the reflected light should appear in a symmetrical location on both corneas. Asymmetrical appearance is suggestive of strabismus. (u) C. Amblyopia, a potential complication of strabismus, is a decrease in visual acuity that results from failure of the retina(s) to receive a clear visual image. (u) D. Afferent papillary defect is identified by assessment of direct and consensual pupil response.

A 23 year-old male with recent upper respiratory symptoms presents complaining of chest pain. His pain is worse lying down and better sitting up and leaning forward. Electrocardiogram shows widespread ST segment elevation. Which of the following is the most likely physical examination finding in this patient? A. Elevated blood pressure B. Subungual hematoma C. Diastolic murmur D. Pericardial friction rub

(u) A. Acute pericarditis is usually not associated with elevated blood pressure. One would expect to see hypertensive pressures in the setting of an aortic dissection. (u) B. Subungual hematomas are usually seen in endocarditis not pericarditis. (u) C. A diastolic murmur in a patient with chest pain would likely be associated with acute aortic regurgitation in the setting of an aortic dissection. (c) D. This patient has symptoms consistent with acute pericarditis and would most likely have a pericardial friction rub on examination.

A 15 year-old male patient presents with oliguria, hematuria, proteinuria, and fatigue following streptococcal pharyngitis 2 weeks ago. Which of the following is the most likely diagnosis? A. Acute pyelonephritis B. Acute glomerulonephritis C. Systemic lupus erythematosus D. Initial onset of type 1 diabetes mellitus

(u) A. Acute pyelonephritis presents with fever, flank pain, urgency, and pyuria. (c) B. Acute glomerulonephritis is a complication that can follow a streptococcal infection after 1 to 3 weeks. (u) C. Systemic lupus erythematosus is not a complication of streptococcal infections, but it can cause rapid progressive glomerulonephritis. (u) D. Diabetes mellitus presents with polyuria, not oliguria or hematuria.

A two month-old infant appeared well until three weeks ago when he became dyspneic and had difficulty feeding. A 4/6 holosystolic murmur is heard at the left lower sternal border in the 3rd ICS. An electrocardiogram (ECG) shows left and right ventricular hypertrophy. Which of the following is the most likely diagnosis? A. Atrial septal defect B. Pulmonary hypertension C. Ventricular septal defect D. Tricuspid insufficiency

(u) A. An ASD usually presents with a soft mid-systolic murmur in the second left ICS with a widely split and fixed S2. (u) B. With pulmonary hypertension the murmur may be most consistent with pulmonary or tricuspid insufficiency. (c) C. This is a classic presentation for a ventricular septal defect. (u) D. Tricuspid insufficiency is a systolic murmur heard best at the 4th ICS that may radiate to the apex.

A 45 year-old female presents complaining of the worst headache of her life. Which of the following is the most appropriate initial diagnostic study? A. Magnetic resonance imaging (MRI) B. Computed tomography (CT) C. Electroencephalogram (EEG) D. Lumbar puncture (LP)

(u) A. An MRI is not as sensitive in detecting an acute cerebral bleed as a CT scan. (c) B. A CT scan is more sensitive in detecting cerebral hemorrhage in the first 24 to 48 hours. (u) C. Electroencephalogram (EEG) is used in the evaluation of seizures and will not assess the presence of cerebral hemorrhage. (u) D. A lumbar puncture is indicated in evaluation of suspected subarachnoid hemorrhage only if the CT scan does not establish the diagnosis.

A 36 year-old patient presents requesting something to help him sleep. He reports that he has always had a problem sleeping, admits to feeling nervous most days for the last 2 years, and that he has always been "uptight" and a "worry wart." During the previous eight months he has frequently felt tense, shaky, sweaty, with palpitations and frequent headaches. He reports being irritable with his 5 year-old son. Which of the following is the best treatment option for this patient? A. Alprazolam (Xanax) B. Haloperidol (Haldol) C. Paroxetine (Paxil) D. Diphenhydramine (Benadryl)

(u) A. Benzodiazapines should not be used in generalized anxiety disorder (GAD) as they only help symptoms short-term and should not be used long-term. (u) B. Antipsychotic medications are not useful in generalized anxiety disorder. (c) C. SSRI's, specifically Paxil, are the mainstay for treatment of generalized anxiety disorder. (u) D. Antihistamines are not useful in the treatment of generalized anxiety disorder.

A 53 year-old woman recently diagnosed with Parkinson's disease has a moderate tremor. She has no other signs of disease at this time. Which of the following is the most appropriate initial treatment? A. lorazepam (Ativan) B. haloperidol (Haldol) C. ramatidine (Flumadine) D. levodopa/carbidopa (Sinemet)

(u) A. Benzodiazepines are used to treat anxiety and insomnia, but are not used to treat Parkinson's disease. (u) B. Haloperidol is used for the treatment of Tourette's syndrome, but is not indicated for the treatment of Parkinson's disease. (u) C. Ramatidine is used in the treatment of viral infections and has no role in the treatment of Parkinson's disease. (c) D. Levodopa/carbidopa is first-line therapy for the treatment of Parkinson's disease.

According to the recent JNC VII guidelines, a 34 year-old male who has type 1 diabetes mellitus and hypertension should be started on which type of antihypertensive agent? A. beta-blocker B. loop diuretic C. ACE inhibitor D. thiazide diuretic

(u) A. Beta blockers could potentially be harmful in a patient with diabetes mellitus. Use a cardioselective beta-blocker to reduce the incidence of hypoglycemia. (u) B. See C for explanation. (c) C. ACE inhibitors are effective in young patients. They are capable of providing protection to the kidney especially in diabetes mellitus. (u) D. See C for explanation

Which of the following medication classes is considered first-line therapy for patients with Raynaud's phenomenon? A. Beta blockers B. Calcium channel blockers C. Central alpha agonists D. Oral nitrates

(u) A. Beta blockers do not cause vasodilation against this vasospasm which makes them less effective as treatment. (c) B. Calcium channel blockers are first line therapy for patients who have uncontrolled symptoms related to Raynaud's phenomenon. They are vasodilating agents which may play a role in preventing the vasospasm that occurs with this disorder. (u) C. See B for explanation. (u) D. Second line agents for Raynaud's phenomenon may include topical but not oral nitrates.

A 38 year-old female is brought to the emergency room with a complaint of paralysis of her arms bilaterally. Her history is significant for a sexual assault two days previously. Her physical examination is unremarkable with normal reflexes. There is no apparent physical explanation for her paralysis. What is the most likely diagnosis? A. Body dysmorphic disorder B. Conversion disorder C. Hypochondriasis D. Factitious disorder

(u) A. Body dysmorphic disorder patients have a preoccupation with an imagined defect in appearance that causes significant distress. (c) B. Patients have abrupt loss of motor or sensory function caused by psychological factors that often precede the physical symptoms (u) C. Hypochondriasis is characterized by six months or more of general, non delusional preoccupation with fears of having a serious disease. (u) D. Patients with factitious disorder fake illness.

A 42 year-old male smoker presents for further evaluation of a 4 cm solitary pulmonary nodule discovered on a recent chest x-ray. Which of the following diagnostic tests is indicated next? A. Bone scan B. Thoractotomy C. Mediastinoscopy D. CT scan of chest

(u) A. Bone scanning is indicated for evaluation of bone metastases, most commonly secondary to cancer of the breast or prostate. (u) B. Diagnostic thoractotomy is indicated for biopsy of the lesion should the CT scan of the chest indicate a suspicious malignant lesion that is inaccessible to thoracoscopy. (u) C. Mediastinoscopy can be utilized to further evaluate any enlarged mediastinal lymph nodes that may be found on the CT scan of the chest, but is not indicated prior to the CT scan. (c) D. A CT scan of the chest is needed to further evaluate the characteristics of the solitary pulmonary nodule and to determine lymph node involvement or presence of multiple lesions.

Which of the following preventive strategies against osteoporosis-associated vertebral fractures has a known side effect of increasing the incidence of hot flashes when used in a perimenopausal female? A. Calcitonin (Miacalcin) nasal spray B. Alendronate (Fosamax) C. Estrogen/progesterone (Prempro)replacement D. Raloxifene (Evista)

(u) A. Calcitonin does not have any estrogen effects on the body and serves as an analgesic when used in the management of vertebral fractures. (u) B. Alendronate is a bisphosphonate that does not have any hormonal effects on the body. (u) C. Estrogen/progesterone replacement has the benefit of maintaining bone and decreasing vertebral fractures but it would improve perimenopausal hot flashes. (c) D. Raloxifene has effects on bone turnover and bone mass and has been shown to decease vertebral fractures. It has anti-estrogen effects on the non-skeletal portions of the body and increases hot flashes in perimenopausal females.

A 56 year-old diabetic male with a three-month history of epigastric pain after eating says he gets full really fast now. His test for Helicobacter pylori is negative. Which of the following is the recommended medication to treat this patient? A. Calcium carbonate (Tums) B. Ranitidine (Zantac) C. Omeprazole (Prilosec) D. Metoclopramide (Reglan)

(u) A. Calcium carbonate has no effect on gastric emptying. (u) B. H2 blockers have no effect on gastric emptying. (u) C. Prilosec has no effect on gastric emptying. (c) D. Metoclopramide accelerates gastric emptying and is the treatment of choice for this patient.

A mother of a newborn infant presents to the office concerned about reducing the risk of sudden infant death syndrome (SIDS). The infant was delivered at 39 weeks gestation weighing 7 pounds 9 ounces. There is no family history of SIDS and this is her first child. Which of the following is appropriate advice to reduce the risk of SIDS? A. Bottle feeding with soy formula B. Offer a pacifier at nap and bedtimes C. Have the infant sleep in the prone position D. Infant should sleep with the parents to allow close observation

(u) A. Bottle feeding with soy formula does not reduce the risk of SIDS. (c) B. Use of a pacifier during sleeping is a current recommendation to decrease the risk of SIDS. (u) C. Sleeping prone has been consistently shown to increase, not decrease, the risk of SIDS. (u) D. Bed sharing with parents has been shown to increase, not decrease, the risk of SIDS. This risk is increased in infants less than 4 months old and when older children are also present in the bed.

Which of the following findings is consistent with thyrotoxicosis? A. bradycardia B. menorrhagia C. nervousness D. constipation

(u) A. Bradycardia, dry skin, constipation, and menorrhagia are typically associated with hypothyroidism. (u) B. See A for explanation. (c) C. Thyrotoxicosis presents with heat intolerance, sweating, nervousness, diarrhea, tachycardia, moist skin, and tremor. (u) D. See A for explanation.

A 74 year-old patient presents with signs and symptoms of heart failure. EKG shows the patient to be in atrial fibrillation at a rate of 80 bpm. Blood pressure is 120/76. The patient denies complaint of palpitations, chest pain, or syncope. Which of the following is the most important long term therapy in this patient? A. verapamil (Calan) B. amiodarone (Cordarone) C. furosemide (Lasix) D. warfarin (Coumadin)

(u) A. Calcium channel blockers are utilized in rate control of atrial fibrillation. This patient's rate is controlled at 80bpm presently. (u) B. Antiarrhythmic therapy may be considered in patients with atrial fibrillation; however anticoagulation therapy must occur first. (u) C. Diuretics may be indicated in the acute treatment of heart failure; however they may not be needed long term. (c) D. Patients with atrial fibrillation have an increased risk for stroke, therefore these patients need anticoagulation with warfarin to an INR of 2.0-3.0.

A 52 year-old male presents with a severely painful, swollen right great toe. He denies recent trauma, but reports several similar episodes of toe pain and swelling over the past two to three years. He has a history of alcohol abuse and hypertension, for which he "takes medication" of unknown type. Examination reveals bright erythema and edema associated with the right first MCP joint. Which of the patient's antihypertensive medications may be contributing to this condition? A. Diltiazem (Cardizem) B. Hydrochlorothiazide (HCTZ) C. Lisinopril (Prinivil) D. Metoprolol (Lopressor)

(u) A. Calcium-channel blockers, such as diltiazem, are not associated with increased risk of hyperuricemia and gout. (c) B. Thiazide diuretics, such as hydrochlorothiazide, are associated with increased risk of hyperuricemia and gout. (u) C. ACE inhibitors, such as lisinopril, are not associated with increased risk of hyperuricemia and gout. (u) D. Beta-blockers, such as metoprolol, are not associated with increased risk of hyperuricemia and gout.

Which of the following is the most effective way for patients with persistent asthma to monitor the severity of their symptoms? A. call the health care provider regularly B. keep a diary of symptoms C. monitor peak flow D. ask a family member to monitor symptoms

(u) A. Calling the health care provider regularly or asking a family member to monitor symptoms is not effective in patients understanding how to manage their asthma symptoms. (u) B. Keeping a diary may be effective for patients to understand their cause of symptoms, but would not be effective in helping to manage their symptoms. (c) C. Monitoring peak flow is the most effective way for the patients and health care providers to manage symptoms and guide treatment. (u) D. See A for explanation.

183. History & Physical/Cardiology Which of the following physical examination findings is consistent with chronic aortic regurgitation? A. Cannon wave visualized on examination of jugular venous pressure B. Decreased blood pressure in the lower extremity compared to upper C. Dependent edema D. Wide pulse pressure

(u) A. Cannon jugular venous pressure is associated with tricuspid stenosis. (u) B. Because the peripheral bed is much larger in the leg than the arm, the BP in the leg may be over 40 mmHg higher than the arm. (u) C. Dependent edema would be associated with right-sided heart failure, aortic regurgitation is associated with leftsided heart failure. (c) D. Major physical examination findings in chronic aortic regurgitation relate to the high stroke volume being ejected into the systemic vascular system with rapid runoff as the regurgitation takes place. This results in a wide pulse pressure.

A person presenting with bleeding esophageal varicies should be treated with which of the following while awaiting arrival of endoscopy? A. Carafate (Sucralfate) B. Octreotide (Sandostatin) C. Omeprazole (Prilosec) D. Enoxaparin (Lovenox)

(u) A. Carafate is not indicated in acute gastrointestinal bleeding. (c) B. Octreotide is a vasoacctive drug used in the treatment of GI bleeding as well as somatostatin, vasopressin, and terlipressin. Somatostatin and octreotide are preferred due to safety and less incidence of serious side effects. (u) C. Omeprazole, a proton pump inhibitor, is not indicated in acute gastrointestinal hemorrhage. (h) D. Enoxaparin will increase bleeding and therefore contraindicated in GI bleeding.

A 28 year-old man comes to the office for his yearly follow up of testicular cancer. He was successfully treated with an orchiectomy two years ago. Which of the following tumor markers can be used to identify early relapse in testicular cancer? A. Carcinoembryonic antigen (CEA) B. Prostate specific antigen (PSA) C. CA 125 glycoprotein D. Alpha fetoprotein (AFP)

(u) A. Carcinoembryonic antigen can be elevated in multiple conditions including colon cancer, gastritis, peptic ulcer disease, diverticulitis, liver disease, chronic obstructive pulmonary disease, and diabetes. (u) B. Prostate specific antigen is a sensitive marker for prostate cancer. (u) C. CA 125 glycoprotein is a marker in epithelial ovarian cancer and can be used for post-treatment surveillance. (c) D. Periodic surveillance of alpha fetoprotein is a sensitive method for detecting early relapse of testicular cancer.

A 23 year-old female with history of palpitations presents for evaluation. She admits to acute onset of rapid heart beating lasting seconds to minutes with associated shortness of breath and chest pain. The patient states she can relieve her symptoms with valsalva. Which of the following is the most appropriate diagnostic study to establish a definitive diagnosis in this patient? A. Cardiac catheterization B. Cardiac MRI C. Chest CT scan D. Electrophysiology study

(u) A. Cardiac catheterization evaluates coronary arteries but has no role in the diagnosis of supraventricular tachycardia. (u) B. Cardiac MRI cannot diagnose and define pathway of supraventricular tachycardia. (u) C. Chest CT scan will not establish definitive diagnosis of supraventricular tachycardia. (c) D. Electrophysiology study is useful in establishing the diagnosis and pathway of complex arrhythmias such as supraventricular tachycardia.

An 80 year-old female presents with syncope and recent fatigue and lightheadedness over the past month. She denies chest pain or dyspnea. Physical examination reveals BP 130/70 mmHg, HR 40 bpm, regular, and RR 16. Electrocardiogram reveals two p waves before each QRS complex. Which of the following is the treatment of choice for this patient? A. Cardio defibrillator insertion B. Atropine as needed C. Permanent dual chamber pacemaker insertion D. Ritalin therapy daily

(u) A. Cardio defibrillators treat ventricular tachycardia and are not indicated in the management of second degree AV block. (u) B. Atropine can be used in the acute management of second degree AV block Mobitz type II, but it should not be used as long-term therapy. (c) C. This patient has findings consistent with symptomatic second degree AV block Mobitz type II for which permanent pacing is the treatment of choice. (u) D. Ritalin therapy is not indicated in the management of second degree heart block.

A 40 year-old female G5P5 complains of small quantities of urine leaking when she coughs, sneezes, or laughs. Her genitourinary examination is unremarkable and her urinalysis is normal. At this time, which of the following is the most appropriate management plan? A. Refer for a cystoscopy. B. Recommend Kegel exercises. C. Refer for surgical correction. D. Recommend hormone replacement therapy.

(u) A. Conservative therapy for stress incontinence should be attempted prior to any evaluation, such as cystoscopy, that might indicate the need for surgical correction. (c) B. Strengthening the pelvic muscles by Kegel exercises and emptying the bladder frequently may resolve the problem. (u) C. See A for explanation. (u) D. There is no indication in the history for hormone replacement therapy and no vaginal atrophy was noted on pelvic examination.

A 27 year-old woman presents with one week of worsening productive cough, dyspnea, fever and malaise. Which of the following physical examination findings would support the diagnosis of lobar pneumonia? A. Decreased tactile fremitus B. Egophony C. Hyperresonance to percussion D. Wheeze

(u) A. Consolidation would increase the transmission of vocal vibrations and manifest as increased tactile fremitus. (c) B. Transmission of vocal sounds through consolidation leads to the changes heard with egophony. (u) C. Dullness, not hyperresonance, would be expected with consolidation. (u) D. Wheezing is associated with narrowing of the small airways as seen in asthma.

An unimmunized child has had a mild fever for several days. Today, the child is brought to the clinic because of the development of a rash. The rash is a pink discrete macular eruption mostly on the face and trunk. Postauricular and suboccipital lymph nodes are palpable. The child has a slight fever, but does not appear sick. Which of the following is the most likely diagnosis? A. Roseola B. Rubeola C. Rubella D. Erythema infectiosum

(u) A. Continued fever after rash onset and lymphadenopathy are not consistent with roseola. (u) B. High fever and lethargy are prominent features of rubeola and help distinguish it from rubella. (c) C. The characteristic lymphadenopathy and lack of systemic symptoms are most consistent with rubella. (u) D. The presence of lymphadenopathy is not consistent with erythema infectiosum, which typically presents with reddened cheeks.

Which of the following medications used in the treatment of supraventricular tachycardia is able to cause sinus arrest and asystole for a few seconds while it breaks the paroxysmal supraventricular tachycardia? A. Digoxin (Lanoxin) B. Adenosine (Adenocard) C. Verapamil (Calan) D. Quinidine (Quinaglute)

(u) A. Digoxin is not used for the acute termination of supraventricular tachycardia. (c) B. Adenosine is an endogenous nucleoside that results in profound (although transient) slowing of the AV conduction and sinus node discharge rate. This agent has a very short half-life of 6 seconds. (u) C. Although verapamil may be used for the termination of acute supraventricular tachycardia, it does not lead to sinus arrest in therapeutic doses. (u) D. Quinidine is rarely used today and is not indicated for the termination of supraventricular tachycardia.

Which of the following is the most likely initial effect on the left ventricle from aortic stenosis? A. Dilitation of the ventricle with diastolic dysfunction B. Wall stiffness due to ischemia from decreased coronary blood flow C. Paradoxical wall motion abnormalities due to increased preload D. Concentric hypertrophy with preserved function

(u) A. Dilitation of the ventricle is a later finding. (u) B. This is not an effect from aortic stenosis but coronary artery blockage. (u) C. Preload is the end-diastolic pressure or volume within the ventricle, ischemic heart disease or infarct would cause paradoxical wall motion abnormalities due to the increased preload. (c) D. Hypertrophy would be the initial changes of the left ventricle as a response to the increased pressure.

A 62 year-old patient with arthritis is on daily naproxen therapy. Which of following medications is used to protect against GI complications? A. diphenoxylate (Lomotil) B. sulfasalazine (Azulfidine) C. metoclopramide (Reglan) D. misoprostol (Cytotec)

(u) A. Diphenoxylate is used in the treatment of diarrhea. (u) B. Sulfasalazine is used in the treatment of inflammatory bowel disease. 31 (u) C. Metaclopramide is used in the treatment of nausea and vomiting. (c) D. Misoprostol is an analog of prostaglandins and used in the treatment of NSAID induced ulcer disease

A patient with history of hypertension and dyslipidemia presents for routine follow up. On funduscopic examination you note moderate sized fluffy white lesions with irregular borders. This is most consistent with which of the following? A. Drusen B. Cotton-wool patches C. Hard exudates D. Preretinal hemorrhages

(u) A. Drusen are tiny to small yellowish round spots with hard or soft edges that are often seen in age-related macular degeneration. (c) B. Cotton-wool patches are fluffy white or grayish ovoid lesions with irregular borders. They are typically moderate in size and seen in patients with hypertension. (u) C. Hard exudates are yellowish bright lesions with well-defined borders. They are often small and round. (u) D. Preretinal hemorrhages obscure the underlying retinal vessels and are seen as a horizontal line of demarcation with plasma above and cells below.

A 45 year-old male presents for a routine physical. His mother and father both had ischemic strokes in their 70's. He does not smoke. His blood pressure is 128/80 mmHg, pulse 78/minutes and regular, respiratory rate of 12/minute. He has no bruits on examination. Which of the following studies should be ordered to further evaluate this patient's risk of stroke? A. Electrocardiogram B. Fasting lipid profile C. Carotid Doppler ultrasound D. MRI with gadolinium

(u) A. ECG does not show risk of stroke. (c) B. This patient has a genetic risk for stroke secondary to family history. Having an elevated LDL cholesterol and low HDL cholesterol puts him at a greater risk for stroke. (u) C. Carotid Doppler US will demonstrate the presence of a plaque in the carotid arteries, but still does not evaluate stroke risk. (u) D. MRI with contrast will only show abnormalities in structure which cannot predict risk of stroke. Often abnormal findings are picked up on MRI that have no relationship to a patient's symptoms.

Which of the following studies is the best initial diagnostic evaluation to estimate ventricular size and hypertrophy? A. Electrocardiogram (ECG) B. Cardiac CT scan C. Echocardiogram D. Myocardial perfusion imaging

(u) A. ECG is not sensitive or reliable to estimate ventricular size and hypertrophy. (u) B. Cardiac CT scan can detect coronary calcification, but is most sensitive to assess disorders of the aorta. (c) C. Echocardiogram provides the safest and most reliable means to evaluate ventricular size, hypertrophy and function. (u) D. Myocardial perfusion imaging is used for measurement of LV ejection fraction and assess regional wall motion abnormalities.

A 30 year-old diabetic female complains of persistent numbness in her right thumb and forefinger that has been awakening her from sleep for the past week. She is right hand dominant and denies any history of activities involving repetitive motion of the hands. Which of the following is the next step in the evaluation of this patient? A. Electromyogram (EMG) B. Nerve conduction velocity (NCT) C. Phalen maneuver D. Finkelstein test

(u) A. EMG may be performed in a patient who is being considered for surgery. (u) B. Nerve conduction test may be performed if operative intervention is being planned. (c) C. The Phalen maneuver is used to reproduce the symptoms of carpal tunnel syndrome by flexion of the wrist. (u) D. The Finkelstein test is performed when deQuervain's tenosynovitis is suspected.

Which of the following would demonstrate rotational misalignment in a patient with a fracture of the fourth metacarpal? A. base of the ring fingernail and index fingernail line up in the partially closed hand B. fingernails of the open hand form an asymmetric arc C. ring finger of the closed hand overlaps the little finger D. ring finger of the open hand is shortened

(u) A. Failure of the planes of the fingernails in the partially closed hand to line up indicates rotational misalignment. (u) B. Fingernails of the open hand normally form an asymmetric arc. (c) C. All fingernails should point to the same spot when the hand is closed. Overlapping of one finger over the other indicates rotational misalignment. (u) D. Shortening of a finger does not indicate rotational misalignment.

What is the recommended method for screening pregnant women for gestational diabetes? A. Fasting blood sugar and 2 hour post prandial B. 50 gram glucose load followed by a blood sugar in 1 hour C. 75 gram glucose load followed by a blood sugar in 2 hours D. 100 gram glucose load followed by a blood sugar at 1 hour, 2 hours, and 3 hours

(u) A. Fasting blood sugar and 2 hour postprandial blood test is used to follow patient with gestational diabetes. (c) B. One hour Glucola is the screening test for gestational diabetes. It is a 50 gram glucose load, with a serum glucose obtained 1 hour after the dose. Normal value is less than 140 mg/dL. (u) C. A 75 gram glucose load is used in non-pregnant patients. (u) D. This describes a three-hour GTT, which is ordered if the 1 hour Glucola is elevated above 140 gm/dL.

A 37 year-old male presents with daytime fatigue and drowsiness. He states that he does not sleep well, with frequent awakenings during the night. He has gained 8 pounds over the past six months, and he complains of palpitations. His wife states that he snores at night. A home nocturnal pulse oximetry indicates that his saturation drops 6% intermittently throughout the night. Which of the following is the most likely diagnosis? A. depression B. narcolepsy C. hypothyroidism D. obstructive sleep apnea

Explanations (u) A. Hypothyroidism, narcolepsy, and depression are not associated with oxyhemoglobin desaturation or snoring. (u) B. See A for Explanation. (u) C. See A for Explanation. (c) D. Obstructive sleep apnea is associated with obesity, nighttime wakening and snoring, hypertension, cardiac dysrhythmias, and oxyhemoglobin desaturation of greater than 4% during hypopnea or apnea.

A 23 year-old female is in active labor and has progressed from 3 cm to 6 cm in the last six hours. Fetal monitoring demonstrates mild repetitive late decelerations. Which of the following is the most likely cause of this finding? A. Fetal hypoxia B. Head compression C. Cord compression D. Uteroplacental insufficiency

(u) A. Fetal hypoxia would be a concern if deep late FHR decelerations were present with absent beat-to-beat variability. (u) B. Early decelerations are due to head compression of the fetus. Pressure on the fetal head causes an alteration in cerebral blood flow causing a central vagal stimulation and subsequent FHR deceleration. The deceleration is a mirror image of the contraction. (u) C. Variable decelerations are from cord compression. The decelerations have a sharp, angular, decline in FHR with duration less than 2 minutes. (c) D. Late decelerations are from uteroplacental insufficiency. The decelerations have a smooth, gradual symmetrical decrease in FHR beginning at or after the peak of the contraction.

Bony and cartilaginous enlargement of distal interphalangeal joints is commonly seen in which of the following medical conditions? A. Rheumatoid arthritis B. Osteoarthritis C. Psoriatic arthritis D. Gouty arthritis

(u) A. Findings in Rheumatoid arthritis include skin nodules, pannus formations and symmetric joint involvement sparing the distal interphalangeal joints. (c) B. Heberden's nodes are commonly seen in primary osteoarthritis. (u) C. Psoriatic arthritis manifests commonly as sacroiliitis. Asymmetrical arthritis occurs in psoriatic arthritis and causes a sausage like appearance to the fingers. It can also mimic the presentation of rheumatoid arthritis. (u) D. Gout commonly affects the great toe, midfoot, ankle, and knee. Tophi may be present which are subcutaneal deposits of monosodium urate crystals.

34 year-old female status-post trans-sphenoidal resection of pituitary adenoma presents with worsening polydipsia of 10-12 liters daily and polyuria within four days of discharge. A urinalysis reveals a specific gravity of 1.004 (1.001-1.035) and shows decreased urine osmolality but is otherwise normal. Labs reveal mild hypernatremia. What is the treatment of choice for this patient? A. Glyburide B. Methylprednisolone C. Desmopressin D. Quinapril

(u) A. Glyburide is not indicated because this patient has diabetes insipidus, not diabetes mellitus. (h) B. Use of steroids in diabetes insipidus will actually worsen renal free water secretion. (c) C. The main treatment for diabetes insipidus is Desmopressin. (u) D. Quinapril is not indicated due to the lack of diabetes mellitus and overt renal failure/insufficiency.

A 32 year-old male presents with an acute onset of pain and swelling to his left ankle. On physical examination the ankle is warm, swollen and erythematous. Evaluation of the synovial fluid reveals only leukocytosis with a low glucose. Which of the following is the most likely diagnosis? A. Gout B. Pseudogout C. Acute rheumatic fever D. Septic arthritis

(u) A. Gout and pseudogout are excluded by the failure to find crystals on synovial fluid analysis. (u) B. See A for explanation. (u) C. Acute rheumatic fever commonly involves multiple joints. (c) D. Leukocytosis and a low synovial glucose are indicative of septic arthritis.

A 33 year-old HIV-positive woman develops an 8mm area of induration following the administration of a purified protein derivative (PPD) test. Her chest radiograph shows no evidence of active tuberculosis (TB) infection. Which of the following is the most appropriate clinical intervention? A. Four-drug regimen for 4 months B. Isoniazid with Rifampin C. Observation only D. Repeat PPD and chest radiograph in 3 months

(u) A. Greater than 5 mm of induration is positive in an HIV-infected patient. A positive PPD and negative chest film is considered latent TB infection and, while requiring treatment, does not require the full four-drug regimen. (c) B. Isoniazid with Rifampin is recommended in HIV positive patients with a positive PPD and a negative chest xray. (u) C. Latent TB infection is associated with a risk of progression to tuberculosis and observation alone is inadequate. (u) D. Repeat screening is not helpful since the diagnosis of latent TB infection has already been established.

A 32 year-old female presents with a seven month history of recurrent, brief episodes of weakness and tingling in the extremities, diplopia, and vertigo. Which of the following is the most likely diagnosis? A. Guillain-Barre syndrome B. Myasthenia gravis C. Multiple sclerosis D. Amyotrophic lateral sclerosis

(u) A. Guillain-Barre syndrome typically presents with progressive weakness that starts peripherally and proceeds centrally over a brief period of time. (u) B. Myasthenia gravis presents with weakness and fatigue in the upper limbs, cranial, and/or trunk musculature. Blurry vision and diplopia are common visual complaints and dysphagia is common. (c) C. Multiple sclerosis is most frequently seen in patients in their twenties and presents with episodes of weakness, paresthesias, and diplopia. (u) D. Amyotrophic lateral sclerosis presents with progressive weakness, fasciculations, and loss of muscle mass.

: (9) 27 57. Scientific Concepts/Infectious Diseases Which of the following clinical pictures best defines acquired immune deficiency syndrome (AIDS)? A. HIV +, CD 4 lymphocyte count of 250 cells/mcL B. Non-Hodgkin lymphoma with or without evidence of HIV infection C. HIV +, community acquired pneumonia (CAP) D. Mycobacterium avium complex with or without evidence of HIV infection

(u) A. HIV infection with a CD 4 lymphocyte count under 200 cells/mcL is diagnostic of AIDS. (u) B. Non-Hodgkin lymphoma with evidence of HIV infection is a definitive AIDS diagnosis. Non-Hodgkin lymphoma can be present in patients who do not have HIV infection. (u) C. Documented HIV infection with Pneumocystis jiroveci pneumonia is diagnostic of AIDS, not CAP. (c) D. Mycobacterium avium complex with or without evidence of HIV infection is considered a definitive AIDS diagnosis.

A 32 year-old female G4P4 with a history of multiple sexual partners is seen in follow-up for recent findings of high grade squamous intraepithelial lesions (HSIL) on a Pap smear. Which of the following is the most appropriate next step? A. HPV DNA testing B. pelvic ultrasound C. repeat Pap smear in 4-6 months D. colposcopy with endocervical curettage

(u) A. HPV DNA testing is recommended for initial evaluation of an ASC-US result on a Pap smear. If HPV testing is positive, colposcopy is then indicated. If negative, the Pap smear is repeated in 12 months. (u) B. A pelvic ultrasound or abdominopelvic CT scan is indicated when repeat abnormalities are seen on a Pap smear, however all diagnostic studies have failed to reveal the source of cellular abnormality. (u) C. Repeat Pap smears in 4-6 months may be done in evaluation of ASC-US Pap smear results after treatment with either appropriate therapy for vaginal infection or intravaginal estrogen therapy for menopausal patients with atrophic changes. (c) D. Colposcopy with endocervical curettage and directed biopsy of suspicious lesions is the appropriate evaluation of a high grade squamous intraepithelial lesions (HSIL) Pap smear result.

A 40 year-old female presents with a Pap smear abnormality revealing atypical glandular cells (AGUS). What is the most appropriate intervention? A. HPV DNA testing B. Colposcopy with endometrial curretage(ECC) C. Repeat Pap smear in 3 months D. Colposcopy and endometrial sampling

(u) A. HPV DNA testing is recommended to further evaluate patients with Pap smears with dysplasia. (u) B. Colposcopy with ECC would be recommended in patients with ASCUS, LGSIL, HGSIL, or squamous cell findings on a Pap smear. (u) C. Repeat Pap smear would be recommended in patients with ASCUS, not AGUS, results on a Pap smear. (c) D. Colposcopy and endometrial sampling are important to perform in patients with AGUS Pap results because glandular cells are associated with squamous and glandular precursor lesions and carcinoma.

Which of the following is the most common cause of conductive hearing loss in an adult patient? A. Head trauma B. Cerumen impaction C. Otosclerosis D. Diabetes mellitus

(u) A. Head trauma and diabetes mellitus are causes of sensorineural hearing loss. (c) B. Cerumen impaction is the most common cause of conductive hearing loss in an adult patient. (u) C. Although a cause of conductive hearing loss, otosclerosis is not the most common cause. (u) D. See A for explanation.

: (4) 79 201. Clinical Intervention/Neurology A 73 year-old female patient was diagnosed 3 years ago with Alzheimer's disease and heart failure. Her 80 year-old husband can no longer help feed and bathe her or manage her medications. Which of the following support services is most appropriate for this patient? A. Hospice care B. Senior center C. Adult day care D. Skilled nursing facility

(u) A. Hospice care is a program to provide palliative care to individuals who are terminally ill and projected to die within 6 months. (u) B. Senior centers are community-based facilities that provide recreational activities and mid-day meals for functional adults. (u) C. Adult day health care is designed to meet the needs of functionally impaired adults in a community setting, but does not usually provide for the higher level of care that is required to feed or bathe a patient. (c) D. A skilled nursing facility is appropriate for patients requiring assistance with activities of daily living (i.e., feeding and bathing) and a higher level of safety assurance.

Which of the following increases the risk of developing testicular cancer? A. Low socioeconomic status B. History of cryptorchidism C. Multiple episodes of epididymitis D. Being of African-American ethnicity

(u) A. High socioeconomic status, not low, is a risk factor. (c) B. The major predisposing risk factor is cryptorchidism unrepaired until after age two. (u) C. Multiple episodes of epididymitis are unrelated to the development of testicular cancer. (u) D. The incidence of testicular cancer is much lower in African-American men than in Caucasian men.

Which of the following is best described as a hard nodule in the periumbilical region? A. Hodgkin's lymphoma B. Gastric bezoar C. Sister Mary Joseph nodule D. Virchow's node

(u) A. Hodgkin's lymphoma will more likely affect intra-abdominal lymph nodes. (u) B. Gastric bezoar can present as a palpable abdominal mass, but is not indicative of intra-abdominal/intra-pelvic cancer. (c) C. The Sister Mary Joseph nodule can occur with metastatic gastric and pancreatic cancers. The nodule is generally minimally to non-tender to palpation. (u) D. Virchow's node is found in cases of metastatic disease, but is represented by a left supraclavicular lymph node.

disregard

disregard

: (8) 6. Clinical Therapeutics/Orthopedics/Rheumatology Which of the following is indicated as the first-line treatment of an initial case of acute gouty arthritis? A. Indomethacin (Indocin) B. Colchicine (Probenicid) C. Methylprednisolone (Medrol) D. Allopurinol (Zyloprim)

(c) A. A nonsteroidal antiinflammatory agent is indicated as the first-line treatment for an initial case of acute gout. (u) B. Colchicine is a useful adjunct to NSAIDs, corticosteroids, or synthetic ACTH for the treatment of severe gout, refractory acute gouty attacks, or as prevention. (u) C. An intra-articular injection of glucocorticoid or a tapered regimen of oral prednisone can be used as an alternative treatment to the first-line treatment of an NSAID. (u) D. Allopurinol is a xanthine oxidase inhibitor used for patients with more than three occurrences per year, gout that is difficult to manage safely (polyarticular gout in a transplant patient or one with renal insufficiency), tophaceous gout, and hyperuricemia. Allopurinol can trigger an episode of gout, especially in the first few months of treatment.

You are examining a patient with right-sided extremity weakness and left-sided weakness of the face. Where is the lesion? A. Brainstem B. Cerebral hemisphere C. Cerebellum D. Basal ganglia

(c) A. A patient with a unilateral sensory or weakness finding on one side of the body and contralateral finding of weakness or sensory loss of the face has a brainstem lesion. (u) B. Patients with cerebral hemisphere lesions may present with motor, sensory, visual or auditory findings depending on the lesion. (u) C. Patients with cerebellum lesions present with ataxia, intention tremor, and dysmetria. (u) D. Patients with basal ganglia lesions present with bradykinesia, akinesia, and loss of postural reflexes.

What examination finding would be expected in a patient with von Willebrand disease? A. Gingival bleeding B. Splenomegaly C. Muscle weakness D. Hemarthrosis

(c) A. A patient with von Willebrand disease most commonly presents with mucosal bleeding seen in epistaxis, gingival bleeding, and menorrhagia. (u) B. Splenomegaly is seen with hemolytic anemia, not with von Willebrand disease. (u) C. Muscle weakness is not seen with von Willebrand disease. (u) D. Hemarthrosis is found in hemophilia, not in von Willebrand disease.

Which of the following is the most appropriate study for diagnosing Hirschsprung disease? A. Rectal biopsy B. Stool leukocyte test C. CT of the abdomen and pelvis D. Fecal occult blood test

(c) A. A rectal biopsy showing the absence of ganglion cells in both the submucosal and muscular layers of the involved bowel is the most appropriate diagnostic study for Hirschsprung disease. (u) B. Stool leukocyte testing can indicate an infectious etiology of diarrhea and is not indicated in the diagnosis of Hirschsprung disease. (u) C. Radiographic examination may show dilated proximal colon and absence of gas in the pelvic colon, but is not diagnostic for Hirschsprung disease. (u) D. Fecal occult blood testing is not indicated in the diagnosis of Hirschsprung disease.

: (28) 140. Diagnostic Studies/Orthopedics/Rheumatology A radiograph reveals a fracture of the proximal third of the ulnar diaphysis with an anterior dislocation of the radial head. Which of the following fractures does this describe? A. Monteggia fracture B. Nightstick fracture C. Chauffeur's fracture D. Smith's fracture

(c) A. A type I Monteggia fracture is a fracture of the proximal third of the ulnar diaphysis with anterior angulation or anterior dislocation of the radial head. (u) B. A nightstick fracture is an isolated fracture of the ulna with nondisplacement or minimal displacement, most commonly from a direct blow to the ulna. (u) C. A chauffeur's fracture is a radial styloid fracture. (u) D. A Smith's fracture is a dorsally angulated fracture of the distal radius.

What is considered the most common physical examination finding for intestinal obstruction? A. Distention B. Fluid wave C. Rigidity D. Tenderness

(c) A. Abdominal distention is the most common hallmark of all kinds of intestinal obstructions though its presence can be variable depending on the duration and exact location of the obstruction. (u) B. Fluid wave is noted with ascites, not intestinal obstruction. (u) C. Rigidity is most often minimal to absent except in the most extreme of late presenting strangulated obstructions and is more commonly found early in the course of peritonitis and related phenomenon. (u) D. Tenderness, like rigidity, is not a predominant finding in obstructed bowel until very late in the course of advancing, untreated cases. At this time distention and symptoms of shock will also be present and command the clinical picture.

Abduction of the shoulder against resistance helps localize pain in which of the following muscles of the shoulder girdle? A. Supraspinatus B. Infraspinatus C. Teres minor D. Subscapularis

(c) A. Abduction against resistance tests the supraspinatus. (u) B. Lateral rotation against resistance tests the infraspinatus and teres minor. (u) C. See B for explanation. (u) D. Medial rotation against resistance tests the subscapularis.

A 47 year-old female presents to the clinic with complaints of prolonged, heavy menses that have been getting progressively worse for 3 years. She denies any pain. On physical examination, enlargement of the uterus with multiple smooth, spherical, firm masses is noted. A CBC is consistent with a mild anemia. Which of the following is the most likely diagnosis? A. Leiomyoma B. Adenomyosis C. Endometriosis D. Endometrial polyps

(c) A. Abnormal uterine bleeding and irregular enlargement of the uterus are most consistent with leiomyoma. Pain is rarely present unless vascular compromise occurs. (u) B. While adenomyosis may present with hypermenorrhea, dysmenorrhea is often also present. Physical examination would reveal the presence of diffuse globular uterine enlargement, not the irregular enlargement as noted in the case presented. (u) C. Endometriosis presents with dyspareunia, dysmenorrhea, and infertility. If the pelvic exam were abnormal, uterine findings would include tender nodules in the cul de sac, not the uterus. (u) D. While endometrial polyps are compatible with the history of abnormal uterine bleeding, the uterus would be normal size without the irregular enlargement noted in the case presented.

In which of the following patients would one most likely find acanthosis nigricans? A. A 55 year-old obese female with hyperinsulinemia B. A 55 year-old male with an enlarged spleen and pancytopenia C. A 24 year-old female with increased Lyme titers D. A 60 year-old male with increased triglycerides

(c) A. Acanthosis nigricans is associated with patients who have hyperinsulinemia. (u) B. Hairy cell leukemia is associated with an enlarged spleen and pancytopenia. Acanthosis nigricans is not a cutaneous manifestation. (u) C. Erythema chronicum migrans is the characteristic lesion associated with Lyme disease. (u) D. Eruptive xanthelasma is associated with increased triglycerides

A 56 year-old female presents complaining of intense left eye pain associated with unilateral headache, nausea, and colored rings around lights. On examination you note decreased visual acuity, a pupil that is fixed and mid-dilated, and ciliary flushing. Which of the following is the most likely diagnosis? A. Acute glaucoma B. Migraine C. Episcleritis D. Acute uveitis

(c) A. Acute glaucoma is an ocular emergency that presents as an acutely painful eye and elevated intraocular pressure. Patients typically complain of acute eye pain associated with unilateral headache, nausea/vomiting, cloudy vision, and colored rings around lights. On exam the pupil is fixed and mid dilated with prominent ciliary flush. (u) B. Migraine headaches have associated unilateral headache and nausea however there would be no pupillary changes. (u) C. Episcleritis is an inflammation of the thin layer of connective tissue between the conjunctiva and sclera. Episcleritis resembles conjunctivitis but is a more localized process and discharge is absent. (u) D. Acute uveitis is frequently due to systemic disorders associated HLA-B27-related conditions ankylosing spondylitis, reactive arthritis, psoriasis, ulcerative colitis, and Crohn's disease. The pupil is usually small, inflammatory cells and flare within the aqueous are present.

: (27) 154. Diagnosis/Orthopedics/Rheumatology A 12 year-old male presents complaining of progressive and worsening nontraumatic left knee and femur pain which began two weeks ago. The patient now refuses to bear weight on the affected limb. Examination reveals pain with passive motion and a temperature of 101 F. Gram stain of a bone aspiration reveals Staphylococcus aureus. Which of the following is the most likely diagnosis? A. Osteomyelitis B. Septic arthritis C. Gaucher disease D. Ewing sarcoma

(c) A. Acute osteomyelitis is characterized by destruction of the bone secondary to an infectious organism, most commonly Staphyloccocus aureus in the metaphysis of long bones. Males are affected more frequently than females and present with increasing pain and disuse of the affected limb. Radiologic findings include soft tissue swelling, narrowing or widening of the joint space, bone destruction, and periosteal reaction. The gold standard diagnostic study is a bone aspiration of the lesion revealing the organism. (u) B. Septic arthritis can present similarly to acute osteomyelitis, however the affected area is limited to the joint and organisms are typically limited to Yersinia and Salmonella. (u) C. Gaucher disease is a glycolipid storage disease that can affect the bone marrow and present similarly to acute osteomyelitis. These patients also have numerous other symptoms that begin in early childhood to late adulthood such as bruising from anemia, hepatomegaly, splenomegaly, and occasionally lung involvement. (u) D. Ewing sarcoma also presents similarly to acute osteomyelitis, however, small round cells are seen microscopically signaling sarcoma, not infection.

A 35 year-old male presents with fever, perineal pain, and dysuria. On physical examination, the patient is toxic-appearing, febrile, and his prostate is very tender to palpation. Laboratory testing reveals leukocytosis, pyuria, and bacteriuria. Which of the following is the treatment of choice for this patient? A. Ampicillin and gentamicin B. Ceftriaxone and doxycline C. Trimethoprim-sulfamethoxazole D. Nitrofurantoin

(c) A. Acute prostatitis is best treated acutely with parenteral antibiotics, such as ampicillin and gentamicin. (u) B. Ceftriaxone and doxycycline are used in the treatment of acute epididymitis due to sexually transmitted infection. (u) C. Trimethoprim-sulfamethoxazole can be used but is second line in toxic patients and is best used after the patient is stable. (u) D. Nitrofurantoin is used in the treatment of acute cystitis and not indicated in acute prostatitis.

A 52 year-old obese female with a history of hypertension, tobacco abuse, and hyperlipidemia presents for routine follow-up. Which of her risk factors for coronary atherosclerosis is not modifiable? A. Age B. High LDL C. Hypertension D. Obesity

(c) A. Age is a non modifiable risk factor, as is family history of premature coronary heart disease (u) B. High LDL is a modifiable risk factor, as is Hypertension, low HDL, obesity, tobacco abuse, physical inactivity (u) C. See B for explanation. (u) D. See B for explanation

Which of the following is the major pathogenetic mechanism that causes asthma? A. Airway inflammation B. Increased pulmonary secretions C. Presence of Ghon complexes D. Irreversible fibrosis

(c) A. Airway inflammation is the major pathogenetic mechanism that leads to the development of asthma. (u) B. Increased pulmonary secretions are the mechanism in chronic bronchitis. (u) C. The presence of Ghon complexes is noted in pulmonary tuberculosis. (u) D. Irreversible fibrosis of the lung parenchyma is associated with interstitial lung diseases.

A 4 year-old boy is sent home from day care for a severe cough following one week of cold symptoms, including sneezing, conjunctivitis, and nocturnal cough. He presents with paroxysms of cough followed by a deep inspiration, and occasional post-tussive emesis. During severe paroxysms, he exhibits transient cyanosis. What is the most appropriate treatment for exposed contacts at his day care center? A. Macrolide prophylaxis B. Isolation C. Observation and treatment only if symptomatic D. Supportive care only

(c) A. All close contacts of a patient with pertussis should be treated with macrolide prophylaxis, regardless of age, immunization history, or symptoms. (u) B. Isolation of contacts is impractical and unnecessary. (u) C. Pertussis is rarely diagnosed before the paroxysmal stage, by which time exposure of contacts to the pathogen is assured. (u) D. While supportive care is essential in those contacts with symptoms, macrolide prophylaxis is mandatory in all contacts to prevent further spread of the illness.

A 76 year-old active female with history of hypertension and hypothyroidism presents with complaints of palpitations and dyspnea on exertion. On examination vital signs are BP 120/80 mmHg, HR 76 bpm, irregular, RR 16. Heart examination reveals an irregularly, irregular rhythm without murmur. Lungs are clear to auscultation and extremities are without edema. Which of the following is the most important medication to initiate for chronic therapy in this patient? A. Warfarin (Coumadin) B. Verapamil (Calan) C. Amiodarone (Cordarone) D. Digoxin (Lanoxin)

(c) A. Anticoagulation is necessary in all patients with atrial fibrillation to prevent thromboembolic events unless there is contraindication. (u) B. This patient currently has a controlled ventricular rates and does not require chronic calcium channel blockers or digoxin at this time. (u) C. Antiarrhythmic therapy may be indicated in some patients with atrial fibrillation, but anticoagulation is indicated in all patients unless there is contraindication. (u) D. See B for explanation.

: (1) 73. Clinical Intervention/Pulmonology A 59 year-old otherwise healthy female develops acute dyspnea and chest pain one week post total abdominal hysterectomy. Which of the following is the next step in the management of this patient? A. Anticoagulation B. Embolectomy C. Thrombolysis D. Inferior vena cava filter

(c) A. Anticoagulation is the treatment of choice in patients with pulmonary embolism with normal ventricular function and no absolute contraindications. (u) B. Embolectomy is not indicated as initial treatment of a pulmonary embolism. (h) C. Thrombolysis is contraindicated in patients within 10 days of having major surgery. (u) D. An inferior vena cava filter is considered in patients with contraindications to anticoagulation therapy or failed anticoagulation therapy.

Antinuclear Antigen (ANA) is most commonly associated with and monitors progress of which of the following disorders? A. Systemic Lupus Erythematosis (SLE) B. Rheumatoid arthritis (RA) C. Osteoarthritis D. Ankylosing spondylitis

(c) A. Antinuclear Antigen is the most helpful screening tool for SLE. (u) B. Although present in rheumatoid arthritis, ANA is not specific. A high rheumatoid factor (RF) is more tellingfor RA. (u) C. Osteoarthritis is not significantly associated with ANA. (u) D. Ankylosing spondylitis is a seronegative disorder and does not reflect RF or ANA levels.

Early clues to impending delirium tremens include A. agitation and decreased cognition. B. visual hallucinations and diaphoresis. C. autonomic hyperactivity and dehydration. D. mental confusion and sensory hyperacuity.

(c) A. Anxiety, decreased cognition, tremulousness, increasing irritability, and hyperactivity are common early clues to impending delirium tremens. (u) B. Mental confusion, tremor, sensory hyperacuity, visual hallucinations, autonomic hyperactivity, diaphoresis, dehydration, electrolyte disturbances, seizures, and cardiovascular abnormalities are common signs and/or symptoms of full-blown delirium tremens. (u) C. See B for explanation. (u) D. See B for explanation.

A 25 year-old female presents to the emergency department after she collapsed at work. Medical history is significant for a mood disorder that causes her to have "wild mood swings and reckless behavior" according to her husband. She was diagnosed a year ago and since has been treated with several medications. Her symptoms today consist of nausea, vomiting, fatigue, tremor, and hyperreflexia. Lab results show an elevated BUN and creatinine, low sodium and elevated drug levels. All other results are normal. Which of the following medications is most likely the cause of her symptoms? A. Lithium (Lithobid) B. Lorazepam (Ativan) C. Carbamazepine (Tegretol) D. Risperidone (Risperdal)

(c) A. Any sodium loss results in increased lithium levels. Signs and symptoms include vomiting and diarrhea which exacerbate the problem. Tremors, muscle weakness, confusion, vertigo, ataxia, hyperreflexia, rigidity, seizures, and coma may also be present. (u) B. See A for explanation. (u) C. See A for explanation. (u) D. See A for explanation.

Which of the following is the most common composition of kidney stones? A. calcium oxalate B. uric acid C. struvite D. calcium phosphate

(c) A. Approximately three fourths of all kidney stones are comprised of calcium oxalate. (u) B. See A for explanation. (u) C. See A for explanation. (u) D. See A for explanation

A 33 year-old presents with sinusitis unresponsive to three various antibiotics over the past four months. Sinus puncture is performed with culture positive for Aspergillus fumigatus. What is the most appropriate treatment for this patient? A. Amphotericin (Amphotericin B) B. Amoxicillin/Clavulanate (Augmentin) C. Ofloxacin (Floxin) D. Cefuroximine (Ceftin)

(c) A. Aspergillus fumigatus is a fungal organism therefore this patient would be most appropriately treated with Amphotericin B which is an anti-fungal medication. (u) B. See A for explanation. (u) C. See A for explanation. (u) D. See A for explanation.

A 4 year-old patient presents with episodic wheezing and a non-productive cough for the last 4 weeks. His symptoms are worse at night. Past medical history reveals a history of atopic dermatitis. Physical examination at this time is unremarkable. Which of the following is the most likely diagnosis? A. Asthma B. Bronchiolitis C. Croup D. Cystic fibrosis

(c) A. Asthma is a chronic inflammatory disorder of the airways. It is characterized by episodic or chronic symptoms of airflow obstruction, breathlessness, cough, wheezing, and chest tightness. The strongest identifiable predisposing factor for the development of asthma is atopy. (u) B. Bronchiolitis is common in infants and young children presenting with acute onset of cough, rhinorrhea, tachypnea, and expiratory wheezes. (u) C. Croup usually presents with a prodrome of upper respiratory tract symptoms followed by onset of a barking cough and stridor. (u) D. Cystic fibrosis is an autosomal recessive disease and is characterized by a chronic cough, sputum production, dyspnea, and wheezing. Steatorrhea, diarrhea, and abdominal pain are also common.

Which of the following conditions is most closely associated with an increased intensity of the P2 heart sound? A. Atrial septal defect B. Aortic stenosis C. Ventricular septal defect D. Mitral valve prolapse

(c) A. Atrial septal defect can cause a left to right shunt with resultant increased volume in the right ventricle. The net result of this is that the P2 heart sound will be accentuated because of the increased blood flow in the right ventricle and increased force of contraction in the right ventricle to remove this blood. Atrial septal defect will also cause a fixed split S2 heart sound. (u) B. Aortic stenosis primarily affects the left ventricle and causes left ventricular hypertrophy. As the left ventricle contracts harder against increased valvular resistance, a systolic ejection murmur occurs. (u) C. Ventricular septal defect most commonly causes a holosystolic murmur as its most notable ausculatory finding. (u) D. Mitral valve prolapse most commonly is associated with a mid-systolic click with or without a systolic heart murmur.

A 52 year-old obese patient with persistent heavy menses undergoes an endometrial biopsy and is diagnosed with atypical adenomatous hyperplasia. What is the next step in the management of this patient? A. Total abdominal hysterectomy B. Observation and endometrial biopsy in 3 months C. Endometrial curettage followed by progesterone daily D. Oral progesterone days 16-25 of the month for 6 months and repeat biopsy

(c) A. Atypical adenomatous hyperplasia contains cellular atypia and mitotic figures in addition to glandular crowding and complexity. This has a 20-30% risk of progression to endometrial cancer and the recommendation is hysterectomy. (h) B. Observation and biopsy again in 3 months would increase the risk of endometrial cancer for this patient. (h) C. Endometrial curettage would remove the hyperplasia and progesterone will decrease the endometrial glandular proliferation. This would be appropriate management in a patient with endometrial hyperplasia without atypia. (h) D. Oral progesterone for 10 days of the month will cause the patient to have a withdrawal bleed every month. This would be an appropriate treatment in a premenopausal patient with endometrial hyperplasia without atypia

A 24 year-old male comes to the clinic with a one week history of pain and swelling that involves the entire right upper extremity. He exercises frequently and has noticed the pain worsening while lifting weights. Examination shows enlarged cutaneous veins over the right anterior chest wall with a palpable cord. His right hand appears dusky. Which of the following is the most likely diagnosis? A. Axillary-subclavian venous thrombosis B. Thromboangiitis obliterans C. Superficial thrombophlebitis of the cephalic vein D. Brachial artery occlusion

(c) A. Axillary-subclavian venous thrombosis can occur in someone who strenuously exercises, has had a central venous catheter or history of venous thrombosis. (u) B. Thromboangiitis obliterans involves the arteries, although the smaller veins can be included and is linked to tobacco use. (u) C. This presentation is not consistent with superficial thrombophlebitis and there is no history of varicosities or previous IVs. (u) D. Symptoms are not consistent with brachial arterial occlusion.

A 4 month-old infant is brought to the clinic by his mother with complaints of a cough for the past 3 weeks. Initially, symptoms included running nose, sneezing and an irritating cough. Over the past week the cough has changed to persistent staccato, paroxysmal forceful coughs ending with a loud inspiration. WBC is 20,0000/mcl with 72% lymphocytes. Which of the following is the drug of choice for managing this patient? A. Azithromycin (Zithromax) B. Ceftriaxone (Rocephin) C. Ampicillin (Unasyn) D. Gentamicin (Garamycin)

(c) A. Azithromycin terminates respiratory tract carriage of Bordetella pertussis. (u) B. Ceftriaxone does not eradicate Bordetella pertussis. (a) C. Ampicillin may be used for macrolide-intolerant patients however it is not the drug of choice. (u) D. Gentamicin does not eradicate Bordetella pertussis.

Which of the following is the most common type of skin cancer? A. basal cell B. melanoma C. atypical nevi D. squamous cell

(c) A. Basal cell cancer is the most common cause of skin cancer usually occurring on sun-exposed areas. (u) B. While malignant melanoma is the leading cause of death from skin disease, it is not the most common skin cancer. (u) C. Atypical nevi are associated with melanoma. They are diagnosed clinically, not histologically. Any atypical nevi suspected to be melanomas should be removed. (u) D. Squamous cell carcinomas also occurs in sun-exposed areas, but are less frequent than basal cell cancers.

Which of the following animals is the major cause of human rabies in the United States and, therefore, poses the highest risk? A. bats B. rabbits C. rodents D. dogs

(c) A. Bat rabies is found in practically every state and is the most common cause of human rabies in the U.S. (u) B. Rabbits, rodents, and dogs are uncommon causes of human rabies in the United States. (u) C. See B for explanation. (u) D. See B for explanation.

A 21 year-old male college student is admitted to the hospital with suspected meningitis. A lumbar puncture is performed. The results of the cerebrospinal fluid (CSF) analysis reveals an elevated white blood cell count of 5,000/mcL with over 90% neutrophils, a decreased glucose level of 35 mg/dL, and elevated protein level of 150mg/dL. What is the most likely diagnosis based on these results? A. Bacterial meningitis B. Viral meningitis C. Fungal meningitis D. Tuberculous meningitis

(c) A. CSF results with bacterial meningitis reveal an elevated white count with predominance of neutrophils, a low glucose, and an elevated protein level. (u) B. CSF values in patients with viral meningitis are lymphocytic pleocytosis with normal glucose and normal or slightly elevated protein. (u) C. CSF findings in fungal meningitis include lymphocytic pleocytosis, elevated protein, and decreased glucose. (u) D. CSF findings with TB meningitis reveals elevated pressure, lymphocytic pleocytosis, elevated protein, and decreased glucose.

A patient presents with complaint of sudden onset of recurrent episodic vertigo for one week that happens when rolling onto the left side. The patient states that this sensation lasts approximately 30 seconds and then goes away. The patient admits to associated nausea. The patient denies associated hearing difficulties or tinnitus. Which of the following is the most likely diagnosis? A. Benign positional vertigo B. Ménière's disease C. Acoustic neuroma D. Vestibular neuronitis

(c) A. Benign positional vertigo is characterized by the sudden onset of vertigo when rolling onto the affected side or tilting the head up. The typical duration is less than a minute. There can be associated nausea and vomiting. There is no impact on hearing and no associated tinnitus. (u) B. Ménière's disease is characterized by a sudden onset of vertigo that lasts several hours to more than a day. Patients typically have sensorineural hearing loss and tinnitus. (u) C. Acoustic neuroma is characterized by an insidious onset of vertigo with impaired unilateral hearing and the presence of tinnitus. (u) D. Vestibular neuronitis (acute labyrinthitis) has a sudden onset of vertigo lasting hours to two weeks. There is no hearing impairment or tinnitus.

A 16 year-old male presents with complaint of syncope after basketball practice today. Physical examination reveals a systolic murmur along the left sternal border that increases with Valsalva maneuver. An electrocardiogram reveals left ventricular hypertrophy. Echocardiogram shows asymmetric left ventricular hypertrophy with a hypercontractile left ventricle. Which of the following is the initial medication of choice in this patient? A. Metoprolol (Lopressor) B. Losartan (Cozaar) C. Lisinopril (Zestril) D. Hydrochlorothiazide (Diuril)

(c) A. Beta-blockers are the initial drug of choice in a symptomatic patient with hypertrophic cardiomyopathy. (u) B. See A for explanation. (u) C. See A for explanation. (u) D. See A for explanation.

What is the most common anatomical location of anterior bleeding from the nose? A. Kiesselbach plexus B. Maxillary sinus ostia C. Nasal pyramid D. Superior turbinate

(c) A. Bleeding is most common in the anterior septum where a confluence of veins creates a superficial venous plexus known as Kiesselbach plexus. (u) B. See A for explanation. (u) C. See A for explanation. (u) D. See A for explanation.

: (8) 194. Clinical Therapeutics/ENT/Ophthalmology A 23 year-old female presents with a 3-day history of itching and redness of both eyes. She states that her eyes were stuck shut upon awakening. Examination is significant for moderate lid-edema with crusting about the eyelashes. There was no change in visual acuity or significant erythema of the conjunctiva. Which of the following is the definitive treatment for this patient? A. Erythromycin ophthalmic ointment B. Lid scrubs C. Olopatadine HCL drops (Patanol) D. Warm compresses

(c) A. Blepharitis is typically caused by Staphylococcus aureus (S. aureus) and most often preferentially involves the eyelid margins and lash follicles. Erythromycin ointment is more effective than antibiotic eyedrops in treating the lid margins. (u) B. Lid scrubs can be utilized for symptomatic relief of blepharitis but will not treat the underlying infection. (u) C. Olopatadine HCL (Patanol) drops are used in the treatment of allergic conjunctivitis, not blepharitis. (u) D. Application of warm compresses is indicated for the symptomatic relief of acute hordeolum (infection of Zeiss or Meibomian glands).

: (28) 87. History & Physical/Gastrointestinal/Nutritional Which of the following symptoms most commonly indicates diarrhea of an inflammatory etiology? A. Bloody stools B. Large volume stools C. Fatty stools D. Watery stools

(c) A. Bloody diarrhea occurs due to mucosal inflammation and/or erosions/ulcerations and may be caused by infection, inflammation, or ischemic enterocolitis. (u) B. Large volume indicates anatomical orgin (small intestine) but not etiology. (u) C. Fatty stools usually seen in malabsorptive conditions of the small intestine and do not necessarily involve inflammation but the increase in fat remaining in the lumen. (u) D. Watery stools are just an indication of increase amount of water, either osmotic or secretory in nature.

High ventricular filling pressures are indicated by a rise in which of the following? A. Brain natriuretic peptide B. Troponin C. Myoglobin D. Creatinine

(c) A. Brain natriuretic peptide (BNP) is a hormone released from the myocardium when stretched such as with high ventricular filling pressures from CHF. It can indicate or be used as prognostic evaluation in patients with acute CHF. (u) B. Troponin is a marker for cardiac muscle damage as in an acute myocardial infarction or ischemia. (u) C. Myoglobin is a byproduct of muscle destruction or damage from ischemia or other causes of skeletal muscle damage. Myoglobin may rise with high ventricular pressures, however it is nonspecific. (u) D. Creatinine is a marker of renal function.

Upon auscultation of a patient's lungs, there are harsh, hollow breath sounds which have a long inspiratory component in the region of the suprasternal notch. Throughout the periphery of the lung fields, softer breath sounds are heard. Which of the following best describes these findings? A. Normal B. Asthmatic C. Atelectasis D. Foreign body

(c) A. Bronchial breath sounds are normally heard near the sternum and vesicular breath sounds are heard over the periphery of the lungs in a healthy, normal patient. (u) B. Breath sounds in an asthmatic patient are usually obscured by wheezing. (u) C. Breath sounds are usually absent over an area of atelectasis. (u) D. Foreign body aspiration can present with stridor, wheezing or decreased breath sounds depending on where it has lodged.

Which of the following medications used in the management of anxiety has a delayed onset of action? A. buspirone (BuSpar) B. diphenhydramine (Benadryl) C. lorazepam (Ativan) D. butalbital (Fiorinal)

(c) A. Buspirone takes several days to weeks for it to have clinical activity. (u) B. Diphenhydramine works as a histamine blocker and will cause sedation immediately because of its anticholinergic effects. (u) C. Lorazepam is an anxiolytic medication that has an immediate onset of activity. (u) D. Butalbital is a short to intermediate-acting barbiturate that has immediate activity.

Which of the following physical examination findings is present in a 12 week singleton pregnancy? A. Chadwick's sign B. Uterus palpable at the level of the umbilicus C. Blood pressure lower than non-pregnant state D. Hyperreflexia

(c) A. Chadwick's sign is a bluish discoloration of the vagina early in pregnancy; it usually appears by 12 weeks of gestation. (u) B. The uterus is palpable at the level of the umbilicus at 20 weeks in a singleton pregnancy. (u) C. Blood pressure lower than non-pregnant state occurs in the second trimester. (u) D. Hyperreflexia is an uncommon finding and may occur with preeclampsia.

An 80 year-old female presents with pain in her vertebral column. Radiography reveals compression fracture of T12 that is consistent with osteoporotic compression fracture. Which of the following treatment modalities has the potential to cause analgesia of the fracture site with its use? A. Calcitonin (Miacalcin) nasal spray B. Alendronate (Fosamax) C. Raloxifene (Evista) D. Combined estrogen and progesterone (Prempro) therapy

(c) A. Calcitonin has the ability to cause analgesia when used for acute compression fracture of the vertebral body. (u) B. Alendronate is effective in building new bone for a patient with osteoporosis but has no associated analgesic effect. (u) C. Raloxifene is a selective estrogen receptor modulator and has positive effects on bone density when used to treat osteoporosis. Raloxifene, however, has no analgesic properties. (u) D. Combined hormonal therapy may have positive effects on bone density but it has no analgesic properties.

Which of the following clinical manifestations is common in candidal vulvovaginitis? A. Extreme vulvar irritation B. Firm, painless ulcer C. Tender lymphadenopathy D. Purulent discharge

(c) A. Candida infection presents with pruritus, vulvovaginal erythema, and white, cheese-like (curd) discharge that may be malodorous. (u) B. A firm painless ulcer is seen in syphilis. (u) C. Tender lymphadenopathy is associated with bacterial infections and is not a feature of candidal vulvovaginitis. (u) D. Purulent discharge is noted in gonorrhea.

Which of the following antihypertensive agents is considered to have both alpha- and beta-blocker activities? A. carvedilol (Coreg) B. hydralazine (Apresoline) C. minoxidil (Loniten) D. spironolactone (Aldactone)

(c) A. Carvedilol has both alpha- and beta-blocker activities. (u) B. Hydralazine and minoxidil are considered vasodilators. (u) C. See B for explanation. (u) D. Spironolactone is a potassium-sparing diuretic.

Which of the following is the most common radiographic presentation of lung abscess? A. cavitation B. pleural thickening C. hilar mass D. hyperinflation

(c) A. Cavitation is seen with lung abscess or progressive primary tuberculosis. (u) B. Pleural thickening is noted in mesothelioma. (u) C. Hilar and mediastinal abnormalities are common on chest radiography in patients with lung cancer. (u) D. Hyperinflation is the main clinical feature in emphysema.

When the diagnosis of gonococcal urethritis is confirmed, which of the following is the treatment of choice? A. Ceftriaxone (Rocephin) B. Amoxicillin (Amoxil) C. Penicillin G benzathine (Bicillin LA) D. Doxycycline (Vibramycin)

(c) A. Ceftriaxone is recommended therapy for gonococcal urethritis. (u) B. Effective single-dose regimens for uncomplicated gonococcal urethritis include cefixime or one of the fluoroquinolones, but NOT amoxicillin due to penicillin-resistant strains of gonorrhea. (u) C. Penicillin G benzathine is the treatment of choice for primary syphilis. (u) D. Doxycycline is the treatment of choice for Chlamydia urethritis, not gonococcal urethritis.

A 30 year-old patient presents with weight loss, diarrhea, and steatorrhea. Labs reveal that the antiendomysial antibody (AEA) is positive. What is the most likely diagnosis? A. Celiac sprue B. Ulcerative colitis C. Whipple's disease D. Zollinger-Ellison syndrome

(c) A. Celiac sprue is not only characterized by these classic symptoms. The antiendomysial antibody has a 90-95% sensitivity and 90-95% specificity for celiac sprue. (u) B. Ulcerative colitis could be responsible for the symptoms mentioned however antiendomysial antibody (AEA)would not be positive. (u) C. Whipple's disease is an infectious disorder known to cause diarrhea and weight loss. It also results in arthralgia and symptoms involving both the central nervous and cardiac systems that are not easily missed due to their severity. Diagnosis involves biopsies of the involved tissues looking for PAS-positive macrophages. (u) D. Zollinger-Ellison syndrome (ZE) is a hypersecretory disorder. Diarrhea can occur but will not generally be the predominant symptom.

: (28) 203. Health Maintenance/Cardiology You are evaluating a patient who complains of worsening orthopnea and dyspnea on exertion for the past year. He admits to drinking a 12-pack of beer on a nightly basis for approximately 20 years. Physical examination shows a laterally displaced PMI, audible rales, and an audible S3 on cardiac auscultation. Echocardiogram shows a dilated left ventricular and an ejection fraction of 35%. Which of the following is the most appropriate counseling to slow the progression of this patient's condition? A. Discontinue alcohol use B. Initiate vigorous exercise program C. Recommend immunosuppressive therapy D. Initiate sodium restricted diet

(c) A. Dilated cardiomyopathy is manifested with traditional signs and symptoms of heart failure (most commonly dyspnea). LV dilation and systolic dysfunction (EF < 50%) are essential for diagnosis. Alcohol use should be discontinued, since there is often marked recovery of cardiac function following a period of abstinence in alcoholic cardiomyopathy. 80 (h) B. Exercise should be progressive as with reduced ejection fraction, the potential for ischemia with exertional activity increases exponentially. (u) C. Immunosuppressive therapy is not indicated in chronic dilated cardiomyopathy. (u) D. Sodium restriction is helpful, especially in acute CHF, however alcohol abstinence often leads to marked recovery of cardiac function.

Cognitive loss in Alzheimer's dementia may be delayed with which of the following medications? A. donepezil (Aricept) B. haloperidol (Haldol) C. risperidone (Risperdal) D. zolpidem (Ambien)

(c) A. Donepezil is a reversible cholinesterase inhibitor that leads to increased acetylcholine, which is necessary for learning and memory. (u) B. Haloperidol, risperidone, and other antipsychotics may be used to treat the agitation and behavioral symptoms in patients with dementia, but have significant side effects. (u) C. See B for explanation. (u) D. Zolpidem is used to treat insomnia

A 23 year-old male presents with symptoms of irritative urethral discharge. History reveals recent unprotected intercourse with a new partner. A Gram stain of the urethral discharge is negative for intracellular diplococci. Assuming no allergies, which of the following is the drug of choice? A. Doxycycline B. Penicillin G C. Ceftriaxone D. Amoxicillin

(c) A. Doxycycline 100 mg orally for 7 days is indicated for the treatment of non-gonococcal urethritis. (u) B. Penicillin G is not used in the treatment of Chlamydia urethritis. (u) C. Ceftriaxone is the treatment of choice for uncomplicated gonorrhea. (u) D. Amoxicillin is not used in the treatment of gonococcal or non-gonococcal urethritis.

A 68 year-old male with a history of alcohol abuse presents with coffee-ground emesis. He denies vomiting prior to this episode. What is the most likely cause of his bleeding? A. Erosive gastritis B. Gastric neoplasm C. Mallory-Weiss tear D. Zenker diverticulum

(c) A. Drugs and alcohol are the most common causes of upper gastrointestinal bleeding. (u) B. Gastric neoplasm is also uncommon (less than 1% of upper GI bleeding) but should be ruled out). (u) C. Mallory-Weiss tears are lacerations of the gastroesophageal junction causing 5-10% of cases of upper GI bleeding. Many patients report a history of heavy alcohol use or retching. (u) D. Peptic ulcer disease is a common cause of upper GI bleeding but is becoming less common with the increasing irradication of H. pylori infections.

: (28) 125. History & Physical/Endocrinology A 42 year-old female presents with complaints of ongoing fatigue, "always feeling cold", and mild constipation. During your diagnostic evaluation her TSH result is found to be elevated. Which of the following skin findings would you expect on examination? A. Dry, rough B. Darkened, velvety C. Flushed, diaphoretic D. Deeply tanned

(c) A. Dry, rough skin is consistent with hypothyroidism, which matches the patient's other signs and symptoms. Findings can also include thin brittle nails, thinning of the hair, puffy face and eyelids, and occasionally yellowing of the skin (carotenemia). (u) B. Darkened, velvety skin describes acanthosis nigricans, which is associated with insulin resistance and consequently diabetes. (u) C. Flushed, diaphoretic skin is consistent with hyperthyroidism. (u) D. Deeply tanned skin would be expected in the skin examination of Addison disease (chronic adrenal insufficiency).

: (8) 134. History & Physical/Pulmonology Which of the following physical exam findings would be most likely in a patient with a pleural effusion? A. Dull to percussion with decreased tactile fremitus B. Hyperresonant to percussion with decreased tactile fremitus C. Dull to percussion with increased tactile fremitus D. Hyperresonant to percussion with increased tactile fremitus

(c) A. Due to the increased density presented by the effusion the percussion note will be dulled and transmission of speech vibration will be reduced. In contrast, emphysema with its increased lung volume will cause hyperresonance and increased fremitus. (u) B. See A for explanation. (u) C. See A for explanation. (u) D. See A for explanation.

A 55 year-old man with a history of chronic bronchitis presents with two days of increased dyspnea and cough with worsening purulent sputum production. He is currently using inhaled albuterol as needed. In addition to systemic corticosteroids, what pharmacologic agent is warranted at this time for treatment of this patient? A. Antibiotic B. Inhaled corticosteroid C. Long acting beta-agonist D. Theophylline

(c) A. Empiric antibiotic treatment is indicated in the treatment of acute exacerbations of COPD if there are sputum changes suggestive of bacterial infection, such as increased quantity and purulence. (u) B. Inhaled corticosteroids are not indicated in the management of acute exacerbations of COPD. (u) C. Long acting beta-agonists are not indicated in the management of acute exacerbations of COPD. (u) D. Theophylline is rarely used in the management of COPD and has no place in the management of acute exacerbations of COPD.

Normal hemoglobin A is made of what combination of heme and globin chains? A. 2-alphas and 2-betas B. 4-gammas C. 2-alphas and 2-gammas D. 4-betas

(c) A. Hemoglobin A1 is composed of two alpha and two beta chains. (u) B. Barts Hemoglobin-Hydrops Fetalis has 4-gamma chains and is incompatible with life. (u) C. Fetal hemoglobin consists of 2-alpha and 2-gamma chains. (u) D. Hemoglobin H consists of 4-beta chains and is nonfunctional.

Which of the following is the most accurate method to differentiate benign from malignant thyroid nodules? A. Fine needle aspiration biopsy B. Ultrasound of the thyroid gland C. Measurement of thyroid antibodies D. Radionuclide iodine uptake

(c) A. Fine needle aspiration biopsy is the best method used in the evaluation of thyroid cancer. Fine needle aspiration biopsy can be done on an outpatient basis and can be done under ultrasound guidance for patients who have small thyroid nodules. (u) B. Ultrasound of the thyroid gland is performed in order to help to guide the practitioner in performing a fine needle aspiration biopsy. It can also be used to differentiate solid from cystic masses but it cannot predict which nodules are cancer and which are benign. (u) C. Measurement of thyroid antibodies is most helpful in patients who have subclinical hypothyroidism or overt hyperthyroidism. There is a lot of controversy regarding whether treatment should be given to patients with subclinical disease and most providers will use thyroid antibodies to determine whether treatment should be given or delayed. (u) D. Radionuclide iodine uptake is performed to assess the activity of the thyroid gland. This test is most helpful to determine whether the gland is metabolically active (as seen with Graves' disease) or whether the gland is just releasing preformed thyroid hormone (subacute thyroiditis). The test provides a percentage of activity rather than a picture (thyroid scan) provides a picture of whether a nodule is "hot" or cold."

Which of the following types of hip fracture has the highest risk for avascular necrosis and nonunion? A. Femoral neck B. Intertrochanetric C. Subtrochanteric D. Greater trochanteric

(c) A. Fractures involving the femoral neck typically disrupt the blood supply to the femoral head and may cause avascular necrosis and nonunion. (u) B. Intertrochanteric fractures usually do not cause avascular necrosis and nonunion due to the capsule and blood supply to the femoral head remaining intact. (u) C. Subtrochanteric fractures usually do not cause avascular necrosis and nonunion due to the capsule and blood supply to the femoral head remaining intact. (u) D. Greater trochanteric fractures usually do not cause avascular necrosis and nonunion due to the capsule and blood supply to the femoral head remaining intact

A 14 year-old patient, who fell on his outstretched hand, complains of pain along his entire arm. There is point tenderness and swelling over the midshaft of the radius. There is significant pain with limited flexion of the elbow joint. An x-ray will most likely show which of the following fractures? A. Galeazzi's B. Scaphoid C. Colles' D. Smith's

(c) A. Galeazzi's fracture/dislocation involves a fracture of the mid or distal radial shaft with distal radioulnar joint dislocation. (u) B. A scaphoid fracture is a fracture of the scaphoid bone and would not cause pain in the elbow joint. (u) C. A Colles' fracture of the distal radius has a characteristic "silver fork" deformity, but does not involve the elbow joint. (u) D. A Smith's fracture is the reverse of a Colles' fracture, with volar angulation of the distal radius, but does not involve the elbow joint.

: (8) 187. Diagnosis/Gastrointestinal/Nutritional A 35 year-old female 2 months status-post gastric bypass surgery develops abdominal cramps, diarrhea, palpitations, sweating and nausea shortly after eating. Which of the following is the most likely diagnosis? A. Dumping syndrome B. Inflammatory bowel disease C. Surgical adhesions D. Zollinger-Ellison syndrome

(c) A. Gastric bypass puts this patient at risk for dumping syndrome. Her symptoms represent both cardiovascular and gastrointestinal reactions which occur after eating. The underlying effect is due to the stomach's inability to regulate its rate of emptying. (u) B. IBD does not present in acute attacks after meals and usually does not present with autonomic symptoms. (u) C. Small bowel obstruction due to peritoneal adhesions is a possibility due to her history of abdominal surgery. However, obstruction presents with abdominal pain, nausea and vomiting which may progress and are not associated with meals. (u) D. A gastrinoma leads to PUD refractory to large amounts of antacids and standard doses of H2 blockers.

A 36 year-old patient with cardiomyopathy secondary to viral myocarditis develops fatigue, increasing dyspnea, and lower extremity edema over the past 3 days. He denies fever. A chest x-ray shows no significant increase in heart size, but reveals prominence of the superior pulmonary vessels. Based on these clinical findings, which of the following is the most likely diagnosis? A. Heart failure B. Subacute bacterial endocarditis C. Pulmonary embolus D. Pneumonia

(c) A. Given the presence of cardiomyopathy, the patient's heart has decreased functional reserve. The symptoms and chest x-ray findings are typical of congestive heart failure. (u) B. Endocarditis occurs as a result of infection that primarily occurs in the blood stream. Endocarditis would present with signs of infection or seeding rather than signs of heart failure. (u) C. Pulmonary embolus usually presents with an acute onset of chest pain, severe dyspnea, and anxiety. (u) D. Pneumonia is less likely since there is no fever and edema is not usually associated with pneumonia.

A 40 year-old G3P3003 female presents complaining of dull aching discomfort of her lower extremities, which is worse in the evening. The patient currently works as a waitress. Examination reveals dilated, tortuous veins beneath the skin in the thigh and leg bilaterally. Which of the following is the best initial approach to prevent progression of disease and complications in this patient? A. Compression stockings B. Warfarin (Coumadin) therapy C. Sclerotherapy D. Clopidogrel (Plavix)

(c) A. Graduated compression stockings can be used in patients with early varicosities to prevent progression of the disease and when used with leg elevation complications from varicose veins can be avoided. (u) B. There is no indication for warfarin or clopidogrel therapy in patients with varicose veins. (u) C. Sclerotherapy is not the best initial choice to prevent disease progression and complications. (u) D. See B for explanation.

A 54 year-old female presents to the office for radiographic and laboratory results. The radioactive iodine uptake is elevated while the thyroid hormone levels are increased with TSH levels being suppressed. Which of the following is the most likely diagnosis? A. Graves' disease B. Hashimoto's thyroiditis C. Subacute thyroiditis D. Pituitary failure

(c) A. Graves' disease is associated with an elevated uptake on the radioactive thyroid scan due to an increase in the activity for the thyroid gland. Because the gland is actually making thyroid hormone, the free T4 level will be increased and the TSH will be suppressed as a result of negative feedback to the hypothalamus and pituitary gland. (u) B. Patients with Hashimoto's thyroiditis will have an underactive thyroid gland with a decrease in the radioactive iodine uptake. The thyroid gland is not producing enough thyroid hormones so the free T4 level will be low and the TSH level will be increased as the body tries to stimulate the thyroid gland to produce more thyroid hormone. (u) C. Patients with subacute thyroiditis have an increase in the release of thyroid hormone rather than an increase in the actual activity of the gland. These patients will have a normal or decreased iodine uptake on the radioactive thyroid scan which is the main differentiating feature between this condition and Graves' disease. The free T4 level can be variable and depends upon when in the course of this condition the thyroid hormones are measured. (u) D. Patients with pituitary failure will not be able to produce thyroid stimulating hormone so these patients will have a decreased radioactive thyroid uptake, a low free T4 level and a low TSH.

A 31 year-old female complains of diarrhea. She admits to associated weight loss and steatorrhea, but denies melena or hematochezia. Which of the following is the most likely diagnosis? A. Celiac disease B. Lactose intolerance C. Short bowel syndrome D. Irritable bowel syndrome

(c) A. Greater than 10 grams of fecal fat in 24 hours is most consistent with a malabsorption syndrome such as celiac sprue. (u) B. See A for explanation. (u) C. See A for explanation. (u) D. See A for explanation.

Which of the following is most frequently associated with renal cell carcinoma? A. Hematuria B. Inguinal pain C. Hypocalcemia D. Fever

(c) A. Gross or microscopic hematuria, flank pain, or mass is common in renal cell carcinoma. (u) B. See A for explanation. (u) C. Hypercalcemia, not hypocalcemia, may be noted in patients with renal cell carcinoma. (u) D. Fever is uncommon in renal cell carcinoma and typically only noted with advanced disease.

The source of pain experienced during a migraine headache is a result of activation of which nerve? A. Trigeminal B. Vagus C. Optic D. Occulomotor

(c) A. Headache may result in release of neuropeptides acting as neurotransmitters at trigeminal nerve branches. (u) B. See A for explanation. (u) C. See A for explanation. (u) D. See A for explanation.

A 26 year-old gravida 0 sexually active female presents to the emergency room complaining of colicky pain in her lower abdomen for the past 12 hours. She passed out earlier in the day while trying to have a bowel movement. Her last menstrual period was 6 weeks ago. She has noted vaginal spotting over the last 24 hours. Vital signs show Temp 37 degrees C, BP 96/60mmHg, P 110, R 16, Oxygen Sat. 98%. Abdominal exam is positive for distension and tenderness. Bowel sounds are decreased. Pelvic exam shows cervical motion and adnexal tenderness. Which of the following is the most likely diagnosis? A. Ectopic pregnancy B. Appendicitis C. Crohn's disease D. Pelvic inflammatory disease

(c) A. High suspicion for ectopic pregnancy should be maintained when any possible pregnant woman presents with vaginal bleeding or abdominal pain. (u) B. Appendicitis presents with nausea, vomiting and periumbilical pain that moves to the right lower quadrant of the abdomen. (u) C. Crohn's disease is more common in women and may present with an acute abdomen. However, pelvic examination would be normal. (u) D. In pelvic inflammatory disease the temperature is usually above 38 degrees C and pelvic pain usually follows onset of cessation of menses.

Higher doses of SSRIs are usually required in which of the following conditions? A. Obsessive-compulsive disorders B. Depression C. Manic depression D. Panic disorder

(c) A. Higher doses of SSRIs are needed in the treatment of OCD for a beneficial effect. (u) B. Although patients with depression may occasionally need to have an increase in the SSRI dosage, higher doses are not needed for beneficial effect. (u) C. Patients who are bipolar will often have worsening of their manic symptoms with the use of SSRIs. (u) D. SSRIs are good drugs to treat patients with panic disorder, however, these patients are particularly sensitive to the activating effects of SSRIs, so they should be started initially at small dosages and titrated up slowly.

A 78 year-old male with history of coronary artery disease status post CABG and ischemic cardiomyopathy presents with complaint of progressive dyspnea and orthopnea. He also complains of lower extremity edema. The patient denies fever, chest pain, or cough. On physical examination, vital signs are BP 120/68, HR 75 and regular, RR 22, afebrile. You note the patient to have an S3 heart sound, jugular venous distention, and 2+ lower extremity edema. The patient is admitted and treated. Upon discharge from the hospital, the patient should be educated to monitor which of the following at home? A. Daily weights B. Daily spirometry C. Daily blood glucose D. Daily fat intake

(c) A. Home monitoring of daily weights can alert the health care provider to the early recognition of worsening heart failure. (u) B. Spirometry monitoring is important in a patient with asthma, not heart failure. (u) C. Daily blood glucose monitoring is important in a patient with diabetes, not heart failure. (u) D. Daily fat intake is important, but will not improve his heart failure management.

: (28) 211. History & Physical/Pulmonology Which of the following non-pulmonary physical exam finding would be most likely in a patient with a large, apical lung cancer? A. Horner syndrome B. Ramsay Hunt syndrome C. Bilateral nasal polyps D. Bullous myringitis

(c) A. Horner syndrome, typified by a constricted pupil, partial ptosis and loss of sweating is associated with apical lung cancers. (u) B. See A for explanation. (u) C. See A for explanation. (u) D. See A for explanation.

A 35 year-old patient with lupus is being treated for mild arthralgias and rash with hydroxychloroquine. Which of the following clinical manifestations is the most common side effect of this medication? A. impaired night vision B. jaundice C. proteinuria D. mouth sores

(c) A. Hydroxychloroquine is associated with macular damage, rash and diarrhea. (u) B. Hydroxychloroquine is not associated with hepatoxicity of hemolytic anemia. (u) C. Hydroxychloroquine lacks renal toxicity. (u) D. Mouth sores may be a sign of agranulocytosis in patients on hydroxychloroquine, but retinopathy occurs more commonly.

A 19 year-old male presents to the ED complaining of a sudden onset of dyspnea and left sided chest pain. He denies fever, chills, cough or sore throat. General survey shows that he is 6 feet 2 inches tall and weighs 135 lbs. Vital signs are BP 86/60 mmHg, HR 130 bpm, RR 28, temp. 98.6 degrees F. Which of the following would you likely find on examination of his thorax? A. Left-sided hyperresonance B. Increased tactile fremitus of left base C. Scattered rales throughout D. Increased anterior/posterior diameter

(c) A. Hyperresonance is present with accumulation of air in the pleural space. (u) B. Increased tactile fremitus is present with lung consolidation. (u) C. Mycoplasma pneumonia, common in young adults, presents with scattered rales audible throughout. (u) D. Increased A/P diameter is common in older patients with history of COPD.

A patient is having a routine physical examination. Funduscopic examination reveals AV narrowing and venous nicking. The nasal border of the optic disc appears blurred. Which of the following is the most likely underlying cause? A. Hypertension B. Macular degeneration C. Retinal detachment D. Diabetes mellitus

(c) A. Hypertensive retinopathy may cause AV narrowing and venous nicking due to these blood vessels having increased pressures. (u) B. Macular degeneration is associated with the formation of Drusen and neoproliferation. (u) C. Retinal detachment is observed funduscopically by the retina being displaced from its attachment. Patients will present with complaints of floaters in the eye field or abrupt loss of vision if the detachment is complete. (u) D. Diabetes mellitus is most closely associated with neovascularization and microaneurysms as its primary manifestations.

A 7 year-old boy wets the bed on most nights. Which of the following is the preferred pharmacological agent to decrease the incidence of bed wetting episodes? A. Imipramine (Tofranil) B. Phenytoin (Dilantin) C. Pramipexole (Mirapex) D. Hyoscyamine (Urised)

(c) A. Imipramine is an anti-cholinergic and when given before bedtime has been shown to decrease the incidence of bed wetting. (u) B. Phenytoin is an anticonvulsant and is not used in enuresis. (u) C. Pramipexole is a dopamine agonist used in the treatment of restless leg syndrome. (u) D. Hyoscyamine is an anti-spasmodic used to treat overactive bladder.

A 26 year-old male presents with increased dyspnea with exercise. He has noted a decrease in his exercise tolerance over the past several months. He denies chest pain or skipped heart beats. Echocardiogram reveals left ventricular hypertrophy with asymmetric septal hypertrophy. Ejection fraction is 65%. Which of the following is the most likely presenting history or physical exam finding? A. He has an older brother with the same diagnosis. B. An S3 gallop is heard. C. Patient notes completing a course of adriamycin. D. Elevated jugular venous distension is noted.

(c) A. Hypertrophic cardiomyopathy can be genetic and present in 25% of first degree relatives. (u) B. Dilated cardiomyopathy may present with an S3 gallop due to volume overload. (u) C. Adriamycin chemotherapy can lead to cardiac dysfunction, dilated cardiomyopathy and eventually heart failure. (u) D. Restrictive or dilated cardiomyopathy may present with JVD due to abnormalities of filling.

A 23 year-old male presents with syncope. On physical examination you note a medium-pitched, mid-systolic murmur that decreases with squatting and increases with straining. Which of the following is the most likely diagnosis? A. Hypertrophic cardiomyopathy B. Aortic stenosis C. Mitral regurgitation D. Pulmonic stenosis

(c) A. Hypertrophic cardiomyopathy is characterized by a medium- pitched, mid-systolic murmur that decreases with squatting and increases with straining. (u) B. Straining decreases the intensity of the murmur associated with aortic stenosis and squatting increases the intensity. (u) C. Mitral regurgitation is characterized by a blowing systolic murmur that radiates to the axilla, it is not often associated with syncope. (u) D. Pulmonic stenosis is a harsh systolic murmur with a widely split S2, and no change with maneuvers.

A 32 year-old female presents complaining of spiking fevers. She was seen four weeks ago with a complaint of left ear pain and was treated for otitis media. She continues to have symptoms, but now has pain behind the ear. On examination you note left post auricular tenderness and erythema. Which of the following is the treatment of choice in this patient? A. IV antibiotics B. Mastoidectomy C. IM steroids D. Ventilating tube placement

(c) A. IV antibiotics are the treatment of choice in a patient with mastoiditis. (u) B. Mastoidectomy is reserved for patients with mastoiditis who fail medical therapy. (u) C. IM steroids are not indicated in the treatment of mastoiditis. (u) D. Ventilating tube placement is indicated in patients with auditory tube dysfunction and chronic serous otitis media.

A patient with Type 1 diabetes mellitus was treated for otitis externa of the right ear for 2 weeks with topical ear drops. The patient presents today with persistent, foul aural discharge, granulations in the ear canal, and deep ear pain. Which of the following is the proper treatment at this time? A. ciprofloxacin (Cipro) IV B. cefuroxime (Zinacef) IV C. ampicillin-sulbactam (Unasyn) PO D. azithromycin (Zithromax) PO

(c) A. IV antibiotics directed against Pseudomonas, the most likely etiology, is needed for the treatment of malignant otitis media. (u) B. Cefuroxime, ampicillin-sulbactam, and azithromycin have no activity against Pseudomonas. (u) C. See B for explanation. (u) D. See B for explanation.

: (8) 181. Clinical Therapeutics/Gastrointestinal/Nutritional A 56 year-old male with a history of hypertension and alcohol use presents with mid-epigastric pain and coffeeground emesis since early this morning. The patient denies diarrhea, constipation or blood in his stools. Endoscopic evaluation done after admission revealed peptic ulcer disease with active bleeding. Which of the following is the most appropriate therapy to reduce the risk of rebleeding in this patient? A. IV pantoprazole (Protonix) B. Oral hyoscyamine sulfate (Levsin) C. IV famotidine (Pepcid) D. Oral metoclopramide (Reglan)

(c) A. IV proton pump inhibitors reduce the risk of rebleeding after endoscopic treatment in patients with peptic ulcers that have high risk features such as active bleeding. (u) B. Hyoscyamine is an anticholinergic used as an adjunct in the treatment of peptic ulcer disease and to treat irritable bowel syndrome. It does not have a role in reducing the risk of rebleeding in these patients. (u) C. H2 antagonists do not stop active bleeding and do not reduce the incidence of rebleeding. (u) D. Metoclopramide is used in the management of GERD and nausea and is not indicated in the reduction of rebleeding secondary to peptic ulcer disease.

Which immunoglobulin is the first to respond during the primary immune response for a gram positive bacterial infection? A. IgM B. IgG C. IgA D. IgE

(c) A. IgM is the first immunoglobin to respond during the acute exposure. This immunoglobulin promotes opsonization and phagocytosis. IgG is the immunoglobulin that responds during the secondary exposure. IgE is the immunoglobulin that responds during an allergic response as well as during a parasitic infection. IgA is an antibody found in colostrums and GI secretions. (u) B. See A for explanation. (u) C. See A for explanation. (u) D. See A for explanation.

94. Health Maintenance/Dermatology A 67 year-old male presents for worsening lower extremity edema. He denies chest pain, shortness of breath or history of congestive heart failure. On examination the patient's lungs are clear to auscultation bilaterally and his cardiac exam reveals a regular rate and rhythm. His lower extremity physical examination is significant for 1+ pitting edema to the mid shin, multiple varicosities, and hyperpigmentation without skin breakdown. His ankle/brachial index (ABI) is within normal limits. You explain your concern to the patient about the development of stasis ulcers. Which of the following is the most important preventative step for this patient? A. Compression stockings B. Immobilization of the affected leg C. Diuretic therapy D. Referral to vascular surgeon

(c) A. In the setting of venous insufficiency, compression stockings are the best preventative tool to developing stasis ulcers by reducing edema. (u) B. The patient has venous insufficiency, immobilization of the leg may exacerbate the edema, leading to increased morbidity. The patient should ambulate and use the leg as tolerated. (u) C. The patient has venous insufficiency. Diuretic therapy is not indicated at this time. (u) D. This patient has venous insufficiency, not peripheral artery disease. He has a normal ABI, therefore does not require referral to a vascular surgeon at this time.

When performing a Weber test on a patient with impacted cerumen in the right canal, the sound should be A. referred to the right ear. B. referred to the left ear. C. equal in both ears. D. louder with air conduction.

(c) A. In unilateral conductive hearing loss, the sound is referred to the impaired ear. (u) B. See A for explanation. (u) C. See A for explanation. (u) D. Bone conduction as noted with the Rinne test is louder than air with conductive hearing loss.

A 26 year-old female comes to the office for evaluation of a painful lump on her right buttock for the past week. Initially, it was a firm, tender nodule that has increased in size and tenderness in the past two days. On physical examination of the right buttock, there is a 3-cm fluctuant tender red nodule. Which of the following is the most appropriate initial intervention? A. Incision and drainage B. Mupirocin ointment C. Systemic antibiotics D. Moist compresses

(c) A. Incision and drainage is the mainstay of therapy for abscesses. (u) B. Individuals may be chronic carriers of S. aureus. Mupirocin ointment is effective in eliminating nasal carriage of S. aureus. (u) C. In healthy individuals, incision and drainage is generally adequate therapy for abscesses. Systemic antibiotics may speed the healing of the tissue, and is instituted after incision and drainage of the abscess. (u) D. Application of moist heat can localize or consolidate the abscess when the furuncle is firm and non-fluctuant aiding in the development and drainage of the abscess.

Which of the following types of pleural effusion result from increased production of fluid due to underlying inflammatory conditions? A. Exudate B. Empyema C. Transudate D. Hemothorax

(c) A. Increased production of fluid due to inflammatory or malignant processes results in an exudative pleural effusion. (u) B. Infection in the pleural space causes an empyema. (u) C. Transudates result from increased hydrostatic or decreased oncotic pressures across normal capillaries. (u) D. Bleeding into the pleural space causes a hemothorax.

On physical examination of a pregnant patient, which can be considered a normal finding? A. Increased second heart sound split with inspiration B. Diastolic murmur C. Facial edema D. Hyperreflexia

(c) A. Increased second heart sound split with inspiration is common in pregnancy due to the increased blood flow across the aortic and pulmonic valves. (u) B. Diastolic murmurs in pregnancy should be considered pathological and evaluated further. (u) C. Facial edema in uncommon in pregnancy and if it occurs, the medical provider should consider preeclampsia. (u) D. Hyperreflexia occurs with preeclampsia and does not occur in a normal pregnancy.

: (30) 214. History & Physical/Psychiatry/Behavioral Medicine During a mental status examination of a patient with schizophrenia, you note increased time between the patient's comprehension and expression of thoughts. This finding is an increase in which of the following? A. Thought latency B. Thought disorder C. Thought insertion D. Thought withdrawal

(c) A. Increased thought latency is otherwise known as blocking. It is common in patients with schizophrenia, and defined by a period of time between a thought and its verbal expression in increased. (u) B. A thought disorder is a disorder of thinking that affects language, communication or thought content. (u) C. Thought insertions are delusions that are being implanted in one's mind by others. (u) D. Thought withdrawal consists of delusions where the patient believes that their thoughts are being removed by others.

Which of the following is used to monitor possible recurrence of prostate cancer? A. Prostate specific antigen B. Acid phosphatase C. Transrectal ultrasound D. Bone scan

(c) A. Increasing levels of prostate specific antigen are consistent with progression of disease. (u) B. Acid phosphatase levels may increase in prostate cancer, but are not as sensitive as prostate specific antigen for recurrence of disease. (u) C. Transrectal ultrasound is mainly used for staging of disease and not monitoring of recurrence. (u) D. Radionuclide bone scan is used to detect bony metastases.

: (8) 101. Diagnosis/Hematology A 35 year-old female presents with fatigue and dyspnea on exertion. She has a history of increasingly heavy menstrual periods for the last 8 months. She denies change in stool. Examination reveals HR 102 bpm, TEMP 97.6 F, RESP 20, BP 100/60 mmHg, pallor and cheilosis. Peripheral smear demonstrates a hypochromic microcytic anemia and few target cells. Which of the following is the most likely diagnosis? A. Iron deficiency anemia B. Vitamin B12 deficiency C. Thalassemia D. Sickle cell anemia

(c) A. Iron deficiency anemia is characterized by pallor and cheilosis and is commonly associated with menorrhagia. The peripheral smear will demonstrate hypochromic microcytic anemia and few target cells. (u) B. See A for explanation. (u) C. See A for explanation. (u) D. See A for explanation.

A pregnant 28 year-old female presents at 30 weeks gestation complaining of fatigue and headache. Her vital signs and physical examination are normal. Routine hemoglobin screening is 10.1 g/dL. Peripheral smear shows microcytic, hypochromic red blood cells. Besides the physiologic dilution of pregnancy, what type of anemia is most likely in this patient? A. Iron deficiency B. Folic acid deficiency C. Thalassemia D. Hereditary spherocytosis

(c) A. Iron deficiency anemia is responsible for 95% of anemias during pregnancy. (u) B. Folic acid deficiency anemia is common where nutrition is inadequate. (u) C. Thalassemia is a rare cause of anemia in pregnancy. (u) D. Hereditary spherocytosis is rare during pregnancy.

A 58-year old male presents for a six week follow-up after an acute anterior wall myocardial infarction. He denies chest pain and shortness of breath. Electrocardiogram shows persistent ST segment elevation in the anterior leads. Echocardiogram reveals a sharply delineated area of scar that bulges paradoxically during systole. Which of the following is the most likely diagnosis in this patient? A. Left ventricular aneurysm B. Postinfarction ischemia C. Ischemic cardiomyopathy D. Constrictive pericarditis

(c) A. Left ventricular (LV) aneurysm develops in about 10-20 percent of patients following acute myocardial infarctions, especially anterior wall myocardial infarctions. LV aneurysm is identified by ST segment elevation that is present beyond 4-8 weeks after the acute infarct and a scar that bulges paradoxically during systole on echocardiogram. (u) B. Postinfarction ischemia is recurrent ischemia that is more common in patients with non-ST segment elevation myocardial infarctions and is characterized by postinfarction angina. This patient denies any chest pain. (u) C. Ischemic cardiomyopathy would be characterized by decreased ejection fraction on echocardiogram and wall motion abnormalities. Ischemic cardiomyopathy is not associated with ST segment elevation or bulge of scar on echocardiogram. (u) D. Constrictive pericarditis is characterized by signs and symptoms of right-sided heart failure with increased jugular venous pressures and a septal bounce on echocardiogram.

A 55 year-old female presents to the emergency department with complaints of dyspnea, chest pain and coughing with hemoptysis. Past medical history includes breast cancer 5 years ago, currently in remission. Vital signs are Temp. 98.6 degrees F, BP 150/90 mmHg, P 110 bpm, RR 20. Physical examination shows her right leg swollen with pain on palpation of deep veins. Which of the patient's history or examination findings is most suggestive of a pulmonary embolus (PE)? A. Leg swelling and pain with palpation of deep veins B. Heart rate > 100 C. Hemoptysis D. Past history of cancer

(c) A. Leg swelling and pain with palpation of the deep veins are consistent with a DVT and increase the likelihood of a PE. (u) B. While a tachycardic rate may be present in a patient with a PE, it is not specific for a PE. (u) C. While hemoptysis may be present in a PE, it is not a specific finding. (u) D. History of cancer may place a patient at increased risk for PE, however is not the most suggestive finding in this patient.

Which of the following causes of pneumonia is most likely to be complicated by diarrhea? A. Legionella B. Chlamydophila C. Mycoplasma D. Pneumococcal

(c) A. Legionella is a water-borne pathogen that can cause diarrhea as a complication of its infection. (u) B. Chlamydophila pneumonia is associated with dry cough, low grade fever, and hoarseness. (u) C. Mycoplasma pneumonia is most likely complicated by bullous myringitis, hoarseness, rash, and chronic cough. (u) D. Pneumococcal pneumonia presents with a single rigor, rust colored sputum, and productive cough. There is no diarrhea as part of its infection.

: (28) 186. Clinical Therapeutics/Pulmonology Which of the following is the treatment of choice for pulmonary Legionellosis? A. Azithromycin (Zithromax) B. Ceftriaxone (Rocephin) C. Vancomycin (Vancocin) D. Rifampin (Rifadin)

(c) A. Legionella is an intracellular pathogen and therefore requires a drug that can achieve adequate intracellular concentrations. Azithromycin, along with quinolones and tetracyclines, is considered first line therapy. Rifampin may be used in combination with first-line drugs but never as monotherapy. (u) B. See A for explanation. (u) C. See A for explanation. (u) D. See A for explanation.

A 58 year-old female comes to the office because she noticed a white spot on her tongue. She has a 42 pack/year smoking history. On examination, she has an 8 mm white thickened, keratinized lesion on the ventral tongue. The lesion cannot be removed by rubbing the mucosal surface. Which of the following studies is most appropriate to confirm the suspected diagnosis? A. Incisional biopsy B. Tzanck smear C. Gram stain D. Potassium hydroxide wet prep

(c) A. Leukoplakia is characterized as a white plaque that cannot be removed by wiping the mucosal surface. Tobacco use is strongly associated with an increased incidence of leukoplakia and oral cancer. An incisional biopsy of the lesion is required for histologic evaluation of the tissue. (u) B. Herpes simplex is a viral infection caused by the herpes simplex virus (HSV) group. Vesicles may be visualized on the soft palate, floor of mouth, and tongue. A Tzanck smear is taken from a vesicular lesion and stained for identification of herpes simplex. (u) C. A direct gram stain of a scraping/swab from oral cavity is performed on lesions due to suspected bacterial or yeast infections. (u) D. Oral candidiasis is characterized by creamy curd-like patches that can be easily rubbed off the mucosal surface. The diagnosis can be confirmed by wet preparation using potassium hydroxide which reveals pseudohyphae and yeast forms.

: (27) 59. Clinical Therapeutics/Neurology A 58 year-old male presents with a 4 month history of instability upon ambulation. Examination reveals resistance to passive movements, mild tremor of the lips and a shuffling gait. Which of the following medications is most appropriate for this patient? A. Levadopa/carbidopa (Sinemet) B. Benztropine mesylate (Cogentin) C. Clozapine (Clozaril) D. Risperidone (Risperdal)

(c) A. Levodopa/carbidopa generally helps reduce many symptoms associated with Parkinsonism, including 28 bradykinesia. (u) B. Benztropine mesylate is useful in the treatment of tremor and rigidity, and less effective on bradykinesia. (u) C. Clozapine is useful in treating the behavioral aspects of Parkinsonism. (u) D. Risperidone is useful in treating the behavioral aspects of Parkinsonism.

What scabicide has been associated with neurotoxicity in infants and young children? A. lindane (Kwell) B. crotamiton (Eurax) C. 10% sulfur ointment D. permethrin (Elimite)

(c) A. Lindane (Kwell) is concentrated in the CNS and toxicity from systemic absorption in infants has been reported. (u) B. Crotamiton (Eurax) is not associated with CNS toxicity and is an effective scabicide. Its primary side effects include dermatitis and conjunctivitis. (u) C. Sulfur ointment is no longer used because newer agents have been developed and is not neurotoxic. (u) D. Permethrin (Elimite) is the drug of choice for the treatment of scabies and is not associated with neurotoxicity.

A 46 year-old male with no past medical history presents complaining of chest pain for four hours. The patient admits to feeling very poorly over the past two weeks with fever and upper respiratory symptoms. The patient denies shortness of breath or diaphoresis. On examination the patient appears fatigued. Vital signs reveal a BP of 130/80, HR 90 and regular, RR 14. The patient is afebrile. Labs reveal a Troponin I of 10.33 ug/L (0-0.4ug/L). Cardiac catheterization shows normal coronary arteries and an ejection fraction of 40% with global hypokinesis. Which of the following is the most likely diagnosis? A. myocarditis B. pericarditis C. hypertrophic cardiomyopathy D. coronary artery disease

(c) A. Myocarditis often occurs secondary to acute viral illness and causes cardiac dysfunction. Patients will commonly have a history of a recent febrile illness. Chest pain may mimic that of a myocardial infarction and Troponin I levels maybe elevated in one-third of patients. Contractile dysfunction is seen on catheterization and/or echocardiogram. (u) B. Pericarditis does not typically cause ventricular dysfunction and cardiac enzymes are usually normal. (u) C. Hypertrophic cardiomyopathy is associated with ventricular hypercontractility. (u) D. This patient had normal coronary arteries on cardiac catheterization, no signs of coronary artery disease.

A 27 year-old male presents with gradually worsening low back pain and stiffness for the past two years. His symptoms are worse upon awakening and gradually improve throughout the day. Lumbosacral flexion is less than 50%. Lumbosacral spine films show erosions in the joint line of both sacroiliac joints. HLA-B27 test is positive. Which of the following is the most appropriate first-line medication for this patient? A. Indomethacin (Indocin) B. Methotrexate (Rheumatrex) C. Prednisone (Medrol dose pack) D. Sulfasalazine (Azulfidine)

(c) A. NSAIDs, such as indomethacin, are the mainstay of therapy in ankylosing spondylitis (AS). (u) B. Methotrexate is beneficial in treatment of rheumatoid arthritis, but there is little evidence that it changes the course of disease in ankylosing spondylitis. (u) C. Oral prednisone may be used sparingly in ankylosing spondylitis, but is less useful for this condition because of its association with osteoporosis. (u) D. Sulfasalazine is potentially useful in the treatment of spondyloarthropathies but studies suggest that it is minimally effective in patients with axial disease.

The first step in the treatment of a patient with an intestinal obstruction and no comorbid diseases is A. nasogastric decompression. B. invasive hemodynamic monitoring. C. abdominal exploration. D. administration of antibiotics.

(c) A. Nasogastric decompression is indicated in all but mild cases of obstruction to prevent distal passage of swallowed air and minimize distension. (u) B. Invasive hemodynamic monitoring is needed only if the patient has underlying cardiac, pulmonary, or renal disease. (u) C. Abdominal exploration is considered only if the obstruction does not resolve in 24 to 48 hours or if peritoneal findings, fever, or rapidly progressing abdominal pain occur. (u) D. Antibiotics are given only if surgery is to be done.

: (4) 200. History & Physical/Psychiatry/Behavioral Medicine A 35 year-old female with a recent diagnosis of depression with psychotic features has been placed on a serotonin re-uptake inhibitor and an atypical antidepressant. She presents to the office for a 2 week follow-up complaining of insomnia. She refuses to sit down, continuously paces and appears anxious. This patient is demonstrating which of the following? A. Akathisia B. Acute dystonia C. Catatonia D. Tardive dyskinesia

(c) A. Neuroleptic induced akathisia is a movement syndrome from neuroleptics that usually occurs in the beginning of treatment. Patients appear to be in constant motion and anxious. (u) B. Dystonia is an extra-pyramidal motor disturbance presenting as muscle spasms usually in the eyes, tongue, jaw, or neck usually from high potency neuroleptic drugs. (u) C. Catatonia may be seen in schizophrenia where the patient alternates from muscular rigidity and mental stupor. (u) D. A syndrome of involuntary movements that can involve the tongue and lips, hands, limbs and trunk. It usually develops after months to years on antipsychotic medications.

A female patient presents with weakness and fatigue for the past three months. A CBC was ordered and revealed the following values: RBC 3.8 X 10(6)mcL (4.7-6.1 x 10(6)mcL) HCT 27% (35-45%) Hgb 9.2 gms/dL (12-15.5 gms/dL) MCV 120 fL (80-100 fL) MCH 35 pg (26-34 pg) On physical examination which of the following would be a consistent finding for this type of anemia? A. Paresthesia of the hands and feet B. Hepatosplenomegaly C. Tachycardia D. Jaundice

(c) A. Neurological manifestation is the earliest type most commonly seen with megaloblastic anemia most commonly from vitamin B12 deficiency. (u) B. Hepatosplenomegaly and jaundice are commonly seen in hemolytic anemias. (u) C. This compensatory mechanism is common in any type of anemia due to the hypoxemic effects of the anemia. (u) D. See B for explanation.

: (7) 30. Clinical Therapeutics/Obstetrics/Gynecology Which of the following tocolytic agents works by inhibition of calcium influx? A. Nifedipine (Procardia) B. Terbutaline sulfate (Terbutaline) C. Indomethacin (Indocin) D. Ritodrine (Yutopar)

(c) A. Nifedipine, a calcium channel blocker, prevents calcium entry into muscle cells. It is commonly used for the treatment of preterm labor. (u) B. See A for explanation. (u) C. See A for explanation. (u) D. See A for explanation.

: (28) 195. History & Physical/Dermatology A patient with known pemphigus vulgaris presents to the office for evaluation after skin trauma. You note the superficial detachment of the skin in the area of trauma. This physical exam finding is known as which of the following? A. Nikolsy sign B. Wickham striae C. Koebner's phenomena D. Auspitz's sign

(c) A. Nikolsky sign is the superficial detachment of the skin after pressure or trauma to the affected area as seen in pemphigus. (u) B. See A for explanation. (u) C. See A for explanation. (u) D. See A for explanation.

A 30 year-old female presents with a one to two year history of daily bouts of mid-abdominal crampy pain and bloating that are briefly relieved with defecation. Loose to watery bowel movements occur four to five times daily. She denies any nocturnal symptoms. Previous laboratory studies, abdominal CT scans and a colonoscopy have all been normal. Attempted dietary changes, exercise and probiotics have provided no relief. What is the next step in treatment for this patient? A. Nortriptyline (Pamelor) B. Omeprazole (Prilosec) C. Polyethylene glycol (Miralax) D. Promethazine (Phenergan)

(c) A. Nortriptyline, and other tricyclic antidepressants (TCAs), is an appropriate choice for someone with diarrhea predominant irritable bowel syndrome due to their numerous mechanisms of action. (u) B. Omeprazole, a proton pump inhibitor, works on parietal cells in the stomach lining to reduce production of hydrochloric acid. Though beneficial for peptic ulcer and reflux disease it would provide no relief of symptoms in a patient with IBS. (u) C. Polyethylene glycol is commonly used in constipation predominant IBS but would probably worsen this patient's symptoms. (u) D. The anticholinergic properties of the promethazine might be helpful even though its somnolent effects would be significant and would not be superior to nortriptyline. Promethazine's strength is in nausea control and less often its antihistamine affects.

An increased incidence of breast cancer is associated with A. nulliparity. B. late menarche. C. trauma to the breast. D. early natural menopause

(c) A. Nulliparous women have a 1.5 times higher incidence of breast cancer than multiparous women. (u) B. Early, not late, menarche is associated with an increased risk of breast cancer. (u) C. Previous trauma to the breast is not associated with an increased incidence of breast cancer. (u) D. Late, not early, natural menopause is associated with an increased risk of breast cancer.

Which of the following pathophysiological processes is believed to initiate acute appendicitis? A. Obstruction B. Perforation C. Hemorrhage D. Vascular compromise

(c) A. Obstruction of the appendiceal lumen by lymphoid hyperplasia, a fecalith or foreign body initiates most cases of appendicitis. (u) B. See A for explanation. (u) C. See A for explanation. (u) D. See A for explanation.

Gallstones usually result in biliary symptoms by causing inflammation or obstruction following migration into the common bile duct or A. cystic duct. B. pancreatic duct. C. duodenal ampulla. D. common hepatic duct.

(c) A. Obstruction of the cystic duct by gallstones causes the typical symptom of biliary colic. Once obstructed the gallbladder distends and becomes edematous and inflamed. Gallstones can also migrate into the common bile duct through the cystic duct leading to a condition known as choledocholithiasis. (u) B. Obstruction of the pancreatic duct leads to development of acute pancreatitis. (u) C. The duodenal ampulla is the area where the pancreatic duct and the common bile duct empty into the duodenum. Gallstones do not cause obstruction at this distal site. (u) D. The common hepatic duct from the liver joins the cystic duct from the gallbladder to form the common bile duct. Stone migration occurs along the pathway of the cystic duct to the common bile duct, not along the common hepatic duct.

A 42 year-old male sustained a closed left tibial fracture in a fall two days ago. He was treated with a cast for immobilization. Acutely, he developed severe pain in his left leg. Examination reveals the anterolateral aspect of the leg to be exquisitely tender to palpation. The patient has extreme pain with plantar flexion. What is the most likely diagnosis? A. Compartment syndrome B. Deep venous thrombosis C. Osteomyelitis D. Complex regional pain syndrome

(c) A. One of the earliest signs of compartment syndrome is severe pain that occurs with extension of the involved muscles. (u) B. DVT below the knee usually occurs in the posterior compartment and, while potentially painful, should not produce the degree of pain described. (u) C. The pain of osteomyelitis, while potentially severe, should not be greatly exacerbated by manipulation of the overlying muscles. (u) D. Complex regional pain syndrome can present with pain but it is typically a burning pain and often accompanied by vasomotor symptoms.

: (3) 191. Diagnosis/Orthopedics/Rheumatology A 14 year-old male athlete presents complaining of a painful "bump" on his right lower extremity just below his knee and associated pain during and after activity. Examination of the right leg reveals a prominent tender tibial tubercle. Which of the following is the most likely diagnosis? A. Osgood-Schlatter disease B. Osteochondritis dissecans C. Tibia vara D. Patellofemoral syndrome

(c) A. Osgood-Schlatter disease is a painful swelling over the tibial tubercle, most commonly in a male adolescent athlete. (u) B. Osteochondritis dissecans occurs most commonly in the knee when a focal area of bone and adjacent cartilage are affected by vascular necrosis, causing separation. (u) C. Tibia vara, also known as Blount disease, is characterized by excessive varus deformity and excessive bowing of the legs. (u) D. Patellofemoral syndrome most commonly affects female adolescent athletes and is thought to be secondary to a small patella and slight exaggeration of knee valgus and quadriceps angle.

On rectovaginal examination of a 72 year-old post-menopausal female a 3 cm by 3 cm right adnexal mass is palpated. The rest of her physical examination is unremarkable. Her last gynecological examination was last year and was unremarkable. Which of the following is the most likely diagnosis? A. Ovarian cancer B. Endometrioma C. Uterine cancer D. Functional ovarian cyst

(c) A. Ovaries should not be palpable in the postmenopausal patient. Consider ovarian cancer until proven otherwise. (u) B. An endometrioma is an endometriosis cyst of the ovary. The cyst is filled with thick, chocolate-colored fluid, often called a "chocolate cyst". These cysts occur in menstruating women. (u) C. 90% of patients with endometrial cancer present with vaginal bleeding. The uterus may be enlarged on examination, however, a palpable ovary is not associated with this condition. (u) D. Functional ovarian cysts occur in women who are ovulating.

: (8) 131. Clinical Intervention/Gastrointestinal/Nutritional A 45 year-old female presents with bowel movements associated with bright red blood after being diagnosed with a stage II hemorrhoid 1 month ago. She has used increased fiber and fluid intake as her primary treatment up to this point. Which of the following is the most appropriate next step in managing this patient's condition? A. Rubber band ligation B. Hemorrhoidectomy C. Diltiazem (Cardizem) therapy D. Iron supplementation

(c) A. Patients who have a stage I, II or III hemorrhoid with recurrent bleeding not responsive to conservative therapy should have injection sclerotherapy, rubber band ligation or application of electrocoagulation. Rubber band ligation is easy to use and has a high efficacy rate. (u) B. Hemorrhoidectomy is indicated in patients with a stage III or IV hemorrhoid with chronic bleeding or a stage II hemorrhoid that is acutely thrombosed. (u) C. Diltiazem is not indicated in the management of hemorrhoids and may exacerbate the patient's symptoms by causing constipation. (u) D. There is rarely significant bleeding secondary to hemorrhoid disease that necessitates iron supplementation. Additionally, this will not treat the underlying disorder.

: (8) 45 108. Clinical Therapeutics/Cardiology You are discharging a patient who had a myocardial infarction with a post infarction ejection fraction of 35%. You place the patient on the following medications: atorvastatin (Lipitor), metoprolol (Toprol-XL), aspirin, and nitroglycerin spray. Which of the following is also indicated in the patient provided there are no contraindications? A. Enalapril (Vasotec) B. Felodipine (Plendil) C. Furosemide (Lasix) D. Valsartan (Diovan)

(c) A. Patients who sustain substantial myocardial damage often experience subsequent progressive LV dilation and dysfunction, leading to clinical heart failure and reduced long-term survival. In patients with ejection fractions less than 40% long-term ACE inhibitor therapy prevents LV dilation and the onset of heart failure and prolongs survival. (u) B. See A for explanation. (u) C. See A for explanation. (u) D. See A for explanation.

A patient was treated for community acquired pneumonia with amoxicillin-clavulanate (Augmentin). On day 7 of therapy he develops fulminate diarrhea. The diarrhea is described as greenish and foul-smelling. He admits to associated abdominal cramps. Which of the following is the treatment of choice for this patient? A. Metronidazole (Flagyl) B. Diphenoxylate/atropine (Lomotil) C. Clindamycin (Cleocin) D. Ciprofloxacin (Cipro)

(c) A. Patients with C. difficile colitis should be treated with Flagyl for 10-14 days following cessation of the diarrhea-inducing antibiotics. (u) B. See A for explanation. (u) C. See A for explanation. (u) D. See A for explanation.

Endotracheal intubation should be performed with caution in patients with which of the following underlying conditions due to the propensity to cause subluxation of C1 on C2? A. Rheumatoid arthritis B. Osteoarthritis C. Gout D. Pseudogout

(c) A. Patients with advanced rheumatoid arthritis will have synovitis of the atlantoaxial joint (C1-C2) which may damage the transverse ligament of the atlas, producing forward displacement of the atlas on the axis (atlantoaxial subluxation). (u) B. Although patients with osteoarthritis may have neck pain and stiffness, there is no predilection for the atlantoaxial joints. (u) C. Patients with gout are likely to have involvement of peripheral joints rather than spinal joints. (u) D. Patients with pseudogout are more likely to have involvement of the knees, wrist, shoulder, ankle, elbow, and hands rather than the cervical spine.

Which of the following leads to retropatellar pain? A. increased Q angle B. increased quadriceps tone and strength C. Osgood-Schlatter Disease D. hamstring stretching

(c) A. Patients with an increased Q angle have more force directed laterally during knee flexion and are at greater risk of patellofemoral pain syndrome. (u) B. Quadriceps strengthening aids in restoring patellofemoral tracking. (u) C. Osgood-Schlatter disease involves the tibial tubercle, not the patellofemoral joint. (u) D. Hamstring stretching aids in restoring patellofemoral tracking.

A 45 year-old female presents with a ten pound weight loss and recurrent greasy stools mixed with diarrhea. The patient notes that these symptoms are worse with certain foods. Which of the following laboratory tests should initially be ordered? A. anti-endomysial antibodies B. anti-mitochondrial antibodies C. anti-glomerular basement membrane antibodies D. anti-phospholipid antibodies

(c) A. Patients with celiac sprue disease are likely to have anti-endomysial antibody formation. (u) B. Anti-mitochondrial antibodies are present with hypothyroid disease and other autoimmune disease processes. (u) C. Anti-glomerular basement disease occurs in the kidney and lungs and these antibodies are positive in patients with Goodpasture's disease. (u) D. Anti-phospholipid antibodies are seen in patients who have systemic lupus erythematosus.

A 3 year-old presents with a 24-hour history of diarrhea. The patient is afebrile and the stool is noted to be loose and watery. No blood is noted in the stool. Fecal WBC is negative. Which of the following is the most likely diagnosis? A. viral gastroenteritis B. toxic megacolon C. ulcerative colitis D. intussusception

(c) A. Patients with viral gastroenteritis are afebrile and noted to have loose, watery, non-bloody diarrhea. Fecal WBC is negative. (u) B. Toxic megacolon is a complication of ulcerative colitis and presents with diarrhea, fever, tachycardia, and leukocytosis. (u) C. Ulcerative colitis presents with bloody diarrhea, abdominal pain, and fever. (u) D. Intussusception is more common in children age 1-2 years and presents with crampy abdominal pain, vomiting, and currant jelly stool.

Annual blood pressure determinations should be obtained beginning at the age of A. 3 years. B. 5 years. C. 12 years. D. 18 years..

(c) A. Periodic measurements of blood pressure should be part of routine preventive health assessments beginning at the age of 3 years. (u) B. See A for explanation. (u) C. See A for explanation. (u) D. See A for explanation

Which of the following treatments of constipation should be used with extreme caution in patients who have chronic renal insufficiency? A. milk of magnesium B. psyllium (Metamucil) C. docusate sodium (Colace) D. lactulose (Chronulac)

(c) A. Patients with chronic renal insufficiency have difficulty excreting magnesium and hypermagnesemia almost always occurs in a patient with chronic renal insufficiency. (u) B. Psyllium is a fiber rich product that helps to bulk up the stool. It can be safely used in the constipated renal patient. (u) C. Docusate sodium is a stool softener that does not accumulate in the patient with renal insufficiency so it can be safely used. (u) D. Lactulose is an agent that is an unabsorbable sugar that can be safely used to treat constipation in the patient with chronic renal insufficiency.

Which of the following types of infection is most likely to benefit from hyperbaric oxygen therapy? A. Clostridial infection B. Group A beta hemolytic streptococcus C. Serratia marcescens D. Methicillin resistant Staphylococcus aureus

(c) A. Patients with clostridium myonecrosis (gas gangrene) will benefit from hyperbaric oxygen therapy, penicillin therapy, and radical surgical excision. (u) B. Patients with group A beta hemolytic strep, staph aureus, and community acquired methicillin resistant staph aureus will benefit from antibiotic therapy but these organisms are not anaerobic so hyperbaric oxygen therapy is not useful. (u) C. See B for explanation. (u) D. See B for explanation.

A 24 year-old female presents complaining of palpitations described as occasional "skipped" beats. The patient denies chest pain, lightheadedness, syncope, or dyspnea. On examination you note a midsystolic click without murmur. Which of the following is the most likely diagnosis in this patient? A. Mitral valve prolapse B. Aortic stenosis C. Atrial septal defect D. Pulmonic stenosis

(c) A. Patients with mitral valve prolapse will often present with complaint of palpitations. Auscultation would reveal a mid-systolic click with or without a late systolic murmur. (u) B. Aortic stenosis presents with a systolic murmur with no click. Patients may complain of chest pain, dyspnea or syncope. (u) C. Atrial septal defect is not associated with a midsystolic click. (u) D. Pulmonic stenosis is not associated with a midsystolic click.

A 55 year-old non-smoking male presents with a hemoglobin of 18.5 g/dl and a hematocrit of 56%. Which of the following physical examination findings is the most likely to be noted with this patient? A. splenomegaly B. cheilosis C. purpura D. decreased vibratory sense

(c) A. Patients with polycythemia vera present with elevated hemoglobin and hematocrit. On physical examination plethora, engorged retinal veins, and splenomegaly are common. (u) B. Cheilosis is noted in iron deficiency anemia. (u) C. Purpura is typically noted in bleeding disorders. (u) D. Decreased vibratory sense is noted in vitamin B12 deficiency.

Patients with primary adrenal insufficiency will have which of the following electrolyte abnormalities? A. Hyperkalemia B. Hypernatremia C. Hypercalcemia D. Hyperosmolality

(c) A. Patients with primary adrenal insufficiency will not have the adrenal gland producing cortisol, aldosterone, and the sex hormones. As a result of the lack of the mineralocorticoid aldosterone, the kidney will not save sodium and will instead save potassium. (u) B. Patients with primary adrenal failure will lack production of aldosterone and will not be able to save sodium. (u) C. Patients with hyperparathyroidism will have an increase in the serum calcium level as a result of the parathyroid gland retaining calcium. (u) D. Increased serum osmolality is the result of diabetes insipidus which results in a dilute urine being produced with resultant increase in the serum osmolality, as compared to the urine osmolality.

Which of the following therapies is recommended for a 13 month-old child with sickle cell disease? A. Folic acid and penicillin V B. Ferrous sulfate and penicillin V C. Folic acid and ferrous sulfate D. Folic acid, ferrous sulfate and penicillin V

(c) A. Patients with sickle cell disease should receive prophylactic penicillin V starting at 2 months of age and folic acid starting at 1 year of age. Ferrous sulfate is not globally recommended for patients with sickle cell disease. (u) B. See A for explanation. (u) C. See A for explanation. (u) D. See A for explanation.

A 68 year-old patient presents after a syncopal episode. The patient has a history of coronary artery disease and ischemic cardiomyopathy. Echocardiogram shows an ejection fraction of 20%. Electrophysiology study reveals inducible sustained ventricular tachycardia from the left ventricle. Which of the following is the most appropriate therapy in this patient? A. implantable defibrillator B. metoprolol (Lopressor) C. radiofrequency ablation D. warfarin (Coumadin)

(c) A. Patients with symptomatic ventricular tachycardia (VT) or sustained VT and left ventricular dysfunction are at increased risk for sudden cardiac death. An implantable defibrillator is the treatment of choice. (u) B. Beta blockers are used in patients with nonsustained VT and normal ventricular function. They may be used as an adjunct to, but not in place of, implantable defibrillator therapy in patients with symptomatic VT or sustained VT. (u) C. Radiofrequency ablation is indicated in patients with outflow tract or fascicular tachycardia, not left sided VT. (u) D. Anticoagulation therapy is indicated in patients with atrial fibrillation not VT.

An infant born at 30 weeks' gestation begins to have respiratory difficulty shortly after birth. Examination reveals rapid, shallow respirations at 80 per minute with associated intercostal retractions, nasal flaring and progressive cyanosis. Chest x-ray reveals the presence of air bronchograms and diffuse bilateral atelectasis. Which of the following is the most likely diagnosis? A. Respiratory distress syndrome B. Spontaneous pneumothorax C. Transient tachypnea syndrome D. Meconium aspiration syndrome

(c) A. Respiratory distress syndrome (hyaline membrane disease) is the most common cause of respiratory distress in a premature infant. This diagnosis is supported by the chest x-ray findings of air bronchograms and diffuse bilateral atelectasis, causing a ground-glass appearance. (u) B. Although spontaneous pneumothorax will present with respiratory distress at birth, the chest x-ray would reveal findings of lung collapse. (u) C. While transient tachypnea syndrome also may present at birth with respiratory distress, the chest x-ray would reveal findings of increased pulmonary vasculature markings, perihilar streaking and fluid in the interlobular fissures. (u) D. Meconium aspiration syndrome usually occurs in term or post-term infants. Typical chest x-ray findings include patchy infiltrates, coarse streaking of both lung fields, increased anteroposterior diameter and flattening of the diaphragm.

A 53 year-old female massage therapist presents with new onset of sudden swelling involving the right elbow. She denies previous episodes similar to this. On physical examination, the patient is afebrile. There is a 4 cm fluid-filled mass that is tender to palpation overlying the tip of the elbow with no evidence of erythema or warmth. Which of the following is the most appropriate intervention? A. Rest and NSAIDs B. Surgical excision C. Incision and drainage D. Aspiration and corticosteroid injection

(c) A. Rest and NSAID is the most appropriate initial intervention in a patient with olecranon bursitis. (u) B. Surgical excision is reserved for chronic bursitis and is not indicated in this acute initial presentation. (u) C. Incision and drainage is not recommended as it may cause a chronic drainage sinus tract. (u) D. Aspiration of the bursa and corticosteroid injection are second-line therapy in a patient with olecranon bursitis who fails rest and NSAIDs.

Patient education for a 23 year-old using oral contraceptives should include which of the following? A. Rifampin may decrease the effectiveness of the oral contraceptives. B. Acetaminophen may decrease the effectiveness of the oral contraceptives. C. Oral contraceptives may provide some protection from coronary artery disease. D. Changing to the "minipill" (progestin only) will inhibit ovulation more consistently than combination oral contraceptives.

(c) A. Rifampin may interfere with the efficacy of the oral contraceptives. (u) B. Acetaminophen levels or effects may be decreased by oral contraceptives. (u) C. Coronary artery disease is a contraindication to the use of oral contraceptives. (u) D. Progestin only oral contraceptives are less effective at inhibiting ovulation than the combination oral contraceptive.

A patient presents with a rash six days after returning from a camping and hiking trip in the woods. The rash is macular and first appeared on the ankles and then the rest of the lower extremities. On exam the physician assistant notes lesions on the soles of the feet and the trunk. Which of the following is the most likely diagnosis? A. Rocky mountain spotted fever B. typhoid fever C. Lyme disease D. Q fever

(c) A. Rocky mountain spotted fever presents with a macular rash on the wrists, ankles, extremities, and trunk. After 5 days the rash appears on the palms and soles. (u) B. The rash of typhoid fever is a faint, salmon-colored, maculopapular rash. The rash is noted primarily on the trunk and chest. The palms and soles are spared. (u) C. Lyme disease presents with a red macule or papule rash which expands slowly with central clearing at the site of the tick bite. Palms and soles are spared. (u) D. Cattle, sheep, and goats transmit Q fever. The rash is non-specific and not seen on the palms or soles.

A 9 year-old boy comes to the office with his mother to discuss treatment for chronic dry skin, and pruritic inflammatory lesions of the flexor surfaces of the neck, hands and wrists. Past medical history is significant for allergic rhinitis in the spring and fall. On physical examination the skin lesions are excoriated, and lichenified with crusted patches. Which of the following prevention strategies can help minimize the symptoms in this patient? A. Avoid rubbing or scratching B. Limit sun exposure C. Take hot baths D. Consume hot liquids

(c) A. Rubbing or scratching plaques can exacerbate the pruritis and lichenification in atopic dermatitis. (u) B. UVA-UVB phototherapy is an effective treatment for atopic dermatitis. (u) C. Hot baths can exacerbate the pruritis in atopic dermatitis. Tepid baths followed by use of unscented emollient creams can help prevent itching and scratching. (u) D. In rosacea, the facial flushing and increased skin temperature often occurs in response to the stimulation of hot liquids in the mouth.

A 66 year-old woman with type 2 diabetes comes to the office because she has had a painful red patch that has been spreading on her right lower leg over the past 3 days. Her temperature is 100.4 degrees F. Physical examination of her right lower leg reveals a 3 cm tender, warm, erythematous, and edematous plaque. Which of the following pathogens is most likely causing this patient's symptoms? A. Staphylococcus aureus B. Haemophilus influenzae C. Proteus mirabilis D. Escherchia coli

(c) A. S. aureus and group A beta-hemolytic streptococci are the most common causes of cellulitis. (u) B. Haemophilus influenzae can cause facial cellulitis in young children. (u) C. Proteus can be one of the pathogens that cause infectious gangrene. (u) D. Escherchia coli can cause crepitant cellulitis. It would not be the most common cause of cellulitis in this patient.

A patient presents with profound itching. Examination reveals short, reddish lesions on the wrists, elbows, and finger webs. Papules are also noted in these areas. There appears to be burrow marks emanating proximal to the finger webs. Which of the following is the most likely diagnosis? A. Scabies B. Body lice C. Rocky Mountain Spotted Fever D. Lyme Disease

(c) A. Scabies is an infestation of a mite that is usually spread by skin to skin contact. Patients present with intractable pruritus, often with minimal cutaneous findings. There may be an associated inflammatory papule or nodule along with a burrow that is sometimes seen early in the course of the infestation. (u) B. Body lice or pediculosis pubis is an infestation of hair-bearing regions, most commonly in the pubic area but may also be seen on hairy parts of the chest, axillae, and upper eyelashes. It is characterized by mild to moderate pruritus, papular urticaria, and excoriation. (u) C. Patients develop Rocky Mountain Spotted Fever after a tick bite. It is classically associated with a sudden onset of fever, severe headache, myalgia, and a characteristic acral exanthem. (u) D. Lyme disease is associated with an oval shaped skin lesion with central clearing known as erythema chronicum migrans. It occurs following a tick bite and may be associated with Bell's palsy, arthralgias, and heart block.

A 20 year-old male presents with pain along the medial tibia. The pain initially began towards the end of soccer practice but now it is present earlier on during practice. Physical exam reveals pain to palpation over the posterior tibialis muscle body. What is the most likely diagnosis? A. shin splint B. stress fracture C. Osgood-Schlatter disease D. patellofemoral pain syndrome

(c) A. Shin splints cause pain over the posterior tibialis muscle body as opposed to discrete pain over the tibia with a stress fracture. (u) B. See A for explanation. (u) C. Osgood-Schlatter disease is an injury occurring at the insertion of the patellar tendon on the tibial tuberosity in a younger age group. (u) D. Patellofemoral pain syndrome is the most common cause of chronic anterior knee pain, more commonly seen in females.

Which of the following is most frequently associated with bladder cancer? A. Hematuria B. Dysuria C. Urgency D. Frequency

(c) A. Significant persistent hematuria >3 RBC/HPF on three urinalyses, a single urinalysis with >100 RBC, or gross hematuria, identifies significant renal or urologic lesions. Bladder cancer usually presents with painless hematuria. (u) B. Dysuria, urgency, and frequency are associated with irritative voiding symptoms associated with cystitis. (u) C. See B for explanation. (u) D. See B for explanation.

An elderly appearing adult male patient is transported to the emergency room with unconsciousness for an underdetermined amount of time. There is no family and the only history is provided by the paramedics. The patient arouses to verbal and painful stimuli. VS: T-97.0 degrees F rectally, P-52 bpm, R-10, BP-95/60 mmHg. Physical examination is unremarkable except for ecchymosis across his extremities. A Foley catheter is inserted draining a small amount of dark brown urine. Urine dipstick reveals 4+ positive hemoglobin and protein. Microscopic urinalysis reveals no RBCs but many renal tubular epithelial cells and renal tubular casts. Drug screen is negative, blood alcohol is 2.5 mg/dL, and creatinine is 4.9 mg/dL. What is the most likely diagnosis? A. Rhabdomyolysis causing acute renal failure B. Obstructive uropathy causing acute renal failure C. Ethanol ingestion causing acute renal failure D. Methanol ingestion causing acute renal failure

(c) A. Since the patient was found unconscious for an undetermined amount of time and the blood alcohol is elevated the patient has been in a state of prolonged immobilization resulting in muscle ischemia resulting in myoglobinuria. This is responsible for turning the dipstick positive without the RBCs seen on the urinary microscopy. The myoglobin causes an acute tubular necrosis resulting in the sloughing of the renal tubular epithelium. Obstructive uropathy does not cause acute tubular necrosis and occurs over time. Methanol ingestion causes visual symptoms, ethylene glycol causes renal failure. (u) B. See A for explanation. (u) C. See A for explanation. (u) D. See A for explanation.

Which of the following would you expect on physical examination in a patient with mitral valve stenosis? A. Systolic blowing murmur B. Opening snap C. Mid-systolic click D. Paradoxically split S2

(u) A. Mitral stenosis is a diastolic, not a systolic murmur. (c) B. Mitral stenosis is characterized by a mid-diastolic opening snap. (u) C. Mid-systolic clicks are noted in mitral valve prolapse, not mitral stenosis. (u) D. Paradoxical splitting of S2 occurs in aortic stenosis not mitral stenosis.

A 60 year-old patient with COPD characteristic of emphysema presents with a cough and increased sputum production. The following information is noted: Temperature 100°F (37.8°C); Respiratory rate 20/min; Heart rate 88 beats/min; pH 7.44; PaO2 75 mmHg; PaCO2 40 mmHg; O2 saturation 92%. Physical examination is remarkable for increased AP diameter, diminished breath sounds without wheezes, rhonchi, or other signs of respiratory distress. Which of the following would be an appropriate treatment for this patient? A. Broad-spectrum antibiotic B. Admission to the hospital C. Oxygen at 6 L/min by nasal cannula D. Brief course of oral theophylline

(c) A. Sputum production is extremely variable from patient to patient, but any increase in sputum with a history of COPD reported by a patient must be regarded as potentially infectious and treated promptly. (u) B. Admission is only warranted if the patient's respiratory status requires ventilatory assistance. This patient's blood gases are unremarkable for a patient with COPD and the patient is not in respiratory distress. (u) C. Oxygen therapy should only be used for severe hypoxemia and should only be given at a low concentration, such as 2 L/min. Higher dose oxygen may stop the hypoxemic ventilatory drive. (u) D. Oral theophylline is considered a secondary bronchodilator. The use of a metered-dose inhaler would be a preferable first-line treatment if this method of treatment were chosen.

A child has been under treatment for attention-deficit hyperactivity disorder (ADHD). No response has occurred with behavioral adaptations. Which of the following categories of medication should this patient be given? A. stimulants B. anxiolytics C. antipsychotics D. antidepressants

(c) A. Stimulants, such as methylphenidate (Ritalin), are effective in 50 to 80% of children with ADHD. (u) B. Anxiolytic agents, such as alprazolam (Xanax), are used in treating anxiety disorders. (u) C. Antipsychotics, such as chlorpromazine (Thorazine), are used in treating psychosis. (u) D. Antidepressants, such as amitriptyline (Elavil), are used to treat depression.

Dental caries are caused by which of the following organisms? A. Streptococcus mutans B. Streptococcus pyogenes C. Staphylococcus epidermidis D. Staphylococcus aureus

(c) A. Streptococcus mutans is the principle organism that helps to demineralize the enamel. (u) B. See A for explanation. (u) C. See A for explanation. (u) D. See A for explanation.

: (5) 76. Scientific Concepts/Urology/Renal Which of the following types of kidney stones occur secondary to infections due to urease-producing bacteria? A. Struvite B. Uric acid C. Calcium oxalate D. Cystine

(c) A. Struvite stones are synonymous with magnesium-ammonium-phosphate stones and are commonly seen in women with recurrent urinary tract infections recalcitrant to appropriate antibiotics. These stones form secondary to urease-producing organisms including Proteus, Pseudomonas, and Providencia, although not E coli. (u) B. See A for explanation. (u) C. See A for explanation. (u) D. See A for explanation.

A 78 year-old male with history of coronary artery disease s/p coronary artery bypass grafting, hypertension, and dyslipidemia presents for routine physical examination. He feels well except for occasional brief episodes of substernal chest pain with exertion that are relieved with rest. He denies associated dyspnea, nausea or diaphoresis. Physical examination reveals a BP of 110/70 mmHg, HR 56 bpm, regular, RR 14, unlabored. Lungs are clear to auscultation, heart is bradycardic, but regular with no S3, S4 or murmur. Electrocardiogram done in the office shows no acute ST-T wave changes. Which therapy is indicated for the acute management of this patient's symptoms? A. Sublingual nitroglycerine B. Metoprolol (Lopressor) C. Verapamil (Calan) D. Lisinopril (Zestril)

(c) A. Sublingual nitroglycerine is the drug of choice for the acute management of chronic stable angina. (u) B. Beta-blockers are preventative and not the first choice for the acute management of chronic stable angina. Beta-blockers may worsen this patient's bradycardia. (u) C. Calcium channel blockers are the third-line antiischemic agent and may also reduce the patient's heart rate. (u) D. ACE inhibitors will not provide acute relief of anginal symptoms.

Which of the following oral hypoglycemic agents when used as monotherapy is most likely to cause hypoglycemia? A. Glipizide (Glucotrol) B. Metformin (Glucophage) C. Pioglitazone (Actos) D. Acarbose (Precose)

(c) A. Sulfonylureas increase insulin levels and predispose patients to hypoglycemia. (u) B. (u) C. (u) D. Acarbose is an alpha glucosidase inhibitor that delays the absorption of carbohydrates in the diet. It does not cause hypoglycemia since it does not drive insulin into the cells.

A 14 year-old male presents with complaint of worsening sore throat for two weeks. He now complains of fever, difficulty swallowing, and difficulty opening his mouth. The patient's mother states his voice seems muffled. On examination his left tonsil is bulging and the uvula is displaced to the right. Which of the following is the most appropriate management? A. Needle aspiration B. Corticosteroid administration C. Nebulized epinephrine administration D. Nasotracheal intubation

(c) A. Surgical drainage by needle aspiration and antibiotic therapy is the treatment of choice for peritonsillar abscess. (u) B. Corticosteroids, nebulized epinephrine, and nasotracheal intubation are not indicated in the treatment of peritonsillar abscesses. (u) C. See B for explanation. (u) D. See B for explanation.

Which of the following clinical manifestations is commonly seen in a patient with rheumatoid arthritis? A. symmetric joint swelling B. presence of Heberden's nodes C. morning stiffness lasting less than 15 minutes D. cervical spondylosis of C3-C7

(c) A. Symmetric joint swelling associated with stiffness, warmth, tenderness, and pain are characteristic of rheumatoid arthritis. (u) B. Heberden's nodes, or bony enlargements of the DIP joints, are seen in osteoarthritis, not rheumatoid arthritis. (u) C. Morning stiffness lasting less than 15 minutes is more characteristic of osteoarthritis. The morning stiffness typically seen with rheumatoid arthritis lasts longer than 1 hour and is a distinguishing feature between the two types of arthritis. (u) D. Cervical spondylosis occurs with osteoarthritis. RA is associated with C1-C2 subluxation.

A 58 year-old patient presents with spells of dizziness which is described as a spinning sensation. This has occurred several times a day for the last month. The patient also complains of some mild hearing loss, fullness, and a blowing sound in the right ear. Which of the following is the most likely diagnosis? A. Meniere's syndrome B. Labyrinthitis C. Benign paroxysmal positioning vertigo D. Vestibular neuronitis

(c) A. The classic findings of Meniere's syndrome consists of episodic vertigo, with discrete vertigo spells lasting 20 minutes to several hours in association with fluctuating low-frequency sensorineural hearing loss, tinnitus, and a sensation of aural pressure. (u) B. Labyrinthitis is an acute onset of continuous, usually severe vertigo lasting several days to a week, accompanied by hearing loss and tinnitus. (u) C. Benign paroxysmal positioning vertigo is a type of vertigo associated with changes in head position, often rolling over in bed. (u) D. Vestibular neuronitis is a paroxysmal, usually single attack of vertigo that occurs without accompanying impairment of auditory function and will persist for several days to weeks before clearing.

A 53 year-old female who is well known to the practice presents to the office complaining of increasing fatigue, constipation, and a weight gain of 10 lb (4.5 kg) over the past year. She also states others have noticed a recent hoarseness to her voice, and she is bothered by "charley horses" in her legs that wake her up at night. Her past medical history is unremarkable except for a history of hyperthyroidism treated by radioactive iodine 5 years ago. She is currently taking no medications and has no known drug allergies. Which of the following is the most likely cause of the patient's symptoms? A. Hypothyroidism B. Hypoparathyroidism C. Vocal cord paralysis D. Radiation thyroiditis

(c) A. The current symptoms, along with the past treatment of hyperthyroidism with radioactive iodine, would indicate hypothyroidism. (u) B. This is a possible later complication of subtotal thyroidectomy, not radioactive iodine therapy. (u) C. This is an immediate complication of subtotal thyroidectomy or injury and does not occur with radioactive iodine therapy. (u) D. Radiation thyroiditis may occur following radiation therapy but there is no history of the patient having previous external beam radiation therapy.

A 13 year-old girl reports two weeks of worsening right knee pain with no history of antecendent injury or recent trauma. She reports frequent episodes of nighttime awakening with knee pain in the past two weeks. Examination of the knee reveals edema and a tender mass over the anterior proximal right tibia. Her knee exam is otherwise within normal limits. Radiographs of the right knee show a lytic mass with a multi-laminated periosteal reaction involving the proximal anterior tibia. What is the most likely diagnosis? A. Ewing sarcoma B. Osteochondroma C. Multiple myeloma D. Osteoid osteoma

(c) A. The distinctive feature of Ewing sarcoma is the radiographic appearance of a periosteal "onion skin" reaction. (u) B. This benign tumor typically presents as a painless mass and appears in plain film radiographs as a stalk or broad-based projection from the surface of the bone. (u) C. The classic radiographic appearance of multiple myeloma is a lytic lesion but this is a condition that is seen in a much older population and is more likely to present with back pain. (u) D. Although the presentation may be similar to Ewing's sarcoma, the radiographs in osteoid osteoma typically show a round lucency surrounded by sclerotic bone.

A 40 year-old male nonsmoker in good health undergoes a routine chest x-ray for an insurance physical. Results show an isolated, well-defined, coin lesion 1 cm in size. Which of the following is the next step in the evaluation of this problem? A. Review old radiographs B. Order chest CT C. Schedule lung biopsy D. Prepare for surgical lung resection

(c) A. The first and most important step in the radiographic evaluation is to review old radiographs to estimate doubling time, an important marker for malignancy. (u) B. See A for explanation. (u) C. See A for explanation. (u) D. See A for explanation.

A 19 year-old female presents with a sore throat for nearly two weeks. She complains of fatigue and a low-grade fever. On physical examination, there is cervical, axillary, and inguinal lymphadenopathy, and mild splenomegaly. On review of the blood smear, which of the following would be expected? A. Atypical lymphocytes B. Hypersegmented neutrophils C. Hypochromic red blood cells D. Schistocytes

(c) A. The hallmark of infectious mononucleosis is the presence of lymphocytosis with atypical large lymphocytes seen in the blood smear. These are larger than normal mature lymphocytes, stain more darkly, and frequently show vacuolated, foamy cytoplasm, and dark chromatin in the nucleus. (u) B. Hypersegmented neutrophils are seen in vitamin B12 deficiency. (u) C. Anemia, if seen in mononucleosis, is normocytic and normochromic. (u) D. Schistocytes are noted in hemolytic anemias.

An 18 year-old female presents to the office with fever, fatigue and sore throat. Physical examination reveals an erythematous pharynx, cervical lymphadenopathy and splenomegaly. CBC reveals an increased white blood cell count with atypical lymphocytes, normal hemoglobin and hematocrit and normal platelet count. What additional laboratory test will help you make the diagnosis in this patient? A. Heterophile test B. Lymph node biopsy C. Serum antibody screening D. Serum transaminase

(c) A. The heterophile test is used for the diagnosis of infectious mononucleosis in children and adults. (u) B. Lymph node biopsy would be done if lymphoma was suspected in this patient. (u) C. Serum antibody screening could be done if one suspected Rubella or toxoplasmosis as the diagnosis in this case. (u) D. Serum transaminases often are elevated in a patient with infectious mono, however, they are not diagnostic for the disease.

A patient states that he has been camping in the mountains of North Carolina for the past two weeks. He presents to the clinic complaining of "flu-like" symptoms for the past 10-14 days however he notes that he started to develop a slight rash on his wrist and ankles about seven days ago. Which of the following tests would provide a confirmatory diagnosis? A. Immunofluorescent assay B. C-reactive protein C. Heterophile agglutination D. Anti-streptolysin O titer

(c) A. The immunofluorescent assay will confirm antibodies to Rickettsia. (u) B. C-reactive protein is a protein found in patients with inflammation during the acute phase illness. It is nonspecific. (u) C. Heterophile agglutination is used to detect antibodies to the Epstein-Barr virus which is responsible for infectious mononucleosis. (u) D. ASO titer is done to detect antibodies to streptococcal infection.

Which of the following interventions is the treatment of choice for actinic keratosis? A. Mohs surgery B. Cryotherapy C. Acid peels D. Radiation therapy

(u) A. Mohs surgery and radiation therapy are not indicated in the treatment for actinic keratosis. (c) B. Cryotherapy is the treatment of choice for isolated superficial actinic keratosis. (u) C. Acid peels can be used to treat actinic keratosis but are not the treatment of choice. (u) D. See A for explanation.

32 year-old female presents to the ED unconscious. Blood glucose obtained from a finger stick reveals the glucose to be 28 mg/dl. Further blood work obtained at IV line placement confirms the glucose finding. Immediate resuscitation is given with D50 and the patient responds appropriately. History taken from the patient is significant for episodes of feeling faint and weak over the past 2-4 weeks, she denies however any medical problems, or the use of any medications. Additional laboratory results from the blood taken in the ED reveal a serum insulin level to be 55 μu/ml (normal less than 22), C-peptide 5.4 ng/ml (normal 0.5-2.0), and proinsulin 1.0 ng/ml (normal 0-0.2). Given these results what is the most likely diagnosis? A. insulinoma B. exogenous insulin administration C. sulfonylurea usage D. glucagonoma

(c) A. The laboratory results confirm the diagnosis of an insulinoma by having an increase in the insulin level, increased C-peptide and proinsulin levels. (u) B. Exogenous insulin administration would be confirmed by the following laboratory results: Insulin levels are increased; C-peptide and proinsulin are decreased. C-peptide comes from the cleavage of proinsulin to insulin. Exogenous administration of insulin by-passes this process. (u) C. Sulfonylurea usage would be confirmed in the history, however the laboratory results would show an increase in the insulin level and C-peptide level but a normal level of the proinsulin. (u) D. Glucagonoma is extremely rare and in addition would present with symptomatology of diabetes mellitus by stimulating glycogenolysis and gluconeogenesis.

A 10 year-old boy was playing with sparklers (magnesium sulfate) and got some of the "sparkle" in his right eye. Which of the following is the most appropriate initial treatment? A. irrigate the eye for at least 20 minutes B. apply Bacitracin ointment and patch the eye C. remove the sparkle with a moistened cotton swab D. protect the eye with a metal shield and refer to an eye ophthalmologist

(c) A. The magnesium from the sparkler combines with tears, producing an alkaline injury and should be treated with prolonged irrigation. Irrigation should be the first step in management of this case. (u) B. See A for explanation. (u) C. See A for explanation. (u) D. See A for explanation.

Which of the following is the most important intervention in acute pancreatitis? A. IV fluid administration B. Antibiotic administration C. Calcium replacement D. Antiemetics

(c) A. The mainstay of management in acute pancreatitis is fluid resuscitation. Isotonic solutions are best to maintain renal perfusion and urine output > 100 ml/hour. (u) B. 90% of patients will recover with supportive measures only. Parenternal antibiotics are not indicated in acute pancreatitis. (u) C. Hypocalcemia is a marker of prognosis, but correction of hypocalcemia does not supercede the importance of fluid resuscitation from the overwhelming inflammatory response. (u) D. Other supportive measures in acute pancreatitis include pain management and antiemetics, but the mainstay is fluid resuscitation.

An elderly female presents for evaluation of exertional syncope, dyspnea, and angina. She admits that previous to these symptoms she had insidious progression of fatigue that caused her to curtail her activities. Which of the following is the most likely diagnosis? A. Aortic stenosis B. Aortic regurgitation C. Mitral stenosis D. Mitral valve prolapse

(c) A. The major symptoms of aortic stenosis are exertional syncope, dyspnea, and angina. Symptoms do not become apparent for a number of years and usually are not present until the valve is narrowed to less than 0.5 cm to 2 cm of valve surface area. (u) B. Patients with aortic regurgitation are likely to complain of an uncomfortable awareness of their heart, especially when lying down. These patients develop sinus tachycardia with exertion and complain of palpitations and head pounding with activity. (u) C. The symptoms related to mitral stenosis are related to increased pulmonary pressure after the left atrium can no longer overcome the outflow obstruction. (u) D. Patients with mitral valve prolapse are typically asymptomatic throughout their lives, although a wide range of symptoms is possible. When symptoms do occur, palpitations from arrhythmias are most common along with lightheadedness. Syncope is not part of this disease process.

Which of the following is the portion of the nephron responsible for the absorption of 90% of the ultrafiltrate? A. Proximal convoluted tubule B. Loop of Henle C. Distal convoluted tubule D. Collecting duct

(c) A. The majority of the ultrafiltrate 90% is reabsorbed in the proximal convoluted tubule. (u) B. The loop of Henle is responsible for the concentration of solutes within the nephron. (u) C. The distal convoluted tubule is responsible for some water and sodium reabsorption. (u) D. The collecting duct is responsible for the final concentration of the urine.

: (27) 96. Scientific Concepts/Orthopedics/Rheumatology Focal ulcerated areas of cartilage with bone eburnation, development of bone cysts and the presence of osteophytes best demonstrates the pathophysiology of which of the following? A. Osteoarthritis B. Osteomalacia C. Osteoporosis D. Osteonecrosis

(c) A. The pathophysiology of osteoarthritis is characterized by focal ulcerated areas of cartilage, bone eburnation, and the development of osteophytes. (u) B. Osteomalacia is characterized by defective bone and cartilage mineralization in children and defective bone 41 mineralization in adults. (u) C. The pathophysiology of osteoporosis is characterized by fractures secondary to low bone density which occurs from bone remodeling. (u) D. The pathophysiology of osteonecrosis is characterized by traumatic causes and nontraumatic causes. Traumatic causes include a direct injury to the blood supply from a hip fracture or subcapital femoral neck fracture while nontraumatic causes include alcohol and corticosteroid use as well as idiopathic causes that lead to necrosis of the bone.

: (27) 164. History & Physical/Endocrinology A 30 year-old female with diabetes mellitus type 1 presents for her annual exam with complaints of intermittent nausea with occasional vomiting, diarrhea and abdominal pain worsening over the past six months. She states she has been irritable lately and feels anxious today. Her menses ceased two months ago. On physical examination she is mildly hypotensive. Urine pregnancy test is negative. Which of the following are you also likely to find on her physical examination? A. Hyperpigmentation over the knuckles B. Increased axillary hair C. Hypopigmented skin lesions D. Weight gain

(c) A. The patient exhibits signs and symptoms consistent with Addison disease (chronic adrenal insufficiency) which can be a complication of diabetes. Skin examination tends to show hyperpigmentation in areas of non-sunexposed skin, as well as sun-exposed areas. This effect is particularly clear in areas of skin creasing such as knuckles, elbows, and posterior neck. (u) B. Axillary hair is sparse in patients with Addison disease. (u) C. See A for explanation. (u) D. Weight loss is consistent with Addison disease.

: (8) 23. History & Physical/Gastrointestinal/Nutritional A 68 year-old woman presents with intermittent crampy abdominal pain and vomiting of 6 hours' duration. She feels bloated but denies rectal bleeding. Examination of the abdomen reveals no palpable mass and you note an upper midline scar from previous abdominal surgery. An abdominal radiograph demonstrates dilated loops of small bowel with air-fluid levels. Which of the following is the most likely cause of this patient's symptoms? A. Adhesions B. Sigmoid volvulus C. Incisional hernia D. Intussusception

(c) A. The patient's surgical history, presenting symptoms and abdominal imaging are consistent with small bowel obstruction. Peritoneal adhesions are the most common cause of small bowel obstruction. (u) B. Sigmoid volvulus or twisting of the sigmoid colon on its mesentery leads to colonic obstruction. This patient does not complain of constipation or report laxative use. Abdominal radiographs would show a dilated colon lacking haustral folds. (u) C. Hernias may occur after abdominal surgery but are usually more chronic in nature and a mass would be palpable especially with maneuvers that increase intra-abdominal pressure. (u) D. Intussusception, although rare in adults, may occur due to small bowel neoplasia but this patient does not complain of bleeding and no mass was palpable on exam. Also a CT scan would usually show a target lesion representing the layers of intussuscepted segment.

: (8) 34. Diagnostic Studies/Endocrinology A patient presents 5 days status-post thyroidectomy with muscle cramping and altered mental status. On physical exam you elicit a positive Chvostek sign. Which of the following is the expected laboratory result for this condition? A. Decreased serum calcium B. Decreased serum phosphate C. Elevated parathyroid hormone D. Elevated serum calcium

(c) A. The patient's symptoms are consistent with hypoparathyroidism. Expected lab results include low serum calcium, high serum phosphate, low urine calcium, normal alkaline phosphatase, and low PTH levels. (u) B. See A for explanation. (u) C. See A for explanation. (u) D. See A for explanation.

A 75 year-old female presents with medial knee pain that worsens with stair climbing. Physical examination reveals swelling and point tenderness inferior and medial to the patella and tenderness overlying the medial tibial plateau. Which of the following is the most likely diagnosis? A. Pes anserine bursitis B. Prepatellar bursitis C. Infrapatellar bursitis D. Trochanteric bursitis

(c) A. The pes anserine bursa underlies the semimembranosus tendon and may become inflamed or painful owing to trauma, overuse, or inflammation. It is a common cause of knee pain and it is often misdiagnosed in adults. (u) B. Prepatellar bursitis causes swelling in the prepatellar area and is worse with kneeling. The prepatellar bursa is superficial and is located over the inferior portion of the patella. (u) C. The infrapatellar bursa is deeper and lies beneath the patellar ligament before its insertion on the tibial tubercle. It has a midline location rather than the medial surface as described in the question. (u) D. Trochanteric bursitis causes hip pain rather than knee pain.

A 45 year-old smoker presents with a sore mouth and increasing difficulty eating for two weeks. Physical examination reveals a 1 cm white lesion on the buccal mucosa that cannot be rubbed off. Which of the following is the most likely diagnosis? A. Oral cancer B. Oral candidiasis C. Aphthous ulcer D. Necrotizing ulcerative gingivitis

(c) A. The presence of leukoplakia in a smoker over the age of 40 should be biopsied to rule out the presence of oral cancer. (u) B. Oral candidiasis presents with white patches. Unlike leukoplakia, the patches easily rub off. (u) C. While aphthous ulcers are commonly found on the buccal mucosa, they are usually 1 to 2 mm round ulcerative lesions. (u) D. Necrotizing ulcerative gingivitis is common in young adults under stress. Clinically, it presents with painful acute gingival inflammation and necrosis.

A 35 year-old female presents with multiple ulcerative lesions on her labia and perineum. A Tzanck preparation of one of the lesions reveals multinucleated giant cells. Which of the following is the most likely diagnosis? A. Herpes Simplex Virus (HSV) B. Molluscum Contagiosum Virus (MCV) C. Human Papilloma Virus (HPV) D. Syphilis

(c) A. The presentation seen on the Tzanck preparation is characteristic of HSV. (u) B. See A for explanation. (u) C. See A for explanation. (u) D. See A for explanation.

A 75 year-old female falls on her outstretched arm. She sustains a humeral mid-shaft fracture. Nerve impingement occurs due to the fracture. What is the most likely physical examination abnormality that will be encountered? A. Inability to extend the wrist against resistance B. Numbness over the deltoid muscle in the shoulder C. Winging of the scapula D. Weakness of the rotator cuff

(c) A. The radial nerve is most likely entrapped by this fracture. Radial nerve damage will cause an inability to extend the wrist against resistance. (u) B. Axillary nerve injury results in numbness over the deltoid muscle; this nerve is more commonly injured in proximal humeral fractures and anterior shoulder dislocations. (u) C. Injury to the long thoracic nerve causes winging of the scapula due to its innervation of the serratus anterior muscle. (u) D. Injury to the subscapular nerve results in weakness and pain of the infraspinatus muscle; this injury is commonly seen in volleyball players from repetitive stress.

The most accurate way to determine the exact degree of spinal curvature in a child with scoliosis is by which of the following? A. Calculation of the Cobb angle B. Measurement of waist asymmetry C. Measurement of rib hump deformity D. Calculation using a scoliometer

(c) A. The scoliotic curve is measured by the Cobb method using AP and lateral x-ray films of the entire length of the spine. (u) B. While waist asymmetry and rib hump deformity may be observed on physical examination of the patient with scoliosis, none of them can be used to determine the exact degree of the spinal curvature. (u) C. See B for explanation. (u) D. A scoliometer or inclinometer measures distortions of the torso and is good for screening angle of rotation, but is not exact to determine exact degree of curvature.

During an influenza epidemic, a 6 year-old male is seen with fever and a severe sore throat. The parents report that his symptoms have not improved despite administration of aspirin. The next day, the parent calls to report that the child has persistent vomiting and increased lethargy. On examination, he is found to be delirious and disoriented with hyperactive reflexes. The liver edge is 3 cm below the right costal margin in the midclavicular line. Which of the following is the most likely diagnosis? A. Reye's syndrome B. Measles encephalitis C. Guillain-Barre syndrome D. Acute bacterial meningitis

(c) A. The suspected influenza associated with development of vomiting, progressive mental status changes, hyperreflexia, and hepatomegaly are consistent with a diagnosis of Reye's syndrome. (u) B. Measles encephalitis is a complication of rubeola, which is not suggested by this patient's presentation. (u) C. Fever is uncommon with Guillain-Barre, which is also characterized by ascending symmetrical weakness of the lower extremities. (u) D. While acute bacterial meningitis might present with acutely evolving symptoms, hepatomegaly is not usually seen.

: (12) 147. Diagnostic Studies/Pulmonology A 7 year-old white male presents with a life-long history of frequent, and commonly complicated, bouts of bronchiectasis and other respiratory tract infections. His past history includes poor weight gain, low exercise endurance and pancreatitis. Examination reveals a barrel chest and hyperresonance to percussion while a mixed 59 obstructive-restrictive pattern is seen on pulmonary function testing. Peribronchial cuffing and increased interstitial markings are noted on chest radiograph. Which of the following tests would confirm the diagnosis? A. Sweat chloride test B. Purified protein derivative test C. Serum ACE level D. Alpha-1 antitrypsin level

(c) A. The sweat chloride test, though not the only diagnostic study, is a commonly ordered test to help solidify the diagnosis of cystic fibrosis. Purified protein derivative is used to assess for tuberculosis, serum ACE levels can be used to diagnose sarcoidosis and alpha-1 antitrypsin levels are checked for deficient states. (u) B. See A for explanation. (u) C. See A for explanation. (u) D. See A for explanation.

A 32 year-old medical transcriptionist presents with burning and tingling in her right wrist and hand for the past month. On physical exam, Phalen's test is positive; however, there is no atrophy of the thenar eminence. Which of the following is the initial step in management of this patient? A. wrist splints B. corticosteroid injection C. surgical referral D. propoxyphene (Darvocet)

(c) A. The treatment of carpal tunnel syndrome is aimed at relieving the pressure on the median nerve. This is best accomplished by having the patient wear a wrist splint during the activities that increase the pressure on the median nerve. (u) B. Corticosteroid injections and surgery are indicated only after a trial of the wrist splint provides no relief. (u) C. See B for explanation. (u) D. Darvocet has no role in the treatment of carpal tunnel syndrome.

A 50 year-old female with a history of coronary artery disease presents to the office requesting medication for an exacerbation of her migraine headaches. She complains of migraines approximately once a month. What medication is contraindicated in this patient? A. Sumatriptan (Imitrex) B. Propranolol (Inderal) C. Droperidol (Inapsine) D. Naproxen sodium (Anaprox)

(c) A. The triptans are contraindicated in patients with coronary artery disease or peripheral vascular disease and should be avoided in patients with an increased risk for stroke. All other drugs listed are not contraindicated. (u) B. See A for explanation. (u) C. See A for explanation. (u) D. See A for explanation.

Routine prophylaxis for patients going to an area of malaria infestation requires the administration of which chemoprophylaxis agent? A. Chloroquine B. Ciprofoxacin C. Tetracycline D. Erythromycin

(c) A. The use of Chloroquine is still recommended as the drug of choice for prophylaxis in areas of non-resistant falciparum malaria. (u) B. The use of quinolones, tetracyclines, and macrolides is not recommended for the prevention of malaria. (u) C. See B for explanation. (u) D. See B for explanation.

Which of the following are characteristics of Cushing's syndrome? A. central obesity, glucose intolerance, and easy bruising B. attacks of severe headaches, hypertension, and glucosuria C. hyperpigmentation, hypoglycemia, and orthostasis D. tetany, hypocalcemia, and cataracts

(c) A. These are classic signs of Cushing's syndrome. (u) B. These are suggestive of pheochromocytoma. (u) C. These are suggestive of Addison's disease. (u) D. These are suggestive of hypoparathyroidism.

A 57 year-old male with history of hypertension presents with acute gouty arthritis. Which of the following antihypertensive classes should be avoided in this patient? A. Thiazide diuretics B. Beta-blockers C. ACE inhibitors D. Calcium channel blockers

(c) A. Thiazide diuretics are common causes of increased urate levels causing gout attacks. (u) B. See A for explanation. (u) C. See A for explanation. (u) D. See A for explanation.

Which of the following is the chief adverse effect of thiazide diuretics? A. Hypokalemia B. Hypernatremia C. Hypocalcemia D. Hypermagnesemia

(c) A. Thiazide diuretics can induce electrolyte changes. Principle among those is hypokalemia. (u) B. Hyponatremia, not hypernatremia may be a complication of thiazide diuretics. (u) C. Thiazide diuretics cause the retention of calcium and would not cause hypocalcemia. (u) D. Thiazide diuretics cause the retention of calcium and do not readily affect magnesium levels.

A 50 year-old male presents to the ER with a complaint of severe headaches. These headaches are unilateral and he describes the headache pain as steady and non-throbbing. He also complains of nasal congestion and rhinorrhea. He also mentions that alcohol often triggers these headaches. What do you recommend for this patient? A. High flow oxygen B. Massage C. Ibuprofen D. Propranolol

(c) A. This a description of a cluster headache (migrainous neuralgia), these headaches respond to oxygen by mask 7-10 L/min for 15 minutes. (u) B. Massage may be effective for patients with tension headaches. (u) C. Ibuprofen is helpful for the treatment of tension type headaches. (u) D. Propranolol is a preventative medication used for migraine headaches.

A 5 year-old boy presents to the office for follow-up of complaints of pruritus in the perianal area. A cellophane tape test is positive. Which of the following is the treatment of choice? A. mebendazole (Vermox) B. metronidazole (Flagyl) C. chloroquine (Aralen) D. paromomycin (Humantin)

(c) A. Treatment of choice for enterobiasis (pinworms) is mebendazole. (u) B. Metronidazole can be used in the treatment of protozoal infections but it is not indicated in the treatment of pinworms. (u) C. Chloroquine is an amebacide that interferes with parasite protein synthesis. Its main indications are in the treatment of malaria and amebiasis. (u) D. Paromomycin is an aminoglycoside that acts directly on amoebas and against normal and pathogenic organisms in the GI tract.

A patient presents with occasional wheezing and chest tightness that occurs approximately once a week and at night only about once a month. Peak expiratory flow is 85% of predicted. Which of the following is the most appropriate initial treatment? A. Albuterol (Proventil) inhaler B. Montelukast (Singular) C. Salmeterol (Serevent) inhaler D. Sustained release theophylline

(c) A. This patient has mild intermittent asthma which is initially treated with inhaled beta 2-agonists as needed. No long-term control medications are indicated. (u) B. Leukotriene modifiers, such as montelukast, may be added to the treatment of uncontrolled asthma as a long-term controller after the initiation of inhaled corticosteroids. (u) C. Long-acting beta 2-agonists, such as salmeterol, are indicated for long-term control of asthma that is categorized as moderate persistent to severe persistent. (u) D. Sustained release theophylline is an alternative treatment for asthma that is at least categorized as mild persistent; however its narrow therapeutic window and side effects limit its use.

A 24 year-old male presents in respiratory distress and appears quite ill. A Gram stain and culture of the sputum reveals gram-positive cocci in clumps and a chest x-ray reveals multiple patchy infiltrates with some cavitations. Which of the following is most likely to also be found in his medical history? A. IV drug abuse B. Alcohol abuse C. Poor dental hygiene D. HIV positive patient

(c) A. This patient has pneumonia caused by Staphylococcus aureus which is commonly associated with a history of intravenous drug use, influenza epidemics and the hospital setting. (u) B. A history of alcohol abuse is commonly seen with pneumonia caused by Klebsiella pneumoniae. (u) C. Poor dental hygiene is associated with pneumonia caused by anaerobes. (u) D. HIV positive patients are most at risk for development of pneumonia caused by Pneumocystis jiroveci.

A 55 year-old female presents with complaints of stiffness, aching, and pain in the muscles of her neck, shoulders, lower back, hips, and thighs. There is no associated weakness associated with the stiffness and achiness. Laboratory evaluation shows an elevated C reactive protein and erythrocyte sedimentation rate. Which of the following medications is used to treat this condition immediately and will also serve to prevent a known complication from this disorder? A. Glucocorticoids B. Cyclophosphamide (Cytoxan) C. Methotrexate (Rheumatrex) D. Azathioprine (Imuran)

(c) A. This patient has polymyalgia rheumatica and treatment with glucocorticoids can relieve discomfort and prevent the associated ischemic temporal arteritis, which threatens vision. (u) B. Cyclophosphamide is an immunosuppressant used in the treatment of acute leukemia. (u) C. Methotrexate is a folate inhibitor used to treat rheumatoid arthritis, not polymyalgia rheumatica. (u) D. Azathioprine is an immunosuppressant that is used to treat rheumtatic disease and inflammatory bowel disease, not polymyalgia rheumatica.

A 69 year-old male with a history of chronic lymphocytic leukemia presents to the clinic complaining of cough, dyspnea and production of copious amounts of foul smelling sputum. Physical examination reveals crackles at the lung bases. Chest x-ray shows dilated and thickened bronchi that appear as ring-like markings. Which of the following is the most likely diagnosis? A. Bronchiectasis B. Tuberculosis C. Adenocarcinoma D. Pulmonary fibrosis

(c) A. This patient has signs and symptoms consistent with bronchiectasis including CXR findings of dilated and thickened bronchi that may appear as tram-tracks or as ring-like markings. (u) B. TB would present with CXR findings in the apical or posterior segments of the upper lobes. (u) C. Radiographic findings of adenocarcinoma include enlarged nodule or mass; persistent opacity, atelectasis or pleural effusion. The sputum would not likely be foul smelling. (u) D. Pulmonary fibrosis does not present with dilated bronchi or ring-like markings on CXR.

A 45 year-old male presents to the Emergency Department complaining of sudden onset of tearing chest pain radiating to his back. On examination the patient is hypertensive and his peripheral pulses are diminished. Electrocardiogram shows no acute ST-T wave changes. Which of the following is the diagnostic study of choice in this patient? A. Computed tomography (CT) scan B. Transthoracic echocardiogram C. Magnetic resonance imaging (MRI) D. Cardiac catheterization

(c) A. This patient has signs and symptoms of acute aortic dissection for which CT scan is the diagnostic study of choice. (u) B. CT scan is better than transthoracic echocardiogram for the diagnosis of acute aortic dissection. Transesophageal echocardiogram (TEE) is a good diagnostic modality, however it is not always available in the acute setting. (u) C. MRI is good in the diagnosis of a chronic aortic dissection, but the longer imaging time and the difficulty in monitoring the patient during the test makes it not the first choice in the setting of an acute dissection. (u) D. Cardiac catheterization is not indicated in the diagnosis of an acute aortic dissection.

An obese 15 year-old male presents with complaint of a limp and right knee pain for two weeks. He denies recent trauma or history of previous injury. Physical examination of the right knee is unremarkable. Examination of the right hip reveals pain with passive range of motion and limited internal rotation and abduction. Flexion of the hip results in external rotation of the thigh. Gait is antalgic with the right hip externally rotated. Which of the following radiographic findings supports the most likely diagnosis? A. Displacement of the femoral epiphysis B. Irregularity and fragmentation of the joint space C. Capsular swelling of the joint D. Dislocation of the hip

(c) A. This patient has slipped capital femoral epiphysis (SCFE) and the classic x-ray findings will demonstrate displacement of the femoral head rotation of the femoral neck anteriorly. (u) B. Irregularity and fragmentation of the joint space is associated with avascular necrosis of the femoral head as seen in Legg-Calve-Perthes disease. This typically occurs in a younger male population and is not associated with the classic externally rotated hip with ambulation seen in SCFE. (u) C. Capsular swelling of the joint may be seen in transient synovitis of the hip but is not associated with SCFE. (u) D. Hip dislocation at this age is associated with major trauma, such as that sustained in a fall from height or dashboard injury. SCFE does not lead to hip dislocation.

A 5 year-old presents with perianal pruritus that is worse at night. Scotch tape is positive for Enterobius vermicularis. The treatment of choice is A. mebendazole (Vermox). B. metronidazole (Flagyl). C. clindamycin (Cleocin). D. thiabendazole (Mintezol).

(c) A. Treatment of choice for pinworms is mebendazole. (u) B. Metronidazole is used in the treatment of amebic dysentery and trichomoniasis. (u) C. Clindamycin is used in the treatment of anaerobic bacterial infections. (u) D. Thiabendazole is used in the treatment of helminth infections.

: (28) 91. Diagnostic Studies/Psychiatry/Behavioral Medicine A 17 year-old female presents with complaints of intermittent abdominal pain associated with frequent episodes of regurgitation of food for the past several months and worsening over the past 12 hours. She maintains a normal weight for her height; however, she seems obsessed with losing weight. On examination she has multiple dental 39 caries, bilateral tenderness of the parotid glands and mild epigastric tenderness. Which of the following findings would you expect to find on laboratory tests to support your suspected diagnosis? A. Hypokalemia B. Hypocalcemia C. Hyperchloremia D. Hypermagnesemia

(c) A. This patient most likely has bulimia nervosa - purging type. Self-induced vomiting is the most common method of purging and this is supported by the physical examination findings noted in this patient. Laboratory findings to support this diagnosis include hypochloremia with subsequent hypokalemia due to renal compensatory mechanisms, hypomagnesemia and metabolic alkalosis. (u) B. See A for explanation. (u) C. See A for explanation. (u) D. See A for explanation.

A 24 year-old female presents with complaints of dysuria associated with fever, malaise, myalgias and headache for the past 3 days. Today she noticed some lesions on her genitalia. Physical examination reveals the presence of multiple clear vesicles and erythematous ulcers with tender bilateral inguinal adenopathy. Which of the following is the most appropriate treatment for this patient? A. valacyclovir (Valtrex) B. metronidazole (Flagyl) C. ceftriaxone (Rocephin) D. benzathine penicillin G (Bicillin)

(c) A. This patient most likely has herpes genitalis which may be treated with oral antivirals, such as valacyclovir, that will reduce the duration of viral shedding and shorten the duration of symptoms. (u) B. Metronidazole is the treatment of choice for Trichomonas vaginitis. (u) C. Ceftriaxone is one of the options to treat uncomplicated gonorrhea or is used as part of the treatment regimen for pelvic inflammatory disease. (u) D. Benzathine penicillin G is the treatment of choice for a patient with syphilis.

A 34 year-old female presents with complaints of a skin rash associated with a low-grade fever, malaise and anorexia. She denies any pruritis. Physical examination reveals the presence of a diffuse maculopapular rash involving the palms and soles associated with generalized lymphadenopathy. Which of the following will confirm the suspected diagnosis? A. VDRL B. patch testing C. acetowhitening D. Gram stain and culture

(c) A. This patient most likely has secondary syphilis. Serological tests, like the VDRL, are usually positive during this stage with high titers. (u) B. Patch testing is utilized to identify a causative agent and confirm a diagnosis of allergic contact dermatitis. (u) C. Acetowhitening helps in the detection of subclinical penile or vulvar warts. (u) D. Gram stain and culture should be performed on lesions suspected to have a bacterial origin.

: (27) 120. History & Physical/Obstetrics/Gynecology A 24 year-old woman presents to the ED with a several day duration of flu-like syndrome, several small tender ulcers on her vulva and an inability to void. She admits to several sexual contacts in the last 20 days. She denies prior sexually transmitted diseases. Examination reveals the presence of tender lymphadenopathy. Which of the following is most likely based upon the patient's physical exam findings? A. Herpes simplex virus B. Chancroid C. Syphilis D. Granuloma inguinale

(c) A. This patient presents with history and physical exam findings suggestive of HSV. This is her first episode so flu-like symptoms are classic, and usually occur within 2 to 3 days post infection. Painful vesicular and ulcerated lesions appear on the vulva, vagina, cervix, or perineal and perinanal skin, often extending to the buttocks. Painful lymphadenopathy is common. Dysuria caused by vulvar lesions or urethral or bladder involvement may lead to urinary retention. (u) B. See A for explanation. (u) C. See A for explanation. (u) D. See A for explanation.

: (28) 126. Diagnostic Studies/Gastrointestinal/Nutritional An 80 year-old woman with a 40 pack-year history of smoking presents with progressive dysphagia for solids over the past 6 months. She states that she has often had to regurgitate her food because it "gets stuck." She has had an unintentional weight loss of 20 pounds. Which of the following is the initial diagnostic test of choice? A. Barium esophagography B. Esophageal manometry C. Esophageal motility studies D. Esophageal pH monitoring

(c) A. This patient's history of smoking, unintentional weight loss, and progressive dysphagia for solids makes esophageal neoplasm most likely. The initial less invasive diagnostic test is barium swallow which will detect lumen narrowing (mechanical) lesions such as cancer. (u) B. Manometry measures esophageal pressures which are reduced in GERD or elevated in certain conditions such as achalasia. (u) C. Motility studies are indicated in dysphagia of both solids and liquids looking for a motor dysfunction. (u) D. Esophageal pH monitoring is not indicated in the evaluation of an esophageal malignancy.

: (8) 36. History & Physical/Cardiology A 59 year-old male with longstanding uncontrolled hypertension presents with nausea and a 5 pound weight gain in the last 2 days. He states "my belly is getting bigger and I can't fasten my pants." Which of the following physical examination findings would be most likely in this patient? A. Ankle edema and elevated jugular venous pressure B. Dry cough and mitral regurgitation murmur C. Hypotension and cyanotic extremities D. S3 heart sound heard along the left sternal border and bilateral wheezing

(c) A. This patient's symptoms indicate right heart failure with bloating. (u) B. Dry cough and mitral regurgitation murmur are symptoms of left-sided failure. (u) C. Hypotension and cyanotic extremities are symptoms of left-sided failure. (u) D. An S3 heart sound heard along the left sternal border and bilateral wheezing are symptoms of left sided failure.

A 24 year-old male presents complaining of a 9 month history of increasing shortness of breath, dyspnea on exertion, and a cough productive of white sputum, mostly in the mornings. He denies orthopnea, PND, peripheral edema, fever, chills, night sweats, recent changes in weight, palpitations, chest pain, food intolerances, or other complaints. Patient has a history of recurrent lung infections. He states that his father had chronic pulmonary problems and died at age 42 from unknown lung disease. The patient denies smoking, alcohol or illicit drug use. On physical examination, the respiratory rate is 22 per minute, pulse of 98 bpm, temperature of 98.7 degrees. Pulmonary exam reveals end-expiratory wheezes bilaterally and hyperresonance to percussion. His cardiac exam is normal. Chest xray shows decreased lung markings. ECG is normal. Pulmonary function tests show an FEV1 63% of expected and residual capacity is 123% of expected. Which of the following is the most likely diagnosis? A. Emphysema B. Pulmonary fibrosis C. Ventricular septal defect D. Congestive heart failure

(c) A. This person has an obstructive lung disease based on PFTs. Emphysema is the most likely diagnosis, and may be related to alpha-1 antitrypsin deficiency based on family history and lack of smoking history and young age. (u) B. The PFTs from a person with pulmonary fibrosis would be consistent with a restrictive pattern. This patient has an obstructive pattern of lung disease. (u) C. Ventricular septal defect will have a systolic murmur associated with it. (u) D. Congestive heart failure might explain some of the symptoms of this patient (increasing shortness of breath and DOE), he denies other common symptoms, such as orthopnea and peripheral edema. CHF should not result in changes in the PFTs.

: (9) 48 116. Diagnosis/Cardiology A 16 year-old athlete with no past medical history collapses after running 50 yards down the field. He is unresponsive, pulseless and cyanotic. Which of the following is the most likely cause of this student's collapse? A. Hypertrophic cardiomyopathy B. Myocardial infarction C. Pulmonary embolism D. Reactive airway disease

(c) A. This presentation is consistent with hypertrophic cardiomyopathy which may initially be difficult to diagnose. Infants but not older children frequently present with signs of CHF. Older children may be asymptomatic, with sudden death as the initial presentation (u) B. See A for explanation. (u) C. See A for explanation. (u) D. See A for explanation.

A 45 year-old female presents with her daughter who is concerned because her mom will not leave the house. The patient describes feeling anxious in crowds and places where there are no easy means of escape. Which of the following is the most likely diagnosis in this patient? A. Agoraphobia B. Generalized anxiety disorder C. Obsessive-compulsive disorder D. Panic attack

(c) A. Though all of the anxiety disorders can have comorbid agoraphobia the case describes only those symptoms inherent to this primary diagnosis. Agoraphobia can occur with generalized anxiety disorder and a panic attack can occur as a result of agoraphobia. (u) B. See A for explanation. (u) C. See A for explanation. (u) D. See A for explanation.

A 55 year-old male presents with a three month history of progressive weakness in the extremities without associated sensory changes or deficits. Examination reveals widespread muscle atrophy, fasciculations and bilateral hyperactive reflexes with Babinski sign. Which of the following is the most likely diagnosis? A. Amyotrophic lateral sclerosis B. Polymyalgia rheumatica C. Myasthenia gravis D. Multiple sclerosis

(c) A. Though some variability in presentation does occur the characteristic progressive weakness without sensory changes and upper and lower motor nerve dysfunction is a hallmark of amyotrophic lateral sclerosis (ALS). (u) B. See A for explanation. (u) C. See A for explanation. (u) D. See A for explanation.

A 48 year-old male with a recent episode of nephrolithiasis comes to the office to discuss prevention of further urinary stone disease. Which of the following dietary recommendations can reduce recurrence of nephrolithiasis? A. Increase fluid intake B. Decrease dietary fiber C. Increase salt intake D. Decrease calcium intake

(c) A. To prevent stone recurrence, the recommendations are to double previous fluid intake. (u) B. Dietary fiber, such as bran, can significantly decrease the risk of stone formation. (u) C. Excess sodium ingestion is a cause of calcium stone disease. (u) D. Dietary recommendations including maintaining a moderate calcium intake. Dietary calcium avoidance can actually increase stone recurrence.

A patient presents complaining of left eye discharge and eyes that were matted shut this morning. The patient denies changes in visual acuity, but states that he is afraid to put his contacts in. On physical examination you note erythematous conjunctivae and mucopurulent discharge of the left eye. The cornea is clear. Which of the following topical agents is the treatment of choice in this patient? A. Aminoglycoside (Tobrex) B. Olopatadine (Patanol) C. Cycloplegic D. Prednisolone acetate

(c) A. Topical aminoglycoside or fluoroquinolones are indicated in contact lens wearers with conjunctivitis to cover for Pseudomonas infection. (u) B. Patanol is indicated in patients with allergic, not bacterial, conjunctivitis. (u) C. Topical cycloplegic agents and corticosteroids are not indicated in the treatment of bacterial conjunctivitis. (u) D. See C for explanation.

A patient is brought to the emergency room with acute onset of dyspnea and tachypnea. He has a long history of alcoholism and was involved in a motor vehicle accident two days ago. He is hypoxic with crackles auscultated bilaterally. Chest radiography reveals diffuse bilateral infiltrates which spare the costophrenic angle and air bronchograms, there was no cardiomegaly or pleural effusion noted. Oxygen saturation is 70%. Which of the following is the most important initial treatment? A. Tracheal intubation B. Bilateral chest tube insertion C. Type-specific packed cells D. Colloid solutions

(c) A. Tracheal intubation with lowest level of PEEP is required to maintain the PaO2 above 60mmHg or SaO2 above 90% in a patient with ARDS. (u) B. See A for explanation. (u) C. Fluids are the preferred treatment initially for hypovolemia. Type-specific packed cells are given when the patient's blood type is identified. Until then O negative packed cells are administered. (u) D. Use of crystalloid solutions are preferred to avoid pulmonary edema.

A 19 year-old college student presents to the student health center complaining of a sore throat and mild flu-like symptoms for 2 days. A rapid strep screen and culture are negative. Symptomatic therapy is instituted. The patient returns 3 weeks later complaining of continued sore throat with increasing fatigue and onset of fever. Physical examination reveals an exudative pharyngitis, tender enlarged posterior cervical lymphadenopathy, and the spleen is palpable. Which of the following is most appropriate as the next step in diagnosis? A. Mono spot B. Repeat strep screen C. Anti-EBV antibodies D. Lymph node biopsy

(c) A. With the presence of exudative pharyngitis associated with posterior cervical lymphadenopathy and splenomegaly, infectious mononucleosis is the most likely diagnosis. A mono spot would most likely be positive since the patient has had symptoms for more than 3 weeks. (u) B. A strep screen tests for the presence of beta-hemolytic Streptococcus group A, which would not account for the splenomegaly. (u) C. Anti-EBV antibodies measure specific antibody titers and are not a cost effective diagnostic test at this time. (h) D. Lymph node biopsy is not indicated and may lead to complications in the setting of acute mononucleosis.

Zostavax (varicella-zoster vaccine) is contraindicated in which of the following groups of patients? A. Chemotherapy patients B. Adults over 60 years of age C. Patients allergic to eggs D. Patients who have recovered from shingles

(c) A. Zostavax is a live attenuated vaccine and is contraindicated in patients with immunodeficiency states, malignancy affecting the bone marrow, pregnant women, and patients taking immunosuppressive medications. (u) B. A single dose of zoster vaccine is recommended for patients over the age of sixty without other contraindications. (u) C. Patients with an anaphylactic reaction to gelatin or neomycin should not receive the Zostavax vaccine. However, egg allergy is not a contraindication to the Zostavax vaccine. (u) D. Patients who have recovered from shingles may still receive the vaccine. This may decrease recurrent varicella zoster and postherpetic neuralgia in these patients.

A 14 year-old male presents to the ED experiencing a severe asthma attack. His respiratory effort is shallow and he is using accessory muscles to breathe. Auscultation of his chest reveals no audible wheezing. Vital signs include BP 90/60 mmHg, P 160 bpm, RR 52. An arterial blood gas (ABG) is ordered. Normal ABG values at your institution are pH 7.35-7.45, CO2 35-45, pO2 80-95. Which of the following ABG findings suggests the poorest prognosis? A. pH = 7.27 pCO2 = 46 pO2 = 56 B. pH = 7.60, pCO2 = 18 pO2 = 80 C. pH = 7.44, pCO2 = 38 pO2 = 90 D. pH = 7.52, pCO2 = 28, pO2 = 80

(c) A. pH = 7.27 pCO2 = 46 pO2 = 56 is associated with the poorest prognosis in this patient. (u) B. See A for explanation. (u) C. See A for explanation. (u) D. See A for explanation.

A patient with advanced AIDS complicated by toxoplasmosis presents with altered mental status, recent onset of seizures, and focal neurologic deficits. Which of the following diagnostic studies is most helpful? A. EEG B. MRI C. lumbar puncture D. Toxoplasma gondii antibody titers

(c) B. An MRI showing multiple isodense or hypodense ring-enhancing mass lesions is the most useful test for such a patient. (h) C. Lumbar puncture is contraindicated secondary to possible mass effect. (u) D. Antibody titers cannot be depended upon since most patients have IgG titers that reflect past infection, significant rises are infrequent, and IgM antibody is rare.

What is the initial treatment of choice for hyperthyroidism in a 10-week pregnant patient? A. No treatment is necessary. B. Propylthiouracil (PTU) C. Radioiodine treatment D. Subtotal thyroidectomy

(h) A. Although thyroid function tests are altered in pregnancy true hyperthyroidism can occur and should be treated. (c) B. This is the initial treatment of choice. (h) C. Radioiodine treatment is contraindicated in pregnancy. (u) D. Subtotal thyroidectomy is an option for pregnant patients during the second or third trimesters. Surgery is not indicated for first-trimester pregnancies.

A 29 year-old patient with idiopathic thrombocytopenia purpura (ITP) is treated with prednisone therapy. Despite therapy, platelet counts remain consistently below 20,000/microliter over the course of 6 weeks. Which of the following is the most appropriate intervention for this patient? A. Aspirin B. Intravenous immunoglobulin C. Danazol (Danocrine) D. Splenectomy

(h) A. Aspirin inhibits platelet function and could lead to significant bleeding and death of this patient. (u) B. Intravenous immunoglobulin can be utilized for short-term treatment, but the platelet count is likely to return to baseline within a month. (u) C. Danazol is typically reserved for ITP that fails to respond to splenectomy. (c) D. Persistently low platelet counts (< 20,000) require effective long-term treatment, and splenectomy is the treatment of choice.

An 18 month-old female presents to the Emergency Department having possibly swallowed a hearing aid battery within the past hour. She is drooling and appears anxious but parents have noticed no stridor or dyspnea. She has no history of previous esophageal injury. Physical examination is unremarkable. Chest radiograph reveals a radiopaque round object at the distal esophagus. Which of the following is the most appropriate treatment option? A. Observation for 24 hours B. Esophagoscopy for removal C. Barium swallow D. Bronchoscopy

(h) A. Batteries must be removed as they can induce mucosal injuries in as little as one hour of contact time. (c) B. Esophagoscopy is the procedure of choice for acutely ingested foreign bodies. (u) C. A barium swallow is a diagnostic option but will not provide treatment. (u) D. Bronchoscopy would be the procedure of choice for an airway foreign body, not esophageal.

A patient taking bleomycin (Blenoxane) should be monitored for which of the following side effects? A.Optic neuritis B. Hyperuricemia C. Encephalopathy D. Pulmonary fibrosis

(u) A. Optic neuritis is a potential side effect of ethambutol, used in the treatment of tuberculosis. (u) B. Hyperuricemia or encephalopathy are not known side effects of bleomycin. (u) C. See B for explanation. (c) D. Pulmonary fibrosis and pulmonary infiltrates are known side effects of bleomycin.

A patient with severe COPD presents to the Emergency Department with a 3 day history of increasing shortness of breath with exertion and cough productive of purulent sputum. An arterial blood gas reveals a pH of 7.25, PaCO2 of 70 mmHg and PaO2 of 50 mmHg. He is started on albuterol nebulizer, nasal oxygen at 2 liters per minute, and an IV is started. After one hour of treatment, his arterial blood gas now reveals a pH of 7.15, PaCO2 100 mmHg and PaO2 of 70 mmHg. Which of the following is the most appropriate next step in his treatment? A. Decrease the oxygen flow rate. B. Administer oral corticosteroids. C. Intubate the patient. D. Administer salmeterol (Serevent)

(h) A. Decreasing the oxygen flow rate would be harmful as it would decrease the amount of oxygen delivered to the patient. (u) B. Administration of steroids is an important treatment modality but this patient is in respiratory failure and needs more immediate therapy. (c) C. This person has increasing respiratory failure as indicated by the raising PaCO2 levels. Intubation is required at this time. (h) D. Long-acting beta agonist therapy such as salmeterol is not utilized for rescue therapy.

A 26 year-old woman comes to the office for her first prenatal visit at 9 weeks gestation. During evaluation, her urinalysis reveals asymptomatic bacteriuria. Which of the following antibiotics is the preferred treatment in this patient? A. Doxycycline (Vibromycin) B. Trimethoprim (Monotrim) C. Nitrofurantoin (Macrobid) D. Erythromycin (Ery-tab)

(h) A. Doxycycline is contraindicated during pregnancy because of dental staining in the exposed child. (u) B. Trimethoprim is a folic acid antagonist, and should be avoided during organogenesis in the first trimester. (c) C. Nitrofurantoin is an effective treatment of asymptomatic bacturia in the pregnant woman. (u) D. Erythromycin is an ineffective treatment for asymptomatic bacteriuria.

A 37 year-old right-handed male presents with daily headaches for the past 2 months. They are worse in the morning and awaken him from sleep. In the past week, he has noticed a tendency to drop things. Neurologic examination reveals upper extremity strength of 4/5 on the right and 5/5 on the left. Which of the following is the most appropriate next step in the evaluation of this patient? A. Lumbar puncture (LP) B. Electroencephalogram (EEG) C. Magnetic resonance imaging (MRI) D. Electromyelogram (EMG)

(h) A. Lumbar puncture results would give information on the cerebrospinal fluid. In the presence of focal neurologic findings, a lumbar puncture would be contraindicated prior to an MRI. (u) B. An EEG is used in the evaluation of seizure disorders. (c) C. MRI is the most appropriate diagnostic study in the evaluation of a suspected intracranial neoplasm. (u) D. EMG is not indicated in the evaluation of a suspected intracranial neoplasm.

According to the Advisory Committee on Immunization Practices, which of the following is the recommended age range for the first or initial MMR (measles, mumps and rubella) vaccination? A. 2-4 months B. 12-15 months C. 4-6 years D. 11-12 year

(h) A. MMR vaccination is not approved for use in infants below the age of 12 months. (c) B. The recommended age range for the initial vaccination against MMR is 12-15 months. (u) C. The second dose of the MMR vaccine is routinely recommended to be administered between 4-6 years of age. (u) D. Any child not previously vaccinated by 11-12 years of age may receive the vaccine. This is considered to be a "catch-up" immunization and is not the recommended age range for the initial vaccination.

An 18 year-old female with diabetes presents to the emergency department with altered level of consciousness, deep breathing and fruity odor to her breath. Which of the following medications is indicated for this patient? A. NPH insulin B. Metformin (Glucophage) C. Regular insulin D. Glyburide (Diabeta)

(h) A. NPH insulin has a slower onset of action than regular insulin, and should not be administered intravenously. Onset is 2-4 hours and in a patient with diabetic ketoacidosis the onset needs to be rapid. (h) B. Metformin and glyburide are oral hypoglycemics that have no role in the emergency treatment of diabetic ketoacidosis. (c) C. Regular insulin has a rapid onset of action when given intravenously and is the initial choice in patients with diabetic ketoacidosis. Following the initial dose, a continuous infusion often promotes a steady, slow fall of glucose levels to normal, which can then be stabilized by decreasing the insulin. (h) D. See B for explanation.

A 16 year-old male presents with increasing pain and swelling of his right scrotum. The right testicle is extremely tender to palpation on examination. A Doppler ultrasound demonstrates decreased blood flow. Which of the following is the most appropriate intervention? A. oral doxycycline B. emergent surgery C. incision and drainage D. scrotal elevation and ice packs

(h) A. Oral doxycycline is the treatment of choice for epididymitis. While epididymitis and testicular torsion present similarly, the Doppler ultrasound in epididymitis would show increased blood flow, not decreased. (c) B. Once a diagnosis of testicular torsion is suspected, emergent surgery is indicated to have the best possible chance of salvaging the testicle (85-97% chance if less than 6 hours). Any other treatment measures delay the definitive treatment and increase the risk of testicular ischemia and infarction. (h) C. Incision and drainage is indicated for treatment of abscesses, not testicular torsion. (h) D. Scrotal elevation and ice packs are indicated for adjunct treatment of epididymitis, not testicular torsion.

Which diagnostic study is considered to be the strategy of choice for symptomatic patients with recurrent ischemia, hemodynamic instability or impaired left ventricular dysfunction? A. Stress echocardiography B. Exercise treadmill testing C. Coronary angiography D. Cardiac magnetic resonance imaging

(h) A. Stress echocardiography should not be performed in the setting of a patient who is acutely symptomatic. (h) B. Exercise treadmill testing should not be performed in the setting of an unstable patient with ongoing cardiac symptoms. (c) C. Coronary or cardiac catheterization is the gold standard technique in the evaluation of patients with significant cardiac symptoms. Anatomical information along with degree of coronary artery blockages are provided and patients may be able to undergo coronary revascularization during or after this procedure. (u) D. Cardiac magnetic resonance imaging has limited availability and is not part of national guidelines for evaluation of the cardiac patient.

: (7) 93. Clinical Therapeutics/Cardiology A 48 year-old male patient presents with a recent onset of anterior chest pain. The pain increases with deep breathing and coughing. He says he slept in the recliner last night as his discomfort worsens when he lies down. He also complains of feeling feverish although did not have a thermometer to check his temperature. An electrocardiogram (ECG) shows ST elevations in the precordial and limb leads. Additionally you note PR depression predominantly over leads I, II and III. Past medical history is significant for pharyngitis diagnosed as mononucleosis about 10 days ago. What is the most appropriate next step in the evaluation of this patient? A. Streptokinase B. Pericardiocentesis C. Indomethacin D. Cardiac catheterization

(h) A. Pericarditis is not treated with thrombolytic therapy. (h) B. Pericardiocentesis is not indicated for inflammation of the pericardial sac unless evidence of tamponade or constrictive effusion is present. (c) C. Acute inflammatory pericarditis presents with anterior pleuritic chest pain that is worse supine than upright. ECG reveals diffuse ST segment elevation with associated PR depression. Viral infections (including Epstein- Barr) are the most common cause of acute pericarditis. Treatment for viral pericarditis is generally symptomatic. ASA or other NSAIDs are usually effective. (u) D. There is no indication for cardiac catheterization in this patient.

A 23 month-old male presents to the office with his father who reports the patient has had an acute onset of severe pharyngitis, fever of 103.5 degrees F and what sounds like harsh, high-pitched breath sounds. His dad states the child has started drooling and seems to be worsening. The child is not presently crying but has muffled voice sounds. The child has not been immunized due to religious reasons. What is the most appropriate next step? A. Administer ceftriaxone (Rocephin) B. Emergent transfer C. Administer racemic epinephrine D. Obtain intravenous access

(h) A. See B for explanation. (c) B. Epiglottitis requires endotracheal intubation to maintain the airway but should be performed only in the operating room or emergency room with a competent physician prepared to place an endotracheal tube or less often to perform a tracheostomy. (u) C. See B for explanation. (h) D. See B for explanation.

A 35 year-old pregnant patient presents with fever, chills, and left-sided flank pain. On physical examination left-sided CVA tenderness is noted. Urinalysis reveals numerous white blood cells and white blood cell casts. Which of the following is the most appropriate treatment? A. Oral ciprofloxacin (Cipro) B. Oral trimethoprim-sulfamethoxazole (Bactrim) C. IV gentamicin (Garamycin) D. IV ceftriaxone (Rocephin)

(h) A. See B for explanation. (h) B. The fluoroquinolones and trimethoprim-sulfamethoxazole are contraindicated in pregnancy. (u) C. Gentamicin is not indicated as first line therapy in the treatment of pyelonephritis in a pregnant patient. (c) D. IV cephalosporins are first line treatment of pyelonephritis in a pregnant patient, followed by oral step-down therapy.

: (26) 97. Clinical Intervention/ENT/Ophthalmology A 28 year-old male presents with a history of eye pain and blepharospasm. He reports being splashed in the eyes with an ammonia-based household cleaner. Which of the following is the most important initial step in the management of this patient? A. Measurement of ocular pH B. Immediate assessment of visual acuity C. Irrigation with 1-2 liters of normal saline D. Immediate slit-lamp evaluation for corneal damage

(h) A. See C for explanation. (h) B. See C for explanation. (c) C. Alkali burn is the most common and most dangerous chemical burn to the eye. Common agents include ammonia-based household cleansers and drain cleaners. The most important and initial management of this presentation includes copious irrigation that begins prior to formal ocular examination. Use of Morgan lens is often required given the frequent finding of blepharospasm. (h) D. See C for explanation.

A 16 year-old male involved in a fight sustained a laceration to his right upper eyelid. He is unable to open his eye, and a possible laceration of the globe is suspected. Which of the following is the next step? A. Use a slit lamp to determine the extent of the injury. B. Use fluorescein strips to determine the extent of injury. C. Apply a metal eye shield and refer to an ophthalmologist. D. Apply antibiotic ointment to the lid and recheck in 24 hours.

(h) A. See C for explanation. (h) B. See C for explanation. (c) C. Protect the eye from any pressure with a rigid metal eye shield and refer for immediate ophthalmologic consultation. Avoid unnecessary actions that would delay treatment or cause further injury. (h) D. See C for explanation.

It is determined that a woman has a nonexistent rubella titer level during her first trimester of pregnancy. When should she receive the rubella vaccine? A. During the first trimester of pregnancy B. During the second trimester of pregnancy C. During the third trimester of pregnancy D. After delivery of the infant

(h) A. See D for explanation. (h) B. See D for explanation. (h) C. See D for explanation. (c) D. The patient should not receive the rubella vaccine during the course of her pregnancy as the possibility of transmission of the rubella virus does exist. During the time that the patient is without protective titer she should avoid anyone with active rubella infection. The proper time to receive the vaccine is after delivery of the infant.

: (7) 35. Clinical Intervention/Hematology A 4 year-old female presents with a headache and new onset confusion. She was diagnosed with a viral illness 2 weeks ago. Examination reveals petechiae and mild scant purpura on her legs bilaterally. The CBC demonstrates a platelet count of 8,000/uL. Which of the following is the most appropriate next step in the assessment of this patient? A. Lumbar puncture B. D-dimer C. PT and PTT D. CT scan of the head

(h) A. See D for explanation. (u) B. See D for explanation. (u) C. See D for explanation. (c) D. The concern for this patient is low platelets and internal bleeding. In this scenario, the patient complaining of a headache would warrant a CT scan to evaluate for an intracranial bleed.

A patient with adrenal insufficiency is taking hydrocortisone 25 mg daily. What should the patient do with the hydrocortisone dose when they develop a minor illness such as a cold? A. Stop the hydrocortisone until the illness resolves. B. Continue the current dose that the patient is taking. C. Increase the dose to 50 mg daily until the illness resolves. D. Increase the dose to 250 mg daily until the illness resolves

(h) A. Stopping the hydrocortisone would cause adrenal crisis. (u) B. See C for explanation. (c) C. To better mimic the normal physiologic response the baseline dose should be doubled for the duration of the illness. Doses should be increased 5-10 fold with major events such as surgery. (u) D. See C for explanation.

: (7) 184. Clinical Intervention/Dermatology A 51 year-old diabetic female is brought to the emergency department with complaints of fever and chills for the past 24 hours. Physical examination reveals a BP 100/70mmHg, P 110bpm, oral T 102 F, R 18 and pulse ox 98% on room air. On her right foot you note an area between the 4th and 5th digits that is erythematous, moist and tender. Lymphadenopathy is present behind the right knee. Which of the followings is the next best step in treatment? A. Provide patient a prescription for an antifungal and have her follow-up with her PCP B. Admit the patient, obtain blood and wound cultures and treat with IV antibiotics C. Obtain blood and wound cultures and send patient home with oral antibiotics and instructions to follow-up with her PCP D. Administer IV antibiotics and send patient home with oral antibiotic and instructions to follow up with her PCP

(h) A. This is a diabetic foot. Failure to recognize and treat appropriately in a timely fashion could cause further morbidity and mortality. (c) B. This is a diabetic foot. Failure to recognize and treat appropriately in a timely fashion could cause further morbidity and mortality. Early admission, cultures and aggressive IV antimicrobial treatment is the standard of care. (u) C. See B for explanation. (u) D. See B for explanation.

A 25 year-old male on a behavioral medicine unit is given haloperidol (Haldol) IM for a violent psychotic outburst. Initially he quiets down, but about an hour later develops confusion, an inability to open his mouth, and a temperature of 40 degrees C. Initial treatment should consist of which of the following? A. Additional Haldol B. Corticosteroid C. Benzodiazepine D. Dantrolene

(h) A. This patient is exhibiting symptoms of neuroleptic malignant syndrome (NMS) and an additional dose of a antipsychotic will be harmful, and potentially deadly with a 20% mortality if untreated. (u) B. Steroids will not reduce the symptoms of NMS. (h) C. Benzodiazepines will not reduce the symptoms of NMS. (c) D. In addition to supportive treatment, the most commonly used medications for neuroleptic malignant syndrome are dantrolene (Dantrium) and bromocriptine (Parlodel).

erral to cardiology for outpatient thallium stress test B. Discharge patient home with nitroglycerine C. Transfer to cardiac catheterization lab D. Monitor with repeat enzymes and ECG in 4-6 hours

(h) A. This patient is not considered stable with his risk factors and symptoms suggestive of a myocardial infarction and therefore should not be discharged from the ED. (h) B. One-fourth to one-half of patients with acute MI will have no typical ECG or enzyme abnormalities. With a clinical scenario that suggests the possibility of ischemia/infarction, the patient should be monitored with serial ECG's to rule out developing events. (u) C. At present there is no ECG evidence of myocardial ischemia or elevated enzymes that would indicate a pure cardiac nature to this problem. Cardiac catheterization may be required if progression indicates a coronary occlusion however would it not be indicated immediately given these findings. (c) D. In one-fourth to one-half of patients with acute MI, the first ECG does not demonstrate typical ST segment changes. Serial ECG's should be obtained to increase diagnostic yield. Serologic identification of myocyte necrosis is another beneficial diagnostic tool. CK-MB is detectable in the blood within 3-6 hours of the onset of the MI. Troponins begin to rise within 4-6 hours and remain elevated for 7-10 days.

: (9) 52. Health Maintenance/ENT/Ophthalmology What is the recommended frequency of retinal screening in a patient with type 2 diabetes mellitus without evidence of retinopathy? A. Examination if symptoms develop B. Every 3-months examination C. Every 6-months examination D. Annual examination

(h) A. This strategy is unacceptable given the high likelihood of lost follow-up or delayed follow-up. (u) B. This frequency is typically indicated for individuals with proliferative retinopathy. (u) C. Screening is recommended at 6-12 month intervals in individuals with mild to moderate non-proliferative retinopathy. (c) D. Annual examination is recommended to ensure continued follow-up and to assess for more aggressive disease.

A 23 year-old male being treated for an acute bacterial prostatitis has been taking antibiotics for less than 24 hours. He presents to the emergency room today with acute urinary retention for 12 hours. Which of the following is the most appropriate next step? A. Insert a Foley catheter. B. Initiate diuretic therapy. C. Schedule for cystoscopy. D. Insert a percutaneous suprapubic tube.

(h) A. Urethral catheterization, or any form of instrumentation is contraindicated in the presence of acute bacterial prostatitis. (h) B. Diuretic therapy is contraindicated in the treatment of acute urinary retention. (u) C. See A for explanation. (c) D. Inserting a percutaneous suprapubic tube is the treatment of choice in a patient with acute bacterial prostatitis who develops acute urinary retention.

A 57 year-old male presents with acute bilateral lower extremity weakness and urinary incontinence that began after he fell earlier today. His examination is significant for bilateral lower extremity sensory deficits and weakness along with decreased rectal sphincter tone. Which of the following is the most appropriate intervention? A. Epidural steroids B. Oral NSAIDs C. Physical therapy D. Surgery

(h) A. While epidural steroids can be effective in treating lumbar disc herniation, in the case of cauda equina syndrome, immediate surgical decompression is mandatory. (h) B. NSAIDs may be beneficial in some cases of lumbar muscle strain and disc herniation. They are not appropriate for management of cauda equina syndrome, immediate surgical decompression is mandatory. (h) C. Physical therapy may be beneficial in some cases of lumbar muscle strain and disc herniation but it is not appropriate for management of cauda equina syndrome. (c) D. Cauda equina syndrome is a rare but serious surgical emergency because the duration of nerve compression is inversely correlated with the likelihood of full neurologic recovery.

Which of the following is most helpful in the diagnosis of a retropharyngeal abscess? A. CBC with differential B. fever and a muffled voice on examination C. CT of the neck with contrast D. history of a recent throat infection

(u) A. A CBC with differential would identify an infection but not specifically a retropharyngeal abscess. (u) B. The presence of fever and a muffled voice on physical exam is not specific for a retropharyngeal abscess. (c) C. CT of the neck is considered the "gold standard" for the diagnosis of a retropharyngeal abscess. (u) D. A recent throat infection is not specific for a retropharyngeal abscess.

An elderly woman presents to your clinic complaining of unilateral facial pain and painful lesions. She also complains of blurred vision in the ipsilateral eye. On examination she has several vesicles on an erythematous base, some of the lesions with crusts. They are distributed in a dermatomal pattern and involve the skin overlying the maxillary region and the tip of her nose. Which of the following is the next most appropriate intervention in the care of this patient? A. KOH prep B. Culture for bacteria C. Referral to an ophthalmologist D. Application of corticosteroids

(u) A. A KOH prep examining for fungal elements is not indicated in this case. (u) B. While bacterial infection may occur along with the viral infection, a bacterial culture is not usually employed. (c) C. Immediate referral to an ophthalmologist is needed when herpes keratitis is suspected, as in this case. A fluorescein stain of the eye might reveal the typical dendritic corneal lesion. (h) D. Application of corticosteroids may cause proliferation of the virus and should only be used by an ophthalmologist.

A 26 year-old presents with two days of a generalized, non-pruritic rash with concurrent low grade fever and sore throat. He is otherwise in good health with no history of dermatologic problems other than acne and a non-painful ulceration at the base of his penis seven months ago that resolved spontaneously. Examination reveals a macularpapular rash spread diffusely over the body including the palms and soles of the feet. Shallow ulcers are noted on the buccal mucosa while the pharynx is moderately erythematous. Which of the following is the most appropriate initial diagnostic study? A. Tzanck smear of lesion B. Serum FTA-Absorption test C. Streptococcus pyogenes culture D. Epstein-Barr IgM titer

(u) A. A Tzanck smear is commonly used for diagnosing herpes virus infections. (c) B. With secondary syphilis, 100% of persons test positive with serum FTA-Absorption test. (u) C. Though some attributes of the case point to a scarlatina- like rash the prior genital lesion and palmar involvement make this less likely. (u) D. Epstein-Barr IgM titer is used in the diagnosis of infections such as mononucleosis which would generally not present with this type of rash unless a penicillin had been given to the patient.

A 29 year-old male is involved in a motor vehicle crash. On the secondary survey it is noted that there is blood at the meatus and the patient is suspected of having a pelvic fracture. The patient is otherwise stable. Which of the following tests should be done to evaluate the urinary system? A. Voiding cystourethrogram (VCUG) B. Intravenous pyelogram (IVP) C. Urethrogram D. Renal arteriography

(u) A. A VCUG is done to evaluate urinary reflux in children. (u) B. An IVP can be done as part of an evaluation for hematuria however it is rarely used today. (c) C. A urethrogram is the only procedure that should be done to evaluate this type of injury as urethral integrity may have been compromised secondary to the pelvic fracture. Blood at the meatus is the most important finding of suspected urethral injury. (u) D. Renal arteriography does not have any place in this scenario as there is no evidence of a renal injury.

A 17 year-old woman presents to the office with recurrent episodes of palpitations and near syncope. Initial ECG was normal. She is concerned about these episodes since they can occur at any time. Which of the following is the most appropriate step to pursue in her evaluation? A. cardiac catheterization B. tilt table testing C. echocardiogram D. Holter monitoring

(u) A. A cardiac catheterization will not be useful since the patient is at low risk for actual coronary artery disease. (u) B. Tilt table testing is useful only in trying to determine vasodepressor syncope that is related to position. (u) C. An echocardiogram shows valves and left ventricle function, not pathways of conduction. (c) D. Holter monitoring will identify the heart rhythm; an event recorder may also be useful in this setting if the Holter monitor is not diagnostic

A 33 year-old female presents for follow-up of her Pap smear that showed cervical dysplasia. Which of the following is the most appropriate diagnostic procedure? A. Cone biopsy B. Aspiration needle biopsy C. Dilation and curettage D. Colposcopy-directed biopsy

(u) A. A cervical cone biopsy may be indicated in further evaluation of this patient, but it is dependent on the results of the colposcopy. (u) B. An aspiration needle biopsy has no role in the evaluation of cervical dysplasia. (u) C. Dilatation and curettage has no role in either the diagnosis or treatment of isolated cervical dysplasia. (c) D. A colposcopy-directed biopsy is the first diagnostic evaluation indicated for cervical dysplasia.

A 50 year-old male with history of alcohol abuse presents with acute, severe epigastric pain radiating to the back. The patient admits to an episode of coffee ground emesis. On examination he is ill-appearing with a rigid, quiet abdomen and rebound tenderness. Which of the following is the most likely diagnosis? A. Abdominal aortic aneurysm B. Perforated duodenal ulcer C. Acute myocardial infarction D. Cholecystitis

(u) A. A patient with an abdominal aortic aneurysm may present with pain radiating to the back, however would not have coffee ground emesis or an acute abdomen. (c) B. Perforation of a duodenal ulcer causes sudden, severe pain, with rebound tenderness and rigid abdomen on physical examination. It is often associated with coffee ground emesis. (u) C. A patient with an acute myocardial infarction may have pain radiating to the back, however would not have hematemesis or an acute abdomen. (u) D. Cholecystitis presents with right upper quadrant pain and is not typically associated with coffee ground emesis or rebound tenderness.

Your patient has just delivered her baby vaginally without difficulty. The patient has a laceration of the vaginal mucosa including the perineal body. You repair it without difficulty. On the chart you document this as what type of tear? A. First degree B. Second degree C. Third degree D. Fourth degree

(u) A. A first degree tear involves the vaginal mucosa or perineal skin, but does not involve the underlying tissue. (c) B. A second degree tear involves the underlying subcutaneous tissues, but not the rectal sphincter or rectal mucosa. (u) C. A third degree tear extends through the rectal sphincter, but not into the rectal mucosa. (u) D. A fourth degree tear extends into the rectal mucosa.

A 28 year-old man presents to the emergency department complaining of sudden onset of shortness of breath associated with sharp right-sided chest pain increased with breathing. On physical examination, respirations are 20 per minute and blood pressure is 120/76 mm Hg. Auscultation of the chest reveals absent breath sounds over the right apex with normal heart sounds. Percussion of the right apex is noted to be hyperresonant. Which of the following is the most likely diagnosis? A. Hemothorax B. Pneumothorax C. Pulmonary embolus D. Foreign body aspiration

(u) A. A hemothorax usually results from trauma. While breath sounds would be absent over the involved area, the percussion note would be dull, not hyperresonant. (c) B. This patient most likely has a spontaneous pneumothorax which is supported by the presenting symptoms of sudden onset of dyspnea and pleuritic chest pain as well as the physical exam findings of absent breath sounds and hyperresonance to percussion. (u) C. While a pulmonary embolism most commonly presents with dyspnea and pain on inspiration, the physical examination is often unremarkable and would not reveal the findings of absent breath sounds and hyperresonance to percussion. (u) D. Foreign body aspiration is most common in children and the elderly. Aspiration of a food bolus is the most common cause in adults which leads to a history of a choking episode followed by dysphagia, wheezing and coughing. Physical examination findings are dependent on the location of the obstruction.

: (31) 92. Scientific Concepts/Hematology Which of the following types of hemoglobin is responsible for the development of sickle cell anemia? A. A B. AS C. F D. SS

(u) A. A is more associated with sickle cell trait. (u) B. AS is associated with sickle cell trait. (u) C. F is fetal hemoglobin and is seen in someone who is successfully undergoing treatment for sickle cell anemia. (c) D. SS is the dominant hemoglobin found in sickle cell anemia.

Bitemporal hemianopia is noted on physical examination in a patient with visual changes over the past 2 years. The central field of vision is spared. The lesion is located in the A. optic nerve. B. optic chiasm. C. temporal optic radiation. D. optic tract.

(u) A. A lesion in the optic nerve would result in loss of vision in the affected eye only and include loss of central vision. (c) B. A lesion in the optic chiasm would result in the loss of vision in the bilateral temporal fields and spare the central field of vision. (u) C. A lesion in the temporal optic radiation would produce superior contralateral quadrantopia. (u) D. A lesion in the optic tract would result in loss of vision in the temporal field of the ipsilateral eye.

: (27) 113. Scientific Concepts/ENT/Ophthalmology A 70 year-old male presents with right-sided vision loss. Examination reveals right homonymous hemianopsia. Which of the following is the location of the suspected lesion? 47 A. Left optic nerve B. Right optic nerve C. Left optic tract D. Right optic tract

(u) A. A lesion of the left optic nerve results in a left eye blindness. (u) B. A lesion of the right optic nerve results in right eye blindness. (c) C. A lesion of the left optic tract results in dysfunction of the right temporal and left nasal visual fields. This leads to right homonymous hemianopsia. (u) D. A lesion of the right optic tract results in left homonymous hemianopsia.

An asymptomatic 60 year-old female with a 30 pack year history of smoking presents to the clinic requesting a chest x-ray to check for lung cancer. Which of the following do you recommend? A. A low-dose helical computed tomography (CT) B. Carcinoembryonic antigen (CEA) C. Serial chest radiographs to identify early stage malignancy D. No testing and referral for smoking cessation

(u) A. A low-dose helical computed tomography is very sensitive but is expensive, has increase false-positive tests and increases over-diagnosis. Mortality benefit remains to be proved. (u) B. CEA lacks clinical validation as a screening for lung cancer. (u) C. No major advisory organization recommends serial radiograph screening for lung cancer. (c) D. No major advisory organization recommends screening for lung cancer.

A 6 year-old boy is brought to the pediatric clinic by his mother for an evaluation of his asthma. He coughs about 3 days out of the week with at least 2-3 nights of coughing. Which of the following would be the most appropriate treatment for this patient? A. Mast cell stabilizer B. Long acting beta agonist C. Leukotriene receptor antagonist D. Low dose inhaled corticosteroid

(u) A. A mast cell stabilizer is an alternative treatment but not the preferred treatment. (u) B. Long acting beta agonist can be used as adjunctive therapy with an anti-inflammatory. (u) C. Leukotriene receptor antagonists are an alternative treatment but not the preferred treatment. (c) D. Low dose inhaled corticosteroids are the preferred treatment for mild persistent asthma.

A 63 year-old female presents with a complaint of chest pressure for one hour, noticed upon awakening. She admits to associated nausea, vomiting, and shortness of breath. 12 lead EKG reveals ST segment elevation in leads II, III, and AVF. Which of the following is the most likely diagnosis? A. Aortic dissection B. Inferior wall myocardial infarction C. Acute pericarditis D. Pulmonary embolus

(u) A. A patient with aortic dissection will complain of tearing, ripping pain. EKG is often normal, but may reveal left ventricular strain pattern. (c) B. Myocardial infarction often presents with chest pressure and associated nausea and vomiting. ST segment elevation in leads II, III, and AVF are classic findings seen in acute inferior wall myocardial infarction. (u) C. Acute pericarditis presents with atypical chest pain and diffuse ST segment elevation. (u) D. Pulmonary embolism often presents with either no EKG changes or sinus tachycardia. Classically described, rarely seen findings include a large S wave in lead I, a Q wave with T wave inversion in lead III, ST segment depression in lead II, T wave inversion in leads V1-V4 and a transient right bundle branch block.

A 60 year-old male nonsmoker with history of coronary artery disease presents with complaint of worsening dyspnea on exertion for three weeks. He admits to orthopnea and lower extremity edema, but denies chest pain, palpitations, and syncope. The patient's last echocardiogram revealed an ejection fraction of 30%. Which of the following would you most likely find on physical examination? A. Pericardial friction rub B. Third heart sound C. Accentuated first heart sound D. Mid-systolic click

(u) A. A pericardial friction rub is a sign of pericarditis, not heart failure. (c) B. An S3 on physical examination is consistent with heart failure. (u) C. An accentuated first heart sound is noted in tachycardia, short PR interval rhythms, increased cardiac output states and mitral stenosis. (u) D. A mid-systolic click is noted in patients with mitral valve prolapse.

Which of the following clinical findings would be seen in a patient with food poisoning caused by Staphylococcus aureus? A. Ingestion of mayonnaise-based salads 48 hours earlier B. Bloody diarrhea with mucus for one week C. Abdominal cramps and vomiting for 48 hours D. High fever for 1 week

(u) A. A preformed toxin causes staphylococcal food poisoning; it has a short incubation period of 1-8 hours. (u) B. Because Staphylococcus aureus does not invade the mucus, blood and mucus are not seen with this noninflammatory cause of food poisoning. (c) C. Abdominal cramps, nausea, vomiting, and watery diarrhea typically last 1-2 days with staphylococcal food poisoning. (u) D. Staphylococcal food poisoning may be associated with low-grade fever or subnormal temperature.

A patient presents with abdominal pain. On deep palpation of the left lower quadrant the patient notes increased pain in the right lower quadrant. This is noted as a positive A. Psoas sign. B. Murphy's sign. C. Rovsing's sign. D. Obturator sign.

(u) A. A psoas sign is positive when the supine patient notes pain with flexion of the hip against resistance. (u) B. A Murphy's sign is positive when the patient experiences abrupt cessation of deep breathing with palpation of the gallbladder. (c) C. A Rovsing's sign is positive when the patient experiences right lower quadrant pain with deep palpation of the left lower quadrant. (u) D. The obturator sign is positive when the supine patient experiences pain when the right leg is flexed at the hip and knee and the hip is externally and internally rotated.

Which of the following is an indication for hospitalization in a patient who has acute bronchiolitis? A. pulse oximetry of 94% on room air B. children between 4-6 months of age C. moderate tachypnea with feeding difficulties D. hyperinflation and interstitial infiltrates on chest x-ray

(u) A. A pulse oximetry reading of 94% on room air is equivalent to a PaO2 of approximately 80 mm Hg which indicates the child is not in severe respiratory distress. (u) B. Children less than 2 months of age require hospitalization. (c) C. Indications for hospitalization include moderate tachypnea with feeding difficulties. (u) D. Hyperinflation and interstitial infiltrates on chest x-ray are frequently seen with acute bronchiolitis and by themselves are not an indication for hospitalization.

A 19 year-old patient was involved in a motor vehicle crash and brought to the emergency department fully immobilized. The patient sustained multiple blunt injuries to the chest and abdomen. During the trauma assessment, there was no blood at the urethral meatus and a Foley catheter was placed. The urine was positive for blood on the dipstick. Which of the following is the most appropriate diagnostic test? A. Retrograde urethrography B. CT scan of abdomen and pelvis C. Serum haptoglobin D. Urine myoglobin

(u) A. A retrograde urethrogram should be performed when blood is found at the external urinary meatus prior to insertion of a catheter. (c) B. CT scan of the abdomen and pelvis is indicated in blunt trauma including those resulting in hematuria or when renal injury is suspected. (u) C. A decreased serum haptoglobin is seen in hemolysis and does not provide information on renal status. (u) D. A positive test for blood in the absence of red blood cells on urine examination suggests myoglobinuria, and should be confirmed by electrophoresis.

An adult male is noted to have a hard nodule in the helix of his left ear that discharges a small amount of chalky, white crystals. He denies any history of trauma. Which of the following is the most likely diagnosis? A. Rheumatoid nodule B. Keloid C. Cutaneous cyst D. Tophus

(u) A. A rheumatoid nodule does not discharge any material unless subjected to repetitive trauma and tend to be wide spread over the body as well. (u) B. The lack of trauma history and the discharge make keloid unlikely. (u) C. A cutaneous cyst will generally be smooth and may have a comedone associated with it. Any discharge tends to be a foul smelling sebaceous material. (c) D. The discharge of crystalline material is typical for a tophus lesion which represent underlying gout.

A 29 year-old female presents for routine prenatal visit at 26 weeks gestation. She has no complaints and has completed all the initial routine obstetrical diagnostic tests to date. Her physical examination and all initial diagnostic evaluations are unremarkable. Which of the following is the most appropriate diagnostic test to order at this time? A. VDRL B. amniocentesis C. maternal serum alpha-fetoprotein D. 1-hour post-Glucola blood glucose

(u) A. A serological test for syphilis, usually the VDRL, is part of the routine obstetrical tests ordered at a patient's initial prenatal visit. (u) B. Genetic testing should be offered routinely to patients over the age of 35. Amniocentesis is usually performed routinely at 16-18 weeks gestation if indicated. (u) C. Maternal serum alpha-fetoprotein testing is routinely done between 15-18 weeks gestation to screen for neural tube defects. (c) D. Glucose screening, usually with a 1-hour Glucola, is routinely performed between 24-28 weeks gestation to evaluate for glucose intolerance.

A 42 year-old male is brought to the emergency department with a stab wound to his right lateral chest wall. On physical examination, the patient is stable with decreased breath sounds on the right with dullness to percussion. An upright chest x-ray reveals the presence of a moderate pleural effusion. Subsequent diagnostic thoracentesis contains bloody aspirate. Which of the following is the next most appropriate intervention? A. Thoracotomy B. Needle aspiration C. Close observation D. Tube thoracostomy

(u) A. A thoracotomy is indicated in a small percentage of patients that do not respond to IV administration of fluids and evacuation of the hemothorax via tube thoracostomy. (u) B. Needle aspiration as treatment for a hemothorax is not recommended as it fails to adequately drain the bloody fluid. (u) C. Close observation is only indicated for small effusions in a patient that remains hemodynamically stable. (c) D. This patient has a hemothorax. Drainage of a hemothorax is best obtained through insertion of a chest tube (tube thoracostomy).

A 40 year-old male presents with several months of abdominal pain and nausea. Endoscopy reveals an irregular 1 x 2 cm area of loss of rugal folds near the antrum. Biopsies show extensive mucosal and submucosal infiltration by Blymphocytes. The patient is given appropriate treatment and the lesion resolves. Which infectious agent is most likely to be associated with these findings? A. Aspergillus fumigatus B. Helicobacter pylori C. Human papilloma virus D. Salmonella typhi

(u) A. A. fumigatus would be associated with pulmonary, not gastrointestinal disease. (c) B. H. Pylori causes gastric mucosal inflammation with PMN's and lymphocytes. Infection causes nausea and abdominal pain. Inflammation may be confined to the superficial gastric epithelium or may extend deeper resulting in varying degrees of gland atrophy. Eradication of H. Pylori with appropriate therapy leads to resolution of the chronic gastritis. (u) C. Human papilloma virus is primarily a urogenital disorder. (u) D. Salmonella typhi infection would present with fevers, malaise, vomiting, and other symptoms. This diagnosis is best made with blood cultures.

A 26 year-old female required 12 units packed red blood cells during a trauma resuscitation and surgical repair of liver and splenic lacerations. The patient is now 6 hours postoperative and has blood oozing from the suture line and IV sites. There is bloody urine in the Foley bag. Laboratory evaluation demonstrates a platelet count of 10,000/microliter, prolonged prothrombin level, and the presence of fibrin split products. Which of the following is the most likely diagnosis? A. acute ABO incompatibility reaction B. disseminated intravascular coagulation C. exacerbation of idiopathic thrombocytopenia D. inadequate repair of the liver lacerations

(u) A. ABO incompatibility results in immediate hemolysis and shock. (c) B. Disseminated intravascular coagulation is characterized by bleeding from many sites as all coagulation factors are consumed and then broken down, leading to decreased fibrinogen level and platelet count, prolonged PT and PTT, and presence of fibrin split products. (u) C. Idiopathic thrombocytopenia is characterized by decreased platelet count, but coagulation factors are normal. (u) D. Bleeding would be localized only and would result in shock if lacerations of the liver were not repaired properly.

A 65 year-old patient with adenocarcinoma of the colon in remission presents for follow-up. Which of the following tumor markers should be monitored? A. AFP B. CEA C. CA 19-9 D. CA-125

(u) A. AFP is used to monitor recurrence of hepatocellular carcinoma. (c) B. CEA is used to monitor recurrence of colon carcinoma. (u) C. CA 19-9 is used to monitor recurrence of pancreatic carcinoma. (u) D. CA-125 is used to monitor recurrence of ovarian carcinoma

A 12 year-old boy presents to the office with pain in his legs with activity gradually becoming worse over the past month. He is unable to ride a bicycle with his friends due to the pain in his legs. Examination of the heart reveals an ejection click and accentuation of the second heart sound. Femoral pulses are weak and delayed compared to the brachial pulses. Blood pressure obtained in both arms is elevated. Chest x-ray reveals rib notching. Which of the following is the most likely diagnosis? A. abdominal aortic aneurysm B. pheochromocytoma C. coarctation of the aorta D. thoracic outlet syndrome

(u) A. Abdominal aortic aneurysm is usually asymptomatic until the patient has dissection or rupture. It is uncommon in a child. (u) B. Pheochromocytoma classically causes paroxysms of hypertension due to catecholamine release from the adrenal medulla, but does not cause variations in blood pressure in the upper and lower extremities. (c) C. Coarctation is a discrete or long segment of narrowing adjacent to the left subclavian artery. As a result of the coarctation, systemic collaterals develop. X-ray findings occur from the dilated and pulsatile intercostal arteries and the "3" is due to the coarctation site with proximal and distal dilations. (u) D. Thoracic outlet syndrome occurs when the brachial plexus, subclavian artery, or subclavian vein becomes compressed in the region of the thoracic outlet. It is the most common cause of acute arterial occlusion in the upper extremity of adults under 40 years old.

A patient presents with nausea, vomiting, and fever for one day. She describes periumbilical abdominal pain which has settled in her right lower quadrant. On examination she has right lower quadrant pain with deep left lower quadrant palpation. This finding is known as which of the following? A. Psoas sign B. Rebound tenderness C. Rovsing's sign D. Obturator sign

(u) A. Abdominal pain with flexion of the right hip against pressure is a psoas sign. (u) B. Right lower quadrant pain with quick withdrawal of the right lower quadrant palpation is rebound tenderness. (c) C. Right lower quadrant pain with deep left lower quadrant palpation is Rovsing's sign. (u) D. Right lower quadrant pain with internal rotation of the right hip while both the hip and knee are flexed is obturator sign.

A 30 year-old female presents to the emergency room having passed out at home 30 minutes prior to arrival. Her last menstrual period was 6 weeks ago and pregnancy test was reported positive 5 days ago. She started having vaginal bleeding last night. Vital signs are BP 70/40 mmHg, P 140 bpm, R 22. She is pale and diaphoretic. The next step in the evaluation of this patient's condition should be which of the following? A. Abdominal ultrasound B. Culdocentesis C. Laparotomy D. Magnetic resonance imaging

(u) A. Abdominal ultrasound is not the test of choice in this patient. (u) B. Culdocentesis has been replaced by transvaginal ultrasound as diagnostic procedure for suspected ectopic pregnancy. (c) C. Laparotomy is indicated with presumptive diagnosis of ectopic pregnancy in an unstable patient. (u) D. MRI is useful in cases of unusual ectopic locations in stable patients.

A 28 year-old male patient is being treated for depression and has been taking paroxetine (Paxil) for the past two and a half months with a marked improvement in symptoms. The patient reports problems with sexual functioning which he believes is related to the medication. Which of the following is an immediate concern with abrupt discontinuation of the medication? A. increased risk of suicide B. diminished sexual functioning C. risk of drug withdrawal symptoms D. worsened depressive symptoms

(u) A. Abrupt discontinuation of an SSRI in a patient who is relatively asymptomatic does not increase the risk of suicide. (u) B. Abrupt discontinuation may improve sexual functioning after the medication has cleared. (c) C. The greatest risk of abrupt withdrawal of short acting SSRIs is a withdrawal syndrome or withdrawal delirium which represents cholinergic rebound. (u) D. Approximately 50% of patients with a first episode of depression are at risk for relapse at some time during their lifetime; however, relapse is not an immediate risk of abrupt discontinuation.

According to the American College of Cardiology/American Heart Association classification of heart failure, which of the following patients fits the Stage B Classification system? A. Asymptomatic patient with no structural disease or patients who are at high risk for the development of heart failure. B. Asymptomatic patient with structural heart disease. C. Symptomatic patient with structural heart disease. D. Patients with refractory symptoms despite intervention.

(u) A. According to the ACC/AHA 2005 guidelines, patients with risk factors for heart disease but who have yet to develop symptoms are categorized as Stage A. These patients have hypertension and lipid disorders treated along with lifestyle modifications. (c) B. According to the ACC/AHA 2005 guidelines, patients with structural heart disease who have not yet experienced symptoms are classified as Stage B. This is the initial stage in which medication therapy other than just ACE inhibitors are recommended. (u) C. According to the ACC/AHA 2005 guidelines, patients with symptoms and structural heart disease are classified as Stage C. (u) D. According to the ACC/AHA 2005 guidelines, patients with refractory symptoms are classified as Stage D.

A 16 day-old male presents in the office with a history of vomiting after feeding for the past 2 days. The vomiting has become progressively worse and the mother describes it as very forceful, sometimes hitting the floor 6 feet away. She says the neonate is always hungry. On physical examination, it is noted that he is 2 ounces below birth weight, and has a small palpable mass (about 1.5 cm) in the epigastrium. The most likely diagnosis is A. achalasia. B. tracheoesophageal fistula. C. pyloric stenosis. D. Meckel's diverticulum.

(u) A. Achalasia is uncommon under the age of 5 and the child presents with retrosternal pain and dysphagia. (u) B. Tracheoesophageal fistula presents with increased secretions, choking, cyanosis, and respiratory distress within the first few hours of life. (c) C. Pyloric stenosis begins between 2 to 4 weeks of age with vomiting that becomes projectile after each feeding. An olive-size mass can often be felt in the epigastrium. (u) D. Meckel's diverticulum presents with painless rectal bleeding. Vomiting is rare unless obstruction has occurred.

A 60 year-old male presents with a slowly developing facial lesion first noticed 4-5 months ago. He describes it as non-painful and non-pruritic but notes it to be extremely scaly. He denies a history of similar lesions or dermatologic disease. Examination reveals a one centimeter, firm nodule at the right temple with heavy keratinization. There is no fluctuance or skin discoloration. Which of the following is the most likely diagnosis? A. Actinic keratosis B. Squamous cell cancer C. Granuloma annulare D. Merkel cell carcinoma

(u) A. Actinic keratosis lesions can be very scaly but are generally flat in appearance not nodular. (c) B. This is a very typical scenario for a squamous cell cancer occurring in a sun exposed area, with slow development and heavy keratinization. (u) C. Granuloma annulare is a self-limited dermatosis occurring primarily on the distal extremities is more common in young adults and children. (u) D. Merkel cell carcinoma lesions are typically discolored and non-keratinizing.

A 38 year-old female with history of coarctation of the aorta repair at the age of two presents with fevers for four weeks. The patient states that she has felt fatigued and achy during this time. Maximum temperature has been 102.1 degrees F. She denies cough, congestion, or other associated symptoms. Physical examination reveals a pale tired appearing female in no acute distress. Heart reveals a new grade III-IV/VI systolic ejection border at the apex, and a II/VI diastolic murmur at the right sternal border. What is the most likely diagnosis? A. Acute myocardial infarction B. Bacterial endocarditis C. Acute pericarditis D. Restrictive cardiomyopathy

(u) A. Acute MI presents with complaint of chest pain, SOB, not with fever and myalgias. (c) B. Bacterial endocarditis presents as febrile illness lasting several days to weeks, commonly with nonspecific symptoms, echocardiogram often reveals vegetations on affected valves. (u) C. Pericarditis does not present with systolic or diastolic murmur or vegetation, more commonly pericardial friction rub would be noted. (u) D. Restrictive cardiomyopathy will show impaired diastolic filling on echocardiogram and is not associated with fever.

An electrocardiogram (ECG) shows a sinus rhythm with varying T wave heights, axis changes every other beat and a wandering baseline. Which of the following is most likely the diagnosis? A. Artifact B. Digoxin toxicity C. Pericardial effusion D. Poor lead placement

(u) A. Artifact could show a wandering baseline, but not the distinct axis changes. (u) B. Digoxin toxicity can cause bidirectional tachycardia, but not electrical alternans. (c) C. This ECG pattern best represents pericardial effusion due to a swinging heart in fluid and is known as electrical alternans. (u) D. Poor lead placement would show different patterns per the leads.

A football player complains of burning pain, numbness, and tingling extending from the left shoulder down into the hand after he tackled a player. These symptoms resolved spontaneously in minutes. Following resolution of the symptoms, he has full strength and normal sensation in the left arm. What is the most likely etiology of his symptoms? A. Acute muscle strain B. Mild concussion C. Stretching of nerve roots and brachial plexus D. Thoracic outlet obstruction

(u) A. Acute cervical muscle strain might result from the mechanism of injury described but symptoms would not quickly resolve. (u) B. Concussion involves generalized symptoms such as loss of consciousness but would not affect one limb exclusively. (c) C. Brachial plexus neurapraxia, commonly called "stinger" injuries, results from stretching of the cervical nerve roots and brachial plexus by a mechanism such as that described in this question. (u) D. Thoracic outlet syndrome is most commonly caused by cervical rib and is usually only symptomatic when the arm is elevated.

A 34 year-old man comes to the urgent care with fever of 102 degrees F, chills, and perineal pain for the past 2 days. In addition, he has difficulty voiding, dysuria, frequency, urgency, and nocturia. Which of the following physical examination findings are consistent with this history? A. Mild suprapubic tenderness B. Scrotal swelling and tenderness C. Prostate tender, warm, swollen D. Costovertebral angle tenderness

(u) A. Acute cystitis symptoms include dysuria, urgency and frequency. On physical exam, the patient is usually afebrile and may have suprapubic tenderness. (u) B. Acute epididymitis is characterized by symptoms of cystitis or urethritis (urethral discharge, pain at the tip of the penis) with pain in the scrotum that may radiate to the flank. (c) C. The diagnosis of acute prostatitis is supported by fever, irritative voiding symptoms, perineal pain and exquisite tenderness of the prostate on rectal exam. (u) D. Acute pyelonephritis is characterized by fever, flank pain and irritative voiding symptoms.

A 54 year-old man comes to the urgent care because he was awoken suddenly from his sleep this morning with severe left flank pain radiating to his left testicle with associated nausea and vomiting. He is afebrile and vital signs are normal. He is constantly moving to find a comfortable position. On physical examination, left flank tenderness is noted with no direct testicular tenderness. Urinalysis reveals a pH of 5.5 and microscopic hematuria, but is otherwise unremarkable. Which of the following is the most likely diagnosis? A. Acute epididymitis B. Appendicitis C. Diverticulitis D. Nephrolithiasis

(u) A. Acute epididymitis is characterized by symptoms of cystitis or urethritis (urethral discharge, pain at the tip of the penis) with pain in the scrotum that may radiate to the flank. (u) B. Appendicitis is associated with fever, abdominal pain and peritoneal signs. (u) C. Diverticulitis presents with left lower quadrant pain that does not radiate to the testicle. The patient would not have hematuria. (c) D. Nephrolithiasis usually presents as a sudden onset of colicky flank pain with associated nausea and vomiting. Urinalysis often reveals gross or microscopic hematuria.

A 52 year-old female presents with complaints of intermittent episodes of dizziness, tinnitus, and hearing loss in the right ear for 6 months. She describes the dizziness as the "room spinning around her," with the episodes typically lasting for 2 to 4 hours. Physical examination reveals horizontal nystagmus and right ear hearing loss, but the remainder of the examination is unremarkable. Which of the following is the most likely diagnosis? A. Acute labyrinthitis B. Positional vertigo C. Acoustic neuroma D. Ménière's syndrome

(u) A. Acute labyrinthitis typically presents with an acute onset of continuous vertigo that lasts several days to a week and is associated with nausea and vomiting. It does not have any associated auditory or neurologic symptoms. (u) B. Positional vertigo occurs following changes in head positioning with very brief, less than 1 minute, episodes. Nystagmus occurs following the position change. (u) C. Acoustic neuroma typically presents with hearing loss and tinnitus. The neuroma grows slowly and central compensatory mechanisms can prevent or minimize the vertigo. Vertigo, when present, is continuous and not episodic. (c) D. Ménière's syndrome usually presents with episodes of vertigo that last from 1 to 8 hours, sensorineural hearing loss and tinnitus.

A 69 year-old female presents to the clinic complaining of fatigue. Physical examination reveals lymphadenopathy, splenomegaly and pale conjunctiva. The remainder of the examination is unremarkable. CBC reveals a normochromic normocytic anemia. White blood cell count is 45,000/mm3, with a differential of 77% mature lymphocytes, 3% eosinophils, 18% segmented neutrophils and 1% basophils and monocytes. Platelets appear adequate in number. What is the most likely diagnosis? A. Acute myelocytic leukemia B. Acute lymphocytic leukemia C. Chronic lymphocytic leukemia D. Chronic myelocytic leukemia

(u) A. Acute leukemias present with circulating blast cells with pancytopenia. (u) B. See A for explanation. (c) C. Chronic lymphocytic leukemia typically occurs after age 50 presenting with lymphocytosis with lymphocytes that appear small and mature. (u) D. See C for explanation.

A 48 year-old male complains of weakness and general malaise for about 2 months. Patient denies any recent illness and does not take any medications. Physical examination reveals a pale looking male in no acute distress. His heart rate is 110 beats/minute without a murmur and his abdominal examination reveals hepatosplenomegaly. A CBC reveals the WBC to be 62,000/microliter, Hgb is 8.3 gms/dl, Hct is 24.6%. A differential reveals a predominance of monoblasts and promyelocytes with Auer rods present. What is the most likely diagnosis? A. Acute lymphocytic leukemia B. Acute myelogenous leukemia C. Chronic lymphocytic leukemia D. Chronic myelogenous leukemia

(u) A. Acute lymphocytic leukemia (ALL) is a leukemia most often seen in children with lymphoblasts predominating. Adults who develop ALL usually have a prolymphocytic cell presentation. (c) B. Acute myelogenous leukemia (AML) is a leukemia that presents in adults with a cell lineage of the immature granulocytic cells seen in the peripheral blood e.g. myeloblasts, promyelocytes. Auer rods are commonly seen in this condition. (u) C. Chronic lymphocytic leukemia (CLL) is a leukemia that presents in older adults with WBC counts up to 100,000/mm3. On peripheral blood smears the cell that predominates is a mature lymphocyte. (u) D. Chronic myelogenous leukemia (CML) is a leukemia that presents in adults with a cell lineage of mature cell lines with a marked increase in basophils and eosinophils. These cells are also known for having the Philadelphia chromosome.

In a patient with metabolic acidosis due to ingestion of methanol, which substance below is considered therapeutic? A. Salicylates B. Paraldehyde C. Ethylene glycol D. Ethanol

(u) A. Salicylates, paraldehyde, ethylene glycol all cause metabolic acidosis with an elevated anion gap. (u) B. See A for explanation. (u) C. See A for explanation. (c) D. Ethanol, along with hemodialysis and supportive measures, is indicated for metabolic acidosis caused by methanol ingestion.

In the treatment of alcohol-related hypoglycemia, thiamine is administered before glucose to prevent which of the following? A. Acute renal failure B. Wernicke's encephalopathy C. Korsakoff psychosis D. Mesenteric thrombosis

(u) A. Acute renal failure is prevented by adequate hydration allowing appropriate blood flow to the kidney. It is prevented by hydration with volume expanders rather than by thiamine and/or glucose. (c) B. Alcoholics are typically deficient in thiamine. If glucose alone is given to treat hypoglycemia, Wernicke's encephalopathy can be precipitated since thiamine is not available for nutritional purposes when glucose is replaced. Therefore, thiamine is always given prior to glucose until a satisfactory diet can be given in order to prevent Wernicke's encephalopathy. (u) C. Korsakoff psychosis is a sequelae of Wernicke's encephalopathy so if thiamine is used to prevent Wernicke's encephalopathy, its sequelae, Korsakoff's psychosis will also be prevented. (u) D. Mesenteric thrombosis is mostly a complication of atherosclerosis or embolism to the mesenteric artery. It is more common in low flow states such as hypotension and can somewhat be prevented by ensuring adequate hydration. It is not dependent on thiamine or glucose, rather it just needs volume expanders.

A 25 year-old female presents with right lower quadrant pain, right flank pain, nausea, and vomiting. Her temperature is 39.6 degrees C. There is right CVA tenderness and RLQ tenderness. Pelvic exam is unremarkable. Urinalysis reveals pH 7.0, trace protein, negative glucose, negative ketones, positive blood, and positive nitrates. Specific gravity is 1.022. Microscopic shows 102 RBCs/HPF, 50-75 WBCs/HPF, rare epithelial cells, and WBC casts. The most likely diagnosis is A. acute salpingitis. B. nephrolithiasis. C. acute pyelonephritis. D. appendicitis.

(u) A. Acute salpingitis would be suggested if pelvic exam abnormalities were present. (u) B. Nephrolithiasis does not usually present with fever or casts. Urinalysis will have RBCs present. (c) C. Acute pyelonephritis presents with flank pain, fever, and generalized muscle tenderness. Urinalysis shows pyuria with leukocyte casts. (u) D. This scenario is consistent with acute pyelonephritis, not acute appendicitis.

A 60 year-old female recently discharged after an 8 day hospital stay for pneumonia presents complaining of pain and redness in her right arm. The patient thinks this was the area where her IV was placed. The patient denies fever or chills. Examination of the area reveals localized induration, erythema and tenderness. There is no edema or streaking noted. Which of the following is the most likely diagnosis? A. Acute thromboembolism B. Thrombophlebitis C. Cellulitis D. Lymphangitis

(u) A. Acute thromboembolism is usually associated with edema of the extremity and warm temperature. Thromboembolism is unusual after thrombophlebitis. (c) B. This patient's signs and symptoms are consistent with thrombophlebitis. Short-term venous catheterization of a superficial arm vein is commonly the cause and thrombophlebitis characterized by dull pain, induration, redness and tenderness along the course of the vein. (u) C. Cellulitis is usually associated with fever, increased warmth over the affected area and associated edema. (u) D. Lymphangitis is associated with fever, malaise, chills, and streaking.

A 25 year-old man presents with odynophagia and dysphagia. On endoscopic examination, small, white, patches with surrounding erythema are noted. Silver stain is positive for hyphae. The best treatment option for this patient is A. acyclovir (Zovirax). B. omeprazole (Prilosec). C. fluconazole (Diflucan). D. penicillin G.

(u) A. Acyclovir is an antiviral used in the treatment of herpes esophagitis. (u) B. Omeprazole is a proton pump inhibitor used in the treatment of gastroesophageal reflux disease with esophageal ulceration and peptic ulcer disease and is not indicated in the treatment of infectious esophagitis. (c) C. The patient has Candida esophagitis and the treatment of choice is fluconazole. (u) D. Penicillin G is an antibiotic and is not effective against fungal infections.

A 25 year-old man presents with odynophagia and dysphagia. On endoscopic examination, small, white, patches withsurrounding erythema of the esophagus are noted. Silver stain is positive for hyphae. The best treatment option for this patient is A. acyclovir (Zovirax). B. omeprazole (Prilosec). C. fluconazole (Diflucan). D. penicillin G.

(u) A. Acyclovir is an antiviral used in the treatment of herpes esophagitis. (u) B. Omeprazole is a proton pump inhibitor used in the treatment of gastroesophageal reflux disease with esophageal ulceration and peptic ulcer disease and is not indicated in the treatment of infectious esophagitis. (c) C. The patient has Candida esophagitis and the treatment of choice is fluconazole. (u) D. Penicillin G is an antibiotic and is not effective against fungal infections.

A 70 year-old presents with headache and neck stiffness. On physical exam, the patient is febrile, Kernig's sign is present, and no rash is noted. A spinal tap reveals a white count of 250/cm3 with 100% neutrophils, total protein 250mg/dL, and glucose 35 mg/dL. Which of the following is the most appropriate treatment? A. Acyclovir (Zovirax) B. Fluconazole (Diflucan) C. Ampicillin and ceftriaxone (Rocephin) D. Penicillin and chloramphenicol (Chloromycetin)

(u) A. Acyclovir is used to treat meningitis secondary to herpes. Viral meningitis presents with increase number of lymphocytes and elevated glucose in the CSF . (u) B. Fluconazole is used to treat fungal meningitis. Fungal meningitis, typically noted in immunocompromised hosts, presents with increase number of lymphocytes in the CSF. (c) C. Ampicillin and ceftriaxone is used to treat bacterial meningitis, secondary to Listeria monocytogenes , which is common in the elderly. Ceftriaxone will cover other common etiologic agents such as Streptococcal pneumonia (u) D. Penicillin and chloramphenicol is used to treat bacterial meningitis, secondary to Neisseria meningitidis. Bacterial meningitis due to N

39. Diagnosis/Endocrinology A 36 year-old patient presents with poor wound healing following sutures to a hand laceration two weeks prior. She had regular periods up to 4 months ago, then they ceased unexpectedly. Examination reveals central obesity, a prominent dorso-cervical fat pad and hirsutism. Which of the following is the most likely diagnosis in this patient? A. Addison disease B. Cushing syndrome C. Paget disease D. Turner syndrome

(u) A. Addison disease or chronic adrenal insufficiency more often presents with weight loss, nausea and vomiting, but can include amenorrhea. (c) B. This patient is exhibiting signs and symptoms of Cushing syndrome or hypercortisolism, which include poor wound healing, central obesity, moon facies, buffalo hump, hirsuitism and amenorrhea. (u) C. Paget disease of the bone (osteitis deformans) is often asymptomatic and usually presents with bone pain. (u) D. Turner syndrome (gonadal dysgenesis) presents with primary amenorrhea, short stature and normal growth hormone levels.

A 30 year-old female presents with amenorrhea, proximal muscle weakness, weight gain, and increased emotional lability. Her physical examination consists of central obesity, full face, and protuberant abdomen with thin extremities. Her blood pressure is 150/96 mmHg. Which of the following is the most likely diagnosis? A. Addison disease B. Hypothyroidism C. Cushing syndrome D. Pituitary insufficiency

(u) A. Addison disease symptoms may include weakness but will show weight loss, anorexia, nausea, anxiety, hyperpigmentation over the knuckles, elbows, knees and sometimes vitiligo. (u) B. Hypothyroidism does not present with central obesity, thin extremities or moon face. (c) C. A classic presentation of cushing syndrome is moon facies, buffalo hump, protuberant abdomen, thin extremities, weakness, back and head ache, hypertension, acne and superficial skin infections. There are typically purple striae and easy bruisability. (u) D. Patients with pituitary insufficiency present with short stature and growth failure. Typically the presentation is at an earlier age.

Which histologic type of lung cancer is typically centrally located? A. Adenocarcinoma B. Bronchoalveolar C. Large cell D. Squamous cell

(u) A. Adenocarcinoma of the lung typically presents as a peripheral lesion. (u) B. Bronchoalveolar carcinoma, actually a subset of adenocarcinoma of the lung, typically presents as a peripheral lesion. (u) C. Large cell lung cancers usually develop as peripheral lesions. (c) D. Most squamous cell lung cancers are centrally located.

You are performing a routine gynecological examination on a 49 year-old female. She states that for the last six months her periods have been getting heavier and lasting for seven days duration. She also tells you that she has been experiencing urinary frequency and constipation. Her abdominal exam is unremarkable. On pelvic examination you feel a slightly enlarged uterus and a left adnexal mass that moves with the uterine fundus. Which of the following is the most likely diagnosis? A. Adenomyosis B. Uterine fibroids C. Ovarian neoplasm D. Diverticular disease in the colon

(u) A. Adenomyosis occurs when endometrial glands and stroma grow into the muscle of the uterus. Patients often have severe secondary dysmenorrhea and menorrhagia; the uterus will be symmetrically enlarged and often "boggy". (c) B. Patients with uterine fibroids often have menorrhagia and/ or metrorrhagia. On examination, the uterus will be larger, and irregularly-shaped. If a mass is present and it moves with the uterus, it is suggestive of a fibroid. (u) C. It is important to consider an ovarian neoplasm in a patient with a palpable adnexal mass, however, most adnexal masses are asymptomatic and not associated with menstrual changes. (u) D. Left-sided bowel disease is more common in older patients and would not be associated with menstrual changes.

Which of the following classes of medications is most likely to cause a persistent cough? A. Tricyclic antidepressants B. Calcium channel blockers C. Beta-adrenoceptor blocking agents D. Angiotensin converting enzyme inhibitors

(u) A. Adverse effects of tricyclic antidepressants are due to blockage of acetylcholine receptors causing such symptoms as blurred vision, dry mouth, urinary retention and constipation. (u) B. Side effects are uncommon with calcium channel blockers, but include constipation, dizziness, headache and a feeling of fatigue. (u) C. Common adverse effects of beta-adrenoceptor blocking agents include bradycardia and central nervous system symptoms, such as fatigue or insomnia. (c) D. Common side effects of angiotensin converting enzyme inhibitors (ACE inhibitors) include a dry cough.

Patients with long-term exposure to silica, coal dust, and asbestos may develop which of the following as complications? A. Airway hyperreactivity B. Epithelial hyperplasia C. Pulmonary fibrosis D. Upper airway obstruction

(u) A. Airway hyperreactivity is classically seen with asthma and while acute exposure to occupational mineral dusts may cause airway hyperreactivity, this does not persist in long-term exposures. (u) B. Epithelial hyperplasia is one of the mechanisms involved in the pathogenesis of chronic bronchitis and is not involved in the pneumoconioses. (c) C. The principal processes in the pathogenesis of this set of diseases is inflammation and subsequent fibrosis. (u) D. The upper airway is not involved in this disease process

Which of the following is a major risk factor for an ectopic pregnancy? A. alcohol intake B. young maternal age C. history of salpingitis D. low dose oral contraceptive use

(u) A. Alcohol intake has not been associated with an increased risk of ectopic pregnancy. (u) B. Advanced maternal age, not younger maternal age, is an established risk factor for ectopic pregnancy. (c) C. A previous history of salpingitis is a major risk factor for ectopic pregnancy since damage to the fallopian tube prevents the fertilized ovum from reaching the uterus prior to implantation. (u) D. Oral contraceptive use prevents ovulation and therefore decreases the over-all risk of pregnancy, including ectopic pregnancies. While high levels of estrogen and progesterone are thought possibly to increase the risk of ectopic pregnancy because these hormones slow the movement of the fertilized egg through the fallopian tube, no proven association has been established.

A male patient presents for a routine physical examination. He denies chronic health problems, regular medication use or previous surgeries. He exercises 4-5 times weekly, does not use tobacco products, and consumes alcohol in moderation. His last intake of alcohol was two weeks ago while on a trip to Mexico. His review of systems and physical examination are both negative. Routine labs were drawn which were significant for the following: Total serum bilirubin 3.5 mg/dL (0.3-1.0 mg/dL), direct bilirubin 0.2 mg/dL (0.1-0.3 mg/dL), AST 35 U/L (0-35 U/L), ALT 30 U/L (0-35 U/L), Alkaline Phosphatase 48 U/L (30-120 U/L) and GGT 12 U/L (1-94 U/L). What is the most likely diagnosis in this patient? A. Alcoholic hepatitis B. Crigler-Najjar syndrome C. Gilbert's syndrome D. Wilson's disease

(u) A. Alcoholic hepatitis presents with elevated ALT and AST. (u) B. The Crigler-Najjar syndrome is typified by an isolated elevation in the unconjugated serum bilirubin and usually causes the bilirubin to be elevated at higher levels (6-45 mg/dL). (c) C. Gilbert's syndrome is a relatively common cause of mild isolated elevations in indirect serum bilirubin. (u) D. Wilson's disease is a disorder of impaired biliary copper excretion which among many other problems will cause a rise in total serum bilirubin but also significant elevations in AST.

Which of the following findings is most consistent with cataracts? A. conjunctival injection B. poorly visualized optic disc C. central visual field loss D. arous senilis

(u) A. Conjunctival injection is associated with conjunctivitis and other inflammatory conditions of the eye. (c) B. Cataracts are caused by opacification of the crystalline lens, and this decreases the amount of light that enters the eye. It is difficult to see through the lens from either direction, and thus, the optic disc is poorly visualized on examination. (u) C. Central field loss is associated with macular degeneration. (u) D. While arcus senilis may be seen in geriatric patients and is not associated with cataracts.

A 63 year-old female with history of diabetes mellitus presents for blood pressure follow-up. At her last two visits her blood pressure was 150/92 and 152/96. Today in the office her blood pressure is 146/92. Recent blood work shows a Sodium 140 mEq/L, Potassium 4.2 mEq/L, BUN of 23 mg/dL, and Creatinine of 1.1 mg/dL. Which of the following is the most appropriate initial medication in this patient? A. Terazosin (Hytrin) B. Atenolol (Tenormin) C. Lisinopril (Zestril) D. Hydrochlorothiazide (HCTZ)

(u) A. Alpha blockers are not the treatment of choice in a diabetic with hypertension. (u) B. Patients with hypertension and diabetes may require a Beta blocker, but it should be added to an ACE inhibitor if the ACE inhibitor is ineffective on its own. (c) C. ACE inhibitors should be part of the initial treatment of hypertension in diabetics because of beneficial effects in diabetic nephropathy and is the most appropriate initial medication. (u) D. Patients with hypertension and diabetes mellitus may require a diuretic, but it should be added to an ACE inhibitor if the ACE inhibitor is ineffective on its own.

When treating a patient with BPH who complains of increased frequency, urgency, decreased force of stream and hesitancy with an alpha-adrenergic blocking agent, the patient should be warned of what possible side effect? A. hypertensive crisis B. postural hypotension C. development of prostate cancer D. development of testicular cancer

(u) A. Alpha-adrenergic agents do not cause hypertension. (c) B. This class of medications can cause postural hypotension as a result of the decrease in peripheral vascular resistance and lower arterial blood pressure by causing relaxation of the arterial and venous smooth muscle. (u) C. Alpha-adrenergic agents have no effect on testosterone levels, which have been suggested as a possible link to the development of prostate cancer. (u) D. Alpha-adrenergic agents have no correlation to the development of testicular cancer.

: (8) 11 12. Diagnostic Studies/Orthopedics/Rheumatology An afebrile 50 year-old male presents with an acute onset of an exquisitely painful first metatarsophalangeal joint. Examination reveals a red and swollen joint with tenderness on palpation. Which of the following test results will most likely be found in this patient? A. Hyperuricemia B. Negative birefringent crystals C. Evidence of a septic joint with needle aspiration D. Calcium pyrophosphate dehydrate crystals

(u) A. Although 75% of patients with gout have hyperuricemia, few patients with hyperuricemia develop gout. (c) B. Negative birefringence of the needle-shaped crystals associated with gout are seen by a yellow colorization on polarized light microscopy. (u) C. A coexisting septic joint is possible, but not typical of gout. (u) D. Calcium pyrophosphate dehydrate crystal deposition is a gout-like syndrome that typically affects the knee and is also known as pseudogout.

A 75 year-old male with chronic hypertension and history of coronary artery disease is assessed in the office for routine evaluation. His hypertension is well-controlled with metoprolol (Lopressor) and hydrochlorothiazide (Diuril). Electrocardiogram reveals U waves. Which of the following is the most likely cause for U waves in this patient? A. Beta blocker therapy B. Hypocalcemia C. Hypokalemia D. Medication noncompliance

(u) A. Although beta blockers slow conduction through the AV node, they are not responsible for causing U waves in this patient. (u) B. Patients maintained on thiazide diuretics actually have a decrease in calciuria and are therefore, not at risk for the development of hypocalcemia. Additionally, calcium disturbances lengthen the QRS complex and may affect the QT interval but, they are not associated with the formation of U waves. (c) C. U waves are associated with hypokalemia and also with the ingestion of certain medications such as quinidine, procainamide, and disopyramide. (u) D. See C for explanation.

A 74 year-old male presents with one month history of right shoulder pain without any known precipitant. His pain involves an area from the right paraspinous musculature to the right deltoid with occasional radiation down the arm. Pain is worse with movement of the shoulder and is not relieved by acetaminophen. He reports numbness of the right index finger and thumb. Physical examination of the shoulder is limited by pain. There is decreased grip strength and absent pinprick sensation in the index finger and thumb. Relexes are normal. What is the most appropriate initial study to obtain? A. EMG with nerve conduction B. MRI of the right shoulder C. Radiographs of the neck D. Radiographs of the right shoulder

(u) A. Although electromyography and nerve conduction studies would be helpful in localizing the nerves involved, this is not an appropriate first-line diagnostic test. (u) B. Although there is pain in the shoulder, this patient presents with cervical radiculopathy and shoulder studies are not appropriate. (c) C. Cervical radiculopathy may be due to intervertebral foraminal osteophytes which will be identified on plain film radiographs of the neck making this the appropriate first step in evaluation. (u) D. Although there is pain in the shoulder, this patient presents with cervical radiculopathy and shoulder studies are not appropriate.

A 60 year-old male with hypertension is brought to the emergency department 30 minutes after the sudden onset of severe chest pain that radiates to his back and arms. His blood pressure is 180/80 mmHg in his left arm; no blood pressure reading can be obtained from the right arm. ECG shows sinus tachycardia with left ventricular hypertrophy. A high pitched decrescendo diastolic murmur is heard along the left mid-sternal border. Which of the following is the most likely diagnosis? A. Acute myocardial infarction B. Aortic dissection C. Pulmonary embolism D. Right subclavian arterial embolus

(u) A. Although included as part of the differential the presentation is not consistent with AMI. ECG may show ST changes and a murmur of mitral regurgitation may be present with papillary muscle rupture. (c) B. This is a classic presentation for aortic dissection. (u) C. Patients will also present with shortness of breath, feelings of impending doom and chest pain that varies with respirations. (u) D. Arterial embolus will present with symptoms related to the location of the occlusion. Pain and paresthesias are usually the earliest symptoms.

A 56 year-old female four days post myocardial infarction presents with a new murmur. On examination the murmur is a grade 3/6 pansystolic murmur radiating to the axilla. She is dyspenic at rest and has rales throughout all her lung fields. Blood pressure is 108/68 mmHg, pulse 70 bpm. Which of the following would be the definitive clinical intervention? A. Intra-aortic balloon counterpulsation B. Mitral valve replacement C. Coronary artery bypass surgery D. Immediate fluid bolus

(u) A. Although part of the primary treatment to reduce mitral regurgitation, it is not definitive. (c) B. MVR is the definitive intervention to correct MR caused by papillary muscle rupture. (u) C. CABG may be necessary if significant blockage is found, but it will not correct the mitral regurgitation. (u) D. A fluid bolus is indicated if the patient is hypotensive.

A 56 year-old male presents to the office with a history of abdominal aortic aneurysm. He was told that he will need ongoing evaluation to assess whether the aneurysm is expanding. What is the recommended study to utilize in this situation? A. plain film of the abdomen B. serial abdominal exam C. ultrasound of the abdomen D. angiography of the abdominal aorta

(u) A. Although some abdominal aortic aneurysms are calcified, abdominal radiography may demonstrate the calcified outline of the aneurysm. However, about 25% of aneurysms are not calcified and cannot be visualized by plain x-ray. (u) B. Serial abdominal exams are not sensitive in detecting progression of abdominal aortic aneurysms. (c) C. An abdominal ultrasound can delineate the transverse and longitudinal dimensions of an abdominal aortic aneurysm and may detect mural thrombus. Abdominal ultrasound is best used to screen patients at risk for the development of this condition. (u) D. Contrast aortography is used commonly for the evaluation of patients with aneurysms before surgery, but it has no role in the serial assessment of patients being followed on a chronic basis.

A newborn is being evaluated for perioral cyanosis while feeding associated with sweating. Vital signs are rectal temperature, 37.8 degrees C (100 degrees F), blood pressure 80/45 mmHg, pulse 180/min, and respirations 40/min. A grade 3/6 harsh systolic ejection murmur with a single loud S2 is heard at the left upper sternal border. Electrocardiogram (ECG) shows right ventricular hypertrophy with right axis deviation. Chest x-ray shows a boot-shaped heart and decreased pulmonary vascular markings. Which of the following is the most likely diagnosis? A. Atrial septal defect B. Total anomalous pulmonary venous return C. Coarctation of the aorta D. Tetralogy of Fallot

(u) A. Although the murmur may be consistent with an ASD with pulmonary hypertension the chest x-ray would not show decreased pulmonary vascular markings. With a large left to right shunt large pulmonary arteries and increased vascularity would be seen. (u) B. The murmur for TAPVR is a soft systolic murmur at the left upper sternal border with a split S2 in addition to a short mid-diastolic murmur at the low left sternal border. (u) C. Cyanosis is usually not the presenting sign for coarctation of the aorta. Infants may present with heart failure, ECG will show evidence of LVH. (c) D. This is a common presentation for tetralogy of fallot.

A 20 year-old male presents with a hard mass on the testicle. There has been no previous infection or trauma to the area. Which of the following is the initial diagnostic evaluation to pursue? A. serum alpha fetoprotein levels B. serum human chorionic gonadotropin hormone C. CT scan of the pelvis D. ultrasound of the testicles

(u) A. Although tumor markers are useful to follow patients with testicular cancer, they are not used as the initial screen. (u) B. See A for explanation. (u) C. CT scanning of the pelvis is most commonly used in the evaluation for metastatic disease and not used in the initial screen for testicular cancer. (c) D. An ultrasound of the testes will enable the clinician to discriminate between testicular tumors and epididymitis, orchitis, hematomas, hydroceles, and infiltrative diseases of the testes.

What is the recommended initial first trimester screening test for fetal aneuploidy? A. Amniocentesis B. Pregnancy-associated plasma protein A (PPA), beta-hCG, and ultrasound of nuchal transparency C. Maternal serum alpha feto protein, beta-hCG, estriol, and inhibin-A D. Level II ultrasound

(u) A. Amniocentesis is a diagnostic test not a screening test. (c) B. Pregnancy-associated plasma protein A (PPA), beta-hCG, and ultrasound of nuchal transparency are screening tests done at 10-13 weeks of gestation. (u) C. Maternal serum alpha feto protein, beta hCG, estriol, and inhibin-A, are included in the quad screen which is a screening test done in the second trimester from 15-20 weeks of gestation. (u) D. Level II ultrasound is done in the second trimester.

A patient presents complaining of right ear pain and itching. On physical examination you note erythema and edema of the right ear canal with purulent exudate. Palpation of the tragus and manipulation of the auricle is painful. The tympanic membrane is not well visualized. Which of the following is the treatment of choice for this patient? A. Amoxicillin B. Aqueous irrigation C. Tympanocentesis D. Polymyxin B sulfate

(u) A. Amoxicillin is the initial treatment of choice in acute otitis media, not otitis externa. (u) B. Aqueous irrigation is not indicated in the treatment of otitis externa. (u) C. Tympanocentesis is not indicated in the treatment of otitis externa. (c) D. Polymyxin B sulfate is the treatment of choice for otitis externa.

A 19 year-old man comes to the office because he has had a fever, frequent urination, urgency, dysuria and scrotal pain for the past 2 days. On physical examination, he has a temperature of 101 degrees F, scrotal swelling, and purulent urethral discharge is visible on penile examination. Gram stain of the discharge shows gram negative intracellular diplococci. Which of the following is the most appropriate antibiotic therapy? A. Ampicillin (Principen) B. Ciprofloxacin (Cipro) C. Ceftriaxone (Rocephin) D. Nitrofurantoin (Macrobid)

(u) A. Ampicillin can be prescribed in conjunction with gentamicin to treat acute pyelonephritis. (u) B. Ciprofloxacin can be prescribed to treat acute cystitis or chronic bacterial prostatitis. (c) C. Ceftriaxone is an effective treatment for acute epididymitis caused by Neisseria gonorrhoeae. (u) D. Nitrofurantoin can be prescribed to treat acute cystitis.

Which of the following is the most effective prophylaxis for traveler's diarrhea? A. Metronidazole B. Ciprofloxacin C. Tetracycline D. Ampicillin

(u) A. Ampicillin, tetracycline, and metronidazole do not cover the common causes of traveler's diarrhea. (c) B. Options for prophylaxis of traveler's diarrhea include norfloxacin, ciprofloxacin, ofloxacin, or trimethoprim/sulfamethoxazole. (u) C. See A for explanation. (u) D. See A for explanation.

: (28) 63 158. Clinical Intervention/Orthopedics/Rheumatology Which of the following is the initial treatment of choice for a Baker's cyst? A. Needle aspiration B. Intra-articular injection of corticosteroid C. Surgical excision D. Incision and drainage

(u) A. See B for explanation. (c) B. Intra-articular injection of corticosteroid to decrease the inflammatory effect is the indicated treatment of a synovial cyst, such as a Baker's cyst. (u) C. See B for explanation. (u) D. See B for explanation.

: (8) 168. Diagnostic Studies/Cardiology A 67 year-old presents as a new patient complaining of progressive dyspnea. Examination reveals 3+ pitting edema of the lower extremities bilaterally and wheezing audible in lower lung fields bilaterally. Which of the following tests is the most appropriate initial study in the patient? A. Electrocardiogram B. Brain natriuretic peptide C. Spiral computed tomography D. Spirometry

(u) A. An electrocardiogram may need to be included in a patient with heart failure however this would not help to differentiate the cause of the dyspnea, and therefore would not be an appropriate initial study. (c) B. The most common complaint of patients with heart failure is progressive shortness of breath. Peripheral edema is most suggestive of right-sided failure which is most frequently caused by left-sided failure. Serum BNP is a powerful prognostic marker that adds to clinical assessment in differentiating dyspnea due to heart failure from non-cardiac causes. (u) C. Spiral CT would be indicated in the work up of acute dyspnea suspected to be due to pulmonary embolism. (u) D. Spirometry would be indicated for the evaluation dyspnea believed to be associated with pulmonary disease (i.e. COPD, emphysema).

A patient presents with a long history of dyspepsia refractory to maximum appropriate therapy. A recent upper GI series revealed multiple gastric and duodenal ulcerations with prominent mucosal folds. What diagnostic study would be confirmatory of this patient's suspected diagnosis? A. EGD with duodenal biopsy B. Serum gastrin level C. Helicobacter pylori serology D. Abdominal CT scan

(u) A. An endoscopic biopsy of the duodenum may be necessary in some cases to ensure the absence of cancer, etc., but a biopsy in this area is not confirmatory for Zollinger-Ellison Syndrome, which is highly suspect, nor does it allow samples of the gastric mucosa to be studied. (c) B. Serum gastrin levels can be elevated for many reasons; however significant elevations in a fasting state as well as with the secretin stimulation test are confirmatory for Zollinger-Ellison syndrome which is highly suspect in this case. (u) C. Helicobacter pylori can cause symptoms refractory to medical management and an infection can occur concomitantly with Zollinger-Ellison syndrome, but it does not confirm the diagnosis of a gastrinoma or similar hypersecretory disease. (u) D. An abdominal CT scan is not specific enough to confirm the diagnosis of Zollinger-Ellison syndrome.

A 30 year-old patient is seen for a non-painful mass on the upper back which has grown slowly over the past year. He denies previous trauma, drainage from the area or history of dermatologic diseases. Examination reveals a four centimeter firm, but highly mobile subcutaneous mass with no overlying skin discoloration or punctum with drainage. Which of the following is the most likely diagnosis? A. Epidermoid cyst B. Hemangioma C. Keratoacanthoma D. Lipoma

(u) A. An epidermoid cyst is a lesion of the dermis, usually with periodic expression of sebaceous material. (u) B. A hemangioma is typically a discolored elevated bed/mass of vessels penetrating to or through the epidermis. (u) C. Keratoacanthoma is rapid in its development, more common in the elderly and confined to the dermis. Resembles squamous cell carcinoma. (c) D. A lipoma is a benign fatty tumor usually developing slowly in the subcutaneous layer and generally remaining small though can become very large.

A 35 year-old patient presents with a sudden onset of fever, dysphonia, drooling, and difficulty drinking a few hours ago. Physical examination reveals a temperature of 102 degrees F. The patient appears ill and is sitting forward. Inspiratory retractions are noted and there is a soft stridor. Which of the following is the most likely diagnosis? A. Angioedema B. Foreign body aspiration C. Epiglottitis D. Bacterial pharyngitis

(u) A. Angioedema would present with swelling of the mouth and upper airway. Patient would not have fever or inspiratory retractions. (u) B. Patients with foreign body aspiration are unlikely to appear acutely ill or be febrile. (c) C. Epiglottitis is characterized by fever, dysphonia, drooling, and dysphagia. Patients often appear ill and sit up leaning forward in an attempt to breathe. (u) D. Bacterial pharyngitis is not associated with stridor or inspiratory retractions.

A 53 year-old female with sudden onset "of the worst headache she has ever had" presents to the emergency department. She has a history of migraines but states that the current headache is not like her usual headaches. Results of her physical examination are unremarkable. Which of the following is the next best step in the evaluation of this patient? A. Angiogram B. CT scan C. Transcranial Doppler D. MRI

(u) A. Angiogram is necessary to define details of aneurysm and anatomic configuration, but not as an initial diagnostic study. (c) B. This patient's history is highly suggestive of subarachnoid hemorrhage. CT is best to screen for intracranial hemorrhage. It is faster than MRI and more sensitive in the first 24 hours. (u) C. Transcranial Doppler can detect cerebral artery vasospasm but cannot detect aneurysm. (u) D. MRI is not as sensitive for an acute bleed, but is appropriate for old bleeds.

: (7) 29. Diagnosis/Gastrointestinal/Nutritional A 36 year-old truck driver presents with a very tender mass that has developed between his buttocks over the past 4 days. Examination shows a small pit-like opening in the midline near the coccyx which is surrounded by an erythematous halo. The area is exquisitely tender and pressure produces purulent drainage from the opening. Which of the following is the most likely diagnosis? A. Anorectal fistula B. Hidradenitis suppurativa C. Perirectal abscess D. Pilonidal cyst

(u) A. Anorectal fistula is associated with perirectal abscess if there is a persistent communication with the crypt after draining. It may create a fistula from the anus to the perirectal skin. (u) B. Hidradenitis is a severe, chronic disabling skin infection of the apocrine sweat glands that can occur in the axilla and groin or perineum area and usually presents with multiple abscesses or sinus tracts. (u) C. Cryptoglandular abscess typically presents with continuous throbbing pain, which is worse with walking. Examination will reveal a tender perianal or rectal mass. Signs of sepsis are possible. (c) D. Pilonidal cyst presents with a tender draining abscess or chronic sinus over the sacrococcygeal or perianal region. It is thought to be due to distended and obstructed hair follicles and rupture into subcutaneous tissues with inspissated hair. On exam the patient will have midline pits near the coccyx or sacrum.

Which of the following would indicate an optic nerve lesion? A. Excessive conjunctival edema B. Ptosis C. Inability to gaze laterally D. Afferent pupillary defect

(u) A. Excessive edema of the conjunctiva is a feature of chemosis. (u) B. Ptosis is not indicative of an optic nerve lesion. (u) C. Inability to gaze laterally would be due to paralysis of the lateral rectus muscle controlled by cranial nerve VI. (c) D. Pupil size, controlled centrally by the Edinger-Westphal nucleus in the midbrain, is primarily based on the afferent light stimulus transmitted via the optic nerve.

A 30 year-old male sustains a blow to his right lateral leg during a soccer game. He complains of pain with weight bearing. Examination reveals tenderness along the lateral aspect of the right lower leg, but no point tenderness over the tibia. There is full active range of motion at the ankle, knee, and hip joints. There is no swelling or tenderness of the ankle or knee joints. Which of the following is the most likely diagnosis? A. Anterior cruciate ligament tear B. Fractured fibula C. Tibial stress fracture D. Gastrocnemius contusion

(u) A. Anterior cruciate ligament injury would have a positive drawer test and mechanism of injury can be due to a direct blow to the knee or as a result of sudden deceleration and rotation of the knee. (c) B. Isolated fibular fractures can occur with direct or indirect trauma to the fibular shaft. X-ray films of the leg are mandatory for any patient with a history of trauma and pain on ambulation to rule out this potentially overlooked injury. (u) C. A tibial stress fracture is more likely to present without acute injury and with tenderness over the tibia rather than the lateral aspect of the leg. (u) D. A contusion does not cause pain with weight bearing.

A patient presents with chest pain. ECG done in the emergency department reveals ST segment elevation in leads II, III, and AVF. This is most consistent with a myocardial infarction in which of the following areas? A. anterior wall B. inferior wall C. posterior wall D. lateral wall

(u) A. Anterior wall myocardial infarction is characterized by ST segment elevation in 1 or more of the precordial (V1-V6) leads. (c) B. Inferior wall myocardial infarction is characterized by ST segment elevation in leads II, III, and AVF. (u) C. Posterior wall myocardial infarction is characterized by ST segment depression in leads V1-V3 and a large R wave in leads V1-V3. (u) D. Lateral wall myocardial infarction is characterized by ST segment elevation in leads I and AVL.

A hospitalized patient is found with confirmed pulseless ventricular tachycardia. IV access is obtained following the second shock given. Which of the following medications is to be administered immediately? A. Amiodarone B. Magnesium C. Atropine D. Epinephrine

(u) A. Antiarrhythmics are given after the third shock and epinephrine has been administered. (u) B. Magnesium is useful for torsades de pointes. (u) C. Atropine may be used for asystole or a slow pulseless electrical activity (PEA) rate. (c) D. Epinephrine should be given as soon as IV access is obtained before or after the second shock.

The best course of action for a patient with a bothersome inflamed pingueculae (pingueculitis) is A. antibiotic drops. B. excision. C. Visine drops. D. no treatment.

(u) A. Antibiotic drops have no benefit with pingueculitis. (h) B. Excision is indicated for a pterygium that is threatening vision. (u) C. Visine drops will not do anything, but artificial tears may be beneficial. (c) D. With pingueculitis, no treatment is necessary; a short course of NSAID drops or steroids may help.

A 22 year-old male presents several hours after sustaining a hand injury when he punched a wall. X-rays of the hand demonstrate fracture of the fifth metacarpal neck with 65 degrees dorsal angulation and a claw hand. What is the most appropriate intervention? A. Antibiotic treatment and ulnar gutter splint immobilization B. Closed reduction and ulnar gutter splint immobilization C. Open reduction and ulnar gutter splint immobilization D. Ulnar gutter splint immobilization only

(u) A. Antibiotic treatment would be appropriate if the skin was broken and the injury was sustained in a fist fight with the potential for introduced oral flora. (u) B. Reduction followed by splinting is recommended for fifth metatarsal fractures with angulation of 15-40 degrees. (c) C. Open reduction is indicated with angulation of greater than 40 degrees. (u) D. With angulation of greater than 15 degrees, reduction should be performed prior to splinting.

A 19 year-old college student complains of a sore throat for over a week, with fever and general malaise. On exam T- 38 degrees C, P-70/minute, RR-20/minute, BP-110/76 mmHg. The skin is warm, dry, and without rash. The TMs have a normal light reflex and the canals are clear. The oropharynx is inflamed, with bilaterally enlarged tonsils, and a small amount of exudate. The neck is supple, with anterior cervical adenopathy. The abdomen is soft, nontender and a spleen tip is palpable. The labs reveal a negative rapid strep screen and positive heterophil antibody. The WBC count is 9,000/microliter with a differential of 40% atypical lymphocytes, 35% lymphocytes, 5% monocytes, 10% eosinophils, and 10% neutrophils. Which of the following is the most appropriate treatment? A. Penicillin B. Erythromycin C. Acetaminophen D. Acyclovir

(u) A. Antibiotics are not indicated in the treatment infectous mononucleosis, or Ebstein-Barr virus (EBV) infections. (u) B. See A for explanation. (c) C. Acetaminophen may be used to treat fever and pain associated with infectious mononucleosis, or EBV infection. (u) D. Acyclovir is not approved for use in treatment of EBV, although it is active against the EBV in vitro and in vivo. It may be used in certain patients with AIDS, but has not been shown to affect the outcome of EBV in these patients.

A 3 month-old male presents with two days of worsening hoarse cough and thick purulent rhinorrhea associated with increasing problems breathing and trouble feeding. Examination reveals a temperature of 100.2 degrees F and respiratory rate of 80/minute with nasal flaring and retractions. Lung examination reveals a prolonged expiratory phase with inspiratory rales. He is tachycardic. Pulse oximetry reveals oxygen saturation of 89%. Chest x-ray reveals hyperinflation with diffuse interstitial infiltrates. Which of the following is the most appropriate intervention? A. Antibiotics B. Hospitalization C. Inhaled corticosteroids D. Racemic epinephrine

(u) A. Antibiotics are utilized to treat bacterial, not viral, illnesses. (c) B. This infant most likely has bronchiolitis. While most cases are mild and can be treated at home, hospitalization is recommended for infants with hypoxia on room air, moderate tachypnea with feeding difficulties and marked respiratory distress with retractions. Additionally hospitalization is recommended for infants less than 2-3 months of age, a history of apnea or an underlying chronic cardiopulmonary disease. (u) C. The use of corticosteroids in children with bronchiolitis has not been studied and does not appear to be helpful. (u) D. Racemic epinephrine is not indicated in the treatment of bronchiolitis.

A patient with advanced AIDS complicated by toxoplasmosis presents with altered mental status, recent onset of seizures, and focal neurologic deficits. Which of the following diagnostic studies is most helpful? A. Toxoplasma gondii antibody titers B. CT scan of the brain C. Lumbar puncture D. MRI of brain

(u) A. Antibody titers cannot be depended upon since most patients have IgG titers that reflect past infection, significant rises are infrequent, and IgM antibody is rare. (u) B. A CT scan may be helpful, but an MRI is the more sensitive test. (h) C. Lumbar puncture is contraindicated secondary to possible mass effect. (c) D. An MRI showing multiple isodense or hypodense ring-enhancing mass lesions is the most useful test for such a patient.

: (11) 212. Clinical Therapeutics/Cardiology You are caring for a 35 year-old patient with a peripherally inserted central catheter (PICC). There is a progressive linear induration that stretches 3 inches proximal from the antecubital fossa with associated erythema and tenderness. The PICC line is removed. Which of the following additional measures should be taken in the treatment of this patient? A. Heparin therapy B. Surgery for ligation and division of the vein C. Naproxyn sodium (Naprosyn) and local heat D. Cephalaxin (Keflex) for 7 days duration

(u) A. Anticoagulation therapy is not indicated unless the disease is rapidly progressing or there is extension into the deep system. (u) B. If the induration is extensive or is progressing toward the cephaloaxillary junction, ligation and division of the vein at the junction of the deep and superficial veins is indicated. (c) C. If the superficial thrombophlebitis is well localized and not near the cephaloaxillary junction, local heat, and non-steroidal anti-inflammatory mediations are usually effective in limiting the process. (u) D. Only with septic superficial thrombophlebitis which is an intravascular abscess would excision of the involved vein as well as broad-spectrum antibiotics and systemic anti-coagulants be indicated.

A 2 year-old female presents with purulent nasal discharge bilaterally with fever and cough for several days. Her mom had taken her out of daycare for a similar occurrence 2 months ago, that was treated with Amoxicillin. Exam further reveals halitosis and periorbital edema. Treatment should be initiated with which of the following? A. Antihistamines B. Ribavirin (Rebetol) C. Intranasal corticosteroids D. Amoxicillin-clavulanate (Augmentin)

(u) A. Antihistamines and intranasal corticosteroids have not been adequately studied in children to prove they make a difference in treating recurrent sinusitis. (u) B. Ribavirin is approved for the treatment of RSV infection. (u) C. See A for explanation. (c) D. High dose amoxicillin-clavulanate is the treatment of choice for resistant bacterial sinusitis, especially in children presenting with risk factors (daycare attendance, previous antibiotic treatment 1-3 months prior, age younger than 2 years).

Which of the following medications is the initial treatment of choice for suppressing the progression of rheumatoid arthritis (RA)? A. Naproxen sodium (Naprosyn) B. Prednisone C. Methotrexate (Rheumatrex) D. Gold salts

(u) A. Antiinflammatory medication may help with the symptoms but does not aid in suppression of RA. (u) B. Corticosteroids are useful in the treatment of symptoms but does not aid in the suppression of disease. (c) C. Methotrexate is the initial treatment choice for RA and aids in suppression of disease. (u) D. Gold salts can be used for an acute RA flare, but do not suppress of the progression of the disease.

Which of the following laboratory tests, if positive, would be most indicative of Crohn's disease? A. Antineutrophil cytoplasmic antibodies (ANCA) B. Antiendomysial antibodies (AEA) C. Antinuclear antibodies (ANA) D. Anti-Saccharomyces cerevisiae antibodies (ASCA)

(u) A. Antineutrophil cytoplasmic antibodies is positive in 60-70% of patients with ulcerative colitis while only 5-10% of Crohn's disease patients have positive results. (u) B. The antiendomysial IgA antibodies are directed against tissue transglutaminase and has a 90-95% specificity for celiac sprue. (u) C. Antinuclear antibodies are elevated in a wide variety of autoimmune disorders and therefore are a nonspecific finding. (c) D. About 60-70% of patients with Crohn's disease are positive for these antibodies which are directed at the cell walls of S. cerevisiae while only 10-15% of ulcerative colitis is found to be positive.

: (9) 53. Diagnostic Studies/Orthopedics/Rheumatology A 50 year-old female presents with a two-month history of fatigue and morning stiffness involving both hands. It is now difficult for her to perform simple tasks at home and work. Examination reveals swelling and warmth of the metacarpophalangeal and proximal interphalangeal joints, sparing the distal interphalangeal joints. Elevation of which of the following laboratory test results would best support the suspected diagnosis? A. Antinuclear antibodies B. Rheumatoid factor C. HLA-B27 D. C-reactive protein

(u) A. Antinuclear antibodies are often elevated in rheumatoid arthritis (RA) but are not specific for the disease. (c) B. A high titer of rheumatoid factor is the most significant diagnostic finding. (u) C. HLA-B27 elevation is seen in ankylosing spondylitis not RA. (u) D. C-reactive protein elevation indicates injury, infection or inflammation generally, but is not specific for RA.

A patient presents with respiratory complaints. Chest x-ray reveals calcification of the hilar nodes with an eggshell pattern. Which of the following occupations is most consistent with these chest x-ray findings? A. building demolitioners B. coal miners C. sandblasters D. farmers

(u) A. Asbestosis is the most likely occupational risk and has an indistinct heart border appearance on CXR described as looking like "ground glass". (u) B. The CXR of a coal miner consists of irregular opacities ranging from a reticular pattern to a nodular pattern. (c) C. Silicosis can occur in sandblasters and produce a CXR appearance of calcification of the hilar nodes with an "eggshell" pattern with long term exposure (u) D. Farmers lung results from spores and produces a hypersensitivity pneumonitis. The CXR would show a patchy fibrosis.

Which of the following is the most common finding of multiple sclerosis? A. facial palsy B. hearing loss C. seizures D. diplopia

(u) A. Facial palsy, hearing loss, and seizures are rare with multiple sclerosis. (u) B. See A for explanation. (u) C. See A for explanation. (c) D. Diplopia due to ophthalmoplegia is the most common presenting complaint in patients with multiple sclerosis.

A mother brings her 6 year-old boy for evaluation of school behavior problems. She says the teacher told her that the boy does not pay attention in class, that he gets up and runs around the room when the rest of the children are listening to a story, and that he seems to be easily distracted by events outside or in the hall. He refuses to remain in his seat during class, and occasionally sits under his desk or crawls around under a table. The teacher told the mother this behavior is interfering with the child's ability to function in the classroom and to learn. The mother states that she has noticed some of these behaviors at home, including his inability to watch his favorite cartoon program all the way through. Which of the following is the most likely diagnosis? A. Antisocial disorder B. Dysthymic mood disorder C. Obsessive-compulsive disorder D. Attention deficit hyperactivity disorder

(u) A. Antisocial behavior disorder is characterized by disregard for rights of others; a defect in the experience of compunction or remorse for harming others. (u) B. Dysthymic mood disorder is characterized by chronic, sad mood occurring for at least 2 years in an adult (one year in a child). Behavioral problems are not part of this disorder. (u) C. Obsessive-compulsive disorder is characterized by recurrent obsessions and compulsions that result in anxiety and disruptive behaviors related to those compulsions. (c) D. Attention deficit hyperactivity disorder is characterized by inattention, including increased distractibility and difficulty sustaining attention; poor impulse control and decreased self-inhibitory capacity; and motor overactivity and motor restlessness, which are pervasive and interfere with the individual's ability to function under normal circumstances.

A patient with a history of severe peptic ulcer disease is 5 weeks status post Billroth I surgery. One week ago he restarted his normal diet and has had the onset of severe nausea, abdominal cramping, and light-headedness that occur approximately thirty minutes after eating. The abdominal exam reveals a healing surgical scar without areas of unusual tenderness or any palpable masses. Which of the following is the most likely diagnosis? A. Anxiety disorder B. Celiac sprue C. Dumping syndrome D. Irritable bowel syndrome

(u) A. Anxiety disorders can cause a wide variety of somatic syndromes such as those mentioned, but generally not with such a straightforward dietary trigger. (u) B. Celiac sprue can cause similar symptoms as those listed, can develop at any age and can be worsened by the ingestion of gluten containing products. The patient's recent surgery makes dumping syndrome a much greater possibility. (c) C. Dumping syndrome typically occurs after Billroth type I surgeries as well as gastric bypass surgeries when the patient attempts to eat a large amount of simple sugars. (u) D. Irritable bowel syndrome is a diagnosis of exclusion and is associated with alternation in bowel habits.

A 32 year-old female complains of severe irritability and emotional lability accompanied by weight gain, breast tenderness, and headache starting mid-cycle each month and abating with the onset of menses. She has tried relaxation therapy, exercise, dietary changes and over the counter pharmacologic interventions with minimal relief of her symptoms. Which of the following prescription medications is the best choice to relieve her symptoms? A. Alprazolam (Xanax) B. Fluoxetine (Prozac) C. Oral contraceptive pill (Ortho-novum 1/35) D. Spironolactone (Aldactone)

(u) A. Anxiolytics have shown to be effective however its potential for dependency makes it not the best choice. (c) B. SSRI's provide symptom improvement for patients with premenstrual syndrome (PMS). (u) C. Studies have found little difference between women taking a low dose birth control and women who do not take pills; currently not recommended for PMS. (u) D. Spironolactone is used for cyclic edema, not the best choice for PMS.

Which of the following valvular heart abnormalities will most likely be seen on echocardiography as a complication of acute myocardial infarction? A. Aortic stenosis B. Aortic regurgitation C. Mitral stenosis D. Mitral regurgitation

(u) A. Aortic stenosis puts additional strain on the left ventricle and contributes to a patient developing an acute myocardial infarction and does not occur as a result of one. (u) B. Aortic regurgitation is not a consequence of acute myocardial infarction and most commonly occurs as a result of an incompetent valve or dilation of the proximal aorta. (u) C. Mitral stenosis most commonly occurs as a complication of rheumatic fever and not because of an acute myocardial infarction. (c) D. In patients with acute myocardial infarction, echocardiogram can show the severity of mitral regurgitation and the presence of ventricular septal defect if one is present. Acute inferior wall myocardial infarction is associated with acute mitral regurgitation due to necrosis of the posterior papillary muscle which is supplied by the right coronary artery.

: (8) 14. Diagnosis/Cardiology A 51 year-old patient presents for follow-up after a recent stroke. Cardiac examination reveals a moderately loud systolic ejection murmur in the 2nd and 3rd interspaces parasternally. The second heart sound is fixed and widely split. Which of the following is the most likely diagnosis in this patient? A. Aortic stenosis B. Mitral valve regurgitation C. Patent foramen ovale D. Ventricular septal defect

(u) A. Aortic stenosis will usually manifest as heart failure or if acute, as angina. In severe aortic stenosis a weak to absent aortic second sound or reversed splitting of the second sound is present. (u) B. Heart sounds associated with mitral regurgitation include normal S1 and a prominent 3rd heart sound. (c) C. A patent foramen ovale is present in 25% of the population and can lead to paradoxic emboli and cerebrovascular events. Suspicion should be highest in patients who have cryptogenic stroke before age 55 years. (u) D. Clinical features depend on the size of the defect and the presence or absence of right ventricular outflow obstruction or increased peripheral vascular resistance. Stroke and fixed S2 split are not representative of a VSD.

A 40 year-old woman presents with 3 months of dry cough and intermittent low-grade fever. She is a non-smoker and has no significant family history or past medical history. A purified protein derivative (PPD) test was recently performed at work and was negative. On physical examination she is afebrile with stable vital signs. Lung auscultation reveals crackles in bilateral upper lobes. Chest x-ray shows hilar and mediastinal adenopathy, mild interstitial disease in the upper lung zones, and several small granulomas in both lungs. What is the most likely diagnosis? A. Asbestosis B. Cryptococcosis C. Sarcoidosis D. Tuberculosis

(u) A. Asbestosis typically presents as interstitial disease in the lower lungs and this patient has no known exposure to asbestos. (u) B. Cryptococcosis typically shows pleural-based nodules on x-ray and this patient has no known risk factors (HIV disease, COPD, chronic steroid use). (c) C. Sarcoidosis classically presents as a vague systemic illness with radiographic evidence of any or all of the following: granulomas, hilar and mediastinal adenopathy and interstitial infiltrate. (u) D. The patient's recent negative PPD makes tuberculosis unlikely.

A 67 year-old female with history of hypertension, diabetes mellitus, and smoking presents to the emergency department with mild expressive aphasia, right facial weakness and mild right arm weakness. She had awakened 60 minutes ago and was speaking to her husband when her speech suddenly became difficult to understand and weakness was noted. Physical examination reveals a blood pressure of 165/85 mm Hg. A CT of the head shows no intracranial hemorrhage. Which of the following is the most appropriate intervention? A. Aspirin B. Warfarin (Coumadin) C. Tissue plasminogen activator (rt-PA) D. Clopidogrel (Plavix)

(u) A. Aspirin is used for preventative purposes and will not resolve a current embolism. (u) B. Warfarin is a preventative medicine that will not help the current situation. (c) C. IV thrombolytic therapy with recombinant tissue plasminogen activator is effective in reducing the neurological deficit in selected patient without CT evidence of intracranial hemorrhage and when administered within 3 hours after onset of ischemic stroke. (u) D. Plavix is a platelet aggregation inhibitor and used for preventative measures.

Which of the following is the optimal therapy for a 76 year-old patient with no allergies who has chronic atrial fibrillation? A. Aspirin B. Clopidogrel (Plavix) C. Warfarin (Coumadin) D. Low molecular weight heparin

(u) A. Aspirin's role to prevent thromboembolism in atrial fibrillation is limited to patients with no risk factors who are under age 65. (u) B. Clopidogrel is not the optimal therapy for patients with atrial fibrillation. (c) C. Patients older than age 75 who have chronic atrial fibrillation should be anticoagulated with warfarin to maintain an INR between 2.5 and 3.0 for optimum therapy unless a contraindication to therapy exists. (u) D. Due to the increased costs and need for parenteral therapy, daily subcutaneous heparin is not first line therapy unless warfarin therapy is contraindicated.

220. Scientific Concepts/Cardiology What is the hallmark finding that is essential to the diagnosis of primary dilated cardiomyopathy? A. Asymmetric septal enlargement B. Atrial enlargement C. Infiltrative myocardial disease D. Systolic dysfunction

(u) A. Asymmetric septal enlargement is consistent with hypertrophic cardiomyopathy. (u) B. Atrial enlargement is not consistent with primary dilated cardiomyopathy. (u) C. Infiltrative myocardial disease is consistent with restrictive cardiomyopathy. (c) D. Systolic dysfunction and left ventricular dilation are essential for the diagnosis of primary dilated cardiomyopathy.

During a routine prenatal visit, the fundal height is found to be at the umbilical level. The number of weeks gestation is estimated to be A. 10 to 12. B. 16 to 18. C. 20 to 22. D. 26 to 28.

(u) A. At 12 weeks, fundal height is palpable just above the pubic symphysis. (u) B. At 16 weeks, fundal height is midway between the pubic symphysis and umbilicus. (c) C. At 20 weeks, fundal height is at the umbilicus. (u) D. At 26 weeks, fundal height is above the umbilicus.

Which of the following side effects is associated with long-term administration of phenytoin (Dilantin)? A. Ataxia B. Hypotension C. Osteomalacia D. Cardiac dysrhythmia

(u) A. Ataxia is associated most often with acute oral overdosage of phenytoin. (u) B. Cardiac dysrhythmia, with or without hypotension, is an expected side effect of rapid IV phenytoin administration. (c) C. Osteomalacia, or demineralization of bone, is a side effect of phenytoin that may occur after chronic administration. (u) D. See B for explanation.

2. Clinical Therapeutics/Urology/Renal In order to help delay the progression to kidney disease in a diabetic patient with proteinuria, which of the following would you consider using to treat the patient's concomitant hypertension? A. Atenolol (Tenormin) B. Amlodipine (Norvasc) C. Accupril (Quinapril) D. Amiloride (Midamor)

(u) A. Atenolol is a beta blocker. Beta blockers are not first line therapy for hypertensive diabetics. (u) B. Amlodipine is a calcium channel blocker. Calcium channel blockers are not first line therapy for hypertensive diabetics. (c) C. Accupril is an ACE inhibitor. Because of their beneficial effects in diabetic neuropathy, ACE inhibitors (and ARBs) should be part of the initial treatment regimen for hypertension in diabetics. (u) D. Amiloride is a potassium-sparing diuretic. An ACE inhibitor is the standard first line therapy for hypertensive diabetics.

A 33 year-old female presents to the office with a complaint of palpitations. There is no history of any significant heart disease in the past and her symptoms begin and end abruptly on their own. Which of the following is the most likely explanation for these symptoms? A. Atrial fibrillation B. Atrial flutter C. Benign supraventricular tachycardia D. Sinus tachycardia

(u) A. Atrial fibrillation, the most common cause of sustained irregular heart rates, tends to present in an older population or in patients that have underlying heart disease such as mitral stenosis. (u) B. Atrial flutter is not commonly seen in young patients without underlying heart disease although it does tend to be intermittent in nature when it does occur. (c) C. Benign supraventricular tachycardia tends to occur in a young patient without pre-existing heart disease. Symptoms begin and end abruptly without therapy and occurrences happen only intermittently. It is the most likely diagnosis in this setting. (u) D. Sinus tachycardia may occur in a young patient without underlying heart disease but the palpitations and tachycardia tends to start and stop gradually rather than abruptly.

: (9) 46. Scientific Concepts/ENT/Ophthalmology A 65 year-old female presents with new onset of right eyelid droop. On examination ptosis, ipsilateral anhydrosis and miosis are noted. What is the underlying mechanism associated with this presentation? A. Autoimmune dysfunction of neuromuscular transmission B. Dysfunction of the 7th cranial nerve C. Disturbance of the cranial sympathetic innervation D. Dysfunction of the 5th cranial nerve

(u) A. Autoimmune dysfunction of the neuromuscular junction typically results in myasthenic syndromes such as, myasthenia gravis or Lambert-Eaton myasthenic syndrome. (u) B. Dysfunction of the 7th cranial nerve results in facial muscle paralysis. (c) C. Horner's Syndrome (ptosis, miosis, anhydrosis) is the result of cranial sympathetic dysfunction that occurs due to nerve injury below the level of entry into the skull. (u) D. Dysfunction of the 5th cranial nerve may result in facial sensory loss and altered motor function of the temporalis and masseter muscles.

An 18 year-old patient has a tibia/fibula fracture following a motorcycle crash. Twelve hours later the patient presents with increased pain despite adequate doses of analgesics and immobilization. Which of the following is the most likely diagnosis? A. avascular necrosis B. myositis ossificans C. compartment syndrome D. reflex sympathetic dystrophy

(u) A. Avascular necrosis is a late complication of fracture resulting from disruption of the blood supply to the bone. (u) B. Myositis ossificans occurs primarily in muscles post-traumatically and may not arise for several months after an injury. (c) C. Compartment syndrome is characterized by a pathological increase of pressure within a closed space and results from edema or bleeding within the compartment. It may occur as an early local complication of fracture. (u) D. Reflex sympathetic dystrophy is characterized by painful wasting of muscles that may be secondary to injury and could occur as a late complication.

A 45 year-old male presents to the clinic complaining of morning sluggishness, daytime fatigue, headaches. He admits to drinking two cocktails each evening. His bed partner reports his loud cyclical snoring, breath cessation and thrashing movements of his extremities during sleep. BMI is 40. Heart examination reveals regular rate and rhythm without S3, S4, or murmur and lungs are clear to auscultation. Polysomnography shows apneic episodes lasting as long as 60 seconds. Which of the following clinical interventions would most likely provide for the acute cessation of apneic episodes? A. Avoidance of alcohol B. Weight loss C. Nasal continuous positive airway pressure D. Supplemental oxygen

(u) A. Avoidance of alcohol is also a necessary step in managing sleep apnea however does not provide immediate relief of apneic episodes. (u) B. Weight loss is also a necessary step in managing sleep apnea however does not provide immediate relief of apneic episodes. (c) C. Nasal CPAP is curative in many patients with sleep apnea. (u) D. Oxygen lessens the severity of nocturnal desaturation but may lengthen the apnea episodes.

The DSM-IV classifies mental disorders by using five axes in completing the process. Axis III is used to identify which of the following? A. Clinical disorders and other conditions that may be the focus of clinical attention B. Any physical disorder or general medical condition that is present in addition to the mental disorder C. The psychosocial and environmental problems that have had a significant contribution to the development or exacerbation of the disorder D. Personality disorders and/or mental retardation

(u) A. Axis I identifies clinical disorders and other conditions that may be the focus of clinical attention. (c) B. Axis III identifies any physical disorder or general medical condition that is present in addition to the mental disorder. (u) C. Axis IV identifies the psychosocial and environmental problems having a significant contribution to the disorder. (u) D. Axis II identifies personality disorders and mental retardation.

Which of the following treatments will most benefit the diabetic patient with two vessel coronary disease? A. Stent placement B. Percutaneous balloon angioplasty C. Medical management D. Coronary artery bypass graft

(u) A. See D for explanation. (u) B. See D for explanation. (u) C. See D for explanation. (c) D. CABG is the treatment of choice in a diabetic with two or three vessel disease.

A 56 year-old female presents to the clinic wanting testing for ovarian cancer. Her best friend was just diagnosed with Stage 4 primary cancer of the ovary. She denies family history of breast or ovarian cancer. According to screening guidelines, which of the following do you recommend? A. BRCA 1-2 gene testing B. A pelvic ultrasound yearly C. Continue her yearly women's health examinations D. CA-125 testing

(u) A. BRCA 1-2 has been found to be associated with ovarian cancers in approximately 5% of cases. However the expression of this gene in BRACA 1-2 carriers are unpredictable. Therefore, routine testing for this gene has limited value at this time. (u) B. Refined imaging techniques offer promise for the future however much work remains before these tests are considered accurate or cost effective screening tests. (c) C. The patient is advised to have regular pelvic exams. The limited prevalence of ovarian cancer and the lack of sensitivity and specificity of current available tests have so far prevented the implementation of routine ovarian cancer screening of the general population. (u) D. The CA-125 recognizes the antigen CA-125 which is present in serous ovarian tumors but not in mucinous or nonepithelial ovarian tumors however the test is not considered a cost effective screening test.

A patient is evaluated in the office with a red eye. The patient awoke with redness and a watery discharge from the eye. The eyelids were not matted together. Examination reveals a palpable preauricular node. Which of the following is the most likely diagnosis? A. bacterial conjunctivitis B. viral conjunctivitis C. allergic conjunctivitis D. gonococcal conjunctivitis

(u) A. Bacterial conjunctivitis is associated with purulent, not watery eye discharge. (c) B. Viral conjunctivitis is associated with copious watery discharge and preauricular adenopathy. (u) C. Allergic conjunctivitis is associated with symptoms limited to the conjunctiva with hyperemia and edema. (u) D. Gonococcal conjunctivitis is associated with copious purulent discharge and no preauricular adenopatthy

: (8) 41. Diagnosis/ENT/Ophthalmology An afebrile 2 year-old female presents with a three-day history of foul smelling, blood-tinged, mucoid drainage from the left nostril. Which of the following is the most likely diagnosis in this patient? A. Acute bacterial sinusitis B. Nasal foreign body C. Acute viral rhinitis D. Allergic rhinitis

(u) A. Bacterial sinusitis typically follows a viral upper respiratory infection with symptoms of cough, nasal drainage and fever. The symptom duration is typically 10-14 days without improvement. (c) B. Nasal foreign body typically presents in children under 3 years of age. The symptoms include mucopurulent drainage, epistaxis, foul odor and nasal obstruction. (u) C. The patient with acute viral rhinitis typically presents with sudden onset of mucoid rhinorrhea, sore throat, fever and cough. (u) D. Allergic rhinitis is associated with recurrent nasal congestion, frequent sneezing and clear rhinorrhea.

119. Diagnostic Studies/Gastrointestinal/Nutritional A 3 week-old infant is evaluated for persistent projectile vomiting described as breast milk without bile or blood. The abdomen is distended before vomiting and a small, mid-epigastric mass is palpable after vomiting. Which of the following is the most appropriate diagnostic study for the evaluation of this patient? A. Barium enema B. Esophageal manometry C. H. pylori stool antigen D. Upper GI contrast radiographs

(u) A. Barium enema is usually both diagnostic and therapeutic for intussusception which usually presents as recurring paroxysms of abdominal pain, vomiting and bloody diarrhea. (u) B. Esophageal manometry is used to measure LES pressure which may be increased in esophageal achalasia. Usually a child over 5 years presents with dysphagia, retrosternal pain and slow eating. (u) C. Positive H. pylori stool antigen is used to test for bacterial infection associated with PUD that presents with pain and bleeding. (c) D. This infant has pyloric stenosis and an upper GI series will reveal a narrowed distal stomach with double tract of barium.

A 66 year-old female presents to your office complaining of progressive difficulty swallowing over the last 6 months. Initially she had difficulty only with meats, but now she has dysphagia with other foods as well. Which of the following is the most appropriate initial diagnostic study? A. Barium swallow B. Endoscopy C. CT scan D. Urea breath test

(u) A. Barium swallow may be used in the evaluation of dysphagia, but does not allow for biopsy to be performed. (c) B. Endoscopy is the diagnostic study of choice in a patient with progressive dysphagia because of its ability to obtain tissue for diagnosis. (u) C. See B for explanation. (u) D. See B for explanation.

: (8) 177. Diagnosis/ENT/Ophthalmology A 29 year-old female presents with 1-day history of severe, continuous dizziness with episodes of nausea and vomiting. She is otherwise healthy with the exception of a recent upper respiratory infection. Which of the following is the most likely cause of her symptoms? A. Benign positional vertigo B. Meniere's disease C. Physiologic vertigo D. Vestibular neuritis

(u) A. Benign positional vertigo occurs with positional change and is of short duration (less than 1 minute). (u) B. Meniere's disease is characterized by episodic vertigo with aural fullness, hearing loss and tinnitus. (u) C. Physiologic vertigo is associated with movement (sea sickness, motion sickness) and is characterized by nausea, increased salivation, vomiting and sweating. (c) D. Vestibular neuritis is characterized by sudden onset of vertigo, nausea and vomiting in the absence of auditory or focal neurologic abnormalities. The symptoms may persist for several days and often follows an acute viral illness.

: (27) 63. History & Physical/Hematology Which of the following physical examination findings is most likely in a patient with anemia, low factor VIII and a prolonged PTT? A. Glossitis B. Arthropathy C. Deep vein thrombosis D. Purpura

(u) A. Glossitis is mostly found in vitamin B12 and folate deficiencies. (c) B. Hemarthrosis is likely in Factor VIII deficiency (Hemophillia A). (u) C. Deep vein thrombosis is not associated with factor VIII deficiency. (u) D. Purpura is not typically found in Factor VIII disorders.

Which of the following medication classes is the recommended treatment for patients who have an anterior wall myocardial infarction with poor left ventricular function? A. Beta blockers B. Calcium channel blockers C. Potassium sparing diuretics D. ACE inhibitors

(u) A. Beta blockers need to be used with caution in a patient with severe left ventricular dysfunction as they will worsen left ventricular contractility and may make this dysfunction worse. They are used, however, in the early stages of chronic heart failure. (u) B. Calcium channel blockers have no proven mortality benefit in patients with myocardial infarctions and left ventricular dysfunction. (u) C. Although potassium sparing diuretics are part of the later stage treatment of congestive heart failure and tend to potentiate the other therapies, they are not first-line therapy in a patient with left ventricular dysfunction. (c) D. ACE inhibitors have been proven to be effective in the therapy of heart failure, especially in the setting of left ventricular dysfunction. They are considered first-line therapy in patients with symptomatic left ventricular systolic function

When utilizing medical treatment for hypertension, which of the following classes of medications should be used with caution in those with elevated potassium levels or intrinsic renal disease? A. Beta blockers B. Calcium channel blockers C. Central alpha agonists D. ACE inhibitors

(u) A. Beta blockers primarily work by decreasing cardiac contraction and slowing the heart rate. There is no association with hyperkalemia. (u) B. Calcium channel blockers (especially the dihydropyridines) act as vasodilators with some effect at lessening cardiac contraction. Their use is not associated with hyperkalemia. (u) C. Central alpha agonists stimulate the alpha receptors in the brain resulting in decreased vessel wall pressures. This process is responsible for causing a lowering of the blood pressure. There is little effect on the kidney and these agents do not cause hyperkalemia. (c) D. ACE inhibitors are recognized as valuable agents in the management of hypertension but they may result in hyperkalemia in patients with intrinsic renal disease. They should be used with caution in patients who are taking potassium-sparing diuretics. These agents reduce glomerular filtration pressure resulting in retention of potassium since less filtration occurs in the kidney.

A 28 year-old female presents with complaint of headaches for the past several months. They typically begin behind her right eye, which she describes as a combination of stabbing and pressure. She states that noise bothers her, and she experiences nausea but no vomiting. She has occasionally awakened with the pain. She admits to being under a lot of stress. Aspirin or acetaminophen does not relieve the pain. Which of the following is the best choice for initial treatment of the acute headache? A. topiramate (Topamax) B. gabapentin (Neurontin) C. propranolol (Inderal) D. sumatriptan (Imitrex)

(u) A. Beta blockers, calcium channel blockers, antiepileptic drugs, and tricyclic antidepressants have been used for the prevention of migraine headaches, but not for the acute treatment. (u) B. See A for explanation. (u) C. See A for explanation. (c) D. 5HT-receptor agonists and ergot alkaloids are effective for the acute treatment of migraine headaches if acetaminophen or nonsteroidal anti-inflammatory drugs are not effective.

: (8) 86. Clinical Intervention/Endocrinology A 30 year-old pregnant female presents with anxiety, palpitations and weight loss. On examination she is diaphoretic, tachycardic and hyperreflexic. The TSH is lower than normal. Which of the following is the treatment of choice in this patient? A. Propranolol (Inderal) B. Methimazole (Tapazole) C. Subtotal thyroidectomy D. Propylthiouracil (PTU)

(u) A. Beta-blockers are routinely given as symptomatic treatment for hyperthyroidism. However, propranolol, the agent of choice, is Category C and contraindicated in the second and third trimesters of pregnancy. (u) B. Methimazole has been associated in pregnancy with a possible increased risk of fetal abnormalities. Propylthiouracil is preferred although methimazole may be tried in low doses if the patient cannot tolerate propylthiouracil. (u) C. Surgery is indicated for pregnant women with thyrotoxicosis or if treatment with PTU is unsuccessful. (c) D. PTU is favored in the management of hyperthyroidism in a pregnant female due to fewer problems in the newborn.

A 26 year-old female has a long history of sexual promiscuity and substance abuse. She frequently expresses anger when she feels abandoned. She also has difficulty in controlling her anger at times. Her past relationships have been intense and short-lived. She has attempted suicide twice in the past 18 months. This patient exhibits which of the following disorders? A. bipolar B. avoidant personality C. borderline personality D. dissociative

(u) A. Bipolar disorder is characterized by swings in mood from extreme elation to severe depression. The characteristics of this patient do not fit the diagnosis of bipolar disorder. (u) B. In avoidant personality disorders, individuals show an exaggerated sensitivity to rejection and severe shyness. (c) C. Borderline personality disorder is characterized by instability of interpersonal relationships, marked impulsivity that is potentially self-damaging, inappropriate, intense anger or control of anger, recurrent suicidal attempts, gestures or threats, and identity disturbances. All of these are exhibited in this patient. (u) D. Dissociative disorder is what is commonly termed "multiple personality disorder." For this diagnosis, the person must display the presence of two or more distinct identities. These personalities recurrently take control of the patient's behavior, there is an inability to recall important personal information, and the disorder is not the effect of a substance or a medical condition.

Which of the following is the leading cause of permanent visual loss in a patient over the age of 75? A. Blepharitis B. Cataracts C. Central retinal artery occlusion D. Macular degeneration

(u) A. Blepharitis is a chronic bilateral inflammatory condition of the lid margins. (u) B. Cataracts are the clouding of the lens sufficient to reduce vision. Most develop slowly as a result of aging, leading to gradual impairment of vision. (u) C. Central retinal artery occlusion presents as a rare cause of sudden profound monocular visual loss. (c) D. Age-related macular degeneration is the leading cause of permanent visual loss in the older population. The exact cause is unknown, but the prevalence increases with each decade over age 50 years.

Maternal blood pressure normally decreases the most during what period of pregnancy? A. First trimester of pregnancy B. Second trimester of pregnancy C. Third trimester of pregnancy D. During labor and delivery

(u) A. Blood pressures during the first trimester will be similar to the non-pregnant state. (c) B. Diastolic blood pressure and the mean arterial pressure reach their nadir at 16-20 weeks of gestation. (u) C. Blood pressures during the third trimester will be similar to the non-pregnant state. (u) D. Blood pressure during labor and delivery often are variable. Blood pressure often rises with pain and apprehension. Blood pressure often decreases with the use of narcotics, epidurals, and significant blood loss.

A patient who appears very anxious enters the office complaining of dizziness with perioral and extremity paresthesias. She vaguely describes some chest discomfort. Physical examination is unremarkable, except for moderate tachypnea with obvious sighing respiration. This clinical picture is most consistent with A. bronchial asthma. B. hyperventilation syndrome. C. spontaneous pneumothorax. D. emphysema.

(u) A. Bronchial asthma attacks are associated with increased dyspnea and prolonged expiration. Patients may use accessory muscles of respiration as part of this acute condition. (c) B. Anxiety may result in hyperventilation that can result in perioral numbness and paresthesias of the extremities. These paresthesias are due to decreased CO2 in the blood stream that results from the hyperventilation. Anxious patients also will have nondescript chest pain as part of this condition and may also complain of dizziness. (u) C. Spontaneous pneumothorax patients will primarily complain of significant chest pain along with their dyspnea. These patients will not have perioral or extremity paresthesias. (u) D. Emphysema alone will not result in hyperventilation or the production of perioral or extremity paresthesias and is a chronic progressive rather than an acute onset condition.

In patients with COPD, which of the following has been shown to decrease rate of malignancy and cardiovascular disease and improve survival? A. bronchodilator therapy B. pulmonary rehabilitation C. oral glucocorticosteroids D. smoking cessation

(u) A. Bronchodilator therapy is used for symptomatic treatment in patients with COPD. (u) B. Pulmonary rehabilitation improves quality of life, dyspnea and exercise capacity. It also has been shown to reduce the rate of hospitalization. (u) C. Chronic use of oral glucocorticosteroids is not recommended because of an unfavorable benefit/risk ratio. (c) D. Smoking cessation has been shown to provide significant improvement in decreasing the rate of decline in pulmonary function.

The bite from which of the following is associated with fever, lacrimation, rhinorrhea, bradycardia, hypertension, and tachyarrhythmias? A. Brown recluse spider B. Black widow spider C. Black flies D. Bedbugs

(u) A. Brown recluse spider bites may cause reactions ranging from mild urticaria to full thickness necrosis but typically, there are no systemic symptoms. (c) B. Black widow spider bites can inject venom that contains a neurotoxin which can produce reactions at the site of the bite along with varying degrees of systemic symptoms. (u) C. Black fly bites produce local reactions as well as fever, nausea, and general lymphadenopathy. (u) D. Bedbugs have nocturnal feedings that produce a linear arrangement of papular urticaria. There are no systemic symptoms associated with their bites.

: (28) 31. Clinical Intervention/Orthopedics/Rheumatology A 20 year-old male is brought to the ED by ambulance for evaluation of a neck injury after diving into shallow water. A radiograph reveals a moderately displaced C1 atlas fracture. Which of the following is the indicated management of this fracture? A. Surgical fusion of C1-C2 B. Halo vest C. Cervical traction D. Cervical collar

(u) A. C1-C2 fusion is indicated for patients with remaining instability, nonunion, or posterior arch disruption after 3-4 months of halo vest treatment. (c) B. Halo vest application is indicated in cases involving a moderately displaced fracture. (u) C. Cervical traction is not indicated for a C1-C2 fracture. Cervical immobilization is indicated. (u) D. Cervical collar immobilization is indicated for a minimally displaced C1 fracture.

A 23 year-old female presents with ongoing arthralgias with intermittent flares of arthritis. She is found to have a malar rash and an abnormal urinalysis. Serum ANA and anti-double-stranded DNA antibodies are present. Which of the following tests should be ordered to assess her risk for thrombotic events and future risk of spontaneous abortion? A. complete blood count B. PT/INR C. bleeding time D. anti-phospholipid antibodies

(u) A. CBC may reveal anemia, leukopenia and thrombocytopenia, but these are not the cause of thrombotic events and spontaneous abortion in SLE. (u) B. The PT/INR should not be altered in SLE. (u) C. Bleeding time measures platelet function, not risk for thrombosis. (c) D. Anti-phospholipid antibodies are present in 25% of SLE patients and may cause thrombotic events and spontaneous abortion.

A 74 year-old male with a history of coronary artery disease and atrial fibrillation presents to the clinic for follow-up of his shortness of breath. Patient's medications include amiodarone (Cordarone) and metoprolol (Lopressor). His chest x-ray reveals patchy ground-glass infiltrates. Which of the following is the most likely diagnosis? A. COPD B. Tuberculosis C. Bronchiectasis D. Pulmonary fibrosis

(u) A. COPD appears as hyperinflation with flattening of the diaphragm on chest radiograph. (u) B. Tuberculosis presents with pulmonary infiltrates on chest radiograph most often apical; cavitations may be seen with progressive primary tuberculosis. (u) C. Chest x-ray in bronchiectasis shows dilated and thickened bronchi that appear as ring-like markings. (c) D. Pulmonary fibrosis presents with ground-glass infiltrates on CXR and is often associated with certain medication use.

A 62 year-old female presents to the emergency room with significant back pain without radiation after lifting a box weighing approximately 15 pounds. She denies any previous trauma or injuries. Past history includes hysterectomy at age 42 and a 49 pack year smoking history. Her current weight is 107 pounds. Lumbo-sacral spine film indicates a spinal compression fracture at level L4. Which of the following tests would you perform to further assess the patient's findings? A. Computed tomography (CT) of the spine B. Magnetic resonance imaging (MRI) of the spine C. DEXA scan D. Technetium-99m bone scan

(u) A. CT of the spine should be reserved for fractures that remain symptomatic or progress after treatment. (u) B. MRI is a good tool in evaluation of union versus non union fractures and should be reserved for fractures that remain symptomatic or progress after treatment. (c) C. Low patient weight, smoking, and early estrogen deficiency are all risk factors for osteoporosis. Spontaneous fractures occurring from lifting with above risk factors should be evaluated for bone density. (u) D. Technetium-99m bone scan is useful in the evaluation of active bone formation (or lack of) and is reserved for fractures that remain symptomatic or progress after treatment.

A patient presents to the office following a syncopal episode. The patient claims that the syncope occurs when he changes position such as rolling over in bed or when he bends over to tie his shoes. Which of the following is the most likely explanation for this presentation? A. Carotid sinus hypersensitivity B. Vasovagal episode C. Subclavian steal syndrome D. Atrial myxoma

(u) A. Carotid sinus hypersensitivity may present with syncope but is usually related to tight collars or when excessively turning the head. (u) B. Vasovagal episodes may occur with syncope as its manifestation but it is not caused by changes in position. (u) C. Subclavian steal syndrome may present with syncope that is related to exercise of the affected arm which results in a decreased pulse when the Adson maneuver is performed. (c) D. Atrial myxoma most commonly presents with sudden onset of symptoms that are typically positional in nature due to the effect that gravity has on the tumor. Myxomas are the most common type of primary cardiac tumor in all age groups and are most commonly found in the atria.

The 35 year-old patient presents after a syncopal episode while throwing a football with his son. Examination reveals regular heart rate and EKG is normal. There were no symptoms prior to the episode. Right radial pulse is decreased. Which of the following is the most likely explanation for the syncope? A. Carotid sinus hypersensitivity B. Vasovagal episode C. Cardiac dysrhythmia D. Subclavian steal syndrome

(u) A. Carotid sinus hypersensitivity typically presents with syncope that is related to turning of the head (such as backing a car out of a driveway) or from tight collars. (u) B. Vasovagal episodes may result in syncope but there would not be abnormal pulse findings. (u) C. Cardiac dysrhythmia may occur in the setting of exercise but this would not produce a decreased pulse on the affected side. (c) D. Subclavian steal syndrome occurs if the subclavian artery is occluded proximal to the origin of the vertebral artery which results in reversal in the direction of blood flow in the ipsilateral vertebral artery. Exercise of the ipsilateral arm may increase demand on the vertebral flow which produces a "subclavian steal".

A 41 year-old female complains of 3 weeks of gradually worsening pain at the base of the thumb and radial aspect of the wrist. She and her husband have been renovating their home for the past 2 months and it has become increasingly difficult for her to hold a hammer. She denies numbness or tingling. She denies any history of previous trauma to the wrist. On examination, there is tenderness over the distal radial styloid and pain reproduced with ulnar deviation of a fist clenched over the abducted thumb. Which of the following is the most likely diagnosis? A. Carpal tunnel syndrome B. deQuervain's tenosynovitis C. Ganglion cyst D. Volar flexor tenosynovitis

(u) A. Carpal tunnel syndrome typically presents with pain and paresthesias in the median nerve distribution. (c) B. deQuervain's tenosynovitis typically results from repetitive activity involving pinching the thumb while moving the wrist. There is often pain and tenderness over the radial styloid and Finkelstein's is positive in this patient. (u) C. Ganglion cysts classically present with a visible or palpable, usually painless swelling over the dorsum of the wrist. (u) D. With volar flexor tenosynovitis, pain is expected with extension of the fingers and localized tenderness of the volar tendon sheaths.

A 42 year-old female experiences pain on the plantar surface of her left foot in the area of the third metatarsal head. The pain is associated with wearing tight shoes and is relieved by removing shoes. Examination reveals a palpable mass and reproduction of pain with deep palpation of the third intermetatarsal space. The patient has tried wearing wider shoes with metatarsal cushions and taking NSAIDS but her symptoms persist. What is the best therapeutic option at this point? A. Casting of the involved foot B. Physical therapy C. Steroid injection D. Surgical excision

(u) A. Casting the foot in a patient with Morton's neuroma is not effective therapy. (u) B. Physical therapy has not been shown to be of benefit in treating Morton's neuroma. (c) C. Steroid injection is the treatment of choice for Morton's neuroma when conservative measures fail. (u) D. Surgical excision is recommended for treatment of Morton's neuroma only if conservative measures and steroid injection have failed.

An elderly patient with a history of hypertension presents with epistaxis. On examination you note blood from both nares and down the posterior oropharynx. Examination of the nasal cavity with an ENT headlamp does not show an area of bleeding. Which of the following is the treatment of choice in this patient? A. Electrical cautery B. Direct pressure on the nose C. Petroleum jelly application D. Posterior nasal packing

(u) A. Cautery and direct pressure are effective in anterior, not posterior epistaxis. (u) B. See A for explanation. (u) C. Petroleum jelly application is not indicated in posterior epistaxis. (c) D. Posterior epistaxis is more common in elderly patients, especially with hypertension. Posterior nasal packing is the treatment of choice.

A 13 year-old boy with leukemia presents with epistaxis for 2 hours. The bleeding site appears to be from Kiesselbach's area. The most appropriate intervention is A. electrocautery of the bleeding site. B. silver nitrate application. C. posterior nasal packing. D. intranasal petrolatum gauze.

(u) A. Cautery is not used because the edges of the cauterized area may begin to bleed. (u) B. Silver nitrate is not used in children because it increases the risk for nasal septal perforation. (u) C. Posterior nasal packing is indicated for posterior bleeds in the inferior meatus. (c) D. Petrolatum gauze will provide pressure to the bleeding point while the cause of bleeding is corrected.

A 2 month-old infant has been diagnosed with pneumonia due to Chlamydia trachomatis. Which of the following is the treatment of choice? A. Ceftriaxone (Rocephin) B. Doxycycline C. Levofloxacin (Levaquin) D. Erythromycin

(u) A. Ceftriaxone is a third-generation cephalosporin that may be safely used in children, however is not indicated for the treatment of Chlamydial pneumonia. (h) B. Doxycycline is a tetracycline and is contraindicated in children under eight years of age secondary to damaging effects on bone and teeth enamel. (h) C. Levofloxacin is a fluoroquinolone and is contraindicated for use in children under 18 years of age secondary to damaging effects that may occur with growing cartilage. (c) D. Erythromycin or sulfisoxazole is the treatment of choice for an infant with Chlamydial pneumonia.

Which of the following medical complications of eating disorders is due to purging (vomiting/laxative abuse)? A. Intestinal obstruction B. Reduced thyroid metabolism C. Amenorrhea D. Electrolyte abnormalities

(u) A. Intestinal obstruction is not caused by vomiting. (u) B. Reduced thyroid metabolism is a complication of anorexia nervosa related to cachexia. (u) C. Amenorrhea occurs as a result of anorexia nervosa with weight loss. (c) D. Electrolyte abnormalities, particularly hypokalemia, hypochloremic alkalosis, and hypomagnesemia may occur as a result of purging.

A 75 year-old patient with history of macular degeneration and hypertension presents with complaint of sudden onset of visual loss in the left eye. The patient denies pain. On examination you note a dome-shaped retina and subretinal fluid that shifts with position changes. Which of the following is the most likely diagnosis in this patient? A. Central retinal vein occlusion B. Acute angle-closure glaucoma C. Acute nongranulomatous anterior uveitis D. Serous retinal detachment

(u) A. Central retinal vein occlusion is characterized by sudden monocular visual loss on examination there would be disc swelling, venous engorgement, cotton-wool spots, and diffuse retinal hemorrhages. (u) B. Acute angle-closure glaucoma is characterized by pain and blurred vision. On examination the eye is red, the cornea is steamy, and the pupil is moderately dilated and nonreactive to light. (u) C. Acute nongranulomatous anterior uveitis presents with acute unilateral eye pain, redness, photophobia, and vision loss. (c) D. Serous retinal detachment is characterized by a dome shaped retina and subretinal fluid that shifts position with posture changes. Serous retinal detachment results from subretinal fluid accumulation which can occur in exudative age-related macular degeneration.

Which of the following is most appropriate to rule out vertebrobasilar insufficiency as a cause of recurrent vertigo? A. Carotid Doppler B. Dix-Hallpike maneuver C. Epply maneuver D. Magnetic resonance imaging

(u) A. Central vertigo can be caused by cerebral ischemia. Since this stems from the vertebral basilar distribution it would not be identifiable on carotid ultrasound studies. (u) B. Dix-Hallpike maneuver is used as a reproducing test for positional vertigo not vertebrobasillar insufficiency. (u) C. Epply maneuver is used a treatment for benign paroxysmal positioning vertigo. (c) D. MRI is the most appropriate of the listed screening tools to differentiate vertebrobasilar insufficiency from other etiologies.

A patient presents for a follow-up visit for chronic hypertension. Which of the following findings may be noted on the fundoscopic examination of this patient? A. cherry-red fovea B. boxcar segmentation of retinal veins C. papilledema D. arteriovenous nicking

(u) A. Cherry-red fovea and boxcar segmentation of the retinal veins are findings seen in central retinal artery occlusion. (u) B. See letter A for explanation. (u) C. Papilledema is noted in conditions causing increased intracranial pressure. (c) D. Arteriovenous nicking is common in chronic hypertension.

A 12 month-old child with tetralogy of Fallot is most likely to have which of the following clinical features? A. Chest pain B. Cyanosis C. Convulsions D. Palpitations

(u) A. Chest pain is not a feature of tetralogy of Fallot. (c) B. Cyanosis is very common in tetralogy of Fallot. (u) C. Convulsions are occasionally seen as part of severe hypoxic spells in infancy rather than a feature of tetralogy of Fallot. (u) D. Palpitations are uncommon in tetralogy of Fallot.

: (8) 176. Diagnostic Studies/Cardiology You are evaluating a trauma patient who was unrestrained during a head-on collision. His chest struck the steering wheel and he is complaining of pain in the pre-cordial region. He is short of breath and complains of a non-productive cough since the injury. Vital signs indicate a heart rate of 140 bpm, respiration rate of 24, and blood pressure of 80/60 mmHg. His electrocardiogram shows nonspecific T wave changes and low QRS voltage. Which imaging modality would be the most appropriate given this history of present illness? A. Chest radiograph B. CT of the chest C. Echocardiogram D. MRI of the chest

(u) A. Chest radiograph can suggest chronic effusion by an enlarged cardiac silhouette but may appear normal in acute situations. (u) B. Cardiac CT and MRI also demonstrate pericardial fluid however, for expediency and cost, echocardiogram should be considered first. (c) C. Echocardiography is the primary method for demonstrating pericardial tamponade. (u) D. Cardiac CT and MRI also demonstrate pericardial fluid however, for expediency and cost, echocardiogram should be considered first.

A 62 year-old homeless patient presents complaining of fever, weight loss, anorexia, night sweats and a chronic cough that recently became productive of purulent sputum that is blood streaked. On physical examination, the patient appears chronically ill and malnourished. Which of the following chest x-ray findings supports your suspected diagnosis? A. Hyperinflation and flat diaphragms B. Interstitial fibrosis and pleural thickening C. Cavitary lesions involving the upper lobes D. "Eggshell" calcification of hilar lymph nodes

(u) A. Chest x-ray findings of hyperinflation and flat diaphragms suggest long-standing chronic obstructive lung disease. (u) B. Interstitial fibrosis and pleural thickening on a chest x-ray are found in cases of interstitial lung disease. (c) C. This patient most likely has tuberculosis. A chest x-ray finding of cavitary lesions involving the upper lobes would support this suspected diagnosis. (u) D. Chest x-ray findings of "eggshell" calcification of hilar lymph nodes strongly supports a diagnosis of silicosis.

: (13) 85. Diagnostic Studies/Urology/Renal A 25 year-old female presents with acute right knee pain with associated dysuria, urgency, and body aches. She admits to an episode of unprotected sex 2 weeks ago. Which of the following organisms is most likely to be found on joint aspiration? A. Chlamydia trachomatis B. Herpes simplex virus C. Neisseria gonorrhea D. Human immunodeficiency virus

(u) A. Chlamydia infection is often cotransmitted with gonorrhea and the patient should be treated for chlamydia as well. However, the organism does not seed the joint and present as monoarticular arthritis. (u) B. The patient needs a full workup to evaluate and treat for and sexually transmitted diseases, including herpes 37 and HIV. However, neither is likely to present as cellulitis of a joint. Herpes, however, can present with its own characteristic skin lesions. (c) C. Gonococcal arthritis usually occurs in otherwise healthy individuals - women more commonly than men, although men who have sex with men are at higher risk. Dysuria and frequency are common with gonococcal infection and low grade fevers and migratory joint pain can follow. Skin lesions may be noted over the knee on physical exam. Analysis of joint fluid is needed to diagnose the disease, which is caused by gram-negative diplococci - Neisseria gonorrhea. (u) D. The patient needs a full workup to evaluate and treat for sexually transmitted diseases, including herpes and HIV. However, neither is likely to present as cellulitis of a joint. Herpes, however, can present with its own characteristic skin lesions.

Which of the following is first-line treatment for acute inflammatory demyelinating polyneuropathy variant of Guillain-Barre? A. Neostigmine B. IV Solu-Medrol C. Phenytoin D. IV immunoglobulin

(u) A. Neostigmine may be used for a more chronic variant for symptoms, but not in acute inflammatory variant. (u) B. Treatment with corticosteroids is ineffective and may prolong recovery time. (u) C. Phenytoin may be used for ongoing neuropathic pain, but is not useful in any type of acute neuropathy. (c) D. IV immunoglobulin is effective in patients with Guillain-Barre.

A 45 year-old male presents with a long history of ulcerative colitis and recent progressive complaints of right upper quadrant pain, weight loss, fever and most recently, a rapid onset of jaundice with pruritus. Labs revealed elevated bilirubin and alkaline phosphatase. Viral serologies were negative. An endoscopic cholangiogram showed areas of stenosis and dilation throughout the bile duct system. What is the most likely diagnosis? A. Choledocholithiasis B. Hepatic carcinoma C. Portal hypertension D. Primary sclerosing cholangitis

(u) A. Choledocholithiasis can most certainly cause elevated bilirubin and other liver function tests when obstruction occurs. It also occurs more often in persons with sclerosed or narrowed bile ducts however it does not generally carry a poor prognosis and can be generally recognized by its typical symptoms. (u) B. Hepatic carcinoma does not cause areas of varied stenosis and dilation in the biliary tree. (u) C. Portal hypertension may present with jaundice but generally has significant ascites as well that helps to distinguish this disorder. It also does not result in the bile duct pattern mentioned. (c) D. Primary sclerosing cholangitis (PSC) results in diffuse intra- and extrahepatic duct sclerosing with dilatation proximal to these areas.

In which of the following conditions would human chorionic gonadotropin (hCG) level be lower than expected for gestational age? A. Choriocarcinoma B. Hydatidiform mole C. Ectopic pregnancy D. Twin gestation

(u) A. Choriocarcinoma is a persistent form of gestational trophoblastic neoplasia and will have an elevated hCG level. (u) B. Quantitative hCG levels are excessively elevated for the gestational age of the pregnancy. (c) C. An ectopic pregnancy will have an abnormally low hCG level because the hCG will not double every 48 hours as a normal pregnancy would. (u) D. A twin gestation will have a higher hCG level because of the presence of two fetuses.

A 60 year-old male complains of progressive fatigue and dyspnea. On examination his lungs are clear to auscultation bilaterally, heart exam reveals regular rate and rhythm without S3, S4 or murmur, and extremities show 1+ edema bilaterally. Chest x-ray reveals cardiomegaly. electrocardiogram shows low voltage, and echocardiogram shows an ejection fraction of 55% with a small, thickened left ventricle that has rapid early filling with diastolic dysfunction. Which of the following is the most likely underlying etiology of this patient's cardiomyopathy? A. Alcoholism B. Myocarditis C. Amyloidosis D. Chronic hypertension Explanations (u) A. Chronic alcohol use is commonly associated with a dilated left ventricle with left ventricular dysfunction. (u) B. Myocarditis is associated with a dilated, not small, left ventricle. (c) C. Amyloidosis is the most common cause of restrictive cardiomyopathy and is associated with a small thickened left ventricle that has rapid early filling with diastolic dysfunction. (u) D. Chronic hypertension is associated with a hypertrophic, hypercontractile left ventricle.

(u) A. Chronic alcohol use is commonly associated with a dilated left ventricle with left ventricular dysfunction. (u) B. Myocarditis is associated with a dilated, not small, left ventricle. (c) C. Amyloidosis is the most common cause of restrictive cardiomyopathy and is associated with a small thickened left ventricle that has rapid early filling with diastolic dysfunction. (u) D. Chronic hypertension is associated with a hypertrophic, hypercontractile left ventricle

A 65 year-old homeless male with a history of pancreatitis is seen in the emergency department for vomiting, upper abdominal pain, back pain and weakness. He is cachetic, pale and jaundiced. A 4-5 cm mass is palpable in the mid to right hypochondrium. What is the most likely diagnosis? A. Chronic cholecystitis B. Carcinoma of head of pancreas C. Fibrolipoma D. Primary biliary cirrhosis

(u) A. Chronic cholecystitis is not typically associated with weight loss or cachexia. There would not be a palpable mass. (c) B. Seventy-five percent of pancreatic cancers are in the head. Risk factors include age, tobacco use, obesity, chronic pancreatitis, family history and previous abdominal radiation. (u) C. Fibrolipoma may present as an abdominal mass, but would not cause weight loss and illness. (u) D. Primary biliary cirrhosis most commonly presents with generalized urticaria and is not associated with an abdominal mass.

: (10) 198. Clinical Therapeutics/Gastrointestinal/Nutritional A 19 year-old female presents with intermittent episodes of loose stools over the past 6 to 8 months. She admits to crampy abdominal pain and bloating that seems to be more associated with her stress level than with food intake. She denies fever, melena, or nocturnal symptoms. Examination reveals normoactive bowel sounds and mild lower abdominal tenderness on palpation without rebound tenderness or organomegaly. Which of the following is the treatment of choice in this patient? A. Cimetidine (Tagamet) B. Metronidazole (Flagyl) 78 C. Metoclopramide (Reglan) D. Dicyclomine (Bentyl)

(u) A. Cimetidine (Tagamet) is used in the management of GERD and PUD not IBS. (u) B. Metronidazole (Flagyl) is indicated in the treatment of Clostridium difficile associated diarrhea, not irritable bowel syndrome. (u) C. Metoclopramide (Reglan) is used in the management of nausea and vomiting, not irritable bowel syndrome. (c) D. Dicyclomine (Bentyl) and other anticholinergic agents can help relieve abdominal symptoms of cramping and bloating associated with irritable bowel syndrome.

A 62 year-old female is admitted to a nursing home during an outbreak of influenza. In review of her records, you note that she did not receive the flu vaccine this year. Which of the following is the most appropriate drug of choice for influenza prophylaxis in this patient? A. Ciprofloxin (Cipro) B. Oseltamivir phosphate (Tamiflu) C. Clarithromycin (Biaxin) D. Alpha-2b interferon (Avonex)

(u) A. Ciprofloxin is indicated for postexposure prophylaxis of anthrax. (c) B. Either oseltamivir or zanamivir are indicated for prophylactic use against influenza A or B. (u) C. Clarithromycin is indicated for prophylaxis against disseminated Mycobacterium avium complex. (u) D. Alpha-2b interferon is indicated for treatment of several disorders, such as chronic hepatitis B & C, but has no role in prophylactic treatment of any condition.

A patient with which of the following is at highest risk for coronary artery disease? A. Congenital heart disease B. Polycystic ovary syndrome C. Acute renal failure D. Diabetes mellitus

(u) A. Congenital heart disease is not an established risk factor for coronary artery disease. (u) B. While patients with polycystic ovary syndrome have hyperinsulimemia, they do not have the same poor prognosis for coronary artery disease as patients with diabetes mellitus. (u) C. Patients with acute renal failure are not at risk for coronary artery disease, although patients with diabetes and chronic renal disease do have this risk. (c) D. Patients with diabetes mellitus are in the same risk category for coronary artery disease as those patients with established atherosclerotic disease.

: (8) 20. Diagnosis/ENT/Ophthalmology A 72 year-old female with a history of diabetes mellitus presents with sudden onset of painless flashing lights and floaters followed by curtain-like vision loss sparing the central vision in the right eye. Which of the following is the most likely diagnosis? A. Classic migraine B. Retinal detachment C. Optic neuritis D. Transient ischemic attack

(u) A. Classic migraine may present with the above symptoms; however, the symptoms are typically bilateral. Additionally, given the advanced age of the patient migraine would be considered less likely. (c) B. Non-traumatic retinal detachment usually presents in an older individual. The classic symptoms include abrupt onset of painless flashing lights and floaters followed by progressive vision loss. (u) C. Optic neuritis may present with the above symptoms; however, the mean age of onset is 32 years of age and the presentation typically involves ocular pain. (u) D. Vertebrobasilar insufficiency may result in dysfunction of the visual cortex with subsequent vision disturbance similar to the above presentation. In contrast the patient may present with additional findings of sensorimotor dysfunction.

A 33 year-old patient returns from a community picnic that was held on a hot July day. The patient developed severe explosive diarrhea, nausea, vomiting and abdominal cramps six hours after having homemade potato salad and hot dogs. Multiple people who ate at this picnic have similar symptoms. There is no associated fever. What is the most likely causative organism for these symptoms? A. Clostridium perfringens B. Campylobacter jejuni C. Shigella D. Staphylococcus aureus

(u) A. Clostridium perfringens has a 12 to 24 hour incubation period and symptoms are limited to diarrhea and crampy abdominal pain but nausea and vomiting are unusual. (u) B. Campylobacter jejuni, the most common cause of acute bacterial diarrhea, has a prodrome period of 12 to 24-hours with headache, fever, and malaise that is followed by crampy abdominal pain and diarrhea. (u) C. Shigella infection causes bacterial dysentery in tropical regions most commonly and transmission is usually from person to person rather than from food-borne spread. Nausea and vomiting are unusual. (c) D. Staphylococcus aureus is an acute illness caused by the ingestion of enterotoxin-producing organism. It is the only common acute ingestion of a preformed toxin. The clinical picture in the question is the classic presentation for this illness. Foods most responsible for this infection are ham, cream, custard pastries, and mayonnaise-based salads.

What nail finding is most consistent with cirrhotic liver disease? A. Clubbing B. Pitting C. Terry's nails D. Beau's lines

(u) A. Clubbing is a bulbous swelling of soft tissue at the nail base most commonly seen in congestive heart disease, interstitial lung disease, lung cancer, irritable bowel syndrome, and malignancies. (u) B. Pitting appears as punctuate depressions of the nail caused by defective layering of the superficial nail plate by the proximal nail matrix. Pitting is usually associated with psoriasis. (c) C. Terry's nails occur when the nail plate turns white with a ground glass appearance, a distal band of reddish brown, and obliteration of the lunula. Terry's nails are seen in liver disease, usually cirrhosis. (u) D. Beau's lines are transverse depressions of the nails and are usually bilateral and result from temporary disruption of proximal nail growth from systemic illness

An 18 year-old female presents with 2-day duration of dysuria, vulvovaginal pruritis, and a frothy clear to white discharge. Which of the following results would be expected? A. clue cells on normal saline preparation B. hyphae and budding yeast on a KOH preparation C. intracellular gram negative diplococci on Gram stain D. mobile flagellated protozoa on a normal saline preparation

(u) A. Clue cells are seen in Gardnerella vaginalis infections, but the vaginal discharge is grayish and has an unpleasant fishy odor. (u) B. Candida infections are diagnosed by demonstrating hyphae and budding yeast on KOH and usually present with a white cottage cheese discharge. (u) C. Gonorrhea presents with a purulent discharge. Gram negative diplococci are not diagnostic in females. (c) D. Trichomonas presents with a frothy discharge, irritative symptoms of pruritus, dysuria, and frequency, and the flagellated protozoa are demonstrated on a saline preparation.

Which of the following is considered to be the treatment of choice for the pruritus that occurs with primary biliary cirrhosis? A. Colchicine (Colzalide) B. Atorvastatin (Lipitor) C. Cholestyramine (Questran) D. Enalapril (Vasotec)

(u) A. Colchicine is used to improve the biochemical abnormalities which may slow the progression of the disease. (u) B. Statins have a role in the management of hyperlipidemia but are not effective in this disease. (c) C. Cholestyramine, a bile salt sequestrant, is able to decrease the pruritus that occurs from the bile stasis and granulomas. (u) D. ACE inhibitors are effective as antihypertensives and in preserving renal function in those with proteinuria, but they have no role in the management of primary biliary cirrhosis.

A patient with a history of chronic venous insufficiency presents for routine follow-up. Which of the following findings is most likely on physical examination? A. Cold lower extremities B. Diminished pulses C. Lower extremity edema D. Palpable cord

(u) A. Cold lower extremities are more commonly seen in peripheral arterial, not venous, disease. (u) B. Diminished pulses are seen in peripheral arterial disease. (c) C. Patients with chronic venous insufficiency will commonly have lower extremity edema. (u) D. A palpable cord is more common in superficial thrombophlebitis.

A 30 year-old female presents to the office complaining of generalized weakness and reduced exercise tolerance that improves with rest. On physical examination you note the presence of bilateral eyelid ptosis, proximal muscle weakness and normal reflexes. What is the most likely diagnosis? A. Lambert-Eaton syndrome B. Organophosphate intoxication C. Multiple sclerosis D. Myasthenia gravis

(u) A. Common symptoms of Lambert-Eaton syndrome are proximal muscle weakness of lower limbs, cranial nerve findings, and depressed or absent reflexes. Patients commonly have a malignancy. (u) B. Patients with organophosphate intoxication have seizures, excessive secretions, wheezing and diaphoresis. (u) C. Patients with multiple sclerosis have multiple lesions in time and space. (c) D. Common symptoms of myasthenia gravis are fatigable weakness, ptosis, diplopia, and proximal muscle weakness. The disease is more common in women in the 2nd and 3rd decade and in men older than 60.

A 25 year-old female with a history of diarrhea presents to the emergency department with Guillian-Barre syndrome. What is the most likely causative pathogen in this patient? A. Shigella dysenteriae B. Yersinia enterocolitica C. Entamoeba histolytica D. Campylobacter jejuni

(u) A. See D for explanation. (u) B. See D for explanation. (u) C. See D for explanation. (c) D. Campylobacter jejuni is a Gram-negative bacteria that has been linked to the subsequent development of Guillain-Barré syndrome (GBS). GBS usually develops two to three weeks after the initial illness.

A 25 year-old female graduate student presents to the student health center for the eighth time in three weeks to be sure she does not have meningitis. She read that there was a student on campus who had meningitis last month, and now she has headaches and is requesting to be tested to make sure she does not have meningitis. She has been evaluated at each visit, and physical examination has been completely normal each time. Which of the following is the most likely diagnosis? A. Conversion disorder B. Hypochondriasis C. Malingering D. Somatization disorder

(u) A. Conversion disorder is characterized by onset of symptoms or deficits mimicking neurologic or medical illness, but the etiology is psychological. (c) B. Hypochondriasis is the chronic preoccupation with the idea of having a serious disease, which is usually not amenable to reassurance (u) C. Malingering is the intentional production or feigning of physical or psychological signs and symptoms for some gain. (u) D. Somatization disorder is characterized by complaints of pain, often related to gastrointestinal and sexual dysfunction, and pseudoneurological symptoms.

Which of the following is described as a harmless triangular nodule in the bulbar conjunctiva on either side of the iris? A. Corneal arcus B. Hordeolum C. Pinguecula D. Xanthelasma

(u) A. Corneal arcus is an extremely common, bilateral, benign peripheral corneal degeneration, associated hyperlipidemia. (u) B. Hordeolum is characterized by a localized red, swollen, acutely tender area on the upper or lower lid. (c) C. Pinguecula is a yellow elevated conjunctival nodule, more commonly on the nasal side, in the area of the palpebral fissure. (u) D. Xanthelasma appear as yellow plaques that occur on the anterior surface of the eyelid, usually bilaterally near the inner angle of the eye.

A 60 year-old male with history of hypertension and hyperlipidemia presents with intermittent chest heaviness for one month. The patient states he has had occasional heaviness in his chest while walking on his treadmill at home or shoveling snow. He also admits to mild dyspnea on exertion. His symptoms are relieved with 2-3 minutes of rest. He denies lightheadedness, syncope, orthopnea or lower extremity edema. Vitals reveal a BP of 130/90, HR 70, regular, RR 14. Cardiac examination revealed a normal S1 and S2, without murmur or rub. Lungs were clear to auscultation. Extremities are without edema. EKG reveals no acute change and cardiac enzymes are negative. Which of the following is the most appropriate next diagnostic study? A. cardiac catheterization B. nuclear exercise stress test C. helical (spiral) CT D. transthoracic Echocardiogram

(u) A. Coronary angiography is indicated in patients with classic stable angina who are severely symptomatic despite medical therapy and are being considered for percutaneous intervention (PCI), patients with troublesome symptoms that are difficult to diagnose, angina symptoms in a patient who has survived sudden cardiac death event, patients with ischemia on noninvasive testings, a stress test is a better initial diagnostic study for this patient. (c) B. This patient has signs and symptoms of classic angina; nuclear stress testing is the most useful noninvasive procedure for diagnosis of ischemic heart disease and evaluation of angina in this patient. (u) C. Helical CT is used in the diagnosis of pulmonary embolism, not in the evaluation of angina. (u) D. Echocardiogram is used in the evaluation of valvular heart disease not in the evaluation of suspected myocardial ischemia.

Which of the following is the most common cause for acute myocardial infarction? A. Occlusion caused by coronary microemboli B. Thrombus development at a site of vascular injury C. Congenital abnormalities D. Severe coronary artery spasm

(u) A. Coronary microemboli occlusion is a rare cause of acute myocardial infarction. (c) B. Acute myocardial infarction occurs when a coronary artery thrombus develops rapidly at a site of vascular injury. In most cases, infarction occurs when an atherosclerotic plaque fissures, ruptures, or ulcerates and when conditions favor thrombogenesis, so that a mural thrombus forms at the site of rupture and leads to coronary artery occlusion. (u) C. Congenital abnormalities are rare causes of acute MI. (u) D. Severe coronary artery spasm is more likely to result in Prinzmetal's angina rather than true infarction.

Use of systemic corticosteroids can cause which of the following adverse effects in the eye? A. Cortical blindness B. Optic atrophy C. Glaucoma D. Papilledema

(u) A. Cortical blindness is a rare adverse effect when prescribing salicylates. (u) B. Optic atrophy can occur as an adverse effect with lead compounds, amebicides, and MAO inhibitors. (c) C. Glaucoma can be caused by the long-term use of steroids. (u) D. Papilledema can be a side effect to many systemic medications.

A 44 year-old female hairdresser presents to the clinic with a two-month history of pain and numbness of her left hand that awakens her from sleep. Your exam reveals electric tingling sensations with percussion over the volar surface of the wrist just proximal to the palmar crease and paresthesias noted within 15 seconds of passively flexing the wrist. Which of the following would be the best initial treatment in this patient? A. Corticosteroid injection B. Surgical intervention C. Nocturnal splinting D. Gabapentin (Neurontin)

(u) A. Corticosteroid use is recommended after initial treatment is unsuccessful. (u) B. Surgical intervention is reserved for patients not responsive to initial/conservative treatment. (c) C. Splinting the affected wrist is the initial treatment choice and maintains a neutral position of the wrist. (u) D. Gabapentin is not indicated in the management of carpal tunnel syndrome.

A 57 year-old male recently on a high protein diet presents with an exquisitely tender, erythematous, warm right great toe. Which of the following is the treatment of choice for this patient? A. Corticosteroids B. Colchicine C. Allopurinol D. Non-steroidal antiinflammatory agents

(u) A. Corticosteroids are effective in acute gout attacks but are reserved for people with non-steroidal anti-inflammatory agent allergies or contraindications. (u) B. Colchicine is not recommended in the treatment of acute gouty arthritis. (u) C. Allopurinol is useful in reducing uric acid levels but is not the treatment of choice in acute gouty arthritis. (c) D. NSAID medications are the treatment of choice for acute gouty attacks.

A previously healthy 8 month-old boy is hospitalized for acute bronchiolitis. He has no known significant past medical or family history. On admission, he exhibits nasal flaring and retractions with a respiratory rate of 68, axillary temperature of 102.0 degrees F and O2 saturation of 86%. Which of the following medications is indicated? A. Prednisolone B. Oxygen C. Ceftriaxone (Rocephin) D. Palivizumab (Synagis)

(u) A. Corticosteroids are not indicated for the treatment of previously healthy infants with bronchiolitis. (c) B. Oxygen is an important supportive therapy for hypoxemic infants with bronchiolitis. Bronchodilators would also be initiated in this patient. (u) C. Antibiotics are not indicated in the treatment of bronchiolitis unless there is a secondary bacterial infection. (u) D. Palivizumab is used only for prevention of RSV infection.

A 34 year-old female, with a past medical history of irritable bowel syndrome and migraines, presents with fatigue, generalized aching and stiffness of the trunk, hip, and shoulder girdles. She complains of pain and tightness in the neck and across the upper posterior shoulders. She complains of poor sleep, but denies depression. Physical examination is unremarkable except for numerous tender points on palpation. Laboratory evaluation was unremarkable. Which of the following is the best intervention for this patient? A. corticosteroids B. supervised exercise program C. hydrocodone (Vicodin) D. refer to endocrinologist

(u) A. Corticosteroids have no role in the management of fibromyalgia. (c) B. A carefully planned and individualized exercise program has been proven effective for the management of fibromyalgia. (u) C. Opioids are not first-line agents in the treatment of fibromyalgia. (u) D. One-third of patients with fibromyalgia are found to have deficiency of growth hormone, but referral to endocrinology is not indicated at this time.

In the neonate, unequal thigh folds may indicate which of the following? A. Coxa vara B. Legg-Calve-Perthes disease C. Developmental hip dysplasia D. Slipped capital femoral epiphysis

(u) A. Coxa vara is a hip deformity that would present with a decrease in the hip angle and a shift of the femoral shaft medially. (u) B. Legg-Calve-Perthes disease presents with a painless limp in children ages 4-10 due to avascular necrosis of the femoral head. (c) C. A dislocated hip displaces proximally in developmental hip dysplasia, causing a shortening of the leg that may present as unequal thigh folds. (u) D. A slipped capital femoral epiphysis is primarily an adolescent disorder with decreased range of motion in abduction and internal rotation of the hip on physical examination.

: (7) 79. History & Physical/ENT/Ophthalmology Which of the following is a characteristic finding on examination of a patient who presents with herpangina? A. Scattered vesicles of the buccal mucosa and fingers B. Erythema and pustular exudates of the tonsillar crypts C. Vesicular lesions of the tonsillar fauces D. Grouped vesicles about the vermillion border

(u) A. Coxsackie virus A16 often presents with scattered vesicles about the mouth along with associated hand and foot lesions (u) B. Erythema and pustular exudates of the tonsillar crypts are typical findings associated with group A β-hemolytic strep infection. (c) C. Herpangina, associated with a number of enteroviruses, typically presents with painful vesicles about the tonsillar fauces. (u) D. Grouped vesicles around the vermillion border is most consistent with herpes simplex labialis.

Closure of the eyelids is mediated by which cranial nerve? A. cranial nerve III B. cranial nerve V C. cranial nerve VII D. cranial nerve IX

(u) A. Cranial nerve III is involved in extraocular movement and controls opening of the eyelids but has no control over closing the eyelids. (u) B. Cranial nerve V controls the motor function of the temporal and masseter muscles and facial sensation. It has no control over eyelid closure. (c) C. Cranial nerve VII controls the motor function of the facial muscles not controlled by cranial nerve V (forehead, eyebrows, mouth, and lips) including closing of the eyelids. (u) D. Cranial nerve IX controls swallowing and the voice.

: (8) 197. Diagnostic Studies/Cardiology A 63 year-old patient was admitted with an acute non-ST elevation myocardial infarction 3 days ago confirmed by elevated CK, CK-MB, troponin I and troponin T. He begins to experience recurrent chest pain. Which laboratory study is most appropriate to evaluate his recurrent chest pain? A. Creatine kinase B. CK-MB C. Troponin D. Myoglobin (Mb)

(u) A. Creatine kinase (CK) levels may rise in response to insults of other etiologies than just myocardial tissue damage. As CK levels are non-specific, they are not appropriate diagnostic test for the evaluation of cardiac chest pain. (c) B. Cardiac specific markers of myocardial infarction include quantitative determinations of CK-MB, troponin I and T. Troponins are more sensitive and specific than CK-MB. All tests should become positive as early as 4-6 hours after onset of a myocardial infarction and should be abnormal by 8-12 hours. Troponins may remain elevated for 5-7 days or longer. CK-MB generally normalizes within 24 hours, thus being more helpful for evaluation of reinfarction. (u) C. As troponins remain elevated for 5-7 days or longer, they are not useful in the evaluation of a patient with chest pain who already had elevated levels. (u) D. Myoglobin levels will elevate sooner than the other options, however they are non-specific so not a useful marker of specific cardiac injury.

Which of the following is the most sensitive to determine whether there is a small effusion in the knee? A. Compress the patella and move it against the femur, noting any crepitus B. Flex the knee to about 90 degrees and palpate for tenderness over the joint line C. Milk the medial aspect of the knee, press lateral margin of the patella, and note a bulge of returning fluid medial to the patella D. Force fluid into space between the patella and the femur, tap the patella over the femur to detect a click

(u) A. Crepitus without pain is not significant and does not indicate an effusion. (u) B. Tenderness over the joint line indicates a meniscal injury, but does not demonstrate an effusion. (c) C. A small bulge of returning fluid after milking fluid upward from the knee is useful for detecting small effusions. (u) D. Ballottement of the patella against the femur is useful for detecting large effusions, but not small ones.

A patient presents with an episode of an expansive, elevated mood during which she cleaned excessively without sleeping. Which of the following is the most likely diagnosis? A. Major depressive disorder B. Bipolar disorder C. Schizoaffective disorder D. Dysthymic disorder

(u) A. Criteria for major depressive disorder do not include elevated expansive moods. (c) B. Bipolar disorder is characterized by episodic mood shifts from depression to manic type moods which is often rapid with depression lasting longer than manic episodes. Bipolar disorder may initially present with a manic episode. (u) C. Schizoaffective disorder presents with a mood disorder and characteristics of schizophrenia. (u) D. Dysthymia has no elevated moods or manic type behaviors.

A patient presents to the office with worsening fatigue, weight loss, and weakness. She notes that she is having recurrent bouts of abdominal pain and has been losing her pubic hair. Patient is found to have orthostatic hypotension. Which of the following conditions is most likely? A. Cushing's syndrome B. Pheochromocytoma C. Primary hyperparathyroidism D. Addison's disease

(u) A. Cushing's syndrome is caused by an increase in the cortisol levels in the body. These patients will have hypertension, buffalo hump, striae, and proximal muscle weakness. (u) B. Pheochromocytoma is caused by an increase in the release of catecholamines from the adrenal medulla. These patients will have episodic hypertension followed by sustained hypertension and bouts of diaphoresis and shakiness. (u) C. Patients with primary hyperparathyroidism are most likely to be asymptomatic. If these patients have symptoms, it is most likely that they will have abdominal pain, renal stones, and bone pain because of the resultant increase in the serum calcium levels. (c) D. Patients with Addison's disease have primary adrenal failure from an autoimmune problem in the adrenal gland or due to hemorrhage into the adrenal gland. These patients are not able to make glucocorticoids, mineralocorticoids, or sex hormones which result in hypotension, hyperpigmentation (from an increase in the ACTH and MSH hormones) and are hyponatremic.

Which of the following is essential to make a diagnosis of cystic fibrosis? A. Positive family history B. Elevated sweat chloride C. Recurrent respiratory infections D. Elevated trypsinogen levels

(u) A. Cystic fibrosis is a genetic disease, but a positive family history in and of itself is not enough to diagnose the condition. (c) B. The diagnosis of cystic fibrosis is made only after an elevated sweat chloride test or demonstration of a genotype consistent with cystic fibrosis. (u) C. While recurrent respiratory infections is a classic presentation of cystic fibrosis, the diagnosis relies on confirmation, as noted in explanation B. (u) D. Trypsinogen levels are used as a neonatal screening test and if elevated should be followed by more definitive testing to confirm the diagnosis.

A male patient complains of chronic dysuria, frequency, and urgency with associated perineal pain. The most likely diagnosis is A. cystitis. B. gonococcal urethritis. C. epididymitis. D. prostatitis.

(u) A. Cystitis is characterized by dysuria without urethral discharge. (u) B. Initially there is burning on urination and serous or milky discharge in gonococcal urethritis. (u) C. Epididymitis is characterized by dysuria, unilateral scrotal pain and swelling. (c) D. Some patients are asymptomatic, but low back or perineal pain, fever, chills, and irritative urinary symptoms are common in prostatitis.

Which of the following is the diagnostic study of choice in the evaluation of Zollinger-Ellison syndrome? A. D-xylose absorption test B. Gastrin level C. 5-hydroxyindoleacetic acid assay (5-HIAA) D. Urea breath test

(u) A. D-xylose absorption test is used in the evaluation of possible gluten-induced enteropathy. (c) B. Zollinger-Ellison Syndrome (ZES) is characterized by elevated levels of serum gastrin resulting from gastrin-secreting tumors usually located in the duodenum or pancreas. (u) C. 5-HIAA levels are elevated in metastatic carcinoid tumors. (u) D. The urea breath test is used to evaluate ulcers caused by H. pylori infection

A 12 month-old in the emergency department is diagnosed with possible viral meningitis. Which of the following cerebral spinal fluid (CSF) laboratory results is most consistent with this diagnosis? A. Decreased CSF glucose level and increased protein B. Decreased CSF total protein level and very few neutrophils C. Increased CSF mononuclear cells and normal glucose D. Increased CSF C-reactive protein and normal glucose

(u) A. Decreased CSF glucose level and increased protein is consistent with bacterial meningitis. (u) B. Decreased CSF total protein level and very few neutrophils is a finding in syphilitic meningitis. (c) C. In aseptic meningitis, CSF shows mainly mononuclear cells within 6-8 hours, glucose is normal and there is normal to lower protein. (u) D. Increased CSF C-reactive protein and normal glucose is found in any inflammatory process affecting the CSF.

Which of the following factors in patients with chronic venous insufficiency predisposes them to development of skin ulcers? A. Increased intravascular oncotic pressure B. Leakage of fibrinogen and growth factors into the interstitial space C. Decreased capillary leakage D. Inherited deficiency of protein C

(u) A. Decreased intravascular oncotic pressure can cause swelling (c) B. Leakage of fibrinogen and growth factors into the interstitial space, leukocyte aggregation and activation, and obliteration of the cutaneous lymphatic network can predispose a patient to skin ulcers (u) C. Increased capillary leakage causes venous insufficiency. (u) D. Inherited deficiency of protein C predisposes patients to thrombosis.

A patient tells you that he is receiving special messages from the TV every night at 7:00 pm. This is an example of which of the following? A. Delusions B. Ideas of reference C. Paranoia D. Suicidal ideation

(u) A. Delusions are false fixed beliefs. (c) B. Ideas of reference are fixed beliefs that people are referring to you and about you through media. (u) C. He is frankly telling you this and not displaying anything that makes you think he is paranoid. (u) D. He is not stating he is having thoughts of harming or killing himself.

A 50 year-old patient presents complaining of headache and left eye pain for 5 hours. The patient admits to decreased vision in the left eye. The patient also complains of associated nausea. Which of the following is most likely on physical examination? A. Neovascularization B. Central vision loss C. Impaired red reflex D. Moderately dilated, nonreactive pupil

(u) A. Neovascularization is seen with diabetic retinopathy not glaucoma. (u) B. Central vision loss is seen with macular degeneration not glaucoma. (u) C. Impaired red reflex is common with cataract not glaucoma. (c) D. The pupil is moderately dilated and nonreactive to light in acute angle closure glaucoma.

A 63 year-old retired engineer presents with one month of difficulty sleeping. He has a hard time staying asleep and says he is just restless. He also states that he has been more forgetful and can't pay attention very well. He lives with his wife. He denies fever, chills, recent trauma, or difficulty walking. When asked about the specifics of his symptoms,he repeatedly replies, "I don't know," without really trying. He states his wife thinks he just sits around the house all day since he retired. His neurological exam is unremarkable, except for some mild psychomotor retardation. Which of the following is the most likely diagnosis? A. Dementia B. Delirium C. Depression D. Dissociative disorder

(u) A. Dementia is a deterioration of selective mental functions including progressive loss of impulse control and an attempt to cover up mental deficiencies. (u) B. Delirium has a rapid onset of symptoms, mental status fluctuations. Anxiety and irritability are common. (c) C. Depression often presents with difficulty thinking and concentrating, lessened sleep, and withdrawal from activities. (u) D. Dissociative disorder is associated with inability to recall important personal information usually of traumatic nature that is too extensive to be explained by ordinary forgetfulness.

A patient presents with edema, which is most noticeable in the hands and face. Laboratory findings include proteinuria, hypoalbuminemia, and hyperlipidemia. The most likely diagnosis is A. congestive heart failure. B. end-stage liver disease. C. nephrotic syndrome. D. malnutrition.

(u) A. Dependent edema is the most typical finding with CHF. Laboratory findings do not generally include proteinuria or hypoalbuminemia. (u) B. Symptoms of end-stage liver disease usually include increased abdominal girth indicating ascites. Hypoalbuminemia can occur as a result of malnutrition or concurrently with nephrotic syndrome. (c) C. Proteinuria, hyperlipidemia, and hypoalbuminemia are consistent with nephrotic syndrome. (u) D. Malnutrition is marked by physical wasting, not edema. Hypoalbuminemia may be seen, but hyperlipidemia is not typical.

Which of the following pathophysiological processes is associated with chronic bronchitis? A. Destruction of the lung parenchyma B. Mucous gland enlargement and goblet cell hyperplasia C. Smooth muscle hypertrophy in the large airways D. Increased mucus adhesion secondary to reduction in the salt and water content of the mucus

(u) A. Destruction of the gas-exchanging structures in the lung is characteristic of emphysema. (c) B. Chronic bronchitis results from the enlargement of mucous glands and goblet cell hypertrophy in the large airways. (u) C. There may be smooth muscle hypertrophy in chronic bronchitis but it is not to the extent as found in asthma and is not an underlying factor in the pathology of chronic bronchitis. (u) D. Abnormal absorption of sodium and a reduced rate of chloride secretion in cystic fibrosis leads to thickening of the mucus and increase in adhesion of the mucus.

Which of the following laboratory assays demonstrates excessive amounts of catecholamine degradation products in the urine? A. dexamethasone suppression B. metanephrine and normetanephrine C. creatinine clearance D. Schilling

(u) A. Dexamethasone Suppression Test is utilized to assess for suppression of ACTH. This test is often utilized to assess for conditions of hypersecretion of glucocorticoids from the adrenal gland. (c) B. Metanephrine and normetanephrine are the degradation products that result from the metabolism of epinephrine and norepinephrine. (u) C. Creatinine Clearance is done to assess glomerular filtration rate. (u) D. The Schilling Test is done to assess for pernicious anemia specifically as it relates to the absorption of Vitamin B12.

A 72 year-old patient with type 2 diabetes treated with a sulfonylurea is brought into the clinic by her daughter, who provides the patient's history. The patient has had a recent upper respiratory infection and excessive diuresis for the past 2 days. She has had decreased oral intake. At the clinic, she becomes increasingly stuporous and lethargic. Which of the following is the most likely diagnosis in this patient? A. Diabetic ketoacidosis B. Hyperglycemic hyperosmolar state C. Hypoglycemic coma secondary to her refusal to eat D. Lactic acidosis secondary to an infectious process

(u) A. Diabetic ketoacidosis is usually preceded by a day or more of polyuria and polydipsia in association with nausea and vomiting in someone receiving insulin. (c) B. Hyperglycemic hyperosmolar state is most common in an elderly patient with mild diabetes. Lethargy and confusion develop as osmolality rises to 300 mosm/kg or higher. (u) C. Although hypoglycemia induced by oral glucose lowering agents is less common, it tends to occur in elderly patients with impaired renal function and is generally associated with longer acting sulfonylureas. (u) D. Lactic acidosis secondary to an infectious process is typically associated with a severe infectious process, which is not definite in this patient.

: (7) 85 218. Diagnostic Studies/Dermatology An 81 year-old long term care facility resident presents to your office with complaint of generalized itching over the last several days. She denies use of any new products or ingestions of new foods or medications. Examination is significant for generalized excoriations and burrows in the web spaces of the hands and feet. Which of the following tests will confirm your diagnosis of this patient? A. No diagnostic testing is needed. The patient can be treated empirically B. Dermascopy C. Skin scrapings D. Shave biopsy

(u) A. Diagnosis of scabies is on occasion treated empirically - typically in residents of long term care facilities. Such residents can also be treated with oral ivermectin, aimed at mass treatment, however skin scrapings are indicated to confirm the diagnosis. (u) B. Scabies cannot be seen or confirmed with dermascopy alone. (c) C. Skin scraping is the diagnostic tool for suspected scabies. A 15 blade is used to scrape multiple, unexcoriated, lesions and mounted on a slide for microscopy. Diagnosis is confirmed by the presence of the organism, ova or feces. (u) D. Shave biopsy is not indicated for scabies. The dermatologic findings of scabies are superficial and shave biopsy sample would be too deep.

A 14-year old male active in sports, has been complaining of intermittent anterior right knee pain for several months.He denies any specific injuries. On examination, there is no erythema, swelling, deformities, joint laxity, or crepitus. Palpation reveals tenderness over the tibial tubercle and bursa of the right knee. This finding is characteristic of which of the following disorders? A. Discoid meniscus B. Osgood-Schlatter disease C. Chondromalacia patella D. Osteochondritis dissecans

(u) A. Discoid meniscus causes clicking over the lateral meniscus during flexion. This disorder is mostly painless and occasionally may cause mild aching or effusion. (c) B. Osgood-Schlatter disease causes pain at the tibial tubercle and it is caused by fragmentation of the tip of the proximal tibial physis. (u) C. Chondromalacia patella may demonstrate exaggerated knee valgus and subluxation. (u) D. Osteochondritis dissecans in older children may have effusions, pain, and locking of the joint caused by a portion of the joint surface softening and a shearing leading to a loose fragment.

An 8 year-old child is brought in by his mother with a two day history of spreading, non-pruritic red rash. The rash was preceded by moderate fever, sore throat and rhinorrhea. Examination reveals a moderately ill appearing child with a fine, macular-papular rash on an erythematous base spread diffusely over the trunk with some accentuation in the skin folds. The face is flush with perioral pallor. There is palpable anterior cervical lymphadenopathy. Which of the following is the most appropriate diagnostic study to establish the diagnosis? A. Shave biopsy B. Epstein-Barr virus Ig G C. Complete blood count D. Throat culture

(u) A. Disorders such as erythema multiforme (EM) minor favors the extremities while the EM major may favor the trunk and have associated oral mucosa involvement but generally has blistering lesions. Direct immunofluorescence studies are negative in these disorders. (u) B. Epstein-Barr virus Ig G would not be helpful in that it screens for prior exposure to this virus or illness such as mononucleosis. (u) C. Though a complete blood count is commonly ordered and may be helpful in stratifying illness in this case it would not be diagnostic as to the etiology. (c) D. Scarlatina rash is due to infection with group A strep. A throat culture would be the most appropriate diagnostic study to establish the diagnosis in this patient.

: (8) 107. Diagnosis/Psychiatry/Behavioral Medicine A 21 year-old female breaks up with her fiancé 1 week prior to her wedding. She is reported missing the next day. She is found in another city far from her home and does not recall how she got there. Which of the following is the most likely diagnosis? A. Dissociative amnesia B. Dissociative identity disorder C. Dissociative fugue D. Depersonalization disorder

(u) A. Dissociative amnesia is an abrupt loss of memory of one or more personal experiences. These experiences are often physically or emotionally traumatic events. (u) B. With dissociative identity disorder the patient has two or more distinct personality states with at least two of these identities recurrently taking control of the person's behavior. (c) C. Dissociative fugue is an abrupt loss of memory regarding personality identify, usually occurring after or during an emotional conflict or trauma. Patients often wander far from home. (u) D. The essential feature of depersonalization disorder is the persistent or recurrent feeling of detachment or estrangement from one's own self.

A 36 year-old male developed a sore throat and was treated with IM penicillin. Within 20 minutes, he felt faint and became dyspneic. Upon entry to the emergency department, he was pale and apprehensive. He had a thready pulse, and systolic blood pressure was 40 mmHg. Which of the following is the most appropriate initial agent to use? A. dopamine B. epinephrine C. hydrocortisone D. diphenhydramine

(u) A. Dopamine is not indicated in the treatment of allergic reactions. (c) B. Epinephrine is the drug of first choice for emergency use and should be given as soon as anaphylactic shock is suspected or diagnosed. (u) C. Hydrocortisone should be given as an adjunct to epinephrine, but it is not the drug of first choice. (u) D. Diphenhydramine should be given as an adjunct to treatment, but it is not the drug of first choice.

A 10 year-old female experiences fever and polyarthralgia. On examination you note a new early diastolic murmur. Laboratory results are positive for antistreptolysin O. The patient has no known drug allergies. Which of the following is the recommended prophylaxis for this condition? A. Doxycycline B. Erythromycin C. Benzathine penicillin G D. Trimethoprim/sulfamethoxazole

(u) A. Doxycycline and Bactrim are not indicated for the prophylaxis of recurrent rheumatic fever. (u) B. Erythromycin is considered second line for prophylaxis of recurrent rheumatic fever in a patient with a penicillin allergy. (c) C. Recurrences of rheumatic fever are most common in patients who have had carditis during their initial episode and in children. The preferred method of prophylaxis is Benzathine penicillin G every four weeks. (u) D. See A for explanation.

Which of the following is the leading cause of injury-related death in children between the ages of 1 and 15? A. Drowning B. Firearms C. House fires D. Motor vehicle injuries

(u) A. Drowning is a frequent, but not the leading, cause of death in early childhood. (u) B. Childhood death due to firearm related injuries is becoming more common, but it is not the leading cause. (u) C. House fires are a common, but not the leading, cause of death in early childhood. (c) D. Motor vehicle injuries are the leading cause of death in children.

A 45 year-old male with Type 1 diabetes presents with the following lipid panel: Total cholesterol 321 mg/dL; Triglycerides 225 mg/dL; HDL 30 mg/dL; LDL 155 mg/dL. The treatment of choice for this patient is A. Nicotinic acid (Niacin). B. Cholestyramine (Questran). C. Gemfibrozil (Lopid). D. Simvastatin (Zocor).

(u) A. Niacin is not indicated in patients with diabetes as it may worsen blood sugar control. (u) B. Cholestyramine is not indicated as it may worsen the triglyceride level in this patient. (u) C. Gemfibrozil should be avoided as it may worsen the LDL level. (c) D. Simvastatin is the drug of choice as it will decreases triglyceride level, decrease LDL, and increase HDL.

: (8) 149. Clinical Intervention/Dermatology A 37 year-old obese, otherwise healthy female presents with complaints of an enlarging painful, red mass on her left medial thigh. She had tried over-the-counter hydrocortisone cream and triple antibiotic ointment without improvement. Examination reveals a 4 cm, warm, tender, fluctuant mass with surrounding erythema and no inguinal lymphadenopathy. Which of the following is the most appropriate clinical intervention for this patient? A. Warm compresses and topical antibiotic therapy B. Incision and drainage, wound culture and oral antibiotic therapy C. Incision and drainage only D. Systemic antibiotic therapy only

(u) A. Early treatment of a furuncle may have responded to warm compresses, however, this patient has a carbuncle with early cellulitis and will need oral antibiotic treatment and incision and drainage. (c) B. The patient has a carbuncle with early cellulitis. The mainstay of treatment is incision and drainage with wound culture followed by administration of oral antibiotics, primarily for the early cellulitis. (u) C. Incision and drainage might be acceptable for an isolated furuncle or carbuncle but this patient also has early cellulitis, which requires wound culture and treatment with systemic antibiotics. (u) D. Systemic antibiotic therapy may be helpful for the early cellulitis but will likely not penetrate the carbuncle and irradicate the infection. Wound cultures are also needed to determine sensitivity and specificity and to monitor response of the therapy.

Which of the following would provide the most specific information regarding the functional cardiac status in a patient with chronic heart failure? A. Electrocardiogram B. Chest x-ray C. Serum electrolytes D. Echocardiogram

(u) A. Electrocardiogram offers no specific information of functional status, but may provide clues about the cause. (u) B. A chest x-ray may show findings of chronic heart failure, such as cardiomegaly or pulmonary congestion, but does not reflect cardiac functional status. (u) C. Serum electrolytes may be abnormal, either as a result of heart failure, or as a contributing factor, but they do not indicate functional status. (c) D. Echocardiogram will estimate ejection fraction, which is an indicator of left ventricular function.

Spina bifida occulta is usually detected by which initial diagnostic evaluation? A. Electroencephalogram B. Alpha-fetoprotein levels C. Folic acid levels D. X-ray of the spine

(u) A. Electroencephalogram (EEG) cannot detect or diagnose spina bifida occulta. (u) B. Alpha-fetoprotein, measured at 16-18 weeks of pregnancy, if elevated, would indicate a neural tube defect, such as spina bifida. (u) C. Folic acid has been shown to decrease the incidence of neural tube defects, not as an aid in diagnosis of the defect. (c) D. X-ray or MRI is the definitive test to diagnose spina bifida occulta, showing the vertebral bony defect.

A patient presents with the complaint of irritation of the left eye one day after gardening. He states "I think there is something in my eye." Which of the following findings is consistent with your suspected diagnosis? A. increased intraocular pressure B. rust ring C. hazy cornea D. fluorescein uptake

(u) A. Elevated intraocular pressure is seen with glaucoma. (u) B. Rust ring is seen with metallic foreign bodies. (u) C. Hazy cornea is seen with glaucoma. (c) D. Fluorescein dye uptake is diagnostic for corneal abrasion.

A 65 year-old male with coronary artery disease, hypertension, and diabetes mellitus is admitted with dyspnea and lower extremity edema. The chest x-ray reveals small bilateral pleural effusions. Echocardiogram shows an ejection fraction of 30% with no valvular heart disease. The patient is treated in the hospital with furosemide (Lasix) and lisinopril (Zestril). What education should be given to this patient upon discharge to help prevent readmission? A. Elevate the head of bed at home B. Avoid physical activity C. Monitor daily weights D. Restrict fluid intake

(u) A. Elevation of the head of the bed may help the patient if they have symptoms of dyspnea but it will not help prevent readmission to the hospital with a CHF exacerbation. (u) B. In stable patients increasing physical activity or regular exercise can diminish symptoms. (c) C. Strategies to prevent rehospitalization can include monitoring daily weights, case management and patient education regarding self-adjustment of diuretics. (u) D. Fluid restriction is not helpful in the readmission for CHF.

The finding of egophony is most consistent with A. emphysema. B. atelectasis. C. pneumothorax. D. lobar pneumonia.

(u) A. Emphysema presents with diminished or absent breath sounds and hyperresonance to percussion without egophony. (u) B. Atelectasis most commonly has decreased breath sounds and dullness to percussion without egophony. (u) C. Pneumothorax presents with absent breath sounds, tactile fremitus, and resonance to percussion without egophony. (c) D. Egophony occurs with consolidation caused by lobar pneumonia.

What should be the initial evaluation of the etiology of infertility in a 25 year-old female who has been trying to conceive for 1 year? A. Endometrial biopsy B. Hysterosalpingogram C. Serum progesterone levels D. Basal body temperature measurements

(u) A. Endometrial biopsy will identify changes associated with infertility, however it is invasive and often done later in the evaluation. (u) B. Hysterosalpingogram provides information about the internal female genital tract. This is usually done later in the work-up. (u) C. Serum progesterone levels drawn at mid-luteal phase will help determine normal from abnormal cycles. Levels above 15 ng/mL will indicate a normal cycle. (c) D. Basal body temperature measurement is an excellent screening test for ovulation, and will help with the timing of coitus.

A 48 year-old G3P3003 female presents to the office complaining of severe secondary dysmenorrhea and menorrhagia over the last 6 months. On physical examination her uterus feels enlarged and irregular. Which of the following is the most likely diagnosis? A. Endometritis B. Endometriosis C. Uterine leiomyomata D. Endometrial hyperplasia

(u) A. Endometritis is an infection of the endometrium. It occurs most commonly postpartum. (u) B. Endometriosis most commonly presents with dysmenorrhea, dyschezia, and dyspareunia. Usually the uterus is not enlarged. (c) C. Uterine leiomyomata (fibroid) typically presents with severe dysmenorrhea and menorrhagia. An enlarged,irregular uterus is noted on examination. (u) D. Endometrial hyperplasia can cause menorrhagia but is not usually a cause of dysmenorrhea.

What is the treatment of magnesium sulfate toxicity? A. Nifedipine B. Terbutaline C. Potassium carbonate D. Calcium gluconate

(u) A. Nifedipine, a calcium-channel blocker is used to treat both preterm labor and hypertension in pregnancy. It works by inhibiting calcium transport through slow-type channels, causing reduction in systemic and pulmonary vascular resistance and tocolysis. (u) B. Terbutaline is a beta-blocker that is used to treat pre-term labor. (u) C. Potassium carbonate is a treatment for metabolic acidosis, not magnesium sulfate toxicity. (c) D. 10% calcium gluconate is used to treat magnesium sulfate toxicity.

A 34 year-old female presents with recurrent bouts of dizziness, tinnitus, and hearing loss. She states that the episodes are incapacitating and cause her to become nauseous and vomit. The attacks last about one hour and the symptoms disappear after a few days. The last two episodes were treated with meclizine (Antivert) and prochlorperazine (Compazine) at the emergency room. Audiologic testing reveals low-tone frequency hearing loss. Which of the following is the most appropriate long-term management for this patient? A. Epley maneuver B. Diuretics and low-sodium diet C. Broad-spectrum antibiotics and Ibuprofen D. Scopolamine transdermal patch

(u) A. Epley maneuver is used to treat benign paroxysmal positional vertigo. (c) B. Diuretics and a low sodium diet will decreases the endolymphatic pressure in the semicircular canals, which is believed to be elevated in Meniere's disease, and help relieve symptoms. (u) C. Broad-spectrum antibiotics and ibuprofen are used to treat otitis media, not Meniere's disease. (u) D. A scopolamine patch is useful for treatment of a single episode, but not long-term management.

A 19 year-old college student has a severe sore throat and a temperature of 102° F. On examination, there are vesicles on the soft palate with some erythema. The tonsils are not enlarged. There is mild cervical lymph node enlargement. Which of the following is the most likely diagnosis? A. Epstein-Barr virus B. Coxsackie virus group A C. Streptococcus pyogenes D. Mycoplasma pneumoniae

(u) A. Epstein-Barr is accompanied by generalized lymphadenopathy. Exudative pharyngitis and tonsillitis may also occur. (c) B. Coxsackie virus group A is characterized early by petechiae or papules/vesicles on the soft palate that become shallow ulcers in about 3 days. (u) C. Streptococcus pyogenes produces an exudative pharyngitis. (u) D. Mycoplasma pneumoniae is characterized by conjunctivitis, otitis media, and atypical pneumonia.

Small grayish vesicles and punched-out ulcers in the posterior pharynx in a child with pharyngitis is representative of which organism? A. Epstein-Barr virus B. Group C Streptococcus C. Coxsackievirus D. Gonorrhea

(u) A. Epstein-Barr virus presents with enlarged tonsils with exudates and petechiae of the palate. (u) B. Group C Streptococcus presents with a red pharynx and enlarged tonsils with a yellow, blood tinged exudates (c) C. Coxsackievirus presents with small grayish vesicles and punched-out ulcers in the posterior pharynx. (u) D. Neisseria gonorrhea of the pharynx may be asymptomatic

: (8) 71. Clinical Therapeutics/Cardiology An ambulance transports a patient to your facility from the nursing home with anuria and diminished level of consciousness. Paramedics report a blood pressure of 225/130mm Hg. Which of the following medications is most appropriate for this patient? A. Esmolol (Brevibloc) B. Nitroprusside sodium (Nipride) C. Nitroglycerine (Nitrostat) D. Nifedipine (Procardia) 32

(u) A. Esmolol is approved only for treatment of supraventricular techycardia. When used for hypertension it should be reserved for patients in whom there is particular concern about serious adverse events related to beta blockers. (c) B. Parenteral therapy is indicated in most hypertensive emergencies, especially if encephalopathy is present. Nitroprusside lowers blood pressure in seconds by direct arteriolar and venous dilation. (h) C. Excessive reductions in pressure may precipitate coronary, cerebral, or renal ischemia. As such, the use of sublingual or oral fast-acting nitroglycerine preparations should be avoided. (h) D. Excessive reductions in pressure may precipitate coronary, cerebral, or renal ischemia. As such, the use of sublingual or oral fast-acting nifedipine preparations should be avoided.

Which of the following diagnostic studies is indicated in the evaluation of an upper gastrointestinal bleed? A. Esophageal manometry B. Bleeding scan C. Upper endoscopy D. Barium swallow

(u) A. Esophageal manometry is not indicated in the evaluation of upper gastrointestinal bleeding. (u) B. Bleeding scans are most useful to evaluate occult GI bleeding. Radioactive isotope tracing is useful to concentrate bleeding sites at a single location. It is most useful to find bleeding that is occurring in the large or small bowel rather than in the upper GI sites. (c) C. Endoscopy is the evaluation modality of choice in patients with upper GI bleeding. The advantage of this technique is that it can be used for both diagnostic and therapeutic purposes. (u) D. Barium swallow is not indicated in the evaluation of upper gastrointestinal bleeding.

: (28) 209. Health Maintenance/Orthopedics/Rheumatology Which of the following is most appropriate to delay the progression of osteoporosis? A. Estrogen B. Bisphosphonate C. Glucocorticoid D. Parathyroid hormone

(u) A. Estrogen loss is associated with loss in bone mineral density (BMD). Estrogen supplementation slows bone 82 resorption and was previously considered the treatment of choice for postmenopausal osteoporosis. (c) B. Bisphosphonate therapy is considered the current first line oral therapy for treatment. These medications reduce vertebral, hip, and nonvertebral fractures by nearly 50%, especially in the first year of treatment. (h) C. Glucocorticoid therapy, especially in high doses and long duration, causes bone loss and fractures. (u) D. Parathyroid hormone, marketed as teriparatide, is considered an anabolic agent, stimulating bone formation, unlike bisphosphonates which block bone resorption. Although effective, teriparatide was associated with osteosarcoma in rats. There are no reports of osteosarcoma or primary and secondary hyperparathyroidism in humans. The current recommendation for teriparatide is for individuals with moderate to severe osteoporosis for up to two years.

: (27) 60 150. Diagnostic Studies/Cardiology A 35 year-old male status-post patent ductus arteriosus repair at 5 years of age, presents with low-grade fever, fatigue and dyspnea worsening over the past 10 days. Prior to the onset of these symptoms, he was healthy and free of any complaints. Examination is significant for petechiae on the palate, a high pitched holo-systolic murmur heard best at the apex, and splinter hemorrhages on both hands under his fingernails. Which of the following is the most appropriate next step in the evaluation of this patient? A. Cardiac catheterization B. Echocardiogram C. MUGA scan D. Chest radiograph

(u) A. Evidence of endocardial involvement documented by echocardiography is included in the modified Duke criteria for the diagnosis of infectious endocarditis. (c) B. The history is most representative of a patient with infectious endocarditis. The clinical criteria, referred to as the modified Duke criteria include (1) two positive blood cultures for a microorganism that typically causes infective endocarditis or persistent bacteremia (2) evidence of endocardial involvement documented by echocardiography or (3) development of a new regurgitant murmur. Diagnosis is made if one major and one minor criterion or three minor criteria are met. (u) C. Evidence of endocardial involvement documented by echocardiography is included in the modified Duke criteria for the diagnosis of infectious endocarditis. (u) D. Evidence of endocardial involvement documented by echocardiography is included in the modified Duke criteria for the diagnosis of infectious endocarditis.

A 22 year-old male presents to the emergency department complaining of right hand pain after punching a brick wall. His pain is noted at the ulnar aspect of his hand and worsens with touch and movement. On examination, you notice obvious swelling and tenderness over the dorsum of the right hand proximal to the metacarpal phalangeal (MCP) joint of the fifth digit. Skin is intact. X-ray reveals a fracture of the proximal fifth metacarpal with good alignment. Which of the following is the treatment of choice? A. Immediate orthopedic consult for surgical fixation B. Long finger splint extending beyond the MCP C. Ulnar gutter splint to immobilize fracture site D. Referral for casting

(u) A. Fixation is warranted for displaced fractures, angulated fractures beyond 40 degrees, or extensor lag. (u) B. A long finger splint would not immobilize the joint above and below and may not offer enough immobilization. (c) C. An ulnar gutter splint is the most appropriate care in the emergency room. (u) D. Although you will eventually refer the patient for casting, the choice treatment initiated in the emergency room should be immobilization.

: (11) 204. History & Physical/ENT/Ophthalmology Which of the following is a characteristic finding on examination of a patient who presents with an uncomplicated corneal foreign body? A. Fixed pupil B. Decreased visual acuity C. Hyphema D. Ciliary injection

(u) A. Fixed pupil is not associated with minor ocular trauma. (u) B. Uncomplicated corneal foreign body does not characteristically result in visual acuity change. (u) C. Hyphema, the presence of blood in the anterior chamber, is characteristic of penetrating injury or spontaneous rupture of iris root vessel. (c) D. Ciliary injection, engorgement of the vessels about the limbus, is a characteristic finding in patients with corneal foreign body.

69. Clinical Intervention/ENT/Ophthalmology A 21 year-old male presents to the ED after being struck over the left eye by a baseball. The patient reports excruciating pain, decreased vision and nausea. The examination is significant for enophthalmos and flattening of the anterior chamber. In addition to an ophthalmology referral, which of the following is most appropriate in the emergent management of this patient? A. Perform fluorescein staining B. Measure intraocular pressure C. Place rigid shield over eye D. Palpate the orbital rim

(u) A. Fluorescein evaluation would be unnecessary given the signs indicating globe rupture. (h) B. Measurement of intraocular pressure is contraindicated with suspected globe rupture. (c) C. The enophthalmos and flattened anterior chamber is highly suggestive of globe rupture. The eye should be shielded and the patient should be referred for an urgent ophthalmology consultation. (h) D. The examination should be limited with emphasis on protection of the globe and contents.

A 72 year-old patient with a history of hypertension and atrial fibrillation presents with episodes of weakness, numbness, and paresthesias in the right arm. At the same time, she notes speech difficulty and loss of vision in her left eye. These symptoms come on abruptly and clear within minutes. Physical examination is normal except for the previously known arrhythmia. Which of the following is the most likely diagnosis? A. Focal seizure B. Migraine headache C. Hypoglycemic episodes D. Transient ischemic attack

(u) A. Focal seizures usually cause abnormal motor movement rather than weakness or loss of feeling. (u) B. Patients with migraines commonly have a history of episodes since adolescence. (u) C. Hypoglycemic episodes do not present with focal neurological findings. (c) D. This patient's symptoms are consistent with transient ischemia in the carotid territory. Atrial fibrillation is a risk factor for cerebral emboli.

: (7) 137. Clinical Therapeutics/Cardiology A 66 year-old female with a history of diabetes and hypertension presents for routine evaluation. Since beginning her ACE inhibitor and diuretic therapy her blood pressures have averaged 138/85 mmHg. Which of the following is the next best step in the management of this patient? A. Congratulate her for being "at goal" with a blood pressure of less than 140/90 mmHg. B. Increase ACE inhibitor to achieve blood pressure of less than 130/80 mmHg. C. Convert ACE inhibitor to angiotensin receptor blocker. D. Explain blood pressure is a little lower than needed so discontinue diuretic.

(u) A. For a patient with diabetes, she is not at goal. (c) B. JNC VII guidelines set target blood pressure for patients with diabetes or chronic kidney disease at less than 130/80. (u) C. There is no indication to change medication before trying to increase her current dose. (u) D. The blood pressure is not at goal. The medications should be increased rather than discontinued.

During a baseball game, a 22 year-old college student is hit in the right eye by a baseball. He complains of blurry vision in that eye. On physical exam, the physician assistant notes proptosis of the right eye, and limitation of movement in all directions. On CT scan, which of the following is most likely to be seen? A. Fracture of the medial orbital wall B. Prolapse of orbital soft tissue C. Hematoma of the orbit D. Orbital emphysema

(u) A. Fracture of the medial orbital wall is associated with diplopia from medial rectus impingement, orbital emphysema and epistaxis. (u) B. Prolapse of orbital soft tissue, including inferior rectus muscle, inferior oblique muscle, orbital fat, and connective tissue results in enophthalmos, ptosis, diplopia, anesthesia of the ipsilateral cheek and upper lip, and limitation of upward gaze and is seen with fractures of the orbital floor. (c) C. Orbital hemorrhage into the space surrounding the globe following blunt trauma and rupture of the orbital vessels results in increased ocular pressure, proptosis, visual loss, and limitation of movement in all directions. CT reveals a hematoma. (u) D. Orbital emphysema is seen with fractures of the medial orbital wall or floor of the orbit into the maxillary and ethmoid sinuses respectively. It will not lead to proptosis.

A male is found to have an eunuchoid body habitus, gynecomastia, diminished secondary sex characteristics, and small and firm testicles. Which of the following is the most likely diagnosis? A. Fragile X syndrome B. Gaucher disease C. Klinefelter syndrome D. Kallmann syndrome

(u) A. Fragile X syndrome is the second most common inherited form of mental retardation. Affected males show macroorchidism (enlarged testis) after puberty, large ears, and a prominent jaw, high-pitched voice, and mental retardation. (u) B. Patients with Gaucher disease have anemia and thrombocytopenia as its major manifestation. These patients may also have cortical erosions of bone and may have bone pain due to local infarctions. (c) C. Men with Klinefelter syndrome have an extra X chromosome and are phenotypically normal until puberty. After puberty, these men have long legs and arms, a female escutcheon, gynecomastia, and small testes. Low serum testosterone and infertility along with a decrease in executive functioning are common in these men. (u) D. Kallmann's syndrome is the most common cause of congenital isolated gonadotropin deficiency. It is associated with anosmia and hyposmia caused by hypoplasia of the olfactory bulbs. About one-half of these patients will have renal agenesis. There is also an increase in cryptorchism, sensorineural hearing deafness, and cleft lip.

A positive osmotic fragility test is seen in which of the following? A. G6PD deficiency B. Sickle cell anemia C. Hereditiary spherocytosis D. Autoimmune hemolytic anemia

(u) A. G6PD deficiency may result in episodic hemolysis in response to oxidant drugs or infections and is not associated with a positive osmotic fragility test. (u) B. Sickle cell anemia is an abnormal hemoglobinopathy leading to chronic hemolytic anemia and is confirmed with hemoglobin electrophoresis. An osmotic fragility test would be negative. (c) C. Hereditary spherocytosis will have a positive osmotic fragility test secondary to a defective red blood cell membrane. (u) D. Coombs' testing forms the basis for the diagnosis of autoimmune hemolytic disorders.

Which of the following conditions is caused by a disorder in the red blood cell membrane? A. G6PD deficiency B. Folate deficiency C. Sickle cell anemia D. Hereditary spherocytosis

(u) A. G6PD deficiency results in an increased destruction of red cells that leads to anemia. (u) B. Folate is essential in DNA synthesis, lack of folate leads to ineffective erythropoiesis. (u) C. Sickle cell anemia is a hemoglobinopathy which leads to increased red cell destruction. (c) D. The cell membrane defect of spherocytosis leads to hemolysis due to trapping of the cells by the spleen.

A 3 week-old male infant presents with recurrent regurgitation after feeding that has progressed to projectile vomiting in the last few days. The mother states that the child appears hungry all of the time. She denies any diarrhea in the child. Which of the following clinical findings is most likely? A. Bile-stained vomitus B. Hemoccult positive stools C. Olive-sized mass in the right upper abdomen D. Sausage-shaped mass in the upper-mid abdomen

(u) A. Gastric obstruction, such as that seen with pyloric stenosis, causes vomiting that is not bilious. (u) B. Blood-streaked vomitus, but not hemoccult positive stools, may be seen in pyloric stenosis. (c) C. An olive-sized mass may be palpated in the right upper abdomen in pyloric stenosis and if found, is pathognomonic for pyloric stenosis. (u) D. A sausage-shaped mass may be noted in intussusception, not pyloric stenosis.

An 18 year-old patient is diagnosed with bacterial conjunctivitis. Gram stain reveals gram-negative intracellular diplococci. Which of the following is the most appropriate treatment of this infection? A. Gentamicin B. Penicillin C. Bactrim D. Ceftriaxone

(u) A. Gentamicin is used in the treatment of gram negative rods and does not provide coverage of Neisseria gonorrhoeae. (u) B. Penicillin does provide coverage for some Neisseria species but is not considered the treatment of choice for Neisseria gonorrhoeae because of increased resistance. (u) C. Bactrim is used to treat Chlamydia trachoma not Neisseria gonorrhoeae. (c) D. Neisseria gonorrhoeae is confirmed by the Gram stain findings of intracellular gram-negative diplococci. The treatment of choice for Neisseria gonorrhoeae is ceftriaxone.

A 14 year-old male who is overweight presents with complaints of left knee and anteromedial thigh pain for the past month. He states the pain gets better with rest and denies any known trauma. On examination of the gait, a slight limp is noted. X-ray films of the left knee are normal. The most likely diagnosis is A. genu valgum. B. Legg-Calve-Perthes disease. C. Osgood-Schlatter disease. D. slipped capital femoral epiphysis.

(u) A. Genu valgum is a knock-knee deformity of the knees and would be detected on physical examination. (u) B. Legg-Calve-Perthes presents in a younger population. (u) C. Osgood-Schlatter disease is characterized by local pain, swelling, and tenderness to palpation overlying the tibial tubercle, and x-ray findings of tibial tubercle prominence, with or without free bony fragments. (c) D. Slipped capital femoral epiphysis is most common in overweight adolescent males who present with complaints of pain that is referred to the thigh or medial side of the knee associated with a limp. X-ray films of the knee are normal since the condition involves the hip.

A patient with type 2 diabetes mellitus presents for a yearly eye exam. Ophthalmoscopic exam reveals neovascularization. Which of the following is the most likely complication related to this finding? A. Glaucoma B. Cataracts C. Vitreous hemorrhage D. Optic neuritis

(u) A. Glaucoma occurs in about 6% of diabetics. Neovascularization of the iris can cause closed angle glaucoma. (u) B. Cataracts can occur secondary to diabetes, but are not caused by proliferative retinopathy. (c) C. Proliferative retinopathy, as evidenced by neovascularization, is associated with an increased risk of vitreous hemorrhage. (u) D. Optic neuritis is strongly associated with demyelinating disease, like multiple sclerosis.

A 63 year-old male with history of hypertension and tobacco abuse presents complaining of dyspnea on exertion for two weeks. The patient admits to one episode of chest discomfort while shoveling snow which was relieved after five minutes of rest. Vital signs are BP 130/70, HR 68, RR 14. Heart exam reveals regular rate and rhythm, normal S1 and S2, no murmur, gallop, or rub. Lungs are clear to auscultation bilaterally. There is no edema noted. Which of the following is the most appropriate initial diagnostic study for this patient? A. Helical CT scan B. Chest x-ray C. Nuclear stress test D. Cardiac catheterization

(u) A. Helical CT scan aids in the diagnosis of pulmonary embolism, not in the evaluation of angina. (u) B. Chest x-ray is not used as a diagnostic study to evaluate symptoms of angina or coronary heart disease. (c) C. In patients with classic symptoms of angina, nuclear stress testing is the most widely used test for diagnosis of ischemic heart disease. (u) D. Coronary angiography is indicated in patients with classic stable angina who are severely symptomatic despite medical therapy and are being considered for percutaneous intervention (PCI), patients with troublesome symptoms that are difficult to diagnose, angina symptoms in a patient who has survived sudden cardiac death event, patients with ischemia on noninvasive testings.

Which of the following is a systemic manifestation of infective endocarditis? A. Hemarthrosis B. Petechiae C. Cafe au lait spots D. Bronzing of the skin

(u) A. Hemarthrosis is most commonly a consequence of a clotting disorder such as hemophilia. (c) B. Petechiae, splinter hemorrhages, Janeway lesions, and Osler's nodes are systemic manifestations of patients who have infective endocarditis. (u) C. Cafe au lait spots are seen in Neurofibromatosis (von Recklinghausen's syndrome). (u) D. Bronzing of the skin is most commonly associated with hemochromatosis or Addison's disease.

Which of the following diagnostic findings in the urinary sediment is specific for a diagnosis of chronic renal failure? A. Hematuria B. Proteinuria C. Broad waxy casts D. Hyaline casts

(u) A. Hematuria and proteinuria are frequent, but nonspecific, findings in chronic renal failure. (u) B. See A for explanation. (c) C. Broad waxy casts in urinary sediment are a specific finding in chronic renal failure. (u) D. Hyaline casts may be found in normal urine or in states of dehydration.

An 18 year-old woman presents to the clinic complaining of fatigue. She reports a past history of lifelong frequent nosebleeds and bleeding gums. She also has menorrhagia. Her mother and maternal grandfather have a similar bleeding history. Initial lab results are as follows: WBC 9,500/mm3, Hgb 10.9 g/dL, HCT 33%, MCV 69 fL, MCHC 26 pg and platelets 284,000/mm3. Which of the following tests should be ordered to evaluate this patient's diagnosis? A. Hemoglobin electrophoresis B. Bleeding time and platelet aggregometry C. Bone marrow aspiration D. PT and aPTT

(u) A. Hemoglobin electrophoresis would be utilized to evaluate microcytic, hypochromic anemias. (c) B. The patient's presentation is consistent with a congential qualitative platelet disorder, most likely von Willebrand's Disease, necessitating a bleeding time and evaluation of platelet function. (u) C. Bone marrow aspiration is not utilized in the evaluation of qualitative platelet disorders. (u) D. A PT and aPTT would be utilized to evaluate for bleeding consistent with abnormalities with the coagulation cascade.

Which of the following complications of Crohn's disease most frequently requires surgery? A. Hemorrhage B. Intra-abdominal abscess C. Malabsorption D. Perforation

(u) A. Hemorrhage and perforation are uncommon in Crohn's disease. (c) B. Surgery is the treatment of choice for an abscess, which is a common complication of Crohn's disease. (u) C. Malabsorption is often a complication of the surgery itself. (u) D. See A for explanation.

A pregnant female presents at 32 weeks gestation with painless rectal bleeding and a bulging perianal mass when straining which goes away. Which of the following is the most appropriate management of this patient? A. Hemorrhoidectomy B. Metronidazole (Flagyl) C. Psyllium (Metamucil) D. Sclerotherapy

(u) A. Hemorrhoidectomy and sclerotherapy are reserved for severe Grade III and IV hemorrhoids. (u) B. Metronidazole is not indicated in the treatment of hemorrhoids. (c) C. Dietary fiber or psyllium bulk laxatives can be used to decrease straining with defecation. (u) D. See A for explanation.

A patient presents complaining of vague anal discomfort. On examination, the patient is noted to have a few small external hemorrhoids and edema in the anal region. Which of the following is the most appropriate intervention? A. proctoscopy followed by a hemorrhoidectomy B. increased dietary fiber and sitz baths C. hemorrhoidal banding D. inject a sclerosing agent

(u) A. Hemorrhoidectomy should be used for permanently prolapsed internal hemorrhoids. (c) B. Most hemorrhoids respond well to conservative treatment such as fiber and sitz baths. (u) C. Banding and injection of sclerosing agents are used if mild prolapse, enlargement, or intermittent bleeding is present. (u) D. See C for explanation.

A 45 year-old female presents with complaint of lower extremity discomfort. The patient admits to dull aching of the left lower extremity. The discomfort is worse after standing for long periods of time. Examination reveals dilated, tortuous and elongated veins on the medial aspect of the left leg. Pedal pulses are +2/4 bilaterally. There are no skin changes or lower extremity edema noted. Which of the following is the most appropriate initial treatment of choice in this patient? A. heparin B. compression stockings C. furosemide (Lasix) D. thrombectomy

(u) A. Heparin is used in the treatment of deep vein thrombosis not varicose veins. (c) B. This patient has signs and symptoms of varicose veins. Initial treatment with compression stockings may prolong or avoid the need for surgery. (u) C. This patient has no signs of edema or venous insufficiency requiring diuretic therapy. (u) D. Thrombectomy is indicated in a patient with an arterial thrombus, this patient has intact pulses and no pallor.

Which of the following subtypes of viral hepatitis requires the presence of the hepatitis B virus for replication? A. hepatitis A B. hepatitis C C. hepatitis D D. hepatitis E

(u) A. Hepatitis A is transmitted almost exclusively by the fecal-oral route and does not require the presence of the hepatitis B virus for replication. (u) B. While hepatitis C may occur in IV drug users, it does not require the presence of the hepatitis B virus for replication. (c) C. In the United States, hepatitis D is most commonly seen in persons exposed frequently to blood or blood products, such as drug addicts. It requires the presence of hepatitis B virus for its replication and expression. (u) D. Hepatitis E resembles hepatitis A in its enteric mode of transmission and does not require the presence of the hepatitis B virus for replication.

A 55 year-old female presents with a lesion on her face that is painful, bright red, with distinct raised borders. She also is complaining of fever and chills. There is no evidence of any marks which would show a portal of entry. Which of the following is the most likely diagnosis? A. Impetigo B. Erythrasma C. Cellulitis D. Erysipelas

(u) A. Impetigo is a bacterial infection of the skin that is associated with honey-colored crusts. (u) B. Erythrasma is a bacterial infection of the skin that is red in appearance and fluoresces a coral red color under a Wood's lamp. Systemic symptoms are rarely seen and the borders tend to be less distinct. (u) C. Cellulitis has many features of erysipelas but it goes on to involve the subcutaneous tissues. Patients with cellulitis will not have raised borders and demarkation from uninvolved skin is not distinct with this condition. (c) D. Erysipelas is a distinct type of superficial cellulitis with redness, a distinct and raised border, and sharp demarkation from uninvolved skin. It is typically associated with systemic symptoms such as fever and chills. It is caused by group A strep most commonly.

: (10) 138. Clinical Therapeutics/Dermatology An 8 year-old female presents with a 5 day history of a honey crusted rash beneath her nose. Which of the following is the best treatment for this condition? A. Topical triamcinolone (Kenalog) B. Oral fluconazole (Diflucan) C. Topical sertaconazole (Ertaczo) D. Oral cephalexin (Keflex)

(u) A. Impetigo is a bacterial infection that will not resolve with the application of topical corticosteroids. This may 56 exacerbate symptoms. (u) B. Impetigo is a bacterial infection which cannot be treated by an anti-fungal. (u) C. An antifungal is not indicated in the treatment of impetigo. (c) D. Impetigo is an infection of the skin, often caused by S. aureus or S. pyogens, characterized by honey crusted erosions. Treatment includes topical mupirocin followed by oral cephalosporin.

A 60 year-old patient presents with elevated blood pressure and peripheral edema. Laboratory testing reveals a BUN of 58 mg/dl and a creatinine of 4.5 mg/dl, these results are unchanged from six months ago. Urinalysis today is negative except for the following, specific gravity of 1.002, 2+ protein, and trace glucose. Which of the following laboratory findings would be most consistent for this patient? A. Hypercalcemia B. Metabolic alkalosis C. Hypokalemia D. Anemia

(u) A. In a patient with chronic renal failure, typical laboratory findings include hypocalcemia, metabolic acidosis, and hyperkalemia. (u) B. See A for explanation. (u) C. See A for explanation. (c) D. Anemia, due to low erythropoietin, is common in patients with chronic renal failure.

A 66 year-old male presents complaining of 6 month history of progressive blurred vision without associated pain. On examination there is no erythema or injection of the sclera. On funduscopic examination there is an absent red reflex and a cloudy lens. Which of the following is the most likely diagnosis? A. Retinal detachment B. Chronic glaucoma C. Age-related macular degeneration D. Cataract

(u) A. In retinal detachment the retina is seen hanging in the vitreous like a gray cloud. (u) B. In chronic glaucoma there will be slight cupping of the optic disc observed. (u) C. Findings in age-related macular degeneration include drusen, degenerative changes in retinal pigmentation, and subretinal neovascular membrane changes. (c) D. Cataracts present with blurred vision that progress over months to years. On examination the red reflex becomes increasingly difficult to visualize until it is finally absent and the pupil is white.

A 3 year-old presents with profuse watery diarrhea for the past three days. The child vomited twice yesterday, but not today. On examination, the child is febrile, with pulse of 142, respiratory rate of 18, and blood pressure of 60/40 mmHg. The child is alert and responsive, with no focal findings. Which of the following is the most appropriate intervention? A. Antibiotic therapy B. Loperamide (Imodium) C. 3% normal saline IV infusion D. Oral rehydration

(u) A. In the US, infectious gastroenteritis is most frequently due to a virus. Antibiotic therapy may be second-line in cases where the causative organism is bacterial, is identified, and symptoms continue. (u) B. Loperamide may lead to toxic megacolon. (h) C. 3% normal saline infusion may cause hypernatremia and central pontine myelinosis. (c) D. The goal of therapy for a child with severe gastroenteritis and dehydration is to restore fluid loss. Oral rehydration with an appropriate electrolyte solution is the best option if the child is not actively vomiting and is alert enough to take oral fluids.

A 43 year-old asymptomatic diabetic female is found to have an elevated total calcium level of 12.4 mg/dL. Which of the following tests must be assessed in order to evaluate this laboratory abnormality? A. Intact parathyroid hormone B. Serum albumin C. 24 hour urine calcium level D. Complete blood count

(u) A. Intact parathyroid hormone levels are only obtained for patients with true hypercalcemia with an unknown etiology. (c) B. Since approximately 50% of calcium is protein bound, total calcium levels should be interpreted relative to albumin levels. (u) C. Demonstration of excessive calcium in the urine does not provide any additional information regarding the increased serum calcium. (u) D. Complete blood count has no relationship to the serum calcium levels.

A 35 year-old G2P1001 female presents to the office at 11 weeks gestation with vaginal bleeding, mid-lower abdominal cramping, and bilateral lower pelvic discomfort. On examination, bright red blood is seen coming from the cervical os. The cervix is closed. The uterus is 9-11 weeks in size by palpation. Her blood pressure is 120/70 mmHg and her pulse rate is 96. What is the patient's most likely diagnosis? A. Inevitable abortion B. Incomplete abortion C. Threatened abortion D. Missed abortion

(u) A. Inevitable abortion is when the patient presents during the first 20 weeks of pregnancy with bleeding and crampy abdominal pain, also associated with a dilated cervix or a gush of fluid without the passage of the products of conception. (u) B. Incomplete abortion occurs when part of the products of conception are passed from the cervix, the cervix will also be dilated. (c) C. If bleeding occurs in the first 20 weeks of pregnancy and the cervix is closed, threatened abortion is the diagnosis. (u) D. A missed abortion occurs when a patient presents with smaller gestational size by examination than by dates and no fetal heart tones.

Which of the following is a proven risk factor for the development of abdominal aortic aneurysm? A. Infective endocarditis B. Diabetes mellitus C. Cigarette smoking D. Alcohol abuse

(u) A. Infective endocarditis is not associated with the development of abdominal aortic aneurysm. (u) B. Diabetic patients do have a higher rate of atherosclerosis, but there is no clear causal evidence of diabetics being at higher risk for the development of abdominal aortic aneurysm. (c) C. Cigarette smoking is the primary risk factor for the development of aortic aneurysms. (u) D. Alcohol abuse is not related to development of abdominal aortic aneurysm.

Which of the following is the most likely physical examination finding in a patient with Peyronie's disease? A. inflammation of the glans of the penis B. foreskin that cannot be retracted C. chancre on the shaft of the penis D. fibrous band on lateral portion of the penis

(u) A. Inflammation of the glans of the penis is noted in balanitis. (u) B. A foreskin that cannot be retracted is seen in phimosis. (u) C. A chancre on the shaft of the penis is noted in primary syphilis. (c) D. Peyronie disease typically presents with fibrotic areas under the penile skin along with a history of penile curvature during erection

A 72 year-old man presents with acute left lower quadrant abdominal pain. He has nausea, vomiting, and constipation. He has a fever of 101° F and guarding and rebound tenderness in his left lower quadrant. His white blood cell count is elevated. He has no prior history of gastrointestinal disease. Which of the following is the most likely diagnosis? A. Inflammatory bowel disease B. Irritable bowel syndrome C. Viral gastroenteritis D. Acute diverticulitis

(u) A. Inflammatory bowel disease typically presents in a younger population. (u) B. Irritable bowel syndrome is not associated with nausea, vomiting and fever. It usually presents in a younger population. (u) C. Viral gastroenteritis typically does not localize to the left lower quadrant. (c) D. Acute abdominal pain, fever, left lower abdominal tenderness, and leukocytosis are hallmark signs of acute diverticulitis.

A 67 year-old man presents complaining of gradually worsening fatigue and shortness of breath. He is a previous smoker with an 80 pack-year smoking history. He denies chest pain, night sweats, or hemoptysis. On physical examination, you note a very thin male who appears older than his stated age. Lung and heart sounds are barely audible to auscultation. Which of the following interventions is likely to alter the disease course? A. Inhaled bronchodilator therapy B. Inhaled steroid therapy C. Home oxygen D. Theophylline

(u) A. Inhaled bronchodilators afford symptomatic relief for some patients with COPD but do not alter the disease course. (u) B. Inhaled steroid therapy may reduce the number and severity of COPD exacerbations but has not been shown to alter the disease course. (c) C. Home oxygen therapy has been shown to prolong life in patients with COPD and alter the natural history of the disease. (u) D. Theophylline is a third-line agent for treating COPD and will not alter the natural history of the disease

An elderly patient with poorly-controlled Type 2 diabetes and renal disease develops a fever of 102°F orally, productive cough, and dyspnea. Physical examination demonstrates a respiratory rate of 32/min, labored breathing, and rales at the left base. Pulse oximetry is 90%. Which of the following is the next appropriate step in the management of this patient? A. Administer nebulized corticosteroids B. Admit to the hospital C. Oral antimicrobial therapy D. Endotracheal intubation

(u) A. Inhaled corticosteroids are not utilized in the management of community-acquired pneumonia. (c) B. Community acquired pneumonia is the most deadly infectious disease in the U.S. Important risk factors for increased morbidity and mortality include advanced age, alcoholism, comorbid medical conditions, altered mental status, respiratory rate greater than 30 breaths/min, hypotension, and a BUN greater than 30. (u) C. Due to the age of the patient, comorbid diseases, and current signs of respiratory distress, intravenous not oral antimicrobial therapy is indicated. (u) D. Endotracheal intubation is indicated for respiratory failure unresponsive to conservative management.

A 26 year-old monogamous female presents with cyclic pelvic pain that has been increasing over the last 6 months. She complains of significant dysmenorrhea and dyspareunia. She uses condoms for birth control. On physical examination her uterus is retroverted and non-mobile, and she has a palpable adnexal mass on the left side. Her serum pregnancy test is negative. Which of the following is the most likely diagnosis? A. Ovarian cancer B. Endometriosis C. Functional ovarian cyst D. Pelvic inflammatory disease

(u) A. It is important to consider ovarian cancer in a patient with a pelvic mass however, ovarian cancer usually occurs in older women over age 55 and patients are often asymptomatic until the disease is more advanced (c) B. With endometriosis, the uterus is often fixed and retroflexed in the pelvis. The palpable mass is an endometrioma or "chocolate cyst". The patient with endometriosis also often has dysmenorrhea, dyspareunia, and dyschezia. (u) C. Functional ovarian cysts occur from ovulation and usually are not symptomatic. (u) D. With PID the patient will have abdominal tenderness, adnexal tenderness, cervical motion tenderness and an elevated temperature.

A patient presents with a concern about an upcoming trip. He states he is traveling to Mexico and has a question about how to treat diarrhea should it develop. In addition to promoting fluid intake which of the following is recommended? A. kaolin-pectin B. ciprofloxacin C. simethicone D. doxycycline

(u) A. Kaolin-pectin is not indicated in the treatment of traveler's diarrhea. (c) B. Ciprofloxacin is indicated in the treatment of traveler's diarrhea. (u) C. Simethicone is used in the treatment of intestinal gas. (u) D. Doxycycline is not indicated in the treatment of traveler's diarrhea due to increasing resistance.

Which of the following conditions will produce a transudative pleural effusion? A. Kaposi's sarcoma B. Pneumonia C. Cirrhosis D. Mesothelioma

(u) A. Kaposi's sarcoma, pneumonia, or mesothelioma will produce a transudative pleural effusion. (u) B. See A for explanation. (c) C. Transudative pleural effusions result from alteration in the formation of pleural fluid, the absorption of pleural fluid, or both, by systemic factors. Local factors affecting pleural fluid absorption and/or formation produce exudative pleural effusions. (u) D. See A for explanation.

A 19 year-old woman has been consuming up to six beers daily since she was 16. She is now pregnant with her first child, has had little prenatal care, and is due to deliver in four weeks. Of the following, which neonatal problems should you anticipate? A. kernicterus B. hydrocephalus C. low birth weight D. teeth discoloration

(u) A. Kernicterus is typically due to ABO/Rh incompatibility, not fetal alcohol syndrome. (u) B. While a wide variety of disorders, such as infection, tumors, and congenital malformations, may play a role in the development of hydrocephalus, it is not a feature of fetal alcohol syndrome. (c) C. Fetal alcohol syndrome is a common cause of low birth weight. (u) D. Teeth discoloration is common in infants exposed to tetracycline

Upon stroking of the lateral aspect of the sole from the heel to the ball of the foot, the great toe dorsiflexes and the other toes fan. This is a positive A. Kernig's sign. B. Brudzinski's sign. C. Babinski's sign. D. Gower's sign.

(u) A. Kernig's sign is positive when pain is noted on straightening the knee after flexing both the hip and knee. (u) B. Brudzinski's sign occurs with neck flexion resulting in resultant flexion of the hips. It is a sign of meningeal irritation. (c) C. A Babinski test is performed by stroking the lateral aspect of the sole from the heel to the ball of the foot, the great toe dorsiflexes and the other toes fan in a positive test. (u) D. A positive Gower's sign is noted in certain types of muscular dystrophy and is described as children rising to stand by rolling over prone and pushing off the floor with arms while the legs remain extended.

: (27) 34 77. History & Physical/Infectious Diseases An 11 year-old female recent immigrant from Haiti, presents to your office with a 3 day history of mild sore throat, low grade fever and malaise. There had been a recent outbreak of diphtheria in your area. Which of the following physical examination findings would you expect in this patient? A. Koplik spots B. Gray membrane coating the pharynx C. White pharyngeal exudates D. Strawberry tongue

(u) A. Koplik spots are found in measles. (c) B. Pharyngeal diphtheria presents with gray membrane covering the tonsils and pharynx. (u) C. White pharyngeal exudates are found in strep pharyngitis. (u) D. Strawberry tongue is found in Kawasaki disease or scarlet fever.

Which of the following laboratory tests would be elevated in a patient with symptomatic heart failure? A. Lactate dehydrogenase (LDH) B. Troponin I (TnI) C. C-reactive protein (CRP) D. Brain natriuretic peptide (BNP)

(u) A. LDH is not a sensitive marker in patients with heart failure. (u) B. Troponin measurements are specific for myocardial infarctions. (u) C. Increased serum levels of CRP are found in patients with unstable angina and MI. They can be a strong predictor of coronary events. (c) D. BNP is expressed in the ventricles and is a sensitive assay in patients with heart failure.

Which of the following patients is at highest risk for the development of the skin disorder known as erythrasma? A. Lactating women B. Postmenopausal women C. Seniors living in cold northern climate D. People living in tropical climate

(u) A. Lactating women are at increased risk for staph infections in the breast. (u) B. Postmenopausal women are at increased risk for atrophic vaginitis but not for erythrasma. (u) C. Seniors living in cold conditions are at risk for winter itch which is worse with dry heat. (c) D. People living in warm, tropical climate, people wearing occlusive clothing or shoes, obese patients, and those with hyperhidrosis are at increased risk for erythrasma. The diagnosis of this condition is made by demonstration of a coral red fluorescence.

A 2 year-old presents to the emergency department in acute respiratory distress. The parents relate a history of a recent upper respiratory illness that was followed by a sudden onset of barking cough during the night, but this morning they noted increased difficulty breathing. The child is noted to have stridor at rest, but has no evidence of cyanosis. Which of the following is the most appropriate initial intervention? A. Intravenous antibiotics B. Endotracheal intubation C. Inhaled mucolytic agent D. Nebulized racemic epinephrine

(u) A. Laryngotracheobronchitis is caused by viruses, not bacteria, and therefore antibiotic therapy is not indicated. (u) B. If patients fail to respond to initial treatment and progress to impending respiratory failure, endotracheal intubation is then indicated. (u) C. Inhaled mucolytic agents are not indicated in the treatment of laryngotracheobronchitis. (c) D. This patient most likely has laryngotracheobronchitis (viral croup). Treatment with nebulized racemic epinephrine and glucocorticosteroids is indicated for patients with stridor at rest.

Which of the following is the most appropriate intervention in suspected child abuse? A. Arrange for the arrest of the parents. B. Confront the suspected assailant in front of the child. C. Assure the safety of the child, with hospitalization if necessary. D. Contact social service department after discharge of the child.

(u) A. Law enforcement would be contacted after an investigation of the incident(s) by the proper authorities. It is not the responsibility of the medical caregiver to arrange for the arrest, only to report your suspicions. (u) B. Confrontation should be done by the investigators of the incident(s). (c) C. The primary goal should be the safety and well-being of the child. Hospitalization may be the only way the clinician has to remove the child from the care of a possible abusive home if no other recourse is available due to a lack of social services and investigators. (u) D. Any abuse suspicion should be reported prior to the discharge of the child. Discharging the child to the person or people responsible for the abuse may cause greater harm to the child.

Which of the following conditions would cause a positive Kussmaul's sign on physical examination? A. Left ventricular failure B. Pulmonary edema C. Coarctation of the aorta D. Constrictive pericarditis

(u) A. Left ventricular failure results in the back-up of blood into the left atrium and then the pulmonary system so it would not be associated with Kussmaul's sign. (u) B. Pulmonary edema primarily results in increased pulmonary pressures rather than having effects on the venous inflow into the heart. (u) C. Coarctation of the aorta primarily affects outflow from the heart due to the stenosis resulting in delayed and decreased femoral pulses; it has no effect on causing Kussmaul's sign. (c) D. Kussmaul's sign is an increase rather than the normal decrease in the CVP during inspiration. It is most often caused by severe right-sided heart failure; it is a frequent finding in patients with constrictive pericarditis or right ventricular infarction.

: (9) 48. History & Physical/Orthopedics/Rheumatology An obese 12 year-old male presents with a 1 month history of right thigh pain worsened with weight bearing. Examination is normal, however, radiographs reveal a posterior and medial displacement of the femoral head. Which of the following is the most likely diagnosis? A. Legg Calvé Perthes disease B. Developmental dysplasia of the hip C. Slipped capital femoral epiphysis D. Osgood-Schlatter disease

(u) A. Legg Calvé Perthes disease is thought to be secondary to avascular necrosis of the hip in children small for their age with retarded bone growth. (u) B. Developmental dysplasia of the hip is partial or complete dislocation of the hip in the newborn. (c) C. Slipped capital femoral epiphysis is the posterior and usually medially displacement of the femoral head in overweight adolescents during puberty. The growth plate is susceptible to slippage due to the rapidly thickened cartilage physis secondary to growth hormone, the lack of sex hormone secretion for physis stability, mechanic stress secondary to obesity, and the anatomy and hip mechanics. (u) D. Osgood-Schlatter disease is a painful swelling over the tibial tubercle, most commonly in a male adolescent athlete.

A positive Wood's light examination (fluorescence) demonstrates A. viral infection with herpes zoster. B. bacterial infection with Treponema pallidum. C. parasitic infestation with Pediculus humanus. D. mycotic infection with Microsporum canis.

(u) A. Lesions of herpes, Treponema, and Pediculus infections do not fluoresce. (u) B. See A for explanation. (u) C. See A for explanation. (c) D. Microsporum causes tinea capitis and fluoresces blue-green under Wood's light.

A 29 year-old female has been diagnosed with infertility due to anovulation. Her provider suggests using a medication that will block the feedback inhibition of estradiol on the hypothalamus and pituitary leading to an increase in FSH. Which of the following medications is the most appropriate for this patient? A. Leuprolide (Lupron) B. Clomiphene citrate (Clomid) C. Medroxyprogesterone acetate (Provera) D. Metformin (Glucophage)

(u) A. Leuprolide inhibits gonadotropin release suppressing ovarian steroidogenesis and ovulation. (c) B. Clomiphene citrate is the agent of choice for women younger than 36 years of age who need induction of ovulation. (u) C. Medroxyprogesterone acetate inhibits pituitary gonadotropin release, it maintains a pregnancy; used for secondary amenorrhea. (u) D. Metformin decreases hepatic glucose, reduces body weight which in turn can improve ovulatory function in women with PCOS; it is sometimes used as an adjunct with clomiphene citrate in anovulation.

A 60 year-old male presents with difficulty voiding and having to get up twice a night to urinate. On physical examination, the prostate is firm, smooth, and enlarged. Prostate specific antigen level is normal. Which of the following is the best treatment option for this patient? A. Leuprolide (Lupron) B. Terazosin (Hytrin) C. Ciprofloxacin (Cipro) D. Naproxen (Naprosyn)

(u) A. Leuprolide is a luteinizing hormone-releasing hormone agonists used in the treatment of prostate cancer. (c) B. Terazosin is an alpha-blocker and used to treat benign prostatic hyperplasia. (u) C. Ciprofloxacin is an antibiotic used to treat acute prostatitis not benign prostatic hypertrophy. (u) D. Naproxen is an anti-inflammatory agent and not used in the treatment of benign prostatic hypertrophy.

A 36 year-old male who is hospitalized because of severe injuries from a motor vehicle accident develops rapid onset of profound dyspnea. Initial chest x-ray shows a normal heart size with diffuse bilateral infiltrates. Follow-up chest xray shows confluent bilateral infiltrates that spare the costophrenic angles. Which of the following is the best clinical intervention for this patient? A. Provide supplemental oxygen B. Insert chest tube C. Tracheal intubation D. Administer corticosteroids

(u) A. Marked hypoxemia is refractory to treatment with supplemental oxygen in ARDS. (u) B. Chest tube insertion is not indicated in a patient with ARDS. (c) C. Treatment of hypoxemia in acute respiratory distress syndrome (ARDS) usually requires tracheal intubation. (u) D. Routine use of corticosteroids is not recommended in the management of ARDS.

A 54 year-old female comes to your office with the complaint of vaginal itching. Her last menstrual period was three years ago. On examination the patient's vulva is smooth and somewhat shiny; her vaginal mucosa is pale and thin with a mild yellowish discharge with a pH of 6.0. The most likely cause of these symptoms is which of the following? A. Lichen sclerosis B. Atrophic vaginitis C. Contact Dermatitis D. Candidiasis

(u) A. Lichen sclerosis is a benign, chronic inflammatory process and the most common vulvar dermatologic disorder. During the acute phase the lesions appear red or purple and involve the non hair bearing areas of the vulva and perianal areas. Erythema and edema of the skin occur. (c) B. Atrophic vaginitis is typically caused by reduced estrogen levels producing intense itching and thin vaginal mucosa with a resultant yellowish discharge that has a pH > 5.5. (u) C. Contact dermatitis is relatively common with red, edematous skin surfaces and sometimes with vesicles and secondary infection. (u) D. Candidiasis produces a white curdy discharge with a pH of < 4.5.

To relieve dependent edema in a pregnant patient, which of the following should be instituted as treatment? A. Limit fluid intake. B. Elevate the legs. C. Prescribe thiazide diuretics. D. Strict avoidance of sodium.

(u) A. Limitation of fluid is not indicated and may be harmful. (c) B. Dependent edema is a common and rarely serious complication of pregnancy due to impedance of venous return. Leg elevation improves circulation. (u) C. Thiazide diuretics are contraindicated and could be harmful. (u) D. Edema due to impedance of venous return will not respond to sodium restriction.

: (28) 153. Clinical Therapeutics/Urology/Renal A 44 year-old male presents as a new patient. He has a history of hypertension, rheumatoid arthritis, polycystic kidney disease, and seasonal allergies. On review of his current medications, which of the following agents would you consider removing or replacing? A. Lisinopril (Zestril) B. Indomethacin (Indocin) C. Hydrochlorothiazide (HCTZ) D. Cetirizine (Zyrtec)

(u) A. Lisinopril is an ACE inhibitor and is an appropriate therapy for hypertension in the setting of chronic renal insufficiency. (c) B. While this NSAID is appropriate therapy for gouty flares, it has the potential for renal toxicity. Its use should be reviewed and if a suitable replacement cannot be made then dosing may need to be adjusted. (u) C. Hydrochlorothiazide is a thiazide diuretic and is an appropriate therapy for hypertension in the setting of chronic renal insufficiency. (u) D. Cetirizine is therapy for allergic rhinitis and is hepatically metabolized; it should not interfere adversely with renal function.

Which of the following is the most appropriate intervention for a patient suffering from a specific phobia, such as fear of snakes? A. Lithium B. Behavioral therapy C. Insight-oriented therapy D. Electroconvulsive therapy

(u) A. Lithium is indicated for treatment of bipolar disease, not phobias. (c) B. Specific phobias most commonly are treated with behavioral therapy including exposure therapy utilizing systemic desensitization. Hypnosis, supportive therapy and family therapy may also be useful adjunct treatment. (u) C. Insight-oriented therapy, while helpful in allowing patients to understand the origin of their fear and potential reasons for secondary gain, will not help to alleviate the patient's phobic symptoms. (u) D. The primary indication for electroconvulsive therapy (ECT) is in treatment of a major depressive disorder that has not responded to medical management.

Which of the following chest x-ray abnormalities would most likely be seen in a patient with hypersensitivity pneumonitis? A. Lobar consolidation B. Apical infiltration C. Granulomatous inflammation D. Diffuse nodular densities

(u) A. Lobar consolidation is seen in community-acquired pneumonia. (u) B. Apical infiltration is seen in tuberculosis. (u) C. Granulomatous inflammation is seen in sarcoidosis. (c) D. Diffuse nodular densities are seen in hypersensitivity pneumonitis.

In adults and intravenous drug abusers, which of the following bones is most commonly affected with acute osteomyelitis? A. Femur B. Humerus C. Vertebral spine D. Tibia

(u) A. Long bones are most commonly affected with osteomyelitis in children. (u) B. See A for explanation. (c) C. The bones of the vertebral spine are most commonly affected in a patient with osteomyelitis. Organisms reach the well-perfused vertebral body of adults via spinal arteries and quickly spread from the end plate into the disk space and then to the adjacent vertebral body. The infection may originate in the urinary tract and intravenous drug use carries an increased risk of spinal infection (u) D. See A for explanation.

A 23 year-old female presents with a 24-hour history of watery diarrhea, nausea and vomiting after returning from a cruise. On exam, the vitals reveal HR 120, BP 90/60, and RR 20. The abdomen is soft and non-tender, with hyperactive bowel sounds. Which of the following is the most appropriate management at this time? A. loperamide (Imodium) B. IV hydration C. bismuth sulfate (Pepto bismol) D. ciprofloxacin (Cipro)

(u) A. Loperamide is not indicated for acute infectious gastroenteritis in the vomiting patient. (c) B. This is most likely a self-limiting viral gastroenteritis and is treated with hydration and supportive care. (u) C. Bismuth sulfate is useful for treating diarrhea but is not helpful while the patient is vomiting. (u) D. Antibiotics, like ciprofloxacin, are not indicated for viral infections.

A 25 year-old female presents to the emergency department after an episode of substernal chest pain with radiation to the middle of her back that came on suddenly and lasted for about four minutes this morning while in bed. She denies previous episodes. Examination is unremarkable, but she appears jittery. Toxicology screen is positive for cocaine. Which of the following medications is contraindicated in this patient? A. Lorazepam (Ativan) B. Diltiazem (Cardizem) C. Nitroglycerin (Nitrostat) D. Propanolol (Inderal)

(u) A. Lorazepam is not contraindicated and can help with agitation, psychosis or seizures. (u) B. Diltiazem is not contraindicated but does not have a definitive role in treating cocaine toxicity. (u) C. Nitroglycerin is not contraindicated but does not have a definitive role in treating cocaine toxicity. (c) D. Pure Beta blockers, such as propranol, can cause a paradoxical hypertension because of unopposed alpha-adrenergic effects.

Which of the following descriptions is most typical of a patient with acromegaly? A. Mask-like face with decreased blinking and oily skin B. Non-pitting facial edema with dry course hair and dry skin C. Round face and red cheeks with hirsutism D. Coarsened facial features with prognathism and prominent brow

(u) A. Mask-like face with decreased blinking and oily skin is usually seen in a Parkinson's disease. (u) B. Non-pitting facial edema with dry course hair and dry skin is commonly seen with myxedema. (u) C. Round face and red cheeks with hirsutism is commonly seen in Cushing's syndrome (c) D. Coarsened facial features with prognathism and prominent brow are typical findings for acromegaly.

A 56 year-old female with a 35 pack year smoking history presents to the clinic with shortness of breath and cough. On examination, she is thin with no recent weight loss. She appears uncomfortable, breath sounds are diminished without adventitious sounds. Pulmonary function tests show a marked increase in total lung capacity (TLC) and a decreased FEV1. What is the most likely diagnosis for this patient? A. Persistent asthma B. Chronic obstructive pulmonary disease C. Idiopathic fibrosing interstitial pneumonia D. Sarcoidosis

(u) A. Lung function in asthma is evaluated by FEV1/FVC ratio with reduction noted with airflow obstruction. (c) B. Lung volume measurements in COPD reveal a marked increase in residual volume indicative of air trapping. (u) C. Pulmonary function testing in idiopathic fibrosing interstitial pneumonia shows a loss of lung volume with normal to increased airflow rates in interstitial lung disease. (u) D. Restrictive changes with decreased lung volumes and diffusing capacity are common in sarcoidosis.

What is the most common examination finding in a patient diagnosed with mumps? A. Occipital lymph node swelling B. Parotid gland tenderness C. Splenic enlargement D. Testicular tenderness

(u) A. Lymph node swelling can occur secondary to facial edema but can be variable and more often involves the submandibular and cervical lymph node chains. (c) B. Parotid gland tenderness and swelling are the hallmark findings of mumps. (u) C. Splenic enlargement is not a common exam finding even in extra-salivary disease. (u) D. Testicular tenderness can occur in as many as 40% of post-pubescent men but it is not the most common finding considering all cases of mumps.

Which of the following dietary substances interact with monoamine oxidase-inhibitor antidepressant drugs? A. Lysine B. Glycine C. Tyramine D. Phenylalanine

(u) A. Lysine, glycine, and phenylalanine are not known to interact with MAO inhibitors. (u) B. See A for explanation. (c) C. Monoamine oxidase inhibitors are associated with serious food/drug and drug/drug interactions. Patient must restrict intake of foods having a high tyramine content to avoid serious reactions. Tyramine is a precursor to norepinephrine. (u) D. See A for explanation.

The most common pathogen identified in community acquired pneumonia (CAP) is A. Mycoplasma pneumoniae. B. Staphylococcus aureus. C. Legionella pneumophila. D. Streptococcus pneumoniae.

(u) A. M. pneumoniae, S. aureus, Legionella pneumophila are common pathogens for CAP, but do not account for a majority of the cases. (u) B. See A for explanation. (u) C. See A for explanation. (c) D. S. pneumoniae accounts for a majority of cases of community acquired pneumonia.

You are treating a patient for her first episode of major depression. She has no medical problems and there is no family history of psychiatric disorders. In addition to psychotherapy, which class of drugs should be your first choice for the treatment of this patient? A. Monamine oxidase inhibitors (MAOIs) B. Tricyclic antidepressants (TCAs) C. Selective serotonin reuptake inhibitors (SSRIs) D. Serotonin dopamine antagonists (SDAs)

(u) A. MAOIs should not be first choice in the treatment of this patient because of the concern for potentially lethal hypertension and the need for a restrictive diet. (u) B. Although, TCAs are effective in the treatment of depression, they are not considered first line and have a significant risk of death with overdose. (c) C. SSRIs are the first line treatment for depression because of ease of use, safety, and broad spectrum of treatment. (u) D. SDAs or atypical antidepressants are not first line treatment for depression.

A 60 year-old male with history of hypertension presents for routine physical examination. He has no current complaints. Vital signs are BP of 136/70 mmHg, HR 60 bpm, regular, RR 14, unlabored. Heart shows regular rate and rhythm with no S3, S4 or murmur, Lungs are clear to auscultation bilaterally, and the abdomen is soft, nontender. There is a 5cm palpable pulsatile abdominal mass noted. Which of the following is the best initial diagnostic study in this patient? A. Magnetic resonance imaging (MRI) B. Arteriography C. Ultrasound (US) D. Plain film

(u) A. MRI, arteriography, and abdominal flat plate are not indicated in the initial diagnostic evaluation of a patient with a suspected abdominal aortic aneurysm. (u) B. See A for explanation. (c) C. Abdominal ultrasound is the diagnostic study of choice for the initial diagnosis of an abdominal aortic aneurysm. (u) D. See A for explanation.

A 79 year-old female presents with productive cough for 2 days. She has associated fever, chills and shortness of breath. On physical exam, RR 30, BP 90/60, T 101.3. There is no JVD. Lungs reveal crackles at the left lower lobe and decreased breath sounds with dullness to percussion. Heart exam reveals RRR with no S3 or S4. No edema is noted. On chest x-ray the patient has a left sided pleural effusion. Examination of the pleural fluid reveals a decreased glucose and an elevated pleural fluid LDH. Pleural fluid cytology reveals squamous epithelial cells. What is the most likely cause of the patient's effusion? A. malignancy B. bacterial pneumonia C. heart failure D. pulmonary embolus

(u) A. Malignancy is also a leading cause of an exudative pleural effusion, second to bacterial pneumonia. Cytology in this case was normal. (c) B. Bacterial pneumonia is the leading cause of an exudative pleural effusion. (u) C. Pleural effusions in heart failure are transudative, not exudative pleural effusions. (u) D. PE can be both an exudative and transudative classification of effusion, however, dyspnea is usually the most common symptom.

A patient with prostate cancer has a nonpalpable, focal lesion, and the patient is reluctant to have surgery at this time. Which of the following would best monitor disease progression? A. Periodic rectal exams B. Transrectal ultrasonography C. Measurements of serum acid phosphatase D. Measurements of prostate-specific antigen

(u) A. Many prostate carcinomas are contained within the gland, making it difficult to assess progression with a digital examination alone. (u) B. Ultrasonography is used largely for staging disease, not monitoring disease progression. (u) C. Serum acid phosphatase is more predictive of metastatic disease than PSA measurement, but its use has largely been replaced by PSA. (c) D. PSA measurement correlates well with volume and stage of disease and is the recommended examination for monitoring disease progression.

A 45 year-old male presents with abdominal pain and one episode of mild hematemesis, which happened days ago. On physical examination, vital signs are stable and he is in no acute distress. Hemoglobin and hematocrit are unremarkable; endoscopy reveals non-bleeding small superficial ulceration of the duodenal bulb. Rapid urease test is positive. Which of the following is the most appropriate treatment at this time? A. Schedule for a selective vagotomy and antrectomy B. Start an antacid along with omeprazole (Prilosec) C. Schedule elective ulcer excision and start sucralfate (Carafate) D. Start omeprazole (Prilosec) and antibiotic therapy against H. pylori

(u) A. Medical therapy should be initiated prior to any consideration of surgery, which is rarely performed secondary to satisfactory ulcer healing with medical therapy. (u) B. While proton pump inhibitors, such as omeprazole, have excellent results in healing duodenal ulcers, this regimen will not treat the H. pylori infection that is documented by the positive rapid urease test. (u) C. While sucralfate can be utilized as a cytoprotectant agent in treatment of active ulcer disease or in maintenance of healed ulcers, surgery to remove the ulcer is not warranted as initial therapy. (c) D. Treatment goals of H. pylori associated ulcers include eradicating the infection with appropriate antibiotics as well as use of a proton pump inhibitor, such as omeprazole, to promote ulcer healing.

A 23 year-old graduate student presents with sudden onset of severe dizziness, with nausea and vomiting for the past couple of hours. She denies hearing loss or tinnitus. She has had a recent cold. Which of the following is the most likely diagnosis? A. Meniere's disease B. vestibular neuronitis C. benign positional vertigo D. vertebrobasilar insufficiency

(u) A. Meniere's disease is associated with hearing loss, tinnitus, and vertigo that lasts from seconds to hours. (c) B. Vestibular neuronitis or labyrinthitis presents with vertigo, nausea, and vomiting, but not hearing loss or tinnitus. It is related to viral URIs, and develops over several hours, with symptoms worse in the first day, with gradual recovery over several days. (u) C. Benign positional vertigo occurs with changes in position, especially rapid movements of the head. Nausea may occur, but vomiting is not significant. (u) D. Vertebrobasilar insufficiency is usually accompanied by brain stem findings, such as diplopia, dysarthria, or dysphagia, and is not common in this age group.

Which of the following medications inhibits prostaglandin synthesis in a patient with rheumatoid arthritis? A. Methotrexate B. Infliximab (Remicade) C. Probenecid (Benemid) D. Aspirin Explanations

(u) A. Methotrexate inhibits the enzyme dihydrofolate reductase. (u) B. Infliximab (Remicade) neutralizes cytokine tumor necrosing factor. (u) C. Probenecid blocks the tubular reabsorption of filtered urate and is used to reduce serum uric acid. (c) D. Salicylates inhibit the enzymatic production of prostaglandins by inhibiting cyclooxygenase.

A 25 year-old male with a history of asthma presents complaining of increasing episodes of evening and daytime symptoms. He is on a short acting inhaled beta agonist prn. He is presently using his short acting beta agonist on a daily basis. Which of the following is the most appropriate addition to this patient's regimen? A. methylxanthine oxidase inhibitor B. long acting beta agonist inhaler C. leukotriene inhibitor D. inhaled corticosteroid

(u) A. Methylxanthine oxidase inhibitor preparations may have beneficial effects in some patients, but their value is limited due to a narrow therapeutic window and modest efficacy. (u) B. Long acting beta agonist inhalers should not be used in place of anti-inflammatory therapy. (u) C. Leukotriene inhibitors are less desirable alternatives to inhaled corticosteroids. (c) D. According to the stepwise approach for managing asthma by the National Asthma Education and Prevention Program, inhaled corticosteroids are indicated for mild to moderate persistent asthma.

A 16 year-old female who is a competitive swimmer presents with a complaint that her toe nails are thickened and discolored. Examination reveals that the nails have a dry yellow brittle appearance along with a friable surface of the nail plate. Which of the following is the treatment of choice? A. Metronidazole (Flagyl) B. Diflucan C. Terbinafine (Sporonox) D. Acyclovir

(u) A. Metronidazole is not indicated for fungal infections of the nails. (u) B. Diflucan is indicated for dermal fungal infections. (c) C. Oral Terbinafine 250 mg is considered the best treatment for onychomycosis. (u) D. Acyclovir is indicated for viral infections, not fungal infections.

A 55 year-old morbidly obese male is seen in the office for routine examination. He has a history of pulmonary hypertension and cor pulmonale. Examination reveals a visible jugular venous pulse and a systolic flow murmur on the right side of the sternum. Which of the following is the most likely diagnosis? A. mitral insufficiency B. tricuspid insufficiency C. hepatic vein thrombosis D. aneurysm of the thoracic aorta

(u) A. Mitral insufficiency results in the accumulation of blood primarily in the pulmonary system and not the right side of the heart. (c) B. Tricuspid insufficiency will result in blood being put back into the right side of the body with increased jugular pulsation in the neck, along with a palpable venous pulse in the liver. (u) C. Hepatic vein thrombosis or Budd-Chiari syndrome is associated with cirrhosis and liver clotting abnormalities and is not due to right-sided heart failure. (u) D. Thoracic aorta aneurysm results in a widened mediastinum that is fairly asymptomatic until it results in rupture or dissection. These are typically found as incidental findings unless they are symptomatic from dissection or rupture, which causes severe chest pain or a severe tearing sensation in the chest.

A 15 year-old male was seen last week with complaints of sore throat, headache, and mild cough. A diagnosis of URI was made and supportive treatment was initiated. He returns today with complaints of worsening cough and increasing fatigue. At this time, chest x-ray reveals bilateral hilar infiltrates. A WBC count is normal and a cold hemagglutinin titer is elevated. The most likely diagnosis is A. tuberculosis. B. mycoplasma pneumonia. C. pneumococcal pneumonia. D. staphylococcal pneumonia.

(u) A. Most children with pulmonary tuberculosis are asymptomatic with few physical examination findings. The results of the diagnostic studies do not support tuberculosis as the most likely diagnosis. (c) B. The insidious onset of symptoms, the interstitial infiltrate on chest x-ray, and elevated cold hemagglutinin titer make this diagnosis the most likely. (u) C. The clinical presentation of bacterial pneumonias in children is variable, but usually involves fever of acute onset. The WBC count is also usually elevated, making this a less likely diagnosis. (u) D. See C for explanation.

A 13 year-old female presents with a six month history of lower mid-abdominal pain that is spasmodic in nature and radiates to the inner thighs. The pain usually starts within a few hours of the onset of menses and lasts about 2 days. The patient's menarche began 2 years ago. She denies any sexual activity. Physical examination, including pelvic, is unremarkable. Which of the following is the most likely diagnosis? A. ovarian cyst B. endometriosis C. primary dysmenorrhea D. premenstrual syndrome

(u) A. Most ovarian cysts are asymptomatic and found as an incidental finding on physical examination. Symptomatic cysts usually present with acute pain or bleeding secondary to rupture. (u) B. Endometriosis is characterized by pain that usually starts 1-2 weeks before the onset of menses and is relieved at the onset of menstrual flow or shortly after. (c) C. This patient most likely has primary dysmenorrhea supported by onset of pain within 1-2 years after onset of menarche and characteristic symptoms of low, mid-abdominal, spasmodic cramping pain that radiates to the back or inner thighs beginning on the first or second day of menstruation. Pelvic examination fails to reveal any pathological findings. (u) D. Premenstrual syndrome is seen primarily in females that are 25-40 years old. Associated physical and emotional symptoms appear 7-14 days before the onset of menses and resolve once menstruation begins

A 3 year-old patient is brought in with a 10 day history of clear nasal drainage and cough which has now developed into otalgia and fever exceeding 101 degrees F for the last 5 days. Mom denies other chronic medical problems. The patient has had similar complaints three times in her life. Which of the following is the most likely causative organism? A. Mycoplasma pneumoniae B. Pneumocystis jiroveci C. Pseudomonas aeruginosa D. Streptococcus pneumoniae

(u) A. Mycoplasma pneumoniae is a common cause of bronchitis and pneumonia. (u) B. Pneumocystis jiroveci is associated with immunocompromised respiratory illness. (u) C. Pseudomonas aeruginosa would most commonly cause otitis externa. (c) D. Acute otitis media is a bacterial infection of the mucous lined air-containing spaces of the temporal bone most commonly caused by Streptococcus pneumoniae, Haemophilus influenzae, and Streptococcus pyogenes

A 60 year-old male presents with a normochromic, normocytic anemia and splenomegaly. His past history reveals several episodes of bacterial pneumonia in the past year. The WBC count is 43,000 mm3 with 25% segmented neutrophils, 3% blasts, 70% mature lymphocytes, 1% basophils, and 1% eosinophils. This most likely represents A. myelodysplastic syndrome. B. acute lymphocytic leukemia. C. chronic lymphocytic leukemia. D. chronic myelogenous leukemia.

(u) A. Myelodysplastic syndrome may present with this clinical picture, but a CBC would indicate a normal or reduced white cell count. (u) B. A hallmark of acute leukemia is pancytopenia with numerous circulating blasts. (c) C. Chronic lymphocytic leukemia usually occurs after the age of 50 presenting with lymphocytosis > 20,000 mm3 and lymphocytes that appear mature. (u) D. Chronic myelogenous leukemia presents with markedly elevated WBC count (mean 150,000/mm3) with left 42 shifted myeloid series.

: (27) 162. Diagnosis/ENT/Ophthalmology A 64 year-old female presents with right-sided ear fullness and hearing loss. Weber testing lateralizes to the right and Rinne testing shows bone conduction greater than air conduction. Which of the following is the most likely diagnosis? A. Ménière's syndrome B. Presbycusis C. Cholesteatoma D. Cerumen impaction

(u) A. Ménière's syndrome consists of sensorineural hearing loss, tinnitus, vertigo and aural fullness. (u) B. Presbycusis is the most common form of sensorineural hearing loss in elderly. (u) C. Cholesteatoma is often a complication of recurrent otitis externa. May present with aural fullness and mixed hearing loss. (c) D. Cerumen impaction is the most common form of conductive hearing loss and may present with aural fullness or pressure, vertigo and otalgia.

An elderly nursing home resident is admitted with methicillin-resistant Staphylococcus aureus pneumonia. Which of the following is the most appropriate treatment to initiate? A. Nafcillin B. Vancomycin C. Clindamycin D. Doxycycline

(u) A. Nafcillin is classified as a penicillinase-resistant penicillin, however would not be effective against methicillin-resistant strains of Staphylococcus aureus. (c) B. Vancomycin with or without the addition of gentamicin or rifampin and linezolid is the treatment of choice for methicillin-resistant Staphylococcus aureus. (u) C. Clindamycin is primarily utilized in treatment of severe anaerobic infections, but is not indicated for the treatment of methicillin-resistant Staphylococcus aureus. (u) D. Doxycycline is a tetracycline and is only utilized to treat respiratory infections that are susceptible to tetracycline, such as Mycoplasma or Chlamydial pneumonias.

A 67 year-old female presents with progressive pain in her left knee that is worse with activity and relieved with rest. She notes stiffness of the knee that last about 20 minutes after activity is resumed. She exercises regularly. No known drug allergies. On physical exam she is 5' 5", 225 pounds. Her left knee exam reveals mild effusion without erythema or warmth. Radiographs of the left knee reveal medial joint space narrowing and subchondral bone sclerosis. Her sodium is 138 mEq/L, potassium 4.3 mEq/L, bicarbonate 24 mEq/L, chloride 104 mEq/L, BUN 23 mg/dl and creatinine 1.8 mg/dl. Which of the following medications is most appropriate for this patient's worsening pain? A. naprosyn B. prednisone C. acetaminophen D. methotrexate

(u) A. Naprosyn is contraindicated in patients with abnormal renal function. (u) B. Oral prednisone is not appropriate for the long-term management of osteoarthritis. (c) C. Acetaminophen is recommended as first-line pharmacotherapy in patients with osteoarthritis. (u) D. Methotrexate is not indicated for the treatment of osteoarthritis.

A 60 year-old male presents with a two day history of acute, right great toe pain with no trauma or change of activities. He states he is healthy but started a high protein diet three weeks ago to try and lose weight. Examination reveals an erythematous, severely swollen and highly sensitive right great toe with limited range of motion at the metatarsal phalangeal joint. No other joint abnormalities are noted. Which of the following is the most appropriate to confirm the diagnosis? A. Complete blood count B. Joint fluid analysis C. Foot radiograph D. Rheumatoid factor

(u) A. Normal to nonspecific findings will occur on CBC with gouty arthritis lessening this tests clinical utility. (c) B. Joint fluid analysis is needed to confirm the diagnosis of gouty arthritis and rule out other diagnoses such as a septic joint, psoriatic arthritis and other crystalline-associated arthritides. (u) C. Radiographs would only show clinically obvious swelling in cases of acute gouty arthritis. (u) D. Rheumatoid factor is not useful in diagnosing this non-autoimmune process.

The most frequent finding in a person presenting with a brain abscess is A. nuchal rigidity. B. headache. C. seizures. D. vomiting.

(u) A. Nuchal rigidity occurs in approximately 35% of patients with a brain abscess. (c) B. Headache occurs in over 70% of patients with a brain abscess. (u) C. Seizures occur in approximately 35% of patients with a brain abscess. (u) D. Vomiting occurs in approximately 35% of patients with a brain abscess.

: (27) 62. Clinical Therapeutics/Gastrointestinal/Nutritional A 48 year-old male with a history of AIDS presents with complaints of odynophagia and dysphagia for 1 week. The patient denies nausea, vomiting, diarrhea, constipation or melena. Endoscopic evaluation reveals diffuse, linear plaques which are yellowish-white in color and adhere to the esophageal mucosa. Which of the following is the treatment of choice for this patient? A. Nystatin (Mycostatin) suspension B. Oral acyclovir (Zovirax) 29 C. Oral fluconazole (Diflucan) D. Inhaled triamcinolone acetonide (Azmacort)

(u) A. Nystatin suspension is indicated in the management of oropharyngeal candidiasis not esophageal. (u) B. Oral acyclovir is indicated in the management of herpes esophagitis. (c) C. Oral fluconazole is the treatment of choice for a patient with candidal esophagitis. (u) D. Inhaled corticosteroid is used in the management of eosinophilic esophagitis.

: (27) 78. Diagnostic Studies/Cardiology A patient admitted with substernal chest pain undergoes cardiac catheterization. Angiography reveals 98% occlusion of the right coronary artery. All other vessels are 100% patent. Which of the following is the most expected electrocardiogram finding in this patient? A. ST elevation in leads I, avL, V5 and V6 B. ST elevation in leads II, III, and avF C. ST elevation across V2, V3 and V4 D. Tall upright R and T waves in V1 and V2

(u) A. Occlusion of the circumflex artery or diagonal branch of the LAD will affect the lateral wall. This manifests as ST elevation in leads V5 and V6. (c) B. The right coronary artery is responsible for inferior wall of the heart. Inferior wall of the heart is best illustrated on EKG in leads II, III, aVF. (u) C. The anterior wall of the heart is supplied by the left anterior descending. On ECG this is manifested on ECG in leads V2-V4. (u) D. The posterior wall generally has a dual blood supply from the RCA and the circumflex artery. Posterior wall MI is usually manifested by ST depression and tall, upright R and T waves in leads V1-V3

A 57 year-old male was working on his farm, when some manure was slung hitting his left eye. He presents several days after with a red, tearing, painful eye. Fluorescein stain reveals uptake over the cornea looking like a shallow crater. Which of the following interventions would be harmful? A. Ophthalmic antibiotics B. Pressure patch C. Examination for visual acuity D. Copious irrigation

(u) A. Ophthalmic antibiotics and copious irrigation are indicated when treating a patient with a suspected corneal ulcer due to an infectious cause. (c) B. Patching of the eye after abrasion associated with organic material contamination is contraindicated due to increased risk of fungal infection. (u) C. Examination for assessment of visual acuity should be performed. (u) D. See A for explanation.

In a patient with amaurosis fugax what is the most appropriate initial diagnostic study? A. Ophthalmoscopy B. Schiotz tonometry C. MR angiography D. Carotid ultrasound

(u) A. Ophthalmoscopy provides visualization of retina but does not help elucidate the source of the plaque. (u) B. Schiotz tonometry is used to measure intraocular pressure. (u) C. MR angiography is a useful test to identify retinal vascular anatomy but is not the first choice in the search for causes of amaurosis fugax. (c) D. The most common cause of amaurosis fugax is an atherosclerotic plaque in the carotid artery which can be identified with ultrasound.

A 16 year-old male is brought into your office by his girlfriend. She states that "he hasn't been himself lately" and seems to fluctuate from being almost "euphoric" to being depressed and irritable. The patient states that "he is really okay" and that he "just feels a little irritable occasionally." On physical examination his pulse is 120 beats/minute, blood pressure is 180/110 mmHg, he is sweating and his pupils are widely dilated. Which of the following is the most likely diagnosis? A. Opiate abuse B. Acute anxiety attack C. Cocaine intoxication D. Bipolar affective disorder

(u) A. Opiate abuse would be suspected with the presence of euphoria, drowsiness, and constricted pupils. More severe cases present with bradycardia, hypotension, coma, or respiratory arrest. (u) B. While acute anxiety disorder may present with tachycardia, agitation, diaphoresis, and hypertension, this diagnosis should be a diagnosis of exclusion. Failure to diagnose cocaine intoxication could lead to severe consequences. (c) C. Cocaine is a stimulant and presenting clinical manifestations of intoxication include agitation, tachycardia, hypertension, diaphoresis, and dilated pupils. (u) D. While the history may suggest the diagnosis of bipolar affective disorder, the physical exam findings do not support this diagnosis.

A 25 year-old female with irritable bowel syndrome presents with complaint of upper and lower extremity discomfort. The patient has pinpoint muscle tenderness over the shoulder girdle, low back and hips. Which of the following medications is the most appropriate treatment in this patient? A. Hydrocodone (Vicodin) B. Amitriptyline (Elavil) C. Methotrexate (Rheumatrex) D. Naproxen sodium (Naprosyn)

(u) A. Opioid medication is not effective in the treatment of fibromyalgia. (c) B. Amitriptyline is the treatment of choice for fibromyalgia and is typically administered at bedtime. (u) C. Methotrexate is not effective in treating fibromyalgia. (u) D. NSAIDs are generally ineffective in treating fibromyalgia.

A 34 year-old female with a history of asthma presents with complaints of increasing asthma attacks. The patient states she has been well-controlled on albuterol inhaler until one month ago. Since that time she notices that she has had to use her inhaler 3-4 times a week and also has had increasing nighttime use averaging about three episodes in the past month. Spirometry reveals greater than 85% predicted value. Which of the following is the most appropriate intervention at this time? A. Oral prednisone B. Oral theophylline (Theo-Dur) C. Salmeterol (Serevent) inhaler D. Beclomethasone (Qvar)inhaler

(u) A. Oral corticosteroids, such as prednisone, are added to therapy in severe persistent asthma. While a course of oral corticosteroids may be needed for mild exacerbations of asthma, they are not added until inhaled corticosteroids have failed to control the symptoms. (u) B. Due to its safety profile, oral theophylline is now considered a third or fourth line treatment option for asthma. (u) C. Long acting inhaled beta2-agonists, such as salmeterol, are not added to the treatment regimen until the symptoms indicate a moderate persistent asthma. Long acting inhaled beta2-agonists should also not be used in place of inhaled steroids. (c) D. This patient has progressed to mild persistent asthma. In addition to her inhaled beta2-agonist (albuterol), she should be started on an anti-inflammatory agent. Inhaled corticosteroids, such as beclomethasone, are preferred for long-term control.

Which of the following prophylactic interventions is recommended in all individuals with HIV regardless of disease stage? A. Oral polio vaccine B. Pneumococcal vaccine C. Fluconazole for fungal infections D. Isoniazid for tuberculosis reactivation

(u) A. Oral polio vaccine is contraindicated in HIV patients because it is a live attenuated vaccine. (c) B. All HIV-positive individuals should receive prophylaxis against pneumococcal pneumonia. (u) C. Prophylaxis against fungal infection would be indicated for those HIV-positive individuals with CD4 less than 50 cells/μL. (u) D. Isoniazid is indicated for those with positive PPD and normal chest x-ray. Reactivation would require a multidrug regimen.

During the stages of fracture healing which of the following is responsible for producing collagen? A. Osteoclasts B. Chondrocytes C. Glycosaminoglycans D. Fibroblasts

(u) A. Osteoclasts are responsible for removing necrotic bone. (u) B. Chondrocytes make up the articular cartilage. (u) C. Glycosaminoglycans help form the osteon or vascular canal. (c) D. Fibroblasts produce collagen during the inflammation stage of healing.

A 12 year-old male presents with pain in his left leg that is worse at night. Aspirin relieves the pain and the patient denies injury. On examination, there is point tenderness over the tibia, and the patient has a slight limp that favors the left leg. Radiographs show a 1 cm radiolucent nidus surrounded by osteosclerosis. Which of the following is the most likely diagnosis? A. Osteosarcoma B. Legg-Calve-Perthes disease C. Osgood-Schlatter disease D. Osteoid osteoma

(u) A. Osteosarcoma and Ewing sarcoma are malignant bone tumors that present with pain and swelling. No improvement is noted with conservative therapy. (u) B. Legg-Calve-Perthes disease is avascular necrosis of the hip affecting boys ages 4-10. (u) C. Osgood-Schlatter disease is inflammation of the tibial tuberosity affecting mainly boys in the ages of 10-15. Commonly associated bilaterally and due to jumping. (c) D. Osteoid osteoma is a benign tumor in children age 5 to 20, presents with increasing pain, worse at night and relieved by aspirin.

The most definitive treatment for primary enuresis is A. oxybutynin chloride (Ditropan). B. imipramine (Tofranil). C. trimethoprim-sulfamethoxazole (Bactrim). D. desmopressin (DDAVP).

(u) A. Oxybutynin chloride is used for bladder spasms. It cannot be used for children under 5 years of age and is not indicated in primary enuresis. (u) B. Imipramine is an older form of treatment that is moderately effective, but many patients relapse when therapy is stopped. This is no longer considered the treatment of choice. (u) C. TMP-SMX is indicated for urinary tract infections that may cause secondary enuresis, but it is not used in primary enuresis. (c) D. Intranasal desmopressin is effective in 50% of patients treated and is the treatment of choice.

A 62 year-old male presents with complaints of dyspepsia, early satiety, and dysphagia. What diagnostic study would be indicated in his initial workup? A. PET CT scan B. Abdominal ultrasound C. Gastric emptying studies D. Upper endoscopy

(u) A. PET CT scan is used primarily to follow the course of an already diagnosed cancer. (u) B. Abdominal ultrasonography is performed only when pancreatic or biliary disease is suspected. (u) C. Gastric emptying studies are valuable only in patients with recurrent vomiting. (c) D. Upper endoscopy is indicated for patients over the age of 55 presenting with new-onset symptoms of dyspepsia in order to evaluate for gastric cancer or other serious organic disease. Upper endoscopy is the diagnostic study of choice to diagnose gastroduodenal ulcers, erosive esophagitis, and upper GI malignancy.

A 9 year-old boy who has had cold-like symptoms for the past few days is brought to the clinic by his mother who states that her son had gross hematuria this morning. Prior to the cold-like symptoms the boy has been in excellent health. He is up-to-date on all of his immunizations. The patient does not have any edema, hypertension or purpura. Urinalysis reveals the urine to be cola-colored with a 2+ positive protein and 2+ hemoglobin. Microscopic analysis reveals 50-100 RBCs/HPF, no WBCs, bacteria, casts or crystals. What is the most likely diagnosis? A. Post streptococcal glomerulonephritis (PSGN) B. IgA nephropathy C. Minimal change disease (MCD) D. Membranous nephropathy

(u) A. PSGN usually presents 2-3 weeks after a streptococcal infection (pharyngeal or skin) and usually presents with nephritic symptoms (edema, hypertension, cola-colored urine). This is due to trapping of the streptococcal antigen within the glomerulus (c) B. IgA nephropathy presents after an upper respiratory illness with deposition of IgA within the mesangium of the glomerulus. (u) C. MCD is the most common nephrotic presentation (edema, hypoproteinemia, hyperlipidemia, >3.5 gms. of proteinuria in 24 hours) in children following an upper respiratory illness. This patient is not exhibiting any of these signs at this time. (u) D. Membranous nephropathy is the most common cause of adult nephrotic syndrome.

A 13 year-old child presents with a lesion on his right forearm that is occasionally pruritic. On examination, a 2 cm ring of erythema with a scaly border and central clearing is noted. A suspected diagnosis is confirmed by the presence of A. a positive patch test. B. hyphae on a KOH prep. C. gram-positive cocci on Gram stain. D. multinucleated cells on Tzanck smear

(u) A. Patch testing is positive with allergic dermatitis, not fungal disorders. (c) B. Ring-shaped lesions with scaly borders and central clearing are most likely caused by fungal infection. Microscopic examination of scrapings reveals hyphae on KOH prep. (u) C. A positive Gram stain is found with bacterial skin infections, not fungal. (u) D. Multinucleated cells found on Tzanck smear indicate herpes, not fungal infections.

A 55 year-old male presents with complaint of sudden ripping chest pain that radiates into the abdomen. On examination the patient is found to have diminished peripheral pulses and a diastolic murmur. EKG reveals left ventricular hypertrophy. Which of the following is the most likely diagnosis? A. acute myocardial infarction B. pulmonary embolism C. acute pericarditis D. aortic dissection

(u) A. Pain associated with a myocardial infarction is commonly a retrosternal pressure, squeezing, or heaviness. ST segment elevation on EKG would be expected. (u) B. A pulmonary embolism is associated with retrosternal pain; however chest pain is not always present. Patients more commonly will have a sudden onset of dyspnea. PE is not usually associated with a diastolic murmur or diminished pulses. (u) C. Acute pericarditis is characterized by sharp, knife-like pain that is worse with lying supine and better with sitting up and leaning forward. One would expect to find a pericardial friction rub on auscultation. (c) D. Aortic dissection is characterized by a ripping or tearing type pain with radiation to the neck, back or abdomen. Left ventricular hypertrophy is often seen on EKG secondary to longstanding hypertension. A diastolic murmur is often present secondary to aortic insufficiency.

A patient presents complaining of periumbilical pain. Which of the following anatomical sites is this finding associated with? A. Bladder B. Stomach C. Pancreas D. Small bowel

(u) A. Pain from the bladder, uterus, or colon causes hypogastric pain. (u) B. Pain from the stomach, duodenum, or pancreas causes epigastric pain. (u) C. See B for explanation. (c) D. Pain from the small intestine, appendix, or proximal colon causes periumbilical pain.

A 32 year-old female presents to the office with the complaint of worry which she can not control for the last six months. She tells you that she has symptoms at least four times per week consisting of sleep disturbances, difficulty concentrating and irritability. What is the most likely diagnosis? A. Panic disorder B. Generalized anxiety disorder C. Posttraumatic stress disorder D. Obsessive-compulsive disorder

(u) A. Panic disorder typically involves certain situations or phobias rather than generalized symptoms. (c) B. A patient needs to have symptoms for more days than not for six months or more, need 3 of 6 symptoms to diagnose generalized anxiety disorder. (u) C. Posttraumatic stress disorder is characterized by development of symptoms after exposure to traumatic events. (u) D. In obsessive-compulsive disorder patients have symptoms of intrusive thoughts, rituals, preoccupations, and compulsions.

A 65 year-old postmenopausal female presents with complaints of new onset of vaginal bleeding. She relates a history of two episodes of vaginal bleeding during the past month, each lasting about four days. Which of the following diagnostic evaluations is the most reliable to evaluate the suspected diagnosis? A. Pap smear B. endometrial biopsy C. serum prolactin level D. serum FSH and LH levels

(u) A. Pap smears are the diagnostic tool of choice to evaluate for cervical cancer. The Pap smear is not reliable in diagnosing endometrial cancer, but in a small percentage of patients may show atypical endometrial cells which necessitates follow-up with an endometrial biopsy. (c) B. Postmenopausal bleeding is a primary complaint in patients with endometrial cancer. The only reliable method of diagnosis is by endometrial biopsy. (u) C. Serum prolactin levels are utilized in the evaluation of a patient with galactorrhea, amenorrhea or infertility. It is elevated most commonly in patients with a pituitary adenoma. (u) D. Serum FSH and LH levels will be increased due to the patient's menopausal state, but will not help in the evaluation of the postmenopausal bleeding.

: (6) 210. Diagnosis/Gastrointestinal/Nutritional A 33 year-old male returns from an extended camping trip and complains of 5 weeks of mild diarrhea, mostly after breakfast. The stool is described as bulky, greasy, frothy, and malodorous; it is free of blood or pus. He is afebrile and has upper abdominal discomfort, cramps, distention, and excessive flatus. he has lost several pounds. Which of the following is the most likely diagnosis? A. Amebiasis B. Giardiasis C. Schistosomiasis D. Trypanosomiasis

(u) A. Parasitic dysentery more probably would present as febrile illness with blood and/or mucus in watery diarrhea and not associated with meals. (c) B. This is a classic presentation of diarrheal illness due to Giardia and in its chronic state can cause malabsorption and therefore be associated with foul smelling, oily or greasy stools with weight loss. (u) C. Chronic infection by Schistosoma sp. with intestinal involvement presents as abdominal pain, irregular bowel movements and blood in the stool. The clinical syndrome usually presents with fever and chills and organomegaly. (u) D. Febrile illness is due to infection by Trypanosoma sp. (sleeping sickness) which may have CVS and CNS clinical features.

Which of the following is an established risk factor for osteoporosis? A. Parity status B. Carbohydrate intake C. Lactation history D. Low body weight

(u) A. Parity status and lactation history have been shown to be poor predictors of bone mass; therefore, they do not have an established role in the prediction of increased risk for development of osteoporosis. (u) B. Carbohydrate intake has no association to the development of osteoporosis. (u) C. See A for explanation. (c) D. Established risk factors for osteoporosis include low body weight, female sex, advanced age, Caucasian race, and bilateral oophorectomy before menopause without estrogen replacement.

: (11) 10. Diagnosis/Neurology A 48 year-old male presents with a mild tremor that seems to increase with stressful situations and subsides when he has a glass of wine with dinner. The patient does not demonstrate the tremor at rest but it reappears when he reaches for a pen. Which of the following is most likely the cause of this patient's symptoms? A. Parkinson disease B. Essential tremor C. Huntington disease D. Focal torsion dystonia

(u) A. Parkinsonism is characterized by combination of tremor, rigidity, bradykinesia, and postural instability. (c) B. Essential tremor can be familial and cause action related hand tremor, head tremor, or voice tremor. The lower extremities are spared and generally no further neurologic findings are present. (u) C. Huntington disease is characterized by gradual onset of chorea and restlessness, progressing to choreiform movements. (u) D. Focal torsion dystonia causes involuntary spasmodic movements including torsion of neck, oromandibular dystonia, or uncontrolled eye blinking.

A newborn presents with blue discoloration of the peripheral extremities which worsens with exposure to cold and improves with warming. Which of the following is the most likely cause? A. Raynauds phenomenon B. Livedo reticularis C. Erythromelagia D. Acrocyanosis

(u) A. Raynauds phenomenon has a triphasic color response and is exacerbated by cold or emotions. (u) B. Livedo reticularis is characterized by a lacy pattern on the skin of the lower extremities. (u) C. Erythromelagia is red, painful extremities. (c) D. Acrocyanosis is a blue discoloration of the digits, usually of the peripheral extremities in newborns, which worsens with exposure to cold and improves with warming.

A 76 year-old male presents after returning from a Safari in Africa. Seven days ago he experienced chest pressure lasting one hour that did not respond to three sublingual nitroglycerin tablets. There was no ability to have lab work or an EKG. The pain has not returned. If the patient had a non-STEMI myocardial infarction, which of the following studies will still be positive? A. Electrocardiogram B. Myoglobulin C. CK-MB index D. Troponin I Explanations

(u) A. Patients suffering from a non-STEMI myocardial infarction will not develop Q waves and most likely will have a normal EKG five days after an acute event. (u) B. Myoglobulin is a nonspecific enzyme that is released into the circulation after any skeletal muscle damage, including a myocardial infarction. It is the first enzyme that becomes positive in the setting of acute myocardial infarction but its non-specific measurement makes it less useful in the setting of acute myocardial infarction. It returns to baseline within 24 hours after infarction. (u) C. CK-MB index has improved sensitivity for myocardial muscle damage that occurs with acute myocardial infarction but it returns to baseline within 2-3 days after injury. (c) D. Troponin I levels will stay positive for at least one week following myocardial infarction and is the preferred enzyme to measure in this setting.

An adult patient who has undergone a splenectomy secondary to idiopathic thrombocytopenic purpura should receive which prophylactic agent? A. Bicillin 1.2 million units IM every month B. Pneumococcal vaccine IM one time only C. Gamma globulin IM every month D. Hepatitis B vaccine IM series

(u) A. Patients who are being treated for rheumatic fever may receive Bicillin 1.2 million units every month for Group A beta-streptococcal prophylaxis. (c) B. Pneumococcal vaccine should routinely be given to all patients who undergo splenectomy for prophylaxis to Streptococcal pneumoniae. (u) C. Gamma globulin is routinely given to patients for prophylaxis to hepatitis exposure. (u) D. Hepatitis B vaccine has no specific indication in patients after splenectomy.

Which of the following findings is usually associated with Addison's disease? A. Weight gain B. Hypertension C. Increased pigmentation D. High plasma cortisol levels

(u) A. Patients with Addison's disease tend to have anorexia and weight loss. (u) B. Patients with Addison's disease tend to have hypotension. (c) C. Patients with Addison's disease have diffuse tanning over nonexposed and exposed skin due to increased melanocytic factor that is released with adrenocorticotropic hormone. (u) D. Patients with Addison's disease tend to have low plasma cortisol levels.

A 70 year-old male is brought to the office by his wife. She is complaining that her husband has been having great difficulty remembering things, however remains alert. She also complains that he is having difficulty getting to the bathroom on time. On physical examination you notice that the patient's gait is wide-based and slow and he walks without lifting his feet off the floor. You also notice a tremor in his right hand. What is the most likely diagnosis? A. Lewy body dementia B. Parkinson's disease C. Normal pressure hydrocephalus D. Frontotemporal dementia

(u) A. Patients with Lewy body dementia have hallucination, Parkinsonism, fluctuating alertness, and falls. (c) B. Patients with Parkinson's disease have clinical features such as bradykinesia, rest tremor, rigidity, postural instability, autonomic dysfunction and behavioral changes. Of the diseases listed, Parkinson's is the most common. (u) C. Patients with normal pressure hydrocephalus have an abnormal gait (apraxic or ataxic), dementia, and urinary incontinence. (u) D. Patients with frontotemporal dementia have predominant behavioral symptoms, dementia, apraxia, Parkinsonism, and motor neuron disease.

A 19 year-old female presents with complaint of palpitations. On examination you note the patient to have particularly long arms and fingers and a pectus excavatum. She has a history of joint dislocation and a recent ophthalmologic examination revealed ectopic lentis. Which of the following echocardiogram findings would be most consistent with this patient's physical features? A. right atrial enlargement B. aortic root dilation C. pulmonic stenosis D. ventricular septal defect

(u) A. Patients with Marfan's syndrome commonly have mitral valve prolapse and possibly aortic regurgitation. Right atrial enlargement, pulmonic stenosis and ventricular septal defect are not commonly seen. (c) B. This patient has the signs and symptoms consistent with Marfan's syndrome. Ectopia lentis, aortic root dilation and aortic dissection are major criteria for the diagnosis of the disease. (u) C. See A for explanation. (u) D. See A for explanation.

Which of the following is the first-line treatment for a patient with mild syndrome of inappropriate secretion of ADH (SIADH)? A. Sodium supplementation to correct the hyponatremia B. Restriction of free water C. High volume hypertonic saline infusion D. Pituitary ablation via transsphenoidal approach

(u) A. Patients with SIADH will have euvolemic hyponatremia but this is a water problem and not a sodium problem. Supplementation of additional sodium only helps with water retention in the body and can produce edema but it is not valuable in the correction of this hyponatremia. (c) B. Restriction of free water intake is the first-line therapy for patients with euvolemic hyponatremia that is caused by SIADH. Water intake should be restricted to 0.5 to 1 liter per day. A gradual increase in serum sodium will occur over days with this treatment. (h) C. High volume hypertonic saline infusion can result from too rapid correction of hyponatremia by causing central pontine myelinolysis. (h) D. Patients with SIADH are treated medically in order to increase the serum sodium rather than ablation of the pituitary gland which is not indicated for this condition.

: (27) 61. Health Maintenance/Orthopedics/Rheumatology A 13 year-old patient presents to the orthopedic surgeon's office for a follow-up evaluation for scoliosis. The Cobb angle is measured at 65 degrees. Which of the following is indicated at this time? A. Observation for 4 months B. Physical therapy C. Leg length measurements D. Pulmonary function testing

(u) A. Patients with a Cobb angle less than 10 degrees can be managed by observation alone while patients with a Cobb angle less than 20 degrees are typically managed conservatively. (u) B. See D for explanation. (u) C. See D for explanation. (c) D. Patients with thoracic curves of 60 - 90 degrees, those with respiratory complaints, and those with scoliosis from a neuromuscular cause should undergo pulmonary function testing, particularly if surgery is being considered. These patients are at risk for cardiopulmonary compromise and a secondary restrictive lung disease.

In what region of the brain are cerebrovascular lesions most likely to cause post-event depressive symptoms? A. Cerebellum B. Occipital C. Pons D. Temporal

(u) A. See D for explanation. (u) B. See D for explanation. (u) C. See D for explanation. (c) D. Cerebrovascular lesions in the anterior brain regions are more common than the posterior areas. The temporal lobe and the diencephalon are especially prone to post event depression.

An 18 year-old female comes to the clinic with the complaint of increased vaginal discharge and vaginal odor. She also complains of urinary frequency. On physical examination there is evidence of thin, gray, frothy discharge in the vagina. The cervix appears erythematous and the vaginal pH is 6. Which of the following is the most likely diagnosis? A. Candida vaginitis B. Bacterial vaginosis C. Trichomonas vaginitis D. Chlamydia trachomatis

(u) A. Patients with a yeast infection most often present with vulvar/vaginal pruritus, burning, dyspareunia "cottage-cheese" discharge which is odorless, pH is often normal 4-4.5. (u) B. Symptoms from bacterial vaginosis include ivory to gray discharge, thin, homogeneous, adherent, often increased pH 5-6.5 (basic), distinctive "fishy" odor, itching may be present. Malodorous discharge is especially noticeable by the patient after menses or intercourse. (c) C. Signs of trichomonas include: thin frothy or bubbly, pale yellow-green to gray adherent vaginal discharge, can have erythema of vulva and vagina, may have petechiae on the cervix, amine odor may be present, may also complain of dysuria and dyspareunia, pH 5 to 6.5 (basic). (u) D. Patients with Chlamydia are often asymptomatic but may have mucopurulent discharge and cervical inflammation.

A 75 year-old male presents to the ER with the following stroke findings: right-sided hemiparesis (face and hand more affected than leg), homonymous hemianopsia of the right half of both visual fields, and aphasia. Where is the location of his stroke? A. Anterior cerebral artery B. Middle cerebral artery C. Posterior cerebral artery D. Internal carotid artery

(u) A. Patients with anterior cerebral artery stroke will have findings greater in the legs than hands. (c) B. This case is a description of a middle cerebral artery stroke. (u) C. Posterior cerebral artery stroke patients will have midbrain and thalamic or sensory findings. (u) D. Internal carotid artery stroke patients will have amaurosis fugax, visual disturbances and crossed symptoms.

A 69 year-old male presents with complaint of increasing dyspnea over the past 6-8 months. The patient denies cough, chest pain or smoking history. Physical examination reveals inspiratory crackles at the bases and clubbing of the nails. Chest x-ray reveals interstitial fibrosis of the lower lungs, thickened pleura and calcified pleural plaques of the lateral chest wall. Pulmonary function testing shows a restrictive pattern with a decreased diffusing capacity. What information is most likely noted in this patient's history? A. Coal mining B. Silica exposure C. Textile work D. Asbestos exposure

(u) A. Patients with coal miners pneumoconiosis are typically asymptomatic with unremarkable pulmonary function tests. CXR shows small opacities in the upper lungs. (u) B. Patients with a history of silica exposure are also asymptomatic and have unaffected pulmonary function tests. CXR shows small rounded opacities throughout the lung and calcified hilar lymph nodes. (u) C. Textile workers present with an asthma-like disorder with chest tightness, cough and dyspnea that is worse on the first day back to work and improves as the week goes on. (c) D. Asbestos exposure often presents years later with increasing dyspnea and interstitial fibrosis of the lower lungs, thickened pleura and calcified pleura plaques. They will have a restrictive pattern on PFT.

A 24 year-old male presents for routine physical examination. On physical examination, you find that the patient's upper extremity blood pressure is higher than the blood pressure in the lower extremity. Heart exam reveals a late systolic murmur heard best posteriorly. What is the most likely diagnosis in this patient? A. Hypertrophic obstructive cardiomyopathy B. Patent foramen ovale C. Coarctation of the aorta D. Patent ductus arteriosus

(u) A. Patients with hypertrophic obstructive cardiomyopathy do not present with hypertension or weak femoral pulses. (u) B. The murmur associated with patent foramen ovale is a systolic ejection murmur heard in the second and third intercostal spaces and patients do not present with hypertension. (c) C. Coarctation of the aorta commonly presents with higher systolic pressures in the upper extremities than the lower extremities and absent or weak femoral pulses. (u) D. Patent ductus arteriosus is rare in adults and patients are noted to have a continuous rough, machinery murmur.

Which of the following clinical signs or symptoms is most likely to be present following iatrogenic injury during subtotal thyroidectomy? A. Inability to shrug the shoulders B. Hoarseness C. Impaired gag reflex D. Impaired taste sensation anterior two-thirds of tongue

(u) A. Patients with injury to the spinal accessory (Cranial Nerve XI) will be unable to shrug the shoulders against resistance. This nerve is most likely to be injured during carotid endarterectomy and not subtotal thyroidectomy. (c) B. The recurrent laryngeal nerve is most commonly injured during subtotal thyroidectomy. If injured, hoarseness is the most common presentation for this nerve impairment. (u) C. Injury to the glossopharyngeal and vagus nerves are most likely to result in impaired gag reflex. These nerves are most likely to be injured during carotid endarterectomy and not during subtotal thyroidectomy. (u) D. The facial nerve (Cranial Nerve VII) provides taste to the anterior two-thirds of the tongue. It is not at risk for injury during subtotal thyroidectomy.

Which of the following is the most common presentation for an elderly female patient with primary hyperparathyroidism? A. Abdominal pain B. Renal lithiasis C. Acute pancreatitis D. Asymptomatic

(u) A. Patients with primary hyperparathyroidism may have increased calcium levels which result in abdominal pain and constipation but this is not the most common manifestation. (u) B. Patients with primary hyperparathyroidism may have renal lithiasis as a result of the hypercalcemia but it is not the most common presentation. (u) C. Patients with acute pancreatitis may have hypocalcemia as a result of the calcium soaps that are formed in order to help to process the food that is eaten. It is not related to hyperparathyroidism; rather it is most commonly caused by gallstones and alcoholism. (c) D. Patients with primary hyperparathyroidism are most commonly found to have this disease by an incidental finding of hypercalcemia on routine laboratory testing as a result of screening. Up to 0.1% of the adult population has this condition which is most commonly seen in females over age 50.

A patient presents with loss of pigmentation on the back of hands, face, and body folds due to the absence of epidermal melanocytes. There has been improvement with PUVA treatment. Which of the following is the most likely diagnosis? A. Pityriasis alba B. Tinea versicolor C. Vitiligo D. Melasma

(u) A. Pityriasis alba is caused by dermal inflammation that becomes scaly and hypopigmented. (u) B. Tinea versicolor is a fungal infection of the skin presenting as oval or circular lesions (c) C. Vitiligo is the acquired loss of pigmentation due to the absence of epidermal melanocytes presenting on the back of hands, face, or body folds. (u) D. Melasma is an acquired brown hyperpigmentation involving the face and neck in women during their second or third trimester of pregnancy.

A 23 year-old male comes to the office with an intensely itchy rash, especially at night for the past week. His roommates have similar symptoms but have not been evaluated. On physical examination, the wrists, web spaces of the hands, and axillae have thin linear markings, tiny vesicles, and excoriations. Which of the following is the most likely diagnosis? A. Pediculosis B. Folliculitis C. Impetigo D. Scabies

(u) A. Pediculosis is a lice infection of the scalp, trunk or pubic region. Body louse occurs in persons living in overcrowded residences, and is characterized by intense pruritis with resultant deep excoriation of the neck, shoulders, and upper back. Nits are identifiable on hair shafts. (u) B. Folliculitis is an infection of the upper hair follicles characterized by itching, burning and pustules in the hairy regions of the body. (u) C. Impetigo is a contagious infection of the skin caused by staphylococci or streptococci characterized by honey-crusted erythematous erosions of the face or other exposed areas of the body. (c) D. Scabies is an infestation of the mite Sarcoptes scabiei. It is usually spread by skin-to-skin contact. Scabies is characterized by intense pruritic burrows, vesicles and excoriations of the finger webs, wrist creases and axillae.

A decrease in the fetal heart rate (FHR) occurring late during contractions is noted. The FHR returns to the baseline slowly after the uterine contraction. The physician assistant should be alerted to the possibility of A. pelvic dystocia. B. precipitous labor. C. fetal head compression. D. placental insufficiency.

(u) A. Pelvic dystocia, particularly that due to small bony architecture, is the most common cause of passage abnormalities and is not directly associated with FHR decelerations. (u) B. This refers to the length of labor, not decelerations in FHR. (u) C. The drop in FHR is caused by an interference with uterine blood flow to the intervillous space causing an early, not late, deceleration. (c) D. Placental insufficiency is the probable cause of fetal distress resulting in late decelerations.

A 24 year-old sexually active woman presents to the clinic complaining of dysmenorrhea, dyspareunia and backache that occurs premenstrually. Pelvic examination shows pain upon uterine motion and uterosacral nodularity in the posterior vaginal fornix. The definitive diagnosis for this patient requires which of the following? A. Pelvic ultrasound B. CT of the abdomen C. Laparoscopy with biopsy D. CA-125 test

(u) A. Pelvic ultrasound has little value in diagnosing endometriosis. (u) B. CT of the abdomen has little value in diagnosing endometriosis. (c) C. Diagnosis of endometriosis must be made by direct visualization. Laparoscopy with biopsy is the most appropriate diagnostic study in this patient. (u) D. CA-125 may be elevated in endometriosis, but it is not diagnostic.

Which of the following mechanisms leads to a primary pneumothorax? A. Penetrating or blunt trauma forces B. Underlying lung cancer C. Pressure of air in the pleural space exceeds room air pressure D. Rupture of subpleural apical blebs due to high negative intrapleural pressures

(u) A. Penetrating or blunt trauma force is responsible for a traumatic pneumothorax. (u) B. A pneumothorax that results from an underlying lung disease is classified as a secondary pneumothorax. (u) C. When pressure of air in the pleural space exceeds room air pressure, it leads to a tension pneumothorax. (c) D. A primary spontaneous pneumothorax is thought to result from a rupture of subpleural apical blebs secondary to high negative intrapleural pressures.

Which of the following treatments is used to reduce the hepatic complications of hemochromatosis? A. Penicillamine B. Paracentesis C. Albumin D. Phlebotomy

(u) A. Penicillamine is a chelating agent that may be used in the treatment of Wilson's disease in which there is a deficiency of the copper-binding protein ceruloplasmin resulting in impaired copper excretion into the bile. (u) B. Paracentesis may be performed in patients with tense ascites which can symptomatically improve the patient but does not alter long-term prognosis. (u) C. Patients with long-standing liver failure have decreased albumin levels but this is not a treatment normally performed for hemochromatosis. (c) D. Hemochromatosis is an autosomal recessive inherited disorder that causes cirrhosis, diabetes, and bronze pigmentation to the skin due to the abnormal accumulation of iron in tissues. Intensive phlebotomy is the treatment of choice until the iron overload is corrected.

A 20 year-old presents 30 minutes after being struck by a hockey puck in the mouth. On physical examination a central incisor is missing from its socket. The patient has the tooth wrapped in tissue paper and the root appears intact. Which of the following is the most appropriate next step in the treatment of this patient? A. Administration of IM penicillin B. No treatment is warranted C. Place tooth in saline and refer to plastic surgery for reimplantation D. Immediately reimplant the tooth and refer to an oral surgeon

(u) A. Penicillin is not indicated for treatment of an avulsed tooth. (h) B. Reimplantation is warranted as this is a permanent tooth with root intact. Primary teeth are never reimplanted. (u) C. See D for explanation. (c) D. Avulsed permanent teeth should be cleansed, transported in Hanks solution or saline and reimplanted by an oral surgeon within one hour.

A 32 year-old male with history of tobacco abuse presents with an intermittent burning sensation in his chest for six months, worsening over the past 2 weeks. His wife has noticed episodes of coughing at night. He denies dysphagia, weight loss, hematemesis, or melena. His vital signs are all normal and physical examination is unremarkable. Which of the following is the most likely diagnosis? A. Peptic ulcer disease B. Acute gastritis C. Gastroesophageal reflux disease D. Esophageal stricture

(u) A. Peptic ulcer disease typically presents with midepigastric pain and is not usually associated with cough. (u) B. Acute gastritis would have a duration of less than 2 weeks. (c) C. Gastroesophageal reflux disease presents with at least weekly episodes of heartburn and typically occurs after meals and upon reclining. Patients may complain of regurgitation, chronic cough, laryngitis, or sore throat. (u) D. Esophageal stricture develops from long term gastroesophageal reflux disease. Patients with esophageal stricture usually have dysphagia.

A patient presents with abdominal pain in the right lower quadrant, examination reveals increased pain in the right lower quadrant on deep palpation of the left lower quadrant. This commonly known as which of the following? A. Psoas sign B. Murphy's sign C. Rovsing's sign D. Obturator sign

(u) A. Psoas sign is right lower quadrant pain with right leg extension. (u) B. Murphy's sign is seen in liver and gallbladder disease in which the patient abruptly halts deep inspiration due to discomfort as the examiners hand applies pressure to the right upper quadrant. (c) C. A positive Rovsing's sign can be elicited in a patient with appendicitis when increased pain occurs in the right lower quadrant upon palpation of the left lower quadrant. (u) D. Obturator sign is right lower quadrant pain with internal rotation of the hip.

A 52 year-old male with history of hypertension and hyperlipidemia presents with an acute myocardial infarction. Urgent cardiac catheterization is performed and shows a 90% occlusion of the left anterior descending artery. The other arteries have minimal disease. Ejection fraction is 45%. Which of the following is the treatment of choice in this patient? A. Coronary artery bypass grafting (CABG) B. Streptokinase C. Percutaneous coronary intervention (PCI) D. Warfarin (Coumadin)

(u) A. Percutaneous coronary intervention is a better, less invasive alternative to CABG for single vessel coronary artery disease. (h) B. Streptokinase is not commonly used for treatment of acute myocardial infarction because it is ineffective at opening the occluded artery and reducing mortality. Streptokinase would be harmful because it would increase the risk of bleeding. (c) C. Immediate coronary angiography and primary percutaneous coronary intervention have been shown to be superior to thrombolysis. (u) D. Warfarin is used to prevent thrombosis and not for acute treatment.

radiates to his shoulders and is relieved with sitting forward. The patient admits to recent upper respiratory symptoms. On examination vital signs are BP 126/68, HR 86, RR 20, temp 100.3 degrees F. There is no JVD noted. Heart exam reveals regular rate and rhythm with no S3 or S4. There is a friction rub noted. Lungs are clear to auscultation. EKG shows diffuse ST segment elevation. What is the treatment of choice in this patient? A. Pericardiocentesis B. Nitroglycerin C. Percutaneous coronary intervention D. Indomethacin (Indocin)

(u) A. Pericardiocentesis is the treatment of choice in a patient with a pericardial effusion and cardiac tamponade, there is no evidence of either of these in this patient. (u) B. Nitroglycerin is indicated in the treatment of chest pain related to angina. (u) C. Percutaneous coronory intervention is the treatment of choice in a patient with an acute myocardial infarction. (c) D. Indomethacin, a nonsteroidal anti-inflammatory medication, is the treatment of choice in a patient with acute pericarditis.

An African American male is placed on hydroxychlorquine (Plaquenil) for travel to Africa. Within six weeks he presents complaining of fatigue. CBC shows anemia. Which of the following is the diagnostic study of choice in this patient? A. Peripheral smear B. G6PD assay C. Total iron binding capacity D. Hemoglobin electrophoresis

(u) A. Peripheral smear may reveal "bite" cells suggestive of G6PD, but is not diagnostic. (c) B. G6PD assay is the diagnostic study of choice for G6PD deficiency. (u) C. TIBC would be indicated in the evaluation of iron deficiency anemia, and is not indicated in the evaluation of G6PD deficiency. (u) D. Hemoglobin electrophoresis is indicated in the evaluation of hemolytic anemia and is not indicated in the evaluation of G6PD deficiency.

: (6) 136. Diagnosis/Psychiatry/Behavioral Medicine You are evaluating a 21 year-old male patient with a history of drug problems and multiple arrests. He has had many jobs in the past, but does not keep them long, due to repetitive fights at work. He has a long history of violating and disregarding the rights of others. Which of the following is the most likely personality disorder in this patient? A. Borderline B. Antisocial C. Avoidant D. Narcissistic

(u) A. Persons with borderline personality disorder have an unstable self-image. They have chronic feelings of emptiness and fear abandonment. (c) B. Antisocial personality disorder patients often disregard the rights and feelings of others. They often engage in criminal behavior. (u) C. People with avoidant personality disorder are socially inhibited and fear being disliked or criticized. (u) D. Persons with narcissistic personality disorder feel that they are special but are very insecure about themselves. They have a great need for admiration and have a sense of entitlement.

: (8) 51. Clinical Therapeutics/Cardiology A patient presents with complaints of headaches during strenuous exercise. Twenty-four hour ambulatory blood pressure readings indicate he is hypertensive with exercise. A chest radiograph reveals mild cardiomegaly. An echocardiogram shows an ejection fraction of 75% with interventricular septal hypertrophy. Which of the following is the most appropriate initial choice of medications for this patient? A. Phentolamine (Regitine) B. Metoprolol (Lopressor) C. Disopyramide (Norpace) D. Isoproterenol (Isuprel)

(u) A. Phentolamine is a nonselective alpha-adrenergic antagonist and is therefore not a first-line drug of choice. (c) B. Beta-blockers should be the initial drug in symptomatic individuals, especially when dynamic outflow obstruction is noted on the echocardiogram. Diuretics may also be required to decrease pre-load. (u) C. Disopyramide is an antiarrhythmic medication that may be used in conjunction with a beta-blocker, however is not first-line drug of choice. (h) D. Isuprel is a sympathomimetic beta adrenergic agonist that is structurally similar to adrenaline and therefore may exacerbate the symptoms.

What is the recommended treatment for absence (petit mal) seizures? A. Phenytoin (Dilantin) B. Carbamazepine (Tegretol) C. Ethosuximide (Zarontin) D. Gabapentin (Neurontin)

(u) A. Phenytoin is used to treat tonic clonic and partial seizures. (u) B. Carbamazepine is used to treat tonic clonic and partial seizures. (c) C. Ethosuximide, valproic acid, and clonazepam are recommended treatments for absence seizures. (u) D. Gabapentin is used to treat partial seizures.

A 36 year-old male presents for follow-up of his hypertension. The patient is currently on three anti-hypertensive medications without improvement of his blood pressure. On examination his BP is 170/86mmHg and his HR is 60bpm and regular. His heart examination reveals a regular rate and rhythm without S3, S4 or murmur and his lungs are clear to auscultation bilaterally. Abdominal examination reveals a bruit over his left upper abdomen. Which of the following is the most likely underlying etiology for this patient's hypertension? A. Pheochromocytoma B. Renal artery stenosis C. Cushing syndrome D. Coarctation of the aorta

(u) A. Pheochromocytoma is an uncommon cause of hypertension characterized by paroxysms of headache, sweating and palpitations. There are no bruits associated with pheochromocytoma. (c) B. Renal artery stenosis is characterized by hypertension that is resistant to three or more medications and renal artery bruits on examination. (u) C. Cushing syndrome is characterized by "moon" facies, a buffalo hump, a protuberant abdomen, weakness and headache. There are no renal artery bruits associated with Cushing syndrome. (u) D. Coarctation of the aorta is associated with hypertension in the upper extremities and normal or low blood pressure in the lower extremities. There are often weak femoral pulses and a late systolic ejection murmur or associated aortic insufficiency murmur.

A 15 year-old male presents acutely to the office. His legs are cool to the touch. Examination reveals that his pulses and blood pressure are higher in the upper extremities than the lower extremities. Femoral pulses are delayed and weakened. Which of the following is the most likely underlying diagnosis? A. Pheochromocytoma B. Conn's Syndrome C. Cushing's Syndrome D. Coarctation of the aorta

(u) A. Pheochromocytoma is most commonly associated with palpitations and feelings of warmth along with episodic (later sustained) hypertension. (u) B. Conn's Syndrome, also known as primary hyperaldosteronism, is found in a patient with hypertension who has unprovoked hypokalemia. (u) C. Cushing's Syndrome is associated with hypertension but is associated with the typical appearance of sustained elevated cortisol levels such as purple striae, buffalo hump, and central obesity. (c) D. Coarctation of the aorta typically has narrowing of the aorta proximal to the left subclavian artery with resultant high blood pressure in the upper extremities and decreased run off to the lower extremities following this narrowed segment.

A 72 year-old female is being evaluated for recurrent kidney stones. Physical examination reveals no abnormal findings. Laboratory findings show elevated calcium and decreased phosphate levels. Which of the following is the most likely diagnosis? A. pheochromocytoma B. adrenal insufficiency C. hyperparathyroidism D. vitamin D deficiency

(u) A. Pheochromocytoma may lead to hypercalcemia but the patient does not have any signs or symptoms suggestive of pheochromocytoma, such as hypertension, headache, profuse sweating, or weight loss. (u) B. Adrenal insufficiency, Addison's disease, would reveal, in addition to the hypercalcemia, anorexia, nausea and vomiting, weight loss, and cutaneous hyperpigmentation, none of which are evident in this patient. (c) C. The majority of patients with hyperparathyroidism are asymptomatic. Recurrent nephrolithiasis may be one of the presentations of primary hyperparathyroidism. Measurement of parathyroid levels would be the initial laboratory test for the evaluation of hypercalcemia. (a) D. Vitamin D deficiency leads to hypocalcemia.

A 46 year-old female is being evaluated for a new-onset hypertension that was discovered on screening at her workplace. The patient had several readings revealing systolic and diastolic hypertension. Patient is currently on no medications. Physical examination is unremarkable. A complete laboratory evaluation revealed hypokalemia as the only abnormality. Which of the following is the most likely diagnosis for this patient? A. pheochromocytoma B. renal artery stenosis C. coarctation of the aorta D. primary aldosteronism

(u) A. Pheochromocytoma will result in an increase in the production and release of catecholamines, which results in an increase in urinary metanephrines on testing. (u) B. Renal artery stenosis is identified by an abnormal radionuclide uptake on the affected kidney. (u) C. Coarctation of the aorta is identified by delayed and weakened femoral pulses along with a blood pressure in the lower extremities significantly lower than in the upper extremities. (c) D. Primary aldosteronism has an increased aldosterone secretion, which causes the retention of sodium and the loss of potassium. This should be the primary consideration for this patient.

A 36 year-old female comes to the office because a mole on her left calf has changed. On physical examination of the left posterior lower leg, there is a 12 mm, asymmetrical, variegated blue-black macule with raised pink plaque in the upper half of the lesion. Which of the following is the most appropriate clinical management of this lesion? A. Cryosurgery B. Topical retinoids C. Excisional surgery D. Topical chemotherapy

(u) A. Physical destruction of suspected melanoma should never be used because histologic verification of the diagnosis cannot be performed. (u) B. Chronic use of topical retinoids may be effective for superficial solar keratoses. (c) C. Surgical excision of suspected melanoma is necessary for histologic diagnosis and treatment of the lesion. (u) D. Topical chemotherapy, such as 5-Fluorouracil, is an effective treatment for squamous cell carcinoma.

Which of the following physical exam findings is consistent with moderate emphysema? A. Increased tactile fremitus B. Dullness to percussion C. Distant heart sounds D. Deviated trachea

(u) A. Physical examination findings in emphysema include a midline trachea, diffuse hyperresonant to percussion, and decreased tactile fremitus. (u) B. See A for explanation. (c) C. Distant heart sounds are common in emphysema patients due to hyperinflation of the lungs. (u) D. See A for explanation.

Which of the following is a physical examination finding that is consistent with a diagnosis of lobar pneumonia? A. Resonant to percussion B. Late inspiratory crackles C. Decreased tactile fremitus D. Tracheal shift toward uninvolved side

(u) A. Physical examination findings that are consistent with a diagnosis of lobar pneumonia include dullness to percussion, increased tactile fremitus and a trachea that is midline. (c) B. Late inspiratory crackles are a physical examination finding that is consistent with lobar pneumonia. (u) C. See A for explanation. (u) D. See A for explanation.

A 30 year-old male has a history of weakness without pain on the left side of his face for the past 4 days. Physical examination of the face reveals unilateral weakness to the left side, but not complete paralysis. The left eye does not close completely. The most appropriate initial treatment is to A. begin physical therapy. B. refer for surgical ablation. C. reassure patient and provide patient education. D. initiate high dose steroids and recheck in 24 hours.

(u) A. Physical therapy is not indicated for Bell's palsy (u) B. Surgery has not been shown to provide any significant benefit in the treatment of Bell's palsy. (c) C. Bell's palsy is usually a self-limited condition and typically resolves within a few weeks. (u) D. Corticosteroids may be used if paralysis is complete or there is significant pain, but are not first-line.

A 32 year-old male presents with a one-month history of weakness, anorexia, and weight loss. On physical examination, he is hypotensive and his skin has diffuse hyperpigmentation. Plasma cortisol levels drawn at 8 am are low. Which of the following is the most likely diagnosis? A. Pituitary insufficiency B. Addison disease C. Cushing syndrome D. Primary aldosteronism

(u) A. Pituitary insufficiency presents with hypopigmentation, not hyperpigmentation. (c) B. This is the classic presentation of Addison's disease (chronic adrenocortical insufficiency). (u) C. Hypercortisolism (Cushing syndrome) would present with central obesity, hypertension, and elevated cortisol levels. (u) D. Weakness is a symptom of primary aldosteronism, but the patient would be hypertensive and aldosterone levels would be elevated.

: (27) 178. History & Physical/Orthopedics/Rheumatology A 17 year-old female presents to the ED after injuring her knee in a soccer match earlier today. The injury occurred while suddenly decelerating and cutting to run in another direction. She heard a pop in the knee and complains the knee now gives out with any twisting motion. Which of the following tests is the most useful in eliciting the most likely diagnosis? A. Posterior drawer test B. Lachman test C. McMurray test D. Valgus stress test

(u) A. Posterior cruciate ligament injuries are assessed with the posterior drawer test. (c) B. Noncontact anterior cruciate ligament (ACL) injuries occur with deceleration and rotational injury while running, cutting or jumping. Contact ACL injuries involve hyperextension and/or valgus forces from a direct blow to the knee. The Lachman test is considered the most valuable test to assess an ACL injury. (u) C. The McMurray test evaluates meniscal injuries. (u) D. The valgus stress test is used to determine medial collateral ligament injuries.

A 56 year-old male with a 30 pack-year smoking history presents with substernal chest pain. The pain is described as a pressure that radiates to his jaw. The pain has lasted consistently for 30 minutes with variable relief. His current medications include atorvastatin (Lipitor) and glyburide (Micronase). Which of the following aggravating or relieving factors about the pain would support the diagnosis? A. Precipitated by exercise and relieved with rest B. Unrelieved by nitroglycerin C. Aggravated by deep breaths D. Relieved with food

(u) A. Precipitated by exercise and relief with rest is consistent with angina. (c) B. Patient is having a myocardial infarction which is unrelieved by rest or nitroglycerin. (u) C. Aggravated by deep breaths suggests pericarditis. (u) D. Relieved with food suggests peptic ulcer disease.

A 43 year-old female presents with a two year history of frequent episodes of pain and morning stiffness in both hands and wrists. She experiences some symptomatic relief with ibuprofen, but feels that the episodes are becoming more frequent and severe. On examination, you observe joint swelling of several MCP joints on both hands. X-ray of the hands shows joint space narrowing of the MCP joints. In addition to NSAIDs, what is the most appropriate first-line long-term medication to treat this patient? A. Prednisone (Deltasone) B. Infliximab (Enbrel) C. Methotrexate (Rheumatrex) D. Sulfasalazine (Azulfidine)

(u) A. Prednisone can produce impressive clinical improvement in rheumatoid arthritis but the side effects associated with their long-term use limit their utility. (u) B. Tumor necrosis factor inhibitors, such as infliximab, are very effective disease modifying antirheumatic drugs (DMARDS). This class of medication is inappropriate as a first-line medication due to very high cost. (c) C. Methotrexate is the standard first-line medication in the treatment of rheumatoid arthritis. This DMARD is generally effective, well-tolerated, and affordable. (u) D. Sulfasalazine is inexpensive but associated with potentially serious hematologic side effects and is considered a second-line DMARD.

Which of the following is a known complication to prescribing excessive doses of levothyroxine (Synthroid) for patients with hypothyroidism? A. Primary hyperparathyroidism B. Papillary carcinoma of the thyroid gland C. Osteoporosis D. Diabetes Insipidus

(u) A. Primary hyperparathyroidism is not related to excessive doses of levothyroxine. (u) B. Papillary thyroid cancer is the most common thyroid cancer. It is most closely associated with radiation exposure. Levothyroxine is typically given after surgical treatment in order to suppress any thyroid activity in any remaining thyroid tissue. (c) C. Osteoporosis may result from overaggressive therapy with levothyroxine (Synthroid) because of the increased bone turnover that results from increased basal metabolic rate. (u) D. Diabetes insipidus results from lack of production of ADH in the brain or from the kidney not responding to ADH in the circulation. It is not related to levothyroxine therapy.

A 64 year-old male presents complaining of new onset of fatigue, weight gain, constipation, erectile dysfunction, and loss of body hair. Laboratory investigation demonstrates: TSH less than 0.5 microunits/mL (normal range 0.5-5.0 microU/mL); Thyroxine (T4) 2 mcg/dL (normal range 5-12 mcg/dL); Prolactin 10 nanograms/ml (normal A. Primary hypothyroidism B. Excessive dosing of levothyroxine (Synthroid) C. Hypopituitarism D. Subacute thyroiditis

(u) A. Primary hypothyroidism is usually associated with an elevated TSH. (u) B. Excessive replacement of levothyroxine would result in symptoms of hyperthyroidism, not hypopituitarism (c) C. The low trophic and target hormone levels combined with symptoms of hypogonadism indicate this patient has hypopituitarism. (u) D. The T4 level would be elevated in subacute thyroiditis due to excessive release of thyroid hormone.

Which of the following medications is the treatment of choice for patients with chronic gout to prevent recurrence of symptoms during its quiescent phase? A. Probenecid (Benemid) B. Allopurinol (Zyloprim) C. Colchicine D. Indomethacin (Indocin)

(u) A. Probenecid is a uricosuric medication that helps to increase the excretion of uric acid but it does not prevent the formation of uric acid making it less beneficial in chronic gout therapy. It is also not effective in patients with chronic renal disease. (c) B. Allopurinol is the best drug to lower serum urate in overproducers, stone formers, and patients with advanced renal failure. It is a xanthine oxidase inhibitor that is used to prevent the formation of uric acid. (u) C. Colchicine treatment is recommended only in patients who have tophaceous deposits in the skin and is used in the acute rather than the chronic setting of gout. (u) D. Indomethacin is used in the acute management of gout but is not effective in decreasing monosodium urate deposition in the joints.

: (7) 188. Diagnostic Studies/ENT/Ophthalmology A 34 year-old male presents for evaluation after being struck in the left eye by a tennis ball. He reports pain about the eye and numbness of the ipsilateral cheek. The examination is significant for enophthalmos and diplopia noted on upward gaze. Which of the following is the preferred diagnostic test to assess this injury? A. Orbital radiographs B. Sinus radiographs C. MRI of the facial bones D. CT imaging of the orbits

(u) A. Radiographs may be used initially to determine obvious bony abnormalities; however, CT imaging provides more detailed evaluation of the soft tissues of the orbit. (u) B. See A for explanation. (u) C. MRI has not been used regularly due to limited discrimination of bony structures. (c) D. Axial and coronal CT imaging of the orbits provides the most detailed evaluation of the bony and soft tissue structures of the orbits.

A 42 year-old male with a history of constipation presents with complaints of severe pain with defecation described as feeling like he is "tearing apart." He has also noted occasional small amounts of blood on toilet paper. External examination of the rectum is unremarkable and an internal rectal exam cannot be performed due to severe pain when attempted. Which of the following is the most likely diagnosis? A. Proctitis B. Anal fissure C. Rectal prolapse D. Internal hemorrhoids

(u) A. Proctitis is usually caused by anorectal infections that produce symptoms of anorectal discomfort, tenesmus, constipation, and discharge from the rectum. (c) B. Anal fissures are easily diagnosed from history alone with the classic finding of severe pain upon defecation. Constipation is also a common cause of the trauma that leads to development of a fissure. (u) C. Rectal prolapse is commonly seen in elderly females with complaints that include an anal mass, rectal bleeding and a change in bowel habits. (u) D. While internal hemorrhoids may cause rectal bleeding, tearing pain is an uncommon complaint unless there is evidence of thrombosis of irreducible tissue.

A 21 year-old obese woman complains of menstrual irregularity since menarche at age 17. She is 5'5" and weighs 180 pounds. Exam of her face reveals excessive hair growth as well as acne. Her abdomen shows midline hair growth and truncal obesity. A previous pelvic ultrasound shows many small fluid filled ovarian cysts bilaterally. Which of the following is the most appropriate diagnostic study to make the initial diagnosis in this patient? A. Prolactin level B. Endometrial biopsy C. Free testosterone D. Thyroid stimulating hormone

(u) A. Prolactin level will not be elevated in a patient with polycystic ovarian syndrome. (u) B. Endometrial hyperplasia occurs secondary to anovulation, endometrial biopsy is mandatory for follow-up management but is not indicated at diagnosis. (c) C. Hyperandrogenism, as evidenced by elevated free testosterone, supports the diagnosis of polycystic ovarian syndrome. (u) D. An increase of TSH is suggestive of hypothyroidism.

Which of the following ECG findings is consistent with hyperkalemia? A. prolonged QT interval B. delta wave C. peaked T waves D. prominent U waves

(u) A. Prolonged QT interval is seen in hypocalcemia. (u) B. Delta wave is a sign of ventricular preexcitation seen in Wolf-Parkinson-White (WPW) Syndrome. (c) C. Narrowing and peaking of T waves are the beginning EKG changes associated with hyperkalemia. (u) D. Prominent U waves are a sign of prolonged ventricular repolarization seen in hypokalemia.

A 22 year-old female presents to the emergency department with rapid heart rate. She appears quite thin and dehydrated. She denies that she is thin, stating "I am so fat that I can hardly stand myself! That is why I exercise every day." She runs twelve to fifteen miles a day, and on weekends also bicycles forty to fifty miles. Her LMP was six months ago. On exam, she is 5' 6" tall and weighs 98 pounds. Temp 98 degrees F, pulse 100, respirations 18, BP 98/60. EKG shows sinus tachycardia. Laboratory findings include Na 138 mEq/L, K 2.8 mEq/L, Cl 91 mEq/L, BUN 35 mg/dL, Creatinine 1.1 mg/dL. Which of the following is the next most appropriate treatment? A. Propylthiouracil (PTU) and individual psychotherapy B. Hormone replacement and cognitive therapy C. Weight restoration and family therapy D. IV hydration and antidepressant therapy

(u) A. Propylthiouracil is used to treat hyperthyroidism, not anorexia nervosa. (u) B. Hormone replacement is not indicated for the amenorrhea of anorexia nervosa, but nutritional support may help. (c) C. Anorexia nervosa requires a comprehensive, multidisciplinary approach to treatment that integrates medical management, individual psychotherapy, and family therapy. Currently, the best results have been shown with weight restoration accompanied by family therapy for patients with adolescent-onset anorexia nervosa and individual therapy for patients with onset after 18 years of age. Inpatient treatment is often required. (u) D. Antidepressant therapy may be useful in bulimia nervosa.

: (28) 52 127. Clinical Intervention/Cardiology A 34 year-old male presents with an acute onset of fatigue and dyspnea. He has experienced repeated episodes of near-syncope and an unresolved chest discomfort described as a "fluttering" sensation over the past 3 hours. His electrocardiogram reveals no definable p waves and his R-R interval is irregular. His blood pressure is 88/60 mmHg. Which of the following is most appropriate for this patient? A. Initiate warfarin (Coumadin) therapy to an INR target of 2.0 B. Consult for radiofrequency ablation therapy C. Transfer to cardiac catherization lab D. Sedate for synchronized cardioversion

(u) A. Providing the patient is hemodynamically stable, management consists of rate control and anticoagulation. This patient's blood pressure and symptoms indicate he is not stable. (u) B. Radiofrequency ablation would be indicated for the treatment of aberrant pathways as is seen in Wolff- Parkinson-White syndrome. The description of the ECG in this scenario is of atrial fibrillation. (u) C. Heart catheterization would be indicated for evidence of myocardial ischemia or ST elevation MI. This scenario represents a patient in atrial fibrillation. (c) D. The patient has symptoms and ECG findings consistent with atrial fibrillation. Urgent cardioversion is indicated with shock or severe hypotension, pulmonary edema, or ongoing myocardial infarction or ischemia.

The initial manifestation of acute pancreatitis is often A. generalized pruritus. B. epigastric pain. C. epigastric mass. D. anorexia.

(u) A. Pruritus presents as a symptom only when acute pancreatitis occurs as a result of obstruction of the distal common bile duct. (c) B. The typical findings in acute pancreatitis include nausea, vomiting, and abdominal pain. Epigastric pain, generally abrupt in onset, is steady and severe. (u) C. Epigastric mass is more common in pancreatic cancer. (u) D. Anorexia is a common finding with chronic pancreatitis, but uncommon with acute pancreatitis.

Which of the following osteoporosis therapies is so poorly absorbed that it must be taken alone and on an empty stomach? A. raloxifene (Evista) B. alendronate (Fosamax) C. calcitonin (Miacalcin) D. teriparatide (Forteo)

(u) A. Raloxifene, calcitonin, and teriparatide lack significant drug-drug or drug-food interactions. (c) B. Fosamax should not be coadministered with any other medication or food. (u) C. See A for explanation. (u) D. See A for explanation

: (3) 117. Scientific Concepts/Orthopedics/Rheumatology A 40 year-old female presents with a history of chronic musculoskeletal pain and chronic fatigue unrelieved by rest. Palpation produces pain in 11 of the 18 paired trigger points. Which of the following correctly identifies the pathophysiology of her suspected diagnosis? A. An underlying psychiatric diagnosis creating the perception of pain B. Abnormal sensory processing of pain in the central nervous system C. A primary sleep disorder creating nonrestorative sleep followed by pain D. Muscular inflammation due to overuse

(u) A. Psychiatric disorders are common in fibromyalgia but do not appear to be the cause of the disease. (c) B. Current evidence strongly suggests that the pathophysiology of fibromyalgia is the result of abnormal sensory processing of pain in the central nervous system. (u) C. Patients with fibromyalgia most likely have nonrestorative sleep due to a specific brain wave pattern that does not allow them to reach stages 3 and 4 of the sleep cycle. Symptom exacerbation is typically subsequent to a night of poor sleep, but the lack of sleep is not the cause of the pain. (u) D. See B for explanation.

A 22 year-old male received a stab wound in the chest an hour ago. The diagnosis of pericardial tamponade is strongly supported by the presence of A. pulmonary edema. B. wide pulse pressure. C. distended neck veins. D. an early diastolic murmur.

(u) A. Pulmonary edema may result with low output states as seen with myocardial contusions, but it is not strongly suggestive of tamponade. (u) B. Wide pulse pressure is seen in conditions of high stroke volume such as aortic insufficiency or hyperthyroidism. Narrow pulse pressure is seen with cardiac tamponade. (c) C. Cardiac tamponade will manifest with distended neck veins and cold clammy skin. (u) D. The onset of diastolic murmur is suggestive of valvular disease, not tamponade.

A 60 year-old female with history of radiation therapy for the treatment of cancer presents with progressive dyspnea and fatigue. On examination the patient has lower extremity edema, significant ascites, and an elevated jugular venous pressure that does not fall with inspiration. Heart examination reveals a pericardial knock. Echocardiogram shows rapid early filling and reduced mitral inflow velocities with inspiration. Which of the following is the most likely diagnosis in this patient? A. Pulmonary hypertension B. Atrial myxoma C. Constrictive pericarditis D. Tako-Tsubo cardiomyopathy

(u) A. Pulmonary hypertension is usually associated with chest pain, dyspnea, fatigue and syncope. Examination would reveal a narrow splitting of S2 with a loud pulmonic component. Echocardiogram would show increased pulmonary artery pressures, right ventricular enlargement and possibly paradoxical motion of the intraventricular septum. (u) B. Atrial myxoma is characterized by fever, weight loss, malaise, embolization, and a diastolic sound or murmur (tumor plop) on examination. Atrial myxoma would be seen on echocardiogram. (c) C. Constrictive pericarditis is associated with TB, radiation therapy, cardiac surgery, or following viral pericarditis. There is evidence of right-sided heart failure, a positive Kussmaul sign, and a septal bounce and reduced mitral inflow velocities with inspiration on echocardiogram. (u) D. Tako-Tsubo cardiomyopathy (broken heart syndrome) is commonly seen in postmenopausal women who experience signs and symptoms of acute coronary syndrome. Echocardiogram shows left ventricular apical dyskinesia.

A 64 year-old female with a 50 pack year smoking history, presents with worsening dyspnea on exertion, a persistent cough, and increasing oxygen requirement from 2 to 3 liters. She denies any cardiac history. What is the most likely chest x-ray finding in this patient? A. pulmonary vascular congestion B. left lower lobe infiltrate C. apical infiltrates D. hyperinflation with bullae

(u) A. Pulmonary vascular congestion represents congestive heart failure not COPD. (u) B. Left lower lobe infiltrate represent an infectious process, such as pneumonia. (u) C. Apical infiltrates represent an infectious process, such as tuberculosis. (c) D. Hyperinflation with bullae is a consistent finding in patients with emphysema, such as this patient.

A 2 month-old infant presents for a routine health maintenance visit. The mother has been concerned about the infant's hearing since birth. Physical examination reveals no apparent response to a sudden loud sound. Which of the following is the most appropriate diagnostic evaluation? A. audiometry B. tympanometry C. acoustic reflectometry D. auditory-evoked potentials

(u) A. Pure tone audiometry can be used to screen for hearing deficits in children over the age of 3 years. (u) B. Tympanometry is used to identify an effusion as the cause of hearing loss, but in infants over the age of months. (u) C. Acoustic reflectometry measures the spectral gradient of the tympanic membrane, but is not used clinically due to concerns about its reliability. (c) D. Brainstem auditory-evoked potentials evaluate the sensory pathway and identify the site of any anatomical disruption. The test does not require any active response from the patient and is useful in the evaluation of suspected hearing loss in an infant.

What is the pathologic mechanism of Hirschsprung's disease? A. Pyloric circular muscle hypertrophy causing gastric outlet obstruction B. Absence of ganglion cells in the mucosal and muscular layers of the colon C. A defect in the diaphragm leading to protrusion of the abdominal viscera into the thoracic cavity D. Absence of relaxation of the lower esophageal sphincter and lack of peristalsis in the esophageal body

(u) A. Pyloric circular muscle hypertrophy causing gastric outlet obstruction describes pyloric stenosis. (c) B. Hirschsprung's disease results from an absence of ganglion cells in the mucosal and muscular layers of the colon. (u) C. A defect in the diaphragm leading to protrusion of the abdominal viscera into the thoracic cavity describes congenital diaphragmatic hernia. (u) D. Absence of relaxation of the lower esophageal sphincter and lack of peristalsis in the esophageal body describes achalasia of the esophagus.

Which of the following medications decreases the exchange of hydrogen for potassium by inhibiting hydrogen, potassium-ATPase? A. Ranitidine (Zantac) B. Misoprostol (Cytotec) C. Sucralfate (Carafate) D. Omeprazole (Prilosec)

(u) A. Ranitidine is an H2 receptor antagonist. (u) B. Misoprostol is a prostaglandin analogue. (u) C. Sucralfate is a coating agent. (c) D. Omeprazole, a proton pump inhibitor, works by inhibiting hydrogen, potassium-ATPase.

A 72 year-old male presents to the emergency department with crushing chest pain, dyspnea and palpitations for 2 hours in duration. Enzymes are pending and he has been given aspirin and sublingual nitroglycerin. He is rushed to the catheterization lab where they find a totally occluded distal right coronary artery. Which of the following electrocardiogram (ECG) findings supports the diagnosis? A. Q waves in leads I, aVL, V5-V6 B. ST segment elevation in leads II, III, aVF C. Hyperacute T waves in leads I, aVL D. Flipped T waves with repolarization changes in leads V1-V4

(u) A. Q waves in leads I, aVL, V5-V6 represent infarction involving the circumflex artery. (c) B. ST segment elevation in leads II, III, aVF, represents an acute process in the right coronary artery. (u) C. Hyperacute T waves in leads I, aVL can represent the initial changes of an infarction involving the circumflex artery. (u) D. Flipped T waves with repolarization changes in leads V1-V4 can represent early stages of infarction involving the left anterior descending artery.

A 16 year-old female presents to the office with a rash. She was healthy until one week ago when she developed a fever, headache, generalized lymphadenopathy and a rash on her trunk. On physical examination you note an erythematous rash with central clearing. Her left knee is swollen with painful range of motion. She recently returned from Girl Scout Camp. Which of the following is the most likely diagnosis? A. Rocky Mountain spotted fever B. Lyme disease C. Juvenile rheumatoid arthritis D. Babesiosis

(u) A. RMSF usually presents in the early part of the disease with fever, headache, malaise, myalgia, and nausea and vomiting. This is often unable to be distinguished from other viral illnesses. The patient then becomes progressively more ill with vascular infection and injury occuring. (c) B. This presentation is classic for stage 2 (disseminated infection) Lyme disease. Commonly patients have skin involvement, headache, mild stiffness of the neck, fever, chills, migratory musculoskeletal pain, arthralgias, malaise, and fatigue. Lyme disease is the most common vector borne disease in the US and Europe. (u) C. Juvenile rheumatoid arthritis is a persistent non infectious arthritis lasting more than 6 weeks to 3 months after other possible etiologies have been ruled out. (u) D. Babesiosis is a tick borne illness that presents with the most common symptoms being fever, fatigue, malaise, sweats, shaking chills and myalgias.

A 20 year-old male presents with 3 weeks of constitutional and upper respiratory symptoms, including malaise, sore throat, dry cough, and fever. Lung auscultation demonstrates diffuse crackles bilaterally. What is the most likely infectious agent involved? A. Respiratory syncytial virus B. Influenza virus C. Mycoplasma pneumoniae D. Streptococcus pneumoniae

(u) A. RSV is characterized by wheezing on auscultation and the course is typically 3-7 days. (u) B. Influenza pneumonia is characterized by a more precipitous onset and fulminant course. (c) C. The indolent course suggests an atypical pneumonia and Mycoplasma is the most common atypical agent. (u) D. Pneumococcal pneumonia is typically characterized by a more severe illness and more fulminant course.

A 65 year-old female who recently had an anterior MI returns to clinic for follow-up six weeks after. She has no chest pain, but reports decreased exercise tolerance. Electrocardiogram (ECG) shows persistent ST elevation in leads V2-V4. Which of the following is the most likely diagnosis? A. Right ventricular infarction B. Re-occlusion of the right coronary artery C. Pericarditis D. Ventricular aneurysm

(u) A. RV infarction is present in one-third of patients who had an inferior wall MI and typically show ST elevation in V4 along with the inferior leads. (u) B. Occlusion of the right coronary artery would show ST elevation in the inferior ECG leads. (u) C. Pericarditis shows ST elevation in multiple leads with a history consistent of a viral illness or fever. (c) D. Persistent ST elevation in the leads where a previous or recent anterior MI occurred is most likely due to ventricular aneurysm.

: (13) 75. History & Physical/Orthopedics/Rheumatology A 45 year-old female secretary presents with complaints of numbness, tingling, and pain in the thumb, index, and third fingers. On examination the patient experiences numbness and tingling while holding her wrists in a flexed position for 60 seconds. This suggests compression of which of the following nerves? A. Radial B. Ulnar C. Median D. Brachial

(u) A. Radial neuropathy syndrome commonly occurs with lateral epicondylitis. Patients have pain over the brachioradialis, extensor carpi radialis longus, and extensor carpi radialis brevis muscles. On examination, patients have pain with resisted forearm supination and when simultaneously extending the wrist and fingers while the long finger is passively flexed. (u) B. Patients with ulnar nerve compression complain of numbness and tingling of the 4th and 5th digits. (c) C. Carpal tunnel syndrome results from compression of the median nerve that innervates the first three digits. It is associated with pregnancy, amyloidosis, flexor tenosynovitis, overuse phenomenon, acute or chronic inflammatory conditions, traumatic disorders of the wrist, diabetes mellitus, hypothyroidism, and tumors within the carpal tunnel. (u) D. Brachial plexus injury usually occurs after a fall on the shoulder. Symptoms include numbness of the affected arm.

What is the diagnostic modality of choice to diagnose cystic fibrosis (CF)? A. Chest radiograph B. Clinical features C. Sweat chloride concentration testing D. Genotyping

(u) A. Radiographic findings may suggest the diagnosis but are not specific. (u) B. While clinical features may suggest the need for testing they are not useful in confirming the diagnosis. (c) C. The standard for diagnosis is two positive sweat chloride concentration tests obtained on separate days or identification of CF mutations or an abnormal nasal potential difference measurement. (u) D. Genotyping screens for only a fraction of the known CF mutations.

A 2 year-old presents with sudden onset of cough and stridor. On examination the child is afebrile and appears nontoxic with a respiratory rate of 42 breaths per minute. What is the next step in the evaluation of this patient? A. Lateral soft tissue x-ray of the neck B. Indirect laryngoscopy C. Finger sweep D. Chest x-ray

(u) A. See D for explanation. (u) B. See D for explanation. (u) C. See D for explanation. (c) D. Chest x-ray should be done first when foreign body aspiration is suspected

A 55 year-old male with history of hypertension and diabetes mellitus presents to the emergency department. The patient's wife states that the patient developed progressive irritability and confusion today after complaining of a headache. Physical examination reveals a BP of 230/130 mmHg and papilledema. Which of the following is the most accurate diagnosis in this patient? A. Resistant hypertension B. Hypertensive urgency C. Hypertensive emergency D. Malignant hypertension

(u) A. Resistant hypertension is the failure to reach blood pressure control in patients who are compliant with a 3 drug regimen including a diuretic. (u) B. Hypertensive urgency is a systolic BP > 220 or a diastolic BP > 125 in a patient who is asymptomatic or who has disk edema, progressive target organ complications. Hypertensive urgency must be treated within a few hours of presentation. (u) C. Hypertensive emergency is similar to hypertensive urgency, however the BP is significantly elevated and must be lowered within an hour. (c) D. Malignant hypertension is significantly elevated BP with progressive retinopathy, including papilledema, encephalopathy, and headache.

Which of the following is a potential complication of a traumatic hyphema? A. retinal detachment B. glaucoma C. cataract formation D. chronic conjunctivitis

(u) A. Retinal detachment may occur from trauma but not from a hyphema. (c) B. If the trabecular network becomes obstructed from the hyphema then glaucoma may occur. (u) C. Cataracts may be caused by increasing age (most commonly), toxins, systemic disease, smoking, and hereditary, but not by hyphemas. (u) D. Chronic conjunctivitis is not known to be associated with hyphemas.

A two year-old male is brought into a free clinic with a fever of 102 degrees F for two days. Through an interpreter, the mother states that they are from Romania. Examination reveals that the buccal mucosa has a grainy appearance opposite the second molars. Which of the following is the most likely diagnosis? A. Rheumatic fever B. Roseola C. Rubeola D. Rubella Explanations

(u) A. Rheumatic fever causes carditis, arthritis, erythema, subcutaneous nodules, and chorea. (u) B. Roseola, caused by human herpes virus VI, causes a high fever followed by a reticular rash. (c) C. Rubeola or measles causes Koplik spots which are described as the salt-like crystals opposite the second molars. (u) D. Rubella causes a macular rash starting on the face with arthralgias and low-grade fevers.

206. Scientific Concepts/Cardiology Which of the following is the most common cause of arterial embolization? A. Rheumatic heart disease B. Myxoma C. Atrial fibrillation D. Venous thrombosis

(u) A. Rheumatic heart disease is a rare cause of embolization (u) B. Myxoma is a rare cause of embolization. (c) C. Atrial fibrillation is present in 60-70% of patients with arterial emboli and is associated with left atrial appendage thrombus. (u) D. Venous thrombosis may be a cause of embolization paradoxically, but is uncommon.

Which of the following is the most common cause of arterial embolization? A. Rheumatic heart disease B. Myxoma C. Atrial fibrillation D. Venous thrombosis

(u) A. Rheumatic heart disease is a rare cause of embolization (u) B. Myxoma is a rare cause of embolization. (c) C. Atrial fibrillation is present in 60-70% of patients with arterial emboli and is associated with left atrial appendage thrombus. (u) D. Venous thrombosis may be a cause of embolization paradoxically, but is uncommon.

Which of the following histories is most consistent with rheumatoid arthritis (RA)? A. Acute onset, morning stiffness, monoarticular joint swelling and tenderness B. Insidious onset, morning stiffness, symmetrical joint swelling and tenderness C. Insidious onset, morning stiffness, monoarticular joint swelling and tenderness D. Acute onset, morning stiffness, symmetrical joint swelling and tenderness

(u) A. Rheumatoid arthritis has an insidious onset, morning stiffness is typically longer than 30 minutes, and although early RA may have monoarticular presentation, it is typically symmetrical. (c) B. Rheumatoid arthritis has an insidious onset, morning stiffness is typically longer than 30 minutes, and is typically symmetrical in presentation. (u) C. Rheumatoid arthritis has an insidious onset, morning stiffness is typically longer than 30 minutes, and although early RA may have monoarticular presentation, it is typically symmetrical. (u) D. Rheumatoid arthritis has an insidious onset, morning stiffness is typically longer than 30 minutes, and although early RA may have monoarticular presentation, it is typically symmetrical.

A 53 year-old patient presents with severe pain at the base of the thumb and no other finger involvement. The pain is worse with activity and lasts a short period of time following rest. There is no specific history of trauma to the thumb but the patient admits working with her hands as a typist. Which of the following is the most likely diagnosis? A. Rheumatoid arthritis B. Osteoarthritis C. Hemochromatosis D. Pseudogout

(u) A. Rheumatoid arthritis typically involves the MCP and PIP joints of the digits for the second through fifth fingers. The thumb is classically spared. (c) B. The base of the thumb is typically involved with osteoarthritis as are the DIP joints of the other fingers. (u) C. Hemochromatosis classically involves the MCP joints of the second through fifth fingers. (u) D. Pseudogout joint involvement is typically the MCP joints of the second through fifth fingers.

A 28 year-old female, who has experienced occasional painful migratory arthralgias, complains now of a tender, swollen, and hot left ankle. The joint was aspirated and the synovial fluid showed 55,000 WBCs, 75% polymorphonuclear lymphocytes, low glucose level, and no crystals. Which of the following would be the most likely diagnosis? A. Rheumatoid arthritis B. Septic arthritis C. Gouty arthritis D. Osteoarthritis

(u) A. Rheumatoid arthritis usually involves more than one joint, primarily affecting metacarpophalangeal and wrist joints initially. Synovial fluid analysis would show moderate leukocytosis (< 50,000) with higher glucose levels. (c) B. Septic arthritis presents with a large number of WBCs, predominantly polymorphonuclear, and with glucose levels much lower than serum levels. (u) C. Although gout presents as monarticular process, it usually presents acutely without previous migratory arthralgias. Crystals are usually found in synovial analysis. (u) D. Osteoarthritis effusions may be normal or show mild elevations of WBCs in the fluid analysis, but it is rarely monarticular.

: (8) 18. Clinical Therapeutics/Endocrinology Which of the following is the pharmacologic therapy of choice in male patients suffering from mild-moderate gynecomastia? A. Selective serum reuptake inhibitor (SSRIs) B. Selective estrogen receptor modulator (SERMs) C. Testosterone D. Progesterone

(u) A. SSRIs are useful in the treatment of depression and are not indicated in the treatment of gynecomastia. (c) B. SERM therapy is particularly effective in glandular ("lumpy") gynecomastia. Examples of SERMS are tamoxifen and raloxifene. (u) C. Neither testosterone nor progesterone is indicated in the treatment of gynecomastia. (u) D. See C for explanation.

A 44 year-old female presents with ongoing arthralgias and myalgias with intermittent flares of arthritis. She is found to have a malar rash that worsens with sun exposure. She is known to have progressive renal damage and has recurrent infections that are slow to respond to therapy. She takes ibuprofen (Motrin) as needed for her joint pain and takes no other medication. Which of the following tests would be the initial test recommended to screen for this diagnosis? A. Rheumatoid factor B. Antihistone antibodies C. Anti-Smith (Anti-Sm) antibodies D. Anti-nuclear antibodies (ANA)

(u) A. Rheumatoid factor is most commonly performed in the assessment of rheumatoid arthritis and not suspected systemic lupus erythematosus. (u) B. Antihistone antibodies are seen in drug-induced SLE rather than in spontaneously occurring SLE. (u) C. Anti-Smith antibodies and antibodies to the double stranded DNA are fairly specific to SLE but their role is for confirmation rather than screening for SLE. (c) D. ANA's are the best screening test used in the evaluation for SLE.

A patient's EKG reveals widened P waves in lead II and large negative deflection of the P wave in lead V1. Which of the following is the most likely underlying cause for this? A. Right atrial enlargement B. Left atrial enlargement C. Right ventricular hypertrophy D. Left ventricular hypertrophy

(u) A. Right atrial enlargement is found on EKG with peaked P waves in lead II and a large positive deflection on the initial P wave in lead V1. (c) B. Wide P waves in lead II and a deep negative deflection in lead V1 is due to P-mitrale which is caused by left atrial enlargement. (u) C. Right ventricular hypertrophy is noted on the EKG by having a large R wave in lead AvR along with a deep S wave in leads V5 or V6. (u) D. On an EKG, left ventricular hypertrophy is associated with tall R waves in leads V5 and V6, deep S waves in AvR and V1, and tall R waves in AvL and AvF.

What is the EKG manifestation of cardiac end-organ damage due to hypertension? A. Right bundle branch block B. Left ventricular hypertrophy C. Right ventricular hypertrophy D. ST segment elevation in lateral precordial leads

(u) A. Right bundle branch block is caused by a delay in the conduction system in the right ventricle. It may be caused by right ventricular hypertrophy or conditions with higher pulmonic resistance such as cor pulmonale. Hypertension, however, is likely to cause changes in the left ventricle rather than the right ventricle. (c) B. Long-standing hypertension can lead to left ventricular hypertrophy with characteristic changes noted on EKG. (u)C. See A for explanation. (u) D. ST segment elevation is a sign of acute myocardial infarction not hypertension.

Which of the following is the earliest symptom for patients with left ventricular failure? A. Dependent edema B. Dyspnea on exertion C. Congestion D. Chest pain

(u) A. Right ventricular failure is manifested by dependent edema and congestion in the lungs. (c) B. Patients with left ventricular heart failure may be comfortable at rest and may experience their first symptoms with dyspnea with conversation or with mild exertion. (u) C. Right ventricular failure is manifested by congestion in the lungs with cough being a possible manifestation of this congestion. (u) D. Patients who have on-going left ventricular failure may have underlying coronary heart disease. When the heart failure is progressing, chest pain (manifested by angina) and myocardial infarction may be manifestations of ongoing, progressive heart failure. Acute decline in heart failure may lead to acute myocardial infarction but this tends to be a very late symptom.

: (8) 15. Diagnosis/Dermatology A 5 year-old male is brought to the office by his father who reports a 2 day history of low grade fever and coryza. The child awoke this morning with bright red cheeks. Physical examination reveals edematous, confluent plaques over the malar region of the face and reticular rash over the child's extensor surfaces. Which of the following is the most likely diagnosis? A. Rubeola B. Parvovirus B19 C. Respiratory syncytial virus D. Epstein-Barr virus

(u) A. Rubeola (measles) is a highly contagious viral infection characterized by cough, conjunctivitis and coryza. There is an associated rash which is an erythematous maculopapular that spreads centrally and inferiorly. (c) B. This is a classic presentation of Fifth Disease, a childhood exantham associated with human parvovirus B19. (u) C. RSV is a viral infection of the airway that causes annual outbreaks causing pneumonia, bronchiolitis and tracheobronchitis. (u) D. EBV is a common human virus that is present in >90% of the adult population and persists in the host for his/her lifetime. THe most common presentation is a 10-35 year-old with mononucleosis.

: (8) 9. History & Physical/Cardiology S3 and S4 gallops are both signs of which type of cardiac abnormality? A. Atrial non-compliance B. Ventricular septal defect C. Valvular insufficiency D. Ventricular dysfunction

(u) A. S3 and S4 are not indicative of atrial abnormalities. (u) B. Ventricular septal defect is not associated with an S3 and S4 gallop. (u) C. S3 and S4 are not directly correlated to valvular insufficiency. (c) D. S3 may be louder if filling pressure is increased or if ventricular compliance is reduced. S4 is most commonly heard when there is increased resistance to filling because of loss of compliance of the ventricular walls or the increased stroke volume of high-output states.

Which of the following classes of antidepressants is associated with anticholinergic side effects, including cardiac dysrhythmias, dry mouth, sedation, and orthostatic hypotension? A. Selective serotonin reuptake inhibitors B. Monoamine oxidase inhibitors C. Tricyclic antidepressants D. Atypical antidepressants

(u) A. SSRI's do not generally cause anticholinergic side effects. (u) B. MAOI's mainly cause orthostatic hypotension and sympthomimetic effects. (c) C. TCA's have well known anticholinergic effects. (u) D. Atypical antidepressants do not cause anticholinergic side effects.

: (28) 124. Clinical Therapeutics/Orthopedics/Rheumatology Which of the following is indicated as the initial management of spinal stenosis? A. Selective serotonin reuptake inhibitor B. Decompressive laminectomy C. Nonsteroidal antiinflammatory D. Epidural corticosteroid injections 51

(u) A. SSRIs are not indicated in the management of spinal stenosis. (u) B. Decompressive laminectomy is an option when conservative methods fail and the patient's quality of life requires further treatment. (c) C. A nonsteroidal antiinflammatory agent and exercise produces a good response in many patients and is indicated as the initial management option. (u) D. Epidural corticosteroids can produce temporary effects in both the short and long term but are not considered the initial treatment of choice.

: (9) 45. Diagnosis/Psychiatry/Behavioral Medicine A 22 year-old male presents to the behavioral health unit with his family who relate his 5 month history of delusions and hallucinations. Examination of the patient reveals a flattened affect, disorganized speech and poor hygiene. Which of the following is the most likely diagnosis? A. Schizophrenia B. Schizophreniform disorder C. Schizoaffective disorder D. Brief psychotic disorder

(u) A. Schizophrenia is a chronic illness that lasts at least 6 months with exacerbations and remissions. Patients have marked social and occupational impairment. They have acute psychotic episodes with delusions, hallucinations, disorganized speech, bizarre behavior, and flattened affect. (c) B. Schizophreniform disorder is characterized by the same features as schizophrenia except for the total duration of illness being from 1-6 months. Additionally, with schizophreniform, there is no impairment in social or occupation functioning. (u) C. Schizoaffective disorder is a disease condition that combines features of schizophrenia with a mood disorder. Patients are manic, depressed, or have a mixed episode that includes psychotic symptoms. (u) D. Brief psychotic disorder has the sudden onset of delusions, hallucinations, disorganized speech, or bizarre behavior. It lasts at least 1 day but less than 1 month.

A 24 year-old male presents to the emergency room via ambulance. He was found by the police walking naked on the highway. His speech is minimal but disorganized and he appears to be responding to auditory hallucinations. With further questioning, he is preoccupied with the delusion of the FBI listening in on his conversations. His mother tells you that he has been acting bizarre for two months now. What is the most likely diagnosis? A. Schizophrenia B. Schizophreniform disorder C. Schizoaffective disorder D. Schizotypal personality disorder

(u) A. Schizophrenia occurs with the above symptoms of greater than six months duration. (c) B. Schizophreniform disorder is characterized by the same features as schizophrenia except the total duration of the illness is at least one month and less than six months. (u) C. Schizoaffective disorder has the features of both schizophrenia and a mood disorder. (u) D. Patients with schizotypal personality disorder are strikingly odd or strange. They have magical thinking and derealization.

A 42 year-old patient who is being treated for colon cancer with chemotherapy develops nausea and vomiting. Which of the following drugs would be the most effective in controlling the nausea and vomiting? A. scopolamine (Scopace) B. meclizine (Antivert) C. ondansetron (Zofran) D. loperamide (Imodium)

(u) A. Scopolamine and meclizine are effective against motion sickness, but ineffective against substances that act directly on the chemoreceptor trigger zone. (u) B. See A for explanation. (c) C. Ondansetron selectively blocks 5-HT3 receptors in the periphery (visceral afferent fibers) and in the brain (chemoreceptor trigger zone). It is indicated for use in chemotherapy induced nausea and vomiting. (u) D. Loperamide has low anti-emetic potency.

You are seeing 62 year-old African American male for health maintenance. He is a former cigarette smoker with a 40 pack-year history. He quit smoking 10 years ago. He denies cough, hemoptysis, shortness of breath, chest pain, weight loss, or night sweats. What method of screening for lung cancer is appropriate in this patient? A. Chest radiograph B. Spiral CT of the chest C. Sputum cytology D. No screening is recommended

(u) A. Screening chest radiographs have not been shown to improve outcomes when used to screen asymptomatic patients. (u) B. While CT may yet prove valuable for screening, it is cost-prohibitive and has not yet been validated as a screening modality in asymptomatic patients. (u) C. Sputum cytology is not an effective screening tool. (c) D. No routine screening for lung cancer is recommended for asymptomatic smokers or former smokers.

A 72 year-old woman presents to your clinic complaining of constipation. Which of the following presentations would be most concerning? A. Fewer than one stools per week B. Weight loss C. Recurrent hemorrhoids D. Scybala

(u) A. See B for explanation (c) B. Alarm symptoms are concerning for colorectal malignancy and include weight loss, anemia, hematochezia, or positive fecal occult blood test. These symptoms are particularly worrisome in a patient with a family history of colorectal cancer. (u) C. See B for explanation. (u) D. See B for explanation.

At what age should a child's eyes be consistently well aligned? A. Two months B. Six months C. Twelve months D. Eighteen months

(u) A. See B for explanation. (c) B. A child's eyes should be consistently well-aligned by five to six months of age. (u) C. See B for explanation. (u) D. See B for explanation.

Colposcopic examination of the cervix of a 38 year-old woman with a high-grade lesion on Papanicolaou (Pap) smear yields a positive endocervical canal curettage (ECC) as its only abnormality. Which of the following is the most appropriate next step in this patient? A. Repeat the Pap smear in 3 months B. Perform a conization of the cervix C. Repeat the colposcopic examination in 3 months D. No follow-up is required

(u) A. See B for explanation. (c) B. A conization of the cervix is recommended in this case because there is a substantial discrepancy between the screening Pap test and the histologic data from biopsy and ECC. Additionally the ECC is positive for disease in this case. (u) C. See B for explanation. (h) D. See B for explanation.

Lesions of the optic chiasm will produce which of the following conditions? A. Amaurosis fugax B. Bitemporal hemianopia C. Unilateral blindness D. Homonymous hemianopia

(u) A. See B for explanation. (c) B. A lesion of the optic chiasm will produce bitemporal hemianopia. (u) C. See B for explanation. (u) D. See B for explanation.

Which of the following is the treatment of choice for a torus (buckle) fracture involving the distal radius? A. Open reduction and internal fixation B. Ace wrap or anterior splinting C. Closed reduction and casting D. Corticosteroid injection followed by splinting

(u) A. See B for explanation. (c) B. A torus or buckle fracture occurs after a minor fall on the hand. These fractures are very stable and are not as painful as unstable fractures. They heal uneventfully in 3-4 weeks. (u) C. See B for explanation. (u) D. See B for explanation.

A 7 year-old is evaluated for episodes of unresponsiveness in which she stares, blinking into space. Episodes last only 10-20 seconds but may happen several times daily. There are no abnormal movements, sensory loss or headaches reported. Which of the following is considered first-line therapy for this disorder? A. Carbamazepine (Tegretol) B. Ethosuximide (Zarontin) C. Gabepentin (Neurontin) D. Topiramate (Topamax)

(u) A. See B for explanation. (c) B. Absence seizures are usually treated successfully with Ethosuximide. The other agents listed are not indicated and may actually worsen this type of seizure. (u) C. See B for explanation. (u) D. See B for explanation.

: (5) 132. Diagnosis/Endocrinology A 66 year-old male presents with sudden onset of nocturnal pain in the metatarsal phalangeal (MTP) joint of the second digit of his right foot. On examination the area is swollen, red, and exquisitely tender to touch. He remembers one similar episode two years ago after attending his son's wedding. His temperature is mildly elevated at 100.5 F, and he otherwise appears fine. Plain radiograph of the foot reveals a rat-bite sign at the affected joint. Which of the following is the most likely diagnosis? A. Rheumatoid arthritis B. Gouty arthritis C. Osteoarthritis D. Osteomyelitis

(u) A. See B for explanation. (c) B. Gouty arthritis usually presents with nocturnal pain, erythema, swelling and severe tenderness in one joint. The first MTP joint is the most commonly affected, but joints in the foot, ankle, and even knee are susceptible. 54 Alcohol and rich foods often precipitate attacks. A rat-bite formation on plain film (punched out erosions with an overhanging rim of cortical bone) especially when adjacent to soft tissue tophi is diagnostic of gout. (u) C. See B for explanation. (u) D. See B for explanation.

: (8) 171. Diagnosis/Neurology A 33 year-old male presents to the emergency department reporting progressive distal-to-proximal upper and lower extremity weakness over the past 24 hours. Examination reveals no muscular atrophy, but absent patellar and biceps reflexes. Sensory examination over the same area is intact. Which of the following is the most likely diagnosis? A. Muscular dystrophy B. Guillain-Barre syndrome C. Charcot-Marie-Tooth disease D. Myasthenia gravis

(u) A. See B for explanation. (c) B. Guillain-Barre is an acute inflammatory demyelinating disorder characterized by rapidly progressive distal-toproximal muscle weakness that is generally first noticed by gait abnormalities and can progress to affect respiration and swallowing. (u) C. See B for explanation. (u) D. See B for explanation.

: (27) 44 105. Diagnostic Studies/Urology/Renal A 65 year-old male smoker presents with a new onset of hematuria. Which of the following studies would be the most definitive diagnostic study in this patient? A. Ultrasound B. Cystoscopy C. KUB radiograph D. CT of the pelvis

(u) A. See B for explanation. (c) B. Hematuria is the presenting symptom in 85-90% of patients with bladder cancer. Bladder cancer is the second most common urologic cancer and occurs more commonly in men than women. Cigarette smoking is a risk factor in over 60% of patients. The diagnosis and staging of bladder cancer are made by cystoscopy followed by transurethral resection of the bladder tumor. (u) C. See B for explanation. (u) D. See B for explanation.

A urinalysis performed during a routine physical examination on a 43 year-old male reveals 1-2 hyaline casts/HPF. The remainder of the UA is normal. Based upon these results, the physician assistant should A. collect a urine for culture and sensitivity. B. do nothing, since these casts are considered normal. C. refer the patient to a nephrologist. D. schedule the patient for a CT scan.

(u) A. See B for explanation. (c) B. Hyaline casts are not indicative of renal disease. They can be found following strenuous exercise and with concentrated urine or during a febrile illness. (u) C. See B for explanation. (u) D. See B for explanation.

A patient with schizophrenia states that newspapers and the television are constantly talking about him though they never mention his name. What term best fits this description? A. Cosmic identity B. Ideas of reference C. Retrospective falsification D. Thought broadcasting

(u) A. See B for explanation. (c) B. Ideas of reference are part of the abnormal thought content that is hallmark in schizophrenic patients. Cosmic identity refers to a patient's belief that they have physically and mentally fused with the universe while thought broadcasting is the belief that others can read the patient's mind or that their thoughts are broadcast over the radio or similar media. A distorted memory filtered through a person's present emotional state refers to retrospective falsification. (u) C. See B for explanation. (u) D. See B for explanation.

You are evaluating a patient whom you suspect has asthma. You perform spirometry before and after administration of an inhaled short-acting bronchodilator. After administration of the bronchodilator, which of the following spirometry results would suggest reversibility? A. Decrease in FEV1 B. Increase in FEV1 C. Decrease in FVC D. Increase in FVC

(u) A. See B for explanation. (c) B. In asthma, the airway obstruction should be at least partially relieved be a short-acting bronchodilator. This would be reflected in an increased forced expiratory volume in 1 second (FEV1). (u) C. The forced vital capacity (FVC) is not a function of obstruction and is generally normal in early mild asthma or lower than expected in severe or long-standing asthma. Either way, it is not expected to change with administration of a short-acting bronchodilator. (u) D. See C for explanation.

: (8) 27. Diagnosis/Cardiology A 9 year-old presents with increasing shortness of breath while playing basketball recently. On examination, radial pulses are exaggerated while femoral pulsations are weak. Chest radiograph shows rib notching and a mildly enlarged heart. Which of the following is the most likely diagnosis for this patient? A. Atrial septal defect B. Coarctation of the aorta C. Patent ductus arteriosus D. Tetralogy of Fallot

(u) A. See B for explanation. (c) B. In older children, the ECG and chest x-ray usually show left ventricular hypertrophy and a mildly enlarged heart. Rib notching may also be seen in older children (>8 years old) with large collaterals. (u) C. See B for explanation. (u) D. See B for explanation.

A 27 year-old male nonsmoker is diagnosed with upper extremity venous thrombosis. What is the most likely factor abnormality found in this patient? A. Factor VIII B. Factor V C. Factor I D. Factor X

(u) A. See B for explanation. (c) B. In young adults with otherwise unexplained thrombosis, factor V Leiden is the most common associated abnormality. (u) C. See B for explanation. (u) D. See B for explanation.

: (8) 192. Health Maintenance/Neurology The developmental milestone of speaking 1-3 words should be reached by which of the following ages? A. 9 months B. 12 months C. 18 months D. 24 months

(u) A. See B for explanation. (c) B. Infants should be able to speak 1 to 3 words by 1 year of age. (u) C. See B for explanation. (u) D. See B for explanation.

A 59 year-old male with history of hypertension and dyslipidemia presents with complaint of substernal chest pain for two hours. The pain woke him from sleep, does not radiate, and is associated with nausea and diaphoresis. Electrocardiogram reveals ST segment elevation in leads II, III, and AVF. Which of the following walls of the ventricle is most likely at risk? A. Anterior B. Inferior C. Lateral D. Posterior

(u) A. See B for explanation. (c) B. Inferior wall myocardial infarction is characterized by ST segment elevation in leads II, III and AVF. (u) C. See B for explanation. (u) D. See B for explanation.

: (7) 37. Diagnostic Studies/Psychiatry/Behavioral Medicine Which of the following laboratory tests would differentiate seizure from pseudoseizure? A. Urine cortisol B. Serum prolactin C. Dexamethasone suppression D. Fasting blood sugar

(u) A. See B for explanation. (c) B. The serum prolactin level can rise briefly after a seizure and therefore may assist in the determination between true seizure activity and pseudoseizure activity. (u) C. See B for explanation. (u) D. See B for explanation.

A 15 year-old softball player presents after jamming the distal tip of her finger into severe flexion. She is unable to extend the distal phalanx and she has pain on palpation of the distal interphalangeal joint. X-ray of the hand fails to reveal any associated avulsion fracture. Which of the following is the treatment of choice? A. Open reduction and internal fixation B. Continuous extension of the DIP with splinting C. Continuous flexion of the PIP with splinting D. Application of short arm cast

(u) A. See B for explanation. (c) B. The treatment of choice for a tear in the extensor tendon of the finger is continuous extension of the DIP via splinting for 6 to 8 weeks. (u) C. See B for explanation. (u) D. Short arm casting is indicated in wrist and metacarpal injuries but not in DIP extensor injuries.

Which of the following is the most common etiologic agent associated with acute bacterial sinusitis in the adult population? A. Staphylococcus aureus B. Streptococcus pneumoniae C. Pseudomonas aeruginosa D. Mycoplasma pneumoniae

(u) A. See B for explanation. (c) B. The typical pathogens associated with acute bacterial sinusitis are Streptococcus pneumoniae, other streptococci species, and Haemophilus influenzae. Moraxella catarrhalis and Staphylococcus aureus are less common causes. Pseudomonas is a less common cause and would be associated with nosocomial infection often in a critically ill patient. Mycoplasma is not a typical cause of bacterial sinusitis. (u) C. See B for explanation. (u) D. See B for explanation.

A 64 year-old right-handed woman presents to the emergency room. The patient is pleasant and cooperative, but you note that the left side of her mouth has little movement as she talks, resulting in some dysarthric speech. On physical examination, the left side of her mouth droops, eyebrows raise symmetrically, frown is symmetric, and eyes close but left offers little resistance to opening. You suspect which of the following? A. Upper motor neuron damage to CN VII (central facial palsy) in left hemisphere B. Upper motor neuron damage to CN VII (central facial palsy) in right hemisphere C. Lower motor neuron damage to CN VII (Bell's palsy) on the left D. Lower motor neuron damage to CN VII (Bell's palsy) on the right

(u) A. See B for explanation. (c) B. The upper face is controlled by pathways from both sides of the face. Upper motor neuron lesions cross, findings on the left are due to lesions on the right. Lower motor neuron lesions of the face are ipsilateral. (u) C. See B for explanation. (u) D. See B for explanation.

What is the most likely mechanism responsible for retinal hemorrhages and neurologic complications in a patient with infective endocarditis? A. Metabolic acidosis B. Embolization of vegetations C. Hypotension and tachycardia D. Activation of the immune system

(u) A. See B for explanation. (c) B. The vegetations that occur during infective endocarditis can become emboli and can be dispersed throughout the arterial system. (u) C. See B for explanation. (u) D. Glomerulonephritis and arthritis result from activation of the immune system.

Which of the following is the most appropriate management of acute psychosis in a patient with schizophrenia? A. Amitriptyline (Elavil) B. Fluphenazine (Prolixin) C. Lithium (Eskalith) D. Sertraline (Zoloft)

(u) A. See B for explanation. (c) B. There are numerous options used in the management of acute psychosis including benzodiazepines and the typical antipsychotics such as fluphenazine, haloperidol, and olanzapine. Antidepressants and lithium have no benefit in the treatment of acute psychosis. (u) C. See B for explanation. (u) D. See B for explanation.

What term is given to an ocular examination finding where small, irregular pupils are seen that react with near vision but not to light? A. Amaurosis fugax B. Argyll-Robertson C. Anisocoria D. Optic neuritis

(u) A. See B for explanation. (c) B. This finding may be seen in diabetes or in cases of neurosyphilis whereas amaurosis fugax is vision loss due to a central lesion, anisocoria has unequal sized pupils, unilateral loss of vision due to multiple sclerosis. (u) C. See B for explanation. (u) D. See B for explanation.

A 55 year-old female presents to the clinic with lethargy, fatigue, constipation, and menorrhagia. Physical examination reveals an enlarged thyroid, dry skin, and a heart rate of 50 bpm. Laboratory results show a decrease in free T4, and an elevation in TSH. Which of the following is the most likely diagnosis? A. Secondary hypothyroidism B. Primary hypothyroidism C. Primary hyperthyroidism D. Secondary hyperthyroidism

(u) A. See B for explanation. (c) B. This is a classic presentation of symptoms of primary hypothyroidism. Symptoms include weight gain, fatigue, lethargy, depression, weakness, constipation, menorrhagia; and patients often present with a palpable, enlarged thyroid. (u) C. See B for explanation. (u) D. See B for explanation.

Which of the following is recommended to reduce the risk for perinatal transmission of HIV in a patient with a viral load of >1000 copies/mL? A. Vaginal delivery with female condom B. Episiotomy to shorten second stage of labor C. Use of forceps or vacuum extractor to shorten second stage of labor D. Cesarean section prior to onset of labor and rupture of membranes

(u) A. See D for explanation. (u) B. See D for explanation. (u) C. See D for explanation. (c) D. Cesarean section performed prior to the onset of labor and rupture of membranes significantly reduces the risk of perinatal HIV transmission. Planned cesarean section delivery at 38 weeks of gestation to prevent perinatal transmission of HIV is recommended in women with a viral load of >1000 copies/mL.

: (28) 25. Scientific Concepts/Pulmonology A 21 year-old college student presents with a 3 week history of slowly worsening dry cough, generalized fatigue and most recently low-grade fevers. He denies nasal congestion, sore throat and nausea and has no past history of pulmonary disease or tobacco use. He does note that many other dorm residents have had similar symptoms over the past two months. Examination reveals mild inspiratory crackles but is otherwise normal. Chest radiograph is clear and CBC is normal. Which of the following is the most likely causative organism? A. Staphylococcus B. Pseudomonas C. Mycoplasma D. Klebsiella

(u) A. See C for explanation. (u) B. See C for explanation. (c) C. The subacute onset in this demographic of patient along with the lack of more advanced findings on exam and chest radiograph points most strongly to Mycoplasma. The other organisms listed have a generally more acute and severe onset and are associated with patients who have complications or other comorbid conditions. (u) D. See C for explanation.

A patient is evaluated for extreme fears of abandonment and an inability to care for himself throughout his adult life. He avoids disagreements and has difficulty initiating projects or acting on his own thoughts and ideas. Advice and reassurance are sought for even minor daily details. There are no reported suicidal tendencies or signs of self harm. What is the most likely personality disorder? A. Narcissistic B. Borderline C. Dependent D. Obsessive-compulsive

(u) A. See C for explanation. (u) B. See C for explanation. (c) C. These patients are constantly seeking external support and will do even unpleasant things for others to gain approval and nurturing. Narcissitic patients are egotistic and would not seek the opinion of others. Patients with borderline disorder can share some of these traits as they make aggressive efforts to avoid abandonment but the suicidal tendencies, impulsivity and self mutilation differentiate it. Obsessive-compulsive patients do not rely on external support and obey very strict personal rules of perfection and efficiency and choose not to rely on others. (u) D. See C for explanation.

55. History & Physical/Pulmonology While examining a patient's chest you discover an area with absent breath sounds, increased tactile fremitus and egophony. Which of the following conditions do these findings most likely represent? A. Pleural effusion B. Pneumothorax C. Pneumonia D. Pulmonary hypertension

(u) A. See C for explanation. (u) B. See C for explanation. (c) C. These physical exam findings most often correlate with an increased density of the pulmonary tissues. This would most commonly be caused by a consolidation secondary to pneumonia. (u) D. See C for explanation.

A 70 year-old male with history of ischemic cardiomyopathy presents with a syncopal episode. He denies complaints of chest pain, palpitations, or dyspnea. ECG shows no acute ST-T wave changes. Echocardiogram reveals an ejection fraction of 25% with no valvular abnormalities. Which of the following is the most appropriate management for this patient? A. Dual chamber permanent pacemaker B. Diltiazem (Cardizem) C. Implantable cardio defibrillator D. Midodrine (ProAmatine)

(u) A. See C for explanation. (u) B. See C for explanation. (c) C. This patient has ischemic cardiomyopathy and syncope, which is most likely due to ventricular tachycardia. Instertion of a cardio defibrillator is the management of choice in this patient. (u) D. See C for explanation.

: (5) 81. Clinical Intervention/Gastrointestinal/Nutritional A 50 year-old female presents with constipation following an episode of tearing pain associated with bleeding while defecating 4 days ago. She admits to a residual throbbing pain and she is afraid to have a bowel movement. On examination you note what appears to be a crack in the epithelium of the anal verge. Which of the following is the most appropriate initial management for this patient? A. Anal sphincter strengthening exercises B. Injection sclerotherapy C. Fiber supplementation and sitz baths D. Analgesics with incision and drainage

(u) A. See C for explanation. (u) B. See C for explanation. (c) C. This patient has signs and symptoms consistent with an anal fissure. Most patients will respond to fiber supplementation and sitz baths. (u) D. See C for explanation.

A 48 year-old alcoholic, whose last drink was 6 days ago, presents to the clinic complaining of palpitations and intermittent abdominal pain. On physical examination, blood pressure is 170/110 mm Hg, pulse 124/min, respirations 22/min, and temperature 100.4 degrees F. The patient is agitated and excitable. Cardiovascular examination reveals tachycardia without murmurs, gallops, or rubs, and is otherwise normal. Which of the following is the most appropriate intervention? A. start antihypertensive medication B. begin a cardiac work-up for angina C. admit to the hospital for alcohol withdrawal D. consult social services for appropriate disposition

(u) A. See C for explanation. (u) B. See C for explanation. (c) C. This patient is exhibiting the signs and symptoms of alcoholic withdrawal characterized by delirium, autonomic hyperactivity, perceptual distortions, and fluctuating levels of psychomotor activity. Seizures are a common occurrence prior to DTs, but the delirium may occur without preceding seizures. This is a medical emergency, and if untreated, it has a mortality rate of 20%. (u) D. See C for explanation.

A patient presents with signs and symptoms of Cushing's syndrome. Extensive diagnostic evaluation reveals an ACTH-secreting pituitary adenoma. First-line therapy should consist of A. pituitary radiation. B. medical adrenalectomy. C. transsphenoidal resection of the tumor. D. amiloride (Midamor).

(u) A. See C for explanation. (u) B. See C for explanation. (c) C. Transsphenoidal resection of the tumor cures about 80% of patients. The remainder can be given a combination of pituitary radiation and medical adrenalectomy with one or more drugs. If these procedures fail, the last option is bilateral adrenalectomy. (u) D. See C for explanation.

A 54 year-old type 2 diabetic male presents for follow up evaluation of previously diagnosed persistent otitis externa. Early in the disease process, a CT scan was obtained secondary to non-improvement on antibiotics. Results showed osseous erosion of the floor of the ear canal. He has been on ciprofloxacin 1000mg twice daily for two months since the CT scan and currently has no further edema, erythema, or exudate from the external auditory canal or surrounding tissue. Which of the following is an appropriate treatment plan? A. Continue prophylactic antibiotics for an additional 6 weeks B. Immediately discontinue antibiotics C. Obtain gallium scan to ensure reduction of inflammatory process D. Skin swab culture of healthy tissue

(u) A. See C for explanation. (u) B. See C for explanation. (c) C. Treatment of malignant external otitis requires prolonged antipseudomonal antibiotic administration often for several months. To avoid relapse, antibiotics should be continued even in the asymptomatic patient, until gallium scanning indicates a marked reduction in the inflammatory process. (u) D. See C for explanation.

: (28) 217. Diagnosis/Cardiology A 58 year-old male presents for evaluation after an episode of chest pain noted last night after an argument with his wife. The pain lasted about 20 minutes and was described as a tightness and burning sensation in his chest behind the sternum and radiated to the left shoulder and upper arm. He states the pain improved after he rested. The patient states this is the third such episode during stressful situations this month. Which of the following is the most likely diagnosis for this patient? A. Anginal equivalant B. Tietze syndrome C. Stable angina D. Unstable angina

(u) A. See C for explanation. (u) B. Tietze syndrome refers to costochondritis and is not related to stressful situations. (c) C. The diagnosis of angina pectoris depends principally on the history. Stable angina is generally of short duration and subsides completely without residual discomfort. Attacks following a heavy meal or brought on by anger last 12-20 minutes. Attacks lasting more than 30 minutes are unusual and suggest the development of unstable angina, myocardial infarction or an alternative diagnosis. (u) D. See C for explanation.

The most reliable site from which to identify the causative organism in cases of osteomyelitis is the A. base of ulcer. B. blood. C. sinus tract. D. bone.

(u) A. See C for explanation. (u) B. While blood cultures are indicated in acute cases of osteomyelitis, they are only positive in 25-50% of pediatric hematogenous osteomyelitis and 10% of other forms of bone infection. (u) C. Taking specimens for culture from a sinus tract or the base of an ulcer correlate poorly with organisms infecting the bone. (c) D. Samples from needle aspiration of pus in the bone, or from a bone biopsy, are essential to determine the exact causative agent.

: (28) 219. Clinical Therapeutics/Infectious Diseases A 32 year-old male presents to your office 1 week following a camping trip to New England. He complains of low grade fever, chills and myalgias. On physical examination a 3 cm, erythematous, papular lesion with central clearing is identified on the left thigh. Which of the following is most appropriate for this patient? A. Complete blood count (CBC) B. Erythromycin C. Doxycycline D. Lyme titer

(u) A. See C for explantion. (u) B. See C for explanation. (c) C. This is a classic clinical presentation of lyme diseae. First line treatment for lyme disease is doxycycline. (u) D. In cases where patients present with a pathopneumonic lesion consistent with Lyme disease, treatment is recommended and the Lyme titer is not indicated.

A patient presents to the emergency department with right upper quadrant pain over eight hours, nausea, and vomiting. On exam there is a fever of 101.2 degrees F. Ultrasound shows a distended gallbladder. What is the most appropriate management of this patient? A. Oral analgesics B. Diagnostic peritoneal lavage C. Proton pump inhibitors D. Laparoscopic cholecystectomy

(u) A. See D for explanation. (h) B. Diagnostic peritoneal lavage is used to detect intraabdominal bleeding from trauma and not to treat acute cholecystitis and may delay appropriate treatment. (u) C. Proton pump inhibitors are used to treat GERD or PUD. (c) D. The proper treatment for acute cholecystitis is IV fluids, antibiotics, pain control, and surgery. Cholecystectomy is the definitive treatment for acute cholecystitis and laparoscopic cholecystectomy is the procedure of choice.

Which of the following medications is most likely to cause acute tubular necrosis? A. Trimethoprim-sulfamethoxazole (Bactrim) B. Acetaminophen C. Cephalothin (Kefzol) D. Gentamicin

(u) A. See D for explanation. (u) B. Acute renal failure can develop as a result of acetaminophen overdose however this is rare. (u) C. Some first generation cephalosporins may cause renal insufficiency this too is rare. Aminoglycosides are still more nephrotoxic. (c) D. In hospitalized patients up to 25% of patients receiving aminoglycosides sustain some degree of acute tubular necrosis. Gentamicin is one of the most toxic aminoglycosides, streptomycin is the least nephrotoxic of the aminoglycosides.

What is the most common location of anterior nasal epistaxis? A. Middle turbinate B. Posterior ethmoid artery C. Kiesselbach's triangle D. Inferior turbinate

(u) A. See D for explanation. (u) B. See D for explanation. (c) C. 90% of all cases of anterior epistaxis originate from Kiesselbach's triangle. (u) D. The inferior turbinate is the most common location for posterior epistaxis.

A 53 year-old male is seen in the emergency department following a motor vehicle collision in which his knee impacted against the dashboard. The patient has a posterior knee dislocation that is promptly reduced in the emergency department. The patient currently has a palpable pulse in the dorsalis pedis and posterior tibial areas. Which of the following studies is mandatory? A. Anterior plain film of knee B. Sunrise view of the knee C. Measurement of compartment pressures D. Angiography

(u) A. See D for explanation. (u) B. See D for explanation. (u) C. Compartment pressures are performed in cases of suspected compartment syndrome, not to determine the patency of the popliteal artery. (c) D. The popliteal artery is at risk for injury whenever a patient sustains a posterior dislocation of the knee and should be evaluated with an arteriogram despite the presence of pedal pulses.

: (8) 179. Clinical Intervention/Endocrinology A 46 year-old patient with a past medical history of a prolactinoma and parathyroid adenoma is brought to the ED at 6:45 AM by a family member who found her to be diaphoretic and confused. Initial laboratory studies are normal except for a fasting blood glucose of 35 mg/dL. The patient responds rapidly to IV dextrose. Which of the following interventions represents the most appropriate long-term treatment for this patient? A. Encourage small frequent meals B. Discourage alcohol consumption C. Glucagon D. Surgical referral

(u) A. See D for explanation. (u) B. See D for explanation. (u) C. Glucagon is able to treat immediate hypoglycemia as related to the insulinoma but long-term definitive therapy will still be necessary. (c) D. An endocrinologist and/or an oncologist will establish the diagnosis of insulinoma but a surgeon is needed for removal of this lesion which would be curative.

Who is the most likely adult to sexually abuse a child? A. School teacher B. Friend of the family C. Scout leader D. Family member

(u) A. See D for explanation. (u) B. See D for explanation. (u) C. See D for explanation (c) D. Adults within the immediate or extended family perpetrate the most child sexual abuse, usually this is a trusted member of the family.

The birth weight of an infant has usually tripled by A. the second and third month. B. the fourth and fifth month. C. the seventh and eighth month. D. one year.

(u) A. See D for explanation. (u) B. See D for explanation. (u) C. See D for explanation. (c) D. A child triples his/her birthweight by one year.

: (27) 190. History & Physical/Cardiology An 80 year-old male from an assisted living facility is brought in for evaluation of repeated near-syncopal episodes when getting up from a seated position. His past medical history is significant for diabetes and hypertension for which he takes enalapril (Vasotec). Which of the following would you expect on physical examination? A. A drop in systolic blood pressure of at least 5 mmHg and a rise in pulse rate of 30 beats per minute after arising from a supine position. B. A rise in systolic blood pressure of 10 mmHg and a fall in pulse rate of 10 beats per minute after arising from a supine position. C. A rise in systolic blood pressure of 15 mmHg or a fall in heart rate by more than 15 beats per minute after arising from a supine position. D. Drop in systolic blood pressure of at least 20 mmHg and a rise in pulse rate of more than 15 beats per minute immediately upon arising from a supine position.

(u) A. See D for explanation. (u) B. See D for explanation. (u) C. See D for explanation. (c) D. A greater than normal decline (20 mmHg) in BP immediately upon arising from the supine to the standing position is observed. With/without tachycardia depending on the status of autonomic (baroreceptor)

A 56 year-old male, status post myocardial infarction, is noted to have left ventricular hypertrophy and an ejection fraction of 38%. Which of the following medications should be prescribed to prevent the development of heart failure symptoms? A. amlodipine (Norvasc) B. furosemide (Lasix) C. hydrochlorothiazide (HCTZ) D. lisinopril (Zestril)

(u) A. See D for explanation. (u) B. See D for explanation. (u) C. See D for explanation. (c) D. ACE inhibitors have been shown to markedly improve survival and are also recommended for prevention of symptoms in patients at risk for heart failure.

A patient with known relapsing-remitting multiple sclerosis (MS) presents to the hospital with an exacerbation of MS symptoms. What is the recommended treatment? A. Baclofen (Lioresal) B. Interferon C. Glatiramer acetate (Copaxone) D. Methylprednisolone

(u) A. See D for explanation. (u) B. See D for explanation. (u) C. See D for explanation. (c) D. Acute relapses of MS are treated with a short course of IV methylprednisolone followed by oral prednisone. This regimen reduces the severity and shortens the duration of attacks. All other drugs listed are used to reduce the attack rate of relapsing remitting multiple sclerosis.

A known alcoholic presents to the emergency department with altered level of consciousness and a blood glucose level of 35 mg/dL. Which of the following best explains this glucose result? A. Excess pancreatic insulin release B. Rapid carbohydrate discharge into the small bowel C. Agonist insulin-receptor antibody formation D. Hepatic glycogen depletion and impaired gluconeogenesis

(u) A. See D for explanation. (u) B. See D for explanation. (u) C. See D for explanation. (c) D. Alcohol-related hypoglycemia results from hepatic glycogen depletion and impaired gluconeogenesis and not due to antibody formation, excessive insulin release from the pancreas, or rapid release of carbohydrate into the small bowel.

A 58 year-old male who is otherwise healthy presents with chest pain and is found to have left main coronary artery stenosis of 75%. The most important aspect of his management now is A. daily aspirin to prevent MI. B. nitrate therapy for the angina. C. aggressive risk factor reduction. D. referral for coronary artery revascularization.

(u) A. See D for explanation. (u) B. See D for explanation. (u) C. See D for explanation. (c) D. Although medical therapy is important, revascularization is indicated when stenosis of the left main coronary artery is greater than 50%.

A 35 year-old female presents with a long standing complaint of dry, scratchy eyes and dry mouth. She also reports dyspareunia. Labs demonstrate a positive anti-nuclear antibody and Anti-La antibodies. The patient has a prolonged Schirmer test. Which of the following classes of medicines should be avoided in treating this condition? A. Antimalarials B. NSAIDs C. Glucocorticoids D. Anticholinergics

(u) A. See D for explanation. (u) B. See D for explanation. (u) C. See D for explanation. (c) D. Anticholinergics should be avoided in a patient with Sjogrens syndrome. All the other medications may be used to treat the systemic symptoms of Sjogrens.

Which of the following is the treatment of choice for a nursing home patient who has asymptomatic bacteriuria with no history of diabetes or structural abnormalities of the genitourinary tract? A. ciprofloxacin (Cipro) B. sulfamethoxazole-trimethoprim (Bactrim) C. cephalexin (Keflex) D. no treatment is needed

(u) A. See D for explanation. (u) B. See D for explanation. (u) C. See D for explanation. (c) D. Asymptomatic bacteriuria is commonly seen in the geriatric population and no treatment is needed as long as the patient is not diabetic or has no structural abnormalities of the genitourinary tract.

A 23 year-old patient presents with two days of fatigue, headache, fever and pain around the area in which she was bitten by a stray baby raccoon in an unprovoked attack 10 days ago. She cleaned the small wound thoroughly. Which of the following is the most appropriate intervention in this patient? A. Do nothing and treat her symptoms B. Human diploid cell rabies vaccine, 5 injections given all at once now C. Rabies immunoglobulin only D. Rabies immunoglobulins and human diploid cell rabies vaccine given 5 times in a 1-month period

(u) A. See D for explanation. (u) B. See D for explanation. (u) C. See D for explanation. (c) D. Bites by bats, skunks, and raccoons always require this regimen if the animal is not caught and tested.

A 26 year-old female reports progressive distal to proximal spread of extremity weakness over the last 36 hours without fever, headache or syncope. Examination reveals symmetrical, paresis of the hands and feet with loss of the brachioradialis and Achillis reflexes. Biceps and knee reflexes are present but diminished. Sensory exam is normal. What are the most likely findings on cerebral spinal fluid (CSF) analysis? A. Decreased glucose, increased WBC count and decreased protein B. Increased glucose, normal WBC count and normal protein C. Normal glucose, decreased WBC count and elevated protein D. Normal glucose, normal WBC count and elevated protein

(u) A. See D for explanation. (u) B. See D for explanation. (u) C. See D for explanation. (c) D. Guillain-Barre syndrome is typified by progressive symmetrical, distal to proximal spread of weakness and areflexia without fever or sensory deficits. CSF analysis shows elevated protein due to axonal demyelination but no glucose disturbances and no significant or sustained pleocytosis.

A 41 year-old female presents to you for medical screening advice. Her 44 year-old sister passed away recently 18 months after diagnosis of metastatic colon cancer. Which of the following is the most appropriate advice for this patient? A. Double contrast barium enema now and repeat every 5 years if normal B. Rectal occult blood testing annually until age 50 then sigmoidoscopy every 3 years C. Rectal occult blood testing annually until age 50 then screening colonoscopy every 5 years D. Screening colonoscopy now and repeat every 3-5 years if normal

(u) A. See D for explanation. (u) B. See D for explanation. (u) C. See D for explanation. (c) D. Hereditary factors are believed to contribute to up to 30% of colorectal cancers. Relative risk is 3.8 times if the family member's cancer was diagnosed at less than 45 years of age. Recommended screening in a single first degree relative with colorectal cancer diagnosed before age 60 is beginning colonoscopy at age 40 or ten years younger than age at diagnosis of youngest affected first-degree relative. Then if negative, every 5 years.

What patient demographic is at highest risk for the development of anorexia nervosa? A. College aged swimmer B. High school football player C. Married, 50 year old, heterosexual female D. Single, 20 year old, homosexual male

(u) A. See D for explanation. (u) B. See D for explanation. (u) C. See D for explanation. (c) D. Homosexual males are at increased risk for anorexia nervosa due to the strong community desire for slimness. This is not the case in the lesbian community. Wrestlers are at the greatest risk group among athletes along with ballet dancers. Though anorexia nervosa is more common in the female population, its onset is far more typical in the teens to early 20's.

Which of the following occurs as a result of pulmonary hypertension? A. left atrial enlargement B. aortic stenosis C. coronary artery spasm D. right ventricular enlargement

(u) A. See D for explanation. (u) B. See D for explanation. (u) C. See D for explanation. (c) D. In pulmonary hypertension increased resistance within pulmonary circulation causes the right ventricle to work harder and eventually enlarge in response. Other changes that may occur are right atrial enlargement, decreased left ventricular cavity size, and tricuspid regurgitation.

: (27) 103. Clinical Intervention/Obstetrics/Gynecology An 86 year-old female presents complaining of progressive, significant vulvar itching. The itching is temporarily relieved with scratching. On examination you note diffuse involvement of the vulva with very thin, whitish epithelial plaques. In several areas the skin appears cracked and bleeds easily in several areas. Which of the following is the appropriate intervention for this patient? A. Topical conjugated estrogen (Premarin) B. Topical clobetasol (Temovate) C. Ultraviolet light treatment D. Biopsy referral

(u) A. See D for explanation. (u) B. See D for explanation. (u) C. See D for explanation. (c) D. It is important to biopsy multiple areas of vulvar skin involvement to confirm diagnosis and to rule out the presence of vulvar cancer. Patients who are confirmed to have lichen sclerosis with a large acanthotic component should be treated with well-penetrating corticosteroid creams.

: (8) 166. Scientific Concepts/Psychiatry/Behavioral Medicine A 60 year-old male with mental status changes presents with his wife reporting his cognitive decline that has worsened over the last several months. Examination reveals that he is alert and oriented x 2, has a shuffling gait, a reduced arm swing and masked facies. He has no delusions but is reacting to visual hallucinations. His affect is flat, thought process reveals blocking. He has deficits in short- and long-term memory and confabulates when answering questions. A previous medical provider gave the patient an antipsychotic medication which caused catatonia. Which of the following is the most likely etiology of his symptoms? A. Alzheimer's dementia B. Pick's disease C. Vascular dementia D. Lewy body dementia

(u) A. See D for explanation. (u) B. See D for explanation. (u) C. See D for explanation. (c) D. Lewy body disease (LBD) is a dementia similar to Alzheimer's and is characterized by hallucinations, parkinsonian features, and extrapyramidal signs. Patients with LBD lose cholingergic neurons which cause a loss in cognitive functioning and a loss in dopaminergic neurons causes the loss in motor control. Patients with with LBD are very sensitive to neuroleptic and antiemetic medications that effect dopaminergic and cholingergic systems. They respond with catatonia, loss of cognitive function and/or develop life-threatening muscle rigidity.

What laboratory test should be followed routinely every six to twelve months in patients taking lithium? A. Complete blood count B. Calcium C. Potassium D. Thyroid stimulating hormone

(u) A. See D for explanation. (u) B. See D for explanation. (u) C. See D for explanation. (c) D. Lithium induces hypothyroidism because of the decrease in concentration of circulating thyroid hormones.

Which physical examination finding distinguishes allergic rhinitis from other rhinitis etiologies? A. Clear rhinorrhea B. Erythematous pharynx C. Nasal flaring D. Pale nasal turbinates

(u) A. See D for explanation. (u) B. See D for explanation. (u) C. See D for explanation. (c) D. On physical examination, the mucosa of the turbinates is usually pale or violaceous with allergic rhinitis because of venous engorgement in contrast to the erythema of viral rhinitis.

A patient sustained a 6 cm laceration on his anterior tibia that was primarily closed in the emergency department. What is the most appropriate time frame for removal of these sutures? A. 1-2 days B. 3-5 days C. 6-8 days D. 7-14 days

(u) A. See D for explanation. (u) B. See D for explanation. (u) C. See D for explanation. (c) D. Suture removal is based upon the area of the body that was sutured. Facial sutures are placed for 3-4 days, scalp sutures for 5-7 days, trunk sutures are placed for 6-8 days, and sutures on the extremity are placed for 7-14 days. Sutures on the extremities can stay for longer periods of time if the area is under maximal tension.

A 9 year-old male is brought in by his mother who reports the patient has exhibited an extremely negative attitude for the past year. He seems angry much of the time and frequently loses his temper. Arguing over even trivial details is common place and he seems to take delight in annoying his family. His grades and conduct at school remain excellent. He has few friends, though he has never been seen bullying or destroying others' property. What is the most likely diagnosis? A. Attention deficit disorder B. Conduct disorder C. Antisocial personality disorder D. Oppositional defiant disorder

(u) A. See D for explanation. (u) B. See D for explanation. (u) C. See D for explanation. (c) D. Oppositional defiant disorder (ODD) best fits this scenario and is differentiated from conduct disorder by the lack of bullying and the lack of destruction of property. Many children with ODD do drift into conduct disorders over time. His good grades and conduct at school lessen the probability of untreated ADD. Personality disorders (i.e. antisocial personality disorder) can not be diagnosed at this early an age.

115. Clinical Therapeutics/Pulmonology A 28 year-old non-smoking black female presents with an 8 week history of dry cough, fatigue and numerous macular-papular lesions on the face. She denies fever, chills or hemoptysis though she has noticed enlarged, nontender lymph nodes in the neck. Lab tests reveal a normal CBC but her serum ACE level is three times the upper limit of normal. Her chest radiograph reveals bilateral, hilar adenopathy but is otherwise clear. Which of the following medications is most indicated in this patient? A. Antibiotics B. Anticholinergics C. Mast cell stabilizers D. Steroids

(u) A. See D for explanation. (u) B. See D for explanation. (u) C. See D for explanation. (c) D. Oral steroids have been shown to provide the best treatment for sarcoidosis.

: (9) 56. Clinical Therapeutics/Psychiatry/Behavioral Medicine Which of the following laboratory tests should be followed in patients treated with atypical antidepressants? A. CBC B. BUN C. TSH D. FBS

(u) A. See D for explanation. (u) B. See D for explanation. (u) C. See D for explanation. (c) D. Patients who take atypical antidepressants should be monitored for the development of diabetes on a quarterly or semiannual basis.

What laboratory test should be closely monitored in patients on long-term lithium treatment for bipolar disorder? A. ALT B. Calcium C. Lipase D. TSH

(u) A. See D for explanation. (u) B. See D for explanation. (u) C. See D for explanation. (c) D. Patients with bipolar disorder and those on long term lithium therapy are prone to hypothyroidism severe enough to require treatment. Liver and pancreatic complications are not a common concern. Electrolyte/renal issues can arise with poor fluid intake and severe vomiting and diarrhea, calcium does not require routine monitoring.

A 62 year-old male chronic smoker is examined for slowly progressive non-painful vision loss. Visual acuity showed 20/150 in both eyes. Extra-ocular muscles are intact. Pupils are responsive to direct and consensual stimuli however appear hazy. During funduscopic exam you are unable to visualize the optic cup, disk or the vessels. What is the most appropriate diagnosis? A. Anterior uveitis B. Narrow angle glaucoma C. Retinal detachment D. Senile cataract

(u) A. See D for explanation. (u) B. See D for explanation. (u) C. See D for explanation. (c) D. Patients with cataract have gradually progressive blurred vision without pain or redness. Lens opacification usually develops and can be grossly visible. Cataracts are the leading cause of blindness worldwide. They usually occur bilaterally. Senile cataracts is by far the most common type with most persons over the age of 60 having some degree of lens opacity. Cigarette smoking increases the risk of cataract formation.

A 67 year-old male with history of mitral valve stenosis undergoes a mechanical valve replacement. Which of the following is the appropriate duration of anticoagulation therapy if the patient has no other risk factors for thromboembolic events or significant bleeding risks? A. One month B. Three months C. Six months D. Lifelong

(u) A. See D for explanation. (u) B. See D for explanation. (u) C. See D for explanation. (c) D. Patients with mechanical valves require lifelong anticoagulation to prevent thrombosis.

: (27) 90. Clinical Therapeutics/Neurology Which of the following medications is most appropriate for migraine headache prophylaxis? A. Lisinopril (Zestril) B. Ergotamine tartrate (Cafergot) C. Metoclopromide (Reglan) D. Propanolol (Inderal)

(u) A. See D for explanation. (u) B. See D for explanation. (u) C. See D for explanation. (c) D. Propanolol is used as a preventative medication for migraine headaches.

Pulmonary capillary wedge pressure indirectly measures which of the following? A. Right ventricular end-diastolic pressure B. Right atrial filling pressure C. Left ventricular systolic pressure D. Left atrial filling pressure

(u) A. See D for explanation. (u) B. See D for explanation. (u) C. See D for explanation. (c) D. Pulmonary capillary wedge pressure indirectly measures left atrial filling pressures.

In which of the following maternal-fetal blood type pairings should the mother receive Rho-GAM? A. A positive mother, O negative infant B. A negative mother, O negative infant C. AB positive mother, spontaneous abortion D. AB negative mother, spontaneous abortion

(u) A. See D for explanation. (u) B. See D for explanation. (u) C. See D for explanation. (c) D. Rho-GAM is indicated for an unsensitized Rh-negative patient who has had a spontaneous or induced abortion, ectopic pregnancy, or at the time of amniocentesis. It is also indicated at 28 weeks gestation and within 72 hours of delivery of an Rh-positive infant.

: (12) 143. Clinical Therapeutics/Psychiatry/Behavioral Medicine Which of the following classes of medications is considered first-line for the treatment of panic disorder? A. Benzodiazepines B. Monoamine oxidase inhibitors C. Tricyclic drugs D. Selective serotonin re-uptake inhibitors

(u) A. See D for explanation. (u) B. See D for explanation. (u) C. See D for explanation. (c) D. SSRIs are considered 1st line for the treatment of panic disorder because of efficacy and side-effect profile.

You have just stuck yourself with a sharp needle. In order for you to be able to interpret this sensation, which of the following areas must be intact? A. Anterior spinothalamic tract, basal ganglia, and sensory cortex B. Corticospinal tract, medulla, and basal ganglia C. Pyramidal tract, hypothalamus, and sensory cortex D. Lateral spinothalamic tract, thalamus, and sensory cortex

(u) A. See D for explanation. (u) B. See D for explanation. (u) C. See D for explanation. (c) D. Sensory impulses reach the sensory cortex from the spinothalamic tract or the posterior columns. Fibers transmit this to the thalamus which sends impulses to the sensory cortex of the brain

: (6) 208. Clinical Intervention/Psychiatry/Behavioral Medicine A 34 year-old woman who is new to the practice has a long history of medical complaints that include almost every organ system. She describes herself as being ill since she was a child. She takes a total of 25 prescriptions from multiple providers,including prednisone and multiple pain medications, as well as several over-the-counter medications. You find no abnormalities on comprehensive examination. What is the recommended plan for this patient? A. Order a CT of the chest and abdomen B. Admit her to the hospital C. Order a PET scan D. Schedule monthly visits with you

(u) A. See D for explanation. (u) B. See D for explanation. (u) C. See D for explanation. (c) D. The patient has somatization disorder. It is important that one provider coordinates the medical care of patients with somatization disorder. Regular, short appointments are helpful, and brief physical examinations should be performed to assess somatic complaints. Diagnostic evaluations should be limited.

: (8) 128. Clinical Therapeutics/ENT/Ophthalmology A 57 year-old male presents with a 3 day history of progressive ear pain and drainage. Examination of the patient reveals purulent auricular drainage and marked tenderness of the tragus. Which of the following treatment options is most appropriate given this presentation? A. Oral amoxicillin (Amoxil) B. Oral minocycline (Minocin) C. Topical clindamycin (Cleocin) D. Topical ciprofloxacin (Cipro)

(u) A. See D for explanation. (u) B. See D for explanation. (u) C. See D for explanation. (c) D. The treatment of otitis externa involves keeping the external canal clean and dry. Topical antibiotics allow for direct application to area of infection. Acidic preparations with potent coverage against gram negative organisms are preferred due to high rates of Pseudomonas and Proteus.

A 47 year-old perimenopausal female with vasomotor symptoms complains of vulvar itching and copious vaginal discharge with a rancid odor. Physical examination reveals erythema of the vulva and petechiae on the cervix. The pH of the vaginal discharge is five. Which of the following is the recommended treatment for this patient? A. Topical metronidazole (Metrogel) B. Oral fluconazole (Diflucan) C. Topical estradiol (Estrace) cream D. Oral metronidazole (Flagyl)

(u) A. See D for explanation. (u) B. See D for explanation. (u) C. See D for explanation. (c) D. This is a classic description of trichomonas vulvovaginitis. This condition must be treated with oral metronidazole or tinidazole.

: (8) 174. Diagnosis/Dermatology A 68 year-old female presents to your office with a 3 day history of pain around her right rib cage. She denies chest pain, shortness of breath, palpitations, recent injury or trauma. Her past medical history is significant for rheumatoid arthritis for which she has used Etanercept (Enbrel) weekly for 4 years. Physical examination is unremarkable. She returns the following day with grouped, vesicular eruption along the T5 dermatome. Which of the following is the most likely diagnosis? A. Drug reaction B. Contact dermatitis C. Bullous impetigo D. Herpes Zoster

(u) A. See D for explanation. (u) B. See D for explanation. (u) C. See D for explanation. (c) D. This is an immune compromised patient with classic prodromal shingles symptoms followed by vesicular, dermatomal rash.

84. Clinical Therapeutics/Obstetrics/Gynecology An 18 year-old G1 P0 patient at 32 weeks gestation is sent to labor and delivery directly from her obstetrician's office. Her blood pressure on presentation is 162/114 mmHg; she is hyperreflexic and has 3+ protein on random urine dipstick. There is good fetal beat to beat variability with an occasional mild contraction on the uterine monitor. Which of the following would be the benefit of magnesium sulfate in this patient? A. Cessation of contractions B. Lowering of blood pressure C. Stabilization of renal function D. Prevention of seizures

(u) A. See D for explanation. (u) B. See D for explanation. (u) C. See D for explanation. (c) D. This patient has findings consistent with severe preeclampsia. Magnesium sulfate is given to prevent and treat eclamptic convulsions. If the patient continues to have systolic blood pressure measurements >160 mmHg or if diastolic blood pressures exceeds 105-110 mm Hg an antihypertensive medication will need to be added. Magnesium sulfate can also be used to treat preterm labor, however an occasional mild contraction at this gestational age is not unusual and does not need to be treated.

A 16 year-old nulliparous acutely ill female presents with bilateral lower abdominal pain. She has a temperature of 100.4 degrees F and on examination has a tender, enlarged left adnexa. Cervical culture is positive for Chlamydia. Ultrasound reveals a complex tubular structure in the left adnexal area. What is the recommended treatment? A. Outpatient treatment with IM ceftriaxone and oral doxycycline B. Oral doxycycline C. IM procaine penicillin D. Hospitalization with parenteral doxycycline and cefoxitin

(u) A. See D for explanation. (u) B. See D for explanation. (u) C. See D for explanation. (c) D. This patient has pelvic inflammatory disease and most likely a tubo-ovarian abscess. It is recommended that the patient be hospitalized and treated with high-dose IV antibiotic therapy. For patients with tubo-ovarian abscesses, surgical drainage is often necessary.

: (8) 185. Diagnosis/Neurology A 79 year-old nursing home resident with a history of diabetes mellitus and hypertension is transferred to the ED following an episode of confusion and right arm weakness which resolved after 8 hours. The staff recorded a fingerstick glucose of 102 at the onset of symptoms. Vital signs were BP 120/80mmHg, HR 78bpm, RR 18, TEMP 98.8 F. Which of the following is the most likely diagnosis in this patient? A. Subclavian steal syndrome B. Cerebral vascular accident C. Giant cell arteritis D. Transient ischemic attack

(u) A. See D for explanation. (u) B. See D for explanation. (u) C. See D for explanation. (c) D. This patient has symptoms of TIA, which are transient and generally resolve within 24 hours. CVA is determined by the extended duration of the above symptoms. Subclavian steal syndrome is demonstrated by a subclavian bruit, unequal radial pulses and unequal blood pressure readings.

: (18) 112. Diagnosis/Orthopedics/Rheumatology A 22 year-old male with a history of chronic low back pain presents with a four month history of worsening and persistent low back pain, fatigue, and morning stiffness which are relieved with exercise. He also reports intermittent decreased vision and photophobia in his right eye. A lumbar spine series reveals squaring of the vertebral bodies with normal disc space. Which of the following is the most likely diagnosis? A. Spondylolisthesis B. Vertebral compression fracture C. Juvenile rheumatoid arthritis D. Ankylosing spondylitis

(u) A. See D for explanation. (u) B. Vertebral compression fracture is the most frequent finding in osteoporosis. When symptomatic vertebral fractures occur, the pain is localized to the area of the fracture. There are no associated eye findings with vertebral compression fractures. (u) C. Polyarticular onset of juvenile rheumatoid arthritis (JRA) includes symptoms of fever and synovitis of four or more joints. JRA with pauciarticular involves one to four joints and iridocyclitis without other systemic symptoms. (c) D. Ankylosing spondylitis (AS) is characterized by low back pain that persists for more than 3 months. It is often associated with morning stiffness relieved by exercise. Anterior uveitis affects up to 40% of patients with AS. Patients have diminished forward flexion and decreased chest expansion on inspiration. Radiographic evidence includes erosions in the sacroiliac (SI) joint line, pseudowidening, subchondral sclerosis, bony replacement of the SI joints, squaring of the vertebral bodies, and syndesmophytes (bridging of the vertebrae).

A 26 year-old patient is brought to the emergency department after a head on collision. The patient complains of chest pain, dyspnea and cough. Examination reveals the patient to be tachypneic and tachycardic with a narrow pulse pressure. Jugular venous distension is noted. Electrocardiogram reveals nonspecific t wave changes and electrical alternans. Which of the following is the most appropriate management plan for this patient? A. Serial echocardiograms B. Pericardiocentesis C. Cardiac catheterization D. Pericardiectomy

(u) A. Serial echocardiograms would be indicated if a patient had a small pericardial effusion and no intervention was immediately needed. This patient has signs and symptoms of cardiac tamponade and needs immediate intervention. (c) B. Urgent pericardiocentesis is the initial treatment of choice in a patient with cardiac tamponade. (u) C. There is no indication for cardiac catheterization in the management of cardiac tamponade. (u) D. A partial pericardiectomy may be needed in patients with recurrent pericardial effusions that occur secondary to neoplastic disease and uremia, but there is no indication for partial pericardiectomy in the acute management of cardiac tamponade.

: (27) 36 83. Clinical Intervention/Cardiology A patient who was recently diagnosed with viral pericarditis now complains of severe dyspnea and non-productive cough with pain over the precordial region. His vital signs reveal a heart rate 130bpm and respiratory rate 26. Blood pressure is 130/105 mmHg but fluctuates with inspiration resulting in a 20 mmHg decline in the systolic pressure. Which of the following is the most appropriate therapy for this patient? A. Serial echocardiography B. Urgent pericardiocentesis C. Surgery for pericardial window D. Furosemide (Lasix) bolus

(u) A. Serial echocardiography is indicated if no intervention is immediately contemplated. (c) B. This patient has signs and symptoms consistent with pericardial tamponade. Urgent pericardiocentesis is required. (u) C. Recurrent effusion in neoplastic disease or uremia may require pericardial window. For this patient with acute onset tamponade, pericardiocentesis would be corrective. (u) D. Lasix will not affect the collection of fluid in the pericardium.

Which of the following is used to screen for malabsorption disorders of the intestines? A. serum gastrin level B. urea breath test C. fecal eosinophils D. stool fecal fat

(u) A. Serum gastric level is elevated in Zollinger-Ellison syndrome. (u) B. Urea breath test is used in the diagnosis of H. pylori infection. (u) C. Eosinophils are elevated in inflammatory diarrhea due to eosinophilic gastroenteritis. (c) D. Stool fecal fat is the gold standard test for the evaluation of patients with malabsorption of the intestine.

A 50 year-old female presents describing poor concentration, generalized fatigue, oligomenorrhea and severe acne outbreaks over the past year. On examination she exhibits central obesity, a round face, numerous striae and elevated blood pressure. Which of the following is the most appropriate initial diagnostic study? A. Serum thyroid stimulating hormone B. Dexamethasone suppression test C. Oral glucose tolerance test D. Antithyroperoxidase antibodies

(u) A. Serum thyroid stimulating hormone would be more appropriate for hypo- or hyperthyroidism. (c) B. Dexamethasone suppression test as well as midnight serum cortisol level, late night salivary cortisol level, and urinary free cortisol used singly or in combination to diagnose Cushing's syndrome (Hypercortisolism). (u) C. Patients with Cushing's syndrome do have issues with glucose tolerance, however oral glucose tolerance test would not confirm the presence of a secondary cause of the intolerance. (u) D. Antithyroperoxidase antibodies are commonly ordered when Hashimoto thyroiditis is suspected.

A mother brings in her 4 year-old son with complaints that he falls frequently and he "stands funny". The mother also notes that he has lost the ability to easily stand from a seated position. She reports that he met developmental milestones as an infant. Which of the following is the most likely cause? A. Developmental hip dysplasia B. Myasthenia gravis C. Cerebral palsy D. Muscular dystrophy

(u) A. Signs and symptoms of developmental hip dysplasia would have been present more at birth and when beginning to crawl and walk. (u) B. Myasthenia gravis (MG) is less likely due to age and MG typically affects the ocular, pharyngeal and respiratory muscles. (u) C. Cerebral palsy (CP) typically presents with ataxia, spasticity or tightness, and uncoordination. Patients with CP typically do not meet developmental milestones. (c) D. Muscular dystrophy age of onset is by age 5, and begins in the pelvic girdle.

Which of the following signs and symptoms is typically noted in patients with acute cystitis? A. Fever and chills B. CVA tenderness C. Flank pain D. Frequency and dysuria

(u) A. Signs of systemic toxicity, such as fever and chills, are absent in acute cystitis. (u) B. CVA tenderness and flank pain are associated with acute pyelonephritis. (u) C. See B for explanation. (c) D. Irritative voiding symptoms, such as frequency and dysuria, are common in acute cystitis.

: (4) 74. Diagnostic Studies/Dermatology At the urging of his wife a 47 year-old construction worker presents to your office with a skin lesion. He reports he has had a slowly growing nodule on the side of his nose for the past year or two. It recently developed a small central area of erosion. He denies pain, fever, chills, pruritus or other similar lesions. On physical exam the lesion is approximately 1 cm as described above, with a few telangiectatasias. Which of the following is the best diagnostic study for this condition? 33 A. Skin scraping B. Dermascopy C. Excision D. Punch biopsy

(u) A. Skin scraping may not provide adequate sample to determine a diagnosis and thus delay treatment leading to additional mobidity, cosmetic deformity. (u) B. Dermascopy does not allow for tissue sampling and can delay diagnosis and treatment which may lead to additional mobidity, cosmetic deformity. (u) C. Excision is reserved for after confirmation of diagnosis by biopsy. (c) D. This patient has a basal cell carcinoma (BCC), which is the most common form of skin cancer and occurs in sun exposed areas of the skin. Any lesion suspected of being a BCC should be biopsied with a punch biospy.

Which of the following is considered to be the modality of choice for the identification of a pituitary macroadenoma that is suspected on the basis of a visual field deficit? A. Skull x-ray B. PET scan C. CT of the brain D. MRI of the brain

(u) A. Skull x-ray is not the modality of choice for the identification of pituitary macroadenoma. (u) B. PET scan is not the modality of choice for the identification of pituitary macroadenoma. (u) C. CT of the brain is not the optimal imaging technique for evaluation of the pituitary stalk. (c) D. MRI of the brain provides the best visualization of pituitary tumors.

A 32 week preterm infant has an APGAR score of 9 at 5 minutes. Thirty minutes after delivery, tachypnea, retractions, and expiratory grunting are noted. Cyanosis and dyspnea appear with little response to oxygen. Physical examination reveals poor air movement bilaterally. A chest x-ray reveals air bronchograms and a fine reticular granular pattern. Which of the following conditions should be suspected? A. Atelectasis B. Diaphragmatic hernia C. Respiratory distress syndrome D. Pneumothorax

(u) A. Small areas of atelectasis usually are asymptomatic. While larger areas may present with similar clinical findings, the chest x-ray findings are not consistent with atelectasis. (u) B. Chest x-ray in a patient with a diaphragmatic hernia would not show a fine reticular granular pattern. (c) C. Clinical findings of increasing cyanosis unresponsive to oxygen therapy and the characteristic x-ray findings are most consistent with respiratory distress syndrome. (u) D. Chest x-ray in a patient with a pneumothorax would not show a fine reticular granular pattern.

A 53 year-old female status post abdominal hysterectomy 3 days ago suddenly develops pleuritic chest pain and dyspnea. On exam, she is tachycardic and tachypneic with rales in the left lower lobe. A chest x-ray is unremarkable and an EKG reveals sinus tachycardia. Which of the following is the most likely diagnosis? A. Atelectasis B. Pneumothorax C. Pulmonary embolism D. Myocardial infarction

(u) A. Small atelectasis is commonly asymptomatic, while large atelectasis may produce signs of dyspnea and cough. Exam reveals absence of breath sounds in the area involved and dullness to percussion. A chest x-ray would reveal various findings dependent on the location of the atelectasis, but would not be normal. (u) B. While a pneumothorax commonly presents with pleuritic chest pain and dyspnea, exam would reveal the presence of diminished breath sounds and hyperresonance on the involved side. A chest x-ray would reveal presence of a pleural line on the expiratory chest x-ray. (c) C. This patient's risk factors for pulmonary embolism include advanced age, surgery, and prolonged bedrest. While the diagnosis of pulmonary embolism is difficult to make due to nonspecific clinical findings, the most common symptoms include pleuritic chest pain and dyspnea associated with tachypnea. Chest x-ray and EKG are usually normal. (u) D. While a myocardial infarction usually presents with dyspnea, the chest pain is not usually pleuritic in nature. An EKG would commonly reveal ST segment changes which would be consistent with ischemia or infarct.

: (10) 139. Clinical Intervention/Pulmonology A 62 year-old male presents with a right hilar mass. Needle-biopsy of the mass reveals the presence of small-cell carcinoma and a bone scan reveals the presence of scattered hot spots throughout the skeleton. Which of the following is the most appropriate treatment? A. Lobectomy B. Pneumonectomy C. Thoracic radiation therapy D. Combination chemotherapy

(u) A. Small-cell carcinoma of the lung is rarely treatable with surgical resection. Surgery may be indicated as part of the treatment protocol for small peripheral lesions without any evidence of metastasis. (u) B. See A for explanation. (u) C. While thoracic radiation therapy has been shown to be beneficial for patients with limited small-cell lung cancer, no benefit has been observed for patients with extensive disease defined as the presence of metastatic disease. (c) D. Combination chemotherapy is the treatment of choice for a patient with small-cell carcinoma of the lung.

A 62 year-old male presents with a right hilar mass. Needle-biopsy of the mass reveals the presence of small-cell carcinoma and a bone scan reveals the presence of scattered hot spots throughout the skeleton. Which of the following is the most appropriate treatment? A. Lobectomy B. Pneumonectomy C. Thoracic radiation therapy D. Combination chemotherapy

(u) A. Small-cell carcinoma of the lung is rarely treatable with surgical resection. Surgery may be indicated as part of the treatment protocol for small peripheral lesions without any evidence of metastasis. (u) B. See A for explanation. (u) C. While thoracic radiation therapy has been shown to be beneficial for patients with limited small-cell lung cancer, no benefit has been observed for patients with extensive disease defined as the presence of metastatic disease. (c) D. Combination chemotherapy is the treatment of choice for a patient with small-cell carcinoma of the lung.

Which of the following effects result from cigarette smoking and contributes to atherogenesis? A. transient decrease in blood pressure B. polycythemia with relative hyperoxemia C. decreased blood viscosity D. chronic inflammation

(u) A. Smoking leads to a transient increase in blood pressure, not a decrease. (u) B. Smoking induces a hypoxic state, leading to polycythemia. However, because of increased carbon monoxide, there is still a relative hypoxemia rather than increased levels of oxygen. (u) C. Smoking leads to increased blood viscosity, not decreased. (c) D. Cigarette smoking induces a chronic inflammatory state, which is believed to contribute to atherogenesis.

A 58 year-old male presents complaining of anterior right shoulder pain the day after performing extensive yard work. The pain is localized over the anterior proximal humerus with distinct point tenderness. There are no visible abnormalities. The patient has full range of motion and strength with all shoulder movements. The pain is reproduced by asking the patient to resist the examiner during supination of the right elbow. Which of the following is the most likely diagnosis? A. supraspinatus tendonitis B. subacromial bursitis C. rotator cuff tear D. bicipital tendonitis

(u) A. Supraspinatus tendonitis, subacromial bursitis and rotator cuff tear usually present with pain in the area of the deltoid muscle, limited abduction and are reproduced through impingement tests that narrow the space between the acromium and the humerus thereby impinging the supraspinatus tendon. (u) B. See A for explanation. (u) C. See A for explanation. (c) D. Bicipital tendonitis presents with anterior shoulder pain that is reproduced by palpating the tendon in the humeral bicipital groove and through resisted motion of the biceps muscle (elbow flexion or supination).

A 68 year-old male with history of COPD is brought to the emergency department following a motor vehicle collision. On physical examination there is evidence of head trauma. The left side of the chest wall appears to move inward with inspiration and outward with expiration. A chest x-ray reveals multiple rib fractures on the left. Which of the following is the most appropriate intervention? A. Surgical fixation of the fractured ribs B. Application of elastic binders and adhesive tape C. Endotracheal intubation and mechanical ventilation D. Chest physiotherapy that encourages frequent coughing

(u) A. Surgical fixation of flail chest is less reliable than positive-pressure ventilation and is performed only rarely in the United States. (u) B. While application of elastic binders and adhesive tape was historically utilized to stabile the chest, this intervention has been found to decrease chest expansion and worsen lung atelectasis. (c) C. Indications for early endotracheal intubation and mechanical ventilation in treatment of flail chest include patients that are over the age of 65, have comorbid lung disease and associated severe head trauma. Other indications include shock, three or more associated injuries and fracture of eight or more ribs. (u) D. Conservative treatment for mild to moderate flail chest includes analgesic relief of pain, chest physiotherapy that encourages frequent coughing and restriction of fluids to prevent fluid overload, however this patient needs early ventilatory support.

A 26 year-old man is stung by a bee, and shortly thereafter, a wheal develops at the site of the sting. He soon feels flushed and develops hives, rhinorrhea, and tightness in the chest. He is seen in the urgent care center. Immediate therapy should be to A. transfer him to a local hospital emergency department. B. apply a cold compress to site of the sting. C. administer subcutaneous epinephrine. D. administer oral albuterol.

(u) A. Systemic (anaphylactic) reactions can rapidly become life-threatening. Delay in treatment may cause death. (u) B. This is only supportive local therapy and does not address the need to treat the systemic reaction present. (c) C. Epinephrine hydrochloride 1:1000, 0.2 to 0.5 mL subcutaneously is indicated for the initial treatment of this systemic reaction. Additional injections may be given every 20 to 30 minutes if needed. (u) D. Albuterol is indicated in the presence of bronchospasm (suggested by the presence of chest tightness), but would be delivered by an aerosol, not an oral, route.

The main complication with the use of transjugular intrahepatic portosystemic shunt (TIPS) procedure is which of the following? A. increased portal pressures resulting in further esophageal varices B. increased portal pressures resulting in a worsening of cirrhosis C. Budd-Chiari syndrome D. increased risk of encephalopathy

(u) A. TIPS procedures are performed in order to shunt blood away from the liver parenchyma which in essence lowers portal pressures lessening the risk for esophageal varices. (u) B. TIPS procedures, since they cause of bypass of the liver parenchyma, result in a lessening of the blood flow into the liver which does not cause cirrhosis to progress. (u) C. Budd-Chiari syndrome is a thrombosis of the hepatic vein. It is not a common complication of the TIPS procedure. (c) D. TIPS procedures involve the placement of a stent in the liver in order to shunt blood away from the portal vein into the hepatic vein which bypasses the cirrhotic liver parenchyma. Its main complication is encephalopathy from the accumulation of toxic substances in the brain since the liver no longer acts as a filter.

Which hormone is responsible for producing 1,25-dihydroxyvitamin D (calcitriol) in the kidney? A. TSH B. ACTH C. PTH D. CRH

(u) A. TSH is thyroid stimulating hormone and comes from the pituitary gland. It is responsible for the production of T4 within the thyroid. (u) B. ACTH is adrenocorticotropin hormone and comes from the pituitary gland. It is responsible for the production of glucocorticoids within the adrenal gland. (c) C. PTH is parathyroid hormone and comes from the parathyroid glands. It is responsible in making calcitriol which is needed for calcium absorption in the gut. Under normal renal function, 25-hydroxyvitamin D is converted to 1,25-dihydroxyvitamin D in the distal convoluted tubule. (u) D. CRH is corticotropin releasing hormone and comes from the hypothalamus. It is the stimulator hormone to the pituitary gland which ultimately is the precursor to ACTH.

A 15 year-old man comes to the office with acute onset of nausea, severe pain and swelling within the right testis. He has no fever or irritative voiding symptoms. Which of the following scrotal physical examination findings would you expect in this patient? A. Nodular mass within the testis B. High-lying testis C. Transilluminated fluid mass D. Palpation of a soft bag of worms

(u) A. Testicular cancer is characterized by a painless nodular enlargement of the testis typically discovered by the patient. (c) B. Testicular torsion occurs in the 10-20 year age group, and is characterized by acute onset of severe pain and swelling of the testis. Physical examination will reveal a tender high-riding testis. (u) C. A hydrocele is a collection of fluid within the scrotum which can be transilluminated on exam. (u) D. A varicocele refers to varicose veins of the spermatic cord. It feels like a soft bag of worms in the scrotum separate from the testis.

Which of the following hormones primarily inhibits growth hormone secretion from the pituitary gland? A. Testosterone B. Somatostatin C. Dopamine D. Insulin-like growth factor

(u) A. Testosterone has no relationship to growth hormone. (c) B. Somatostatin inhibits the release of growth hormone from the pituitary gland as well as hyperglycemic states. It is therefore useful in the treatment of excessive growth hormone release that occurs with gigantism and acromegaly. (u) C. Dopamine is a vasoactive substance whose main effect is to cause vasoconstriction making it useful in treating hypotension that does not respond to a fluid challenge. (u) D. Insulin-like growth factor parallels the release of growth hormone and growth hormone itself stimulates the release of insulin-like growth factor from the liver and other tissues.

A blacksmith presents to your clinic after feeling like something went in his eye while he was grinding on a piece of metal. You stain the eye with Fluorescein and can visualize uptake with what appears like a deep abrasion. You are unable to visualize any foreign body with your indirect ophthalmoscope. Although he makes an attempt, he complains of some visual loss in that eye. His last Tetanus booster was 4 years ago. What is the most appropriate next step in the management of this patient? A. Administer tetanus B. Prescribe Tetracaine ophthalmic C. Provide reassurance only D. Refer to ophthalmologist

(u) A. Tetanus booster is only required every 5 years. (u) B. See D for explanation. (u) C. See D for explanation. (c) D. Intraoccular foreign body requires emergency treatment by an ophthalmologist. Patients giving a history of something hitting the eye, particularly while hammering on metal or using grinding equipment, must be assessed for the possibility of intraoccular foreign body especially when no corneal foreign body is seen, a corneal or scleral wound is apparent, or there is marked visual loss or media opacity.

Which of the following primitive reflexes should begin to disappear at about 2 months of age in a normal infant? A. Moro B. Grasp C. Tonic neck D. Parachute

(u) A. The Moro reflex starts to disappear at about 5-6 months of age. (c) B. The grasp reflex starts to disappear at about 2-3 months of age. (u) C. The tonic neck reflex starts to disappear at about 6-7 months of age. (u) D. The parachute reflex remains throughout life.

Which of the following tests is the most specific for the diagnosis of syphilis? A. Rapid plasma reagin (RPR) B. Weil-Felix agglutination test C. Venereal Disease Research Laboratory (VDRL) D. Fluorescent treponemal antibody absorption (FTA-ABS)

(u) A. The RPR is a non-specific test. False positives are common. (u) B. The Weil-Felix agglutination test is used for rickettsial infections not syphilis. (u) C. The VDRL if positive must by confirmed with an additional testing because of a large number of false positives including bacterial and viral infections, pregnancy, chronic liver disease, connective tissue disorders. (c) D. The FTA-ABS and the MTA-TP are specific treponemal tests used for the confirmation of syphilis.

A 22 year-old male presents to the ED after sustaining a blow to the knee during football practice. The knee exam demonstrates significant forward translation of the tibia when the knee is in 20 degrees of flexion and the tibia is forced forward while the femur is stabilized. Which of the following knee maneuvers does this represent? A. abduction stress test B. anterior drawer sign C. Lachman test D. McMurray test

(u) A. The abduction stress test is performed to evaluate medial collateral ligament tears while applying valgus stress. (u) B. The anterior drawer sign is performed to evaluate the anterior cruciate ligament; however the patient is supine, hips and knees flexed, and feet are flat on the table. (c) C. The Lachman test is performed to evaluate the anterior cruciate ligament. The knee is placed in 20 degrees of flexion. (u) D. The McMurray test is performed to evaluate medial and lateral meniscal tears while rotating the lower leg internally and externally.

A 63 year-old man comes to the office to discuss treatment for erectile dysfunction. He is interested in learning more about the medication sildenafil (Viagra). He has a history of coronary artery disease, asthma and benign prostatic hyperplasia. Which of the following medications is contraindicated (category X) with Viagra due to the potential drug interaction? A. Enalapril (Vasotec) B. Albuterol C. Finasteride (Proscar) D. Nitroglycerin

(u) A. The additive effect of sildenafil on enalapril can potentiate the anti-hypertensive effect. This is a category C interaction; monitor therapy as dosage adjustments may be required. (u) B. There is no interaction between albuterol and sildenafil. (u) C. There is no interaction between finasteride and sildenafil. (c) D. The additive effect of sildenafil on nitrates can amplify cardiac preload reduction and hypotension. Sildenafil is contraindicated in patients taking nitrates.

A 3 year-old child playing in an abandoned shed is bitten by a black widow spider. The mother rushes the child to the emergency department within 20 minutes of the incident. Which of the following if the best initial intervention? A. Intramuscular steroids B. Administration of antivenin C. Immediate immersion in a cold bath D. Hospital admission for symptomatic care

(u) A. The bite of this spider does not cause an inflammatory reaction; therefore, steroids are not indicated. (u) B. Administration of antivenin should be administered to patients with severe envenomation manifested as seizures, respiratory failure, or hypertension. (u) C. Application of an ice pack initially would be helpful for pain relief, however, immersion in a cold bath may cause hypothermia. (c) D. Hospital admission for symptomatic care should be considered in children, pregnant women, and patients with preexisting cardiovascular disease.

Which of the following is the most common presenting clinical manifestation of breast cancer? A. breast tenderness B. nipple discharge C. nipple retraction D. breast mass

(u) A. The breast mass is usually painless; therefore, breast tenderness would not be present. (u) B. While serous or bloody nipple discharge may be present, it is not the most common presenting clinical manifestation of breast cancer. (u) C. Nipple retraction is a later finding of breast cancer and indicates ductal involvement. (c) D. A breast mass is the most common presenting clinical manifestation of breast cancer found by the patient or health care provider.

A 26 year-old woman requests screening after her boyfriend was treated for a sexually transmitted infection recently. On examination you find a painless vulvar ulcer. Which of the following is the most likely diagnosis? A. Herpes B. Syphilis C. Chancroid D. Granuloma inguinale

(u) A. The classic presentation of herpes is a painful vesicle. (c) B. The primary lesion of syphilis presents as a painless ulcer or chancre. Secondary syphilis presents with a skin rash lymphadenopathy and mucocutaneous lesions. (u) C. Chancroid presents with a painful genital ulcer and tender suppurative inguinal adenopathy. (u) D. Granuloma inguinale presents with raised, red lesions that bleed easily.

Formications are most commonly associated with which of the following? A. Delusional disorder B. Adverse drug reaction C. Alcohol withdrawal D. Obsessive compulsive disorder

(u) A. The diagnosis of a delusional disorder requires the presence of nonbizzarre delusions of at least one months duration that are not attributed to another disorder. (u) B. Adverse drug reactions are not known to cause formication. (c) C. Formications are the sensation of insects crawling on the skin and is commonly associated with delirium tremens from alcohol withdrawal and cocaine addiction. (u) D. Patients with OCD do not have formication with their disorder.

You are evaluating a 67 year-old Asian male in the emergency room for acute onset right eye pain. He states he was at the evening premier of a newly released movie when the pain started. He had acute, profound visual loss in the affected eye. The pain was intense enough for him to leave the theatre before the movie's conclusion, and present to your location. On examination, the eye appears injected (red) and the cornea appears hazy. His pupils are 6 mm on the affected side and 3 mm on the unaffected side. They respond to light on the unaffected side but not on the affected side. On palpation, the globe feels tense. What history question is most relevant to support the diagnosis? A. Contact lens use B. Past sexual contacts C. Recent URI symptoms D. Visualizing halos around street lights

(u) A. The major risk from contact lens wear is bacterial, amebic, or fungal corneal infection, potentially a blinding condition. In this condition the eye may appear red, however the cornea would be clear, and the globe would not be tense. (u) B. Past sexual contacts would be related to pupillary abnormalities associated with neurosyphilis. (u) C. URI symptoms would be considered when associated with conjunctivitis. Pupil size is normal and is the pupillary light response. Intraoccular pressure is normal. (c) D. Primary acute angle-closure glaucoma occurs only with closure of a preexisting narrow anterior chamber angle found in older age groups, hyperopes, inuites, and Asians. Angle closure may be precipitated by pupillary dilation and thus can occur from sitting in a darkened room, at times of stress or, rarely, from pharmacologic mydriasis. The symptoms given are classic for acute angle-closure glaucoma (older age group, Asian, rapid onset of severe pain and profound visual loss with halos around light, red eye, steamy cornea, dilated pupil, hard eye to palpation).

Which of the following drugs is first-line therapy for schizophrenia? A. Chlorpromazine (Thorazine) B. Clozapine (Clozaril) C. Haloperidol (Haldol) D. Olanzapine (Zyprexa)

(u) A. The older, traditional antipsychotic agents, such as haloperidol and chlorpromazine have higher risk of side effects, including acute motor system side effects a long-term risk of tardive dyskinesias, and should not be considered as first-line drugs. (u) B. Clozapine should not be considered a first-line therapy because of its hematopoietic and hepatic side effects. (u) C. See A for explanation. (c) D. Initial pharmacologic therapy of schizophrenia should begin with one of the newer, "atypical" antipsychotic drugs, such as olanzapine, risperidone, quetiapine, ziprasidone, and clozapine because their side effect profile is significantly better than the older drugs, and they may be more effective for negative psychotic symptoms.

A 36 year-old male has a history of recurrent low back pain. When lifting a stack of books yesterday, he experienced sudden, severe pain in the lumbar area. He denies radicular pain. His lower extremity examination is unremarkable, and his back examination is remarkable for paraspinal muscle tenderness and increased pain with flexion at the waist. Which of the following management options should be instituted at this time? A. refer the patient to orthopedic surgery B. continue ordinary activities as tolerated C. confine the patient to bed with traction D. recommend sleeping on a softer mattress

(u) A. The patient can be managed conservatively by a primary care provider since there are no neurological deficits. (c) B. Continuation of activities as tolerated is recommended during the acute phase. After symptoms resolve, an exercise program should be initiated to strengthen the back. (u) C. Traction is an outdated method of treatment, while bed rest for more than a few days will cause muscle atrophy. (u) D. Management for low back strain includes using a firm mattress.

A 45 year-old type 2 diabetic female with history of cholelithiasis presents to the clinic with 2-3 episodes of sudden, severe epigastric pain that radiates to her shoulder. She has associated nausea and vomiting. Temperature is 101 degrees F and she is experiencing chills. Today her eyes appear yellow in color. Which of the following is the most likely diagnosis for this patient A. Postcholecystectomy syndrome B. Cholangitis C. Gastroesophageal reflux disease D. Pancreatic cancer

(u) A. The patient has no history of previous gall bladder surgery. (c) B. Cholangitis is characterized by a history of biliary pain, fever, chills, and jaundice associated with episodes of abdominal pain. (u) C. Gastroesophageal reflux disease (GERD) is characterized by heartburn. Fever and jaundice are not typical features of GERD. (u) D. Pancreatic cancer, although a possibility, is characterized by chronic weight loss, epigastric pain radiating to the back, and occasional jaundice. Fever and chills are not typical features.

: (8) 129. History & Physical/Neurology A 30 year-old male presents with a 1 week history of intermittent left orbital headache. He states that his episodes occur at night and last approximately 1 hour. Examination reveals ptosis, miosis and dyshidrosis. Which of the following would most likely be present in this patient? A. Loss of vision B. Scotoma C. Nasal lacrimation D. Facial muscle spasm

(u) A. The patient is experiencing a cluster headaches; visual changes are not commonly associated. (u) B. Scotoma occurs in migraines and can occur with optic disc inflammation. (c) C. This patient is exhibiting signs and symptoms of cluster headache which often includes nasal lacrimation. (u) D. Facial muscle spasms occur in trigeminal neuralgia.

: (9) 42. Clinical Intervention/Cardiology You are evaluating a patient who was brought in secondary to an acute onset of repeated syncopal episodes. His electrocardiogram (ECG) shows wide QRS complexes with a fixed R-R interval at a rate of 40 bpm. The P waves occur with a fixed P-P interval at a rate of 70 bpm. The PR interval is variable. Which of the following is the most appropriate initial treatment for this condition? A. Balloon angioplasty B. Endarterectomy C. Transthoracic pacemaker D. Unsynchronized cardioversion

(u) A. The patient with a 3rd degree heart block would gain no benefit from balloon angioplasty. (u) B. The patient with a 3rd degree heart block would gain no benefit from endarterectomy. (c) C. The ECG this patient's symptoms are consistent with a third degree heart block. Patients with episodic or chronic infranodal complete heart block require permanent pacing, and temporary pacing is indicated if implantation of a permanent pacemaker is delayed. (h) D. Unsynchronized cardioversion in a patient with third degree heart block can result in a fatal ventricular fibrillation conversion.

A 26 year-old female presents with several pruritic lesions on her dorsal forearms. The lesions are erythematous with vesicles, with a few beginning to weep. She works in a photography laboratory, but denies any other possible exposures. Which of the following is the most useful diagnostic test? A. VDRL serology B. KOH prep C. Patch testing D. Gram's stain

(u) A. VDRL serology is useful in the diagnosis of syphilis, not contact dermatitis. (u) B. KOH prep is used for diagnosis of fungal infections, not contact dermatitis. (c) C. Patch testing with a suspected agent is usually positive in cases of allergic contact dermatitis. (u) D. Gram's stain is useful in the diagnosis of bacterial infections, not contact dermatitis.

225. Diagnostic Studies/Gastrointestinal/Nutritional A 25 year-old woman complains of bloating, abdominal cramps, and loose stools with flatulence. Her symptoms seem to occur 1-2 hours after meals containing any dairy products. Which of the following would be the most appropriate diagnostic test to confirm your suspicion? A. Colonoscopy B. Fecal smear for leukocytes C. Hydrogen breath test D. Serum IgA endomysial antibody

(u) A. The symptoms are due to a malabsorption of a selective carbohydrate (lactose) in the small intestine. Colonoscopy will not elucidate the cause. (u) B. Fecal leukocytes indicate diffuse colonic inflammation and is ordered in more moderate or severe cases of infectious diarrhea (bacterial). This patient's symptoms are associated with particular enzyme deficiency. (c) C. This woman most likely has lactase deficiency as her symptoms occur after dairy ingestion. Hydrogen breath test after administration of lactose will confirm the diagnosis. (u) D. Serum EMA IgA antibody is a screening test for gluten-sensitive enteropathy; symptoms occur due to ingestion of wheat not dairy products.

: (12) 145. History & Physical/Cardiology A 13 year-old patient is hospitalized with a fever of 102.5 F and a rash. After 36 hours the rash has rapidly progressed to enlarging macules that appear ring orcrescent shaped with central clearing. He also complains of multiple arthralgias involving his ankles, knees, and now his elbows. The electrocardiogram shows evidence of a first degree AV block. Labs were significant for an elevated erythrocyte sedimentation rate and leukocytosis. Which of the following physical examination findings would be most likely in this patient? A. Cord-like palpable calf vein B. Diminished lower extremity pulses C. Mitral regurgitation murmur D. Oral cyanosis

(u) A. The symptoms are most reflective of rheumatic fever. A mitral regurgitation murmur would coincide with this diagnosis. (u) B. The symptoms are most reflective of rheumatic fever. A mitral regurgitation murmur would coincide with this diagnosis. (c) C. Jones criteria of rheumatic fever include 1 major (erythema marginatum) and 3 minor (fever, polyarthralgies, prolonged PR interval). Cardiovascular manifestations typically associated with rheumatic fever include mitral regurgitation. (u) D. The symptoms are most reflective of rheumatic fever. A mitral regurgitation murmur would coincide with this diagnosis.

Which of the following drugs is the first choice for insomnia in an elderly patient? A. Barbiturates B. Cholinesterase inhibitors C. Benzodiazepines D. Beta-blockers

(u) A. The use of barbiturates is obsolete and these agents may actually disrupt the sleep cycle. (u) B. Cholinesterase inhibitors are used in the treatment of Alzheimer's, but have no effect on insomnia. (c) C. Benzodiazepines are the drugs of choice for insomnia in the elderly population. (u) D. Beta-blockers have no use in the treatment of insomnia.

A 58 year-old female presents with acute onset of 105 degrees F fever, chills, delirium and tachycardia. Laboratory analysis reveals a TSH of 0.08 mcU/L (0.4-6.0 mcU/L), total T3 of 400 ng/dL (95-190 ng/dL)and a total T4 of 180 mcg/dL (5-11 mcg/dL). What is the initial treatment of choice to normalize this condition? A. Hydrocortisone B. Propylthiouracil C. Digoxin D. Levothyroxine

(u) A. The use of steroids is common in thyroid storm, but it is not the best initial treatment. (c) B. Propylthiouracil is the preferred initial drug in patients with thyroid storm. (u) C. Digoxin is not the treatment of choice for symptoms related to thyroid storm. It could be used if heart failure was associated. (h) D. Levothyroxine would actually worsen this condition.

Which of the following is an independent risk factor for development of a mesothelioma? A. Cigarette smoking B. Asbestos exposure C. Radon gas exposure D. Chronic obstructive lung disease

(u) A. There has not been any evidence of association between cigarette smoking and the development of mesothelioma. (c) B. Studies confirm the association of asbestos exposure to the development of mesothelioma. (u) C. After cigarette smoking, radon gas is the second most common risk factor for development of bronchogenic lung cancer, not mesothelioma. (u) D. Chronic obstructive lung disease is associated with an increased risk of bronchogenic lung cancer, not mesothelioma.

: (27) 123. Health Maintenance/Urology/Renal Your practice routinely screens its male patients for prostate cancer according to US Preventive Services Task Force guidelines. A 71 year-old Caucasian male presents with a 50 pack-year history of tobacco use, previous treatment for prostatitis, and a paternal uncle with prostate cancer at age 65. He currently is sexually active with multiple female partners, eats a high fat diet, and is being treated for benign prostatic hyperplasia. Which of the following factors increases his risk of developing prostate cancer? A. Tobacco use B. Prior prostatitis C. Current BPH D. High-fat diet

(u) A. There is no demonstrated association between prostate cancer and cigarette smoking, sexual activity, prior history of prostatitis or BPH. (u) B. See A for explanation. (u) C. See A for explanation. (c) D. A high-fat diet has been implicated in the development of prostate cancer in some studies. This patient already carries the risk factors of age and family history.

Which of the following strategies promotes improved carbohydrate metabolism and is recommended for all Type 2 diabetic patients? A. Low-carbohydrate, high protein diet B. Routine aerobic exercise C. Metformin (Glucophage) D. Acupuncture

(u) A. There is no evidence that a low-carbohydrate, high protein diet is effective in improving carbohydrate metabolism. (c) B. Routine exercise improves carbohydrate metabolism and insulin sensitivity. (u) C. Metformin is a biguanide that primarily works to decrease gluconeogenesis rather than affecting carbohydrate metabolism. (u) D. There is no evidence that acupuncture improves carbohydrate metabolism.

A 40 year-old patient with a history of recurrent kidney stones presents with acute onset of right flank pain and hematuria. The patient is afebrile and pain is poorly controlled on oral medications. On CT scan a 1 cm stone is noted in the renal pelvis. Which of the following is the most appropriate intervention for this patient? A. Antibiotics B. Shock wave lithotripsy C. Ureterolithotomy D. Fluid hydration

(u) A. There is no indication of infection and antibiotics are not warranted at this time. (c) B. Extracorporeal shock wave lithotripsy is indicated in patients with stones greater than 6 mm in size or intractable pain. (u) C. Ureterolithotomy is an open surgical procedure and therefore very invasive. In this situation an ESWL would be as effective and safer. (u) D. While fluid hydration is indicated, a stone greater than 6 mm will typically not spontaneously pass and surgical therapy is indicated.

When instituting diuretic therapy for patients with heart failure, which of the following is considered the treatment of choice as first-line therapy in a failing kidney due to its improved sodium clearance? A. Hydrochlorothiazide (Diuril) B. Bumetanide (Bumex) C. Spironolactone (Aldactone) D. Acetazolamide (Diamox)

(u) A. Thiazide diuretics may have better hypertension control than the short acting loop diuretics but they are generally ineffective when the glomerular filtration rate falls below 30-40 mL/min. (c) B. Loop diuretics remain active in severe renal insufficiency and are the most effective type of diuretics used in the management of heart failure symptoms. These agents have a rapid onset and result in natriuresis due to their activity in the ascending limb in the Loop of Henle. (u) C. Potassium-sparing diuretics have very weak diuretic properties and are useful as adjunctive therapy in patients with Stage 3 or Stage 4 heart failure who are already on a first-line diuretic and other agents. (u) D. Acetazolamide is a carbonic anhydrase inhibitor and works by causing a metabolic acidosis with loss of bicarbonate in the failing kidney. It is a weak diuretic and does not work in the setting of a failing kidney.

A mother brings in her five year-old boy for his school physical. She voices some concerns about his readiness for school, saying he seems to be socially immature. She has noticed he does not interact with other children well, and that when he plays with them, he has a tendency to "place them" and then run around them as if they were statues. He rarely cries when he is hurt, and he shrugs off any attempt to hug him. He has good attention to details, and will sit and draw the same geometric shapes over and over again, but does not seem interested in learning the alphabet. He avoids eye contact with anyone. Which of the following is the most likely diagnosis? A. Normal 5 year-old B. Social phobia C. Autism D. Avoidant personality

(u) A. This behavior is not normal for a child this age. (u) B. Social phobia is an excessive and persistent fear of social situations in which the person may be scrutinized by others. (c) C. Children with autism do not tend to make eye contact, and even avoid it. They do not accept comfort when hurt and stiffen up when hugged. They do not tend to play with others, and do not tend to imitate grown-ups in play. They approach play in a more mechanical way, using others as props rather than interacting with them. (u) D. Avoidant personality disorder is characterized by timidity, social awkwardness, and a pervasive sense of inadequacy and fear of criticism.

A 66 year-old male patient has both hypertension and benign prostatic hypertrophy. Which of the following medications would most likely benefit both conditions? A. Finasteride (Propecia) B. Verapamil (Calan) C. Propanolol (Inderal) D. Prazosin (Minipress)

(u) A. This drug could help to relieve the symptoms of benign prostatic hypertrophy (BPH) however it has no benefit for hypertension. (u) B. This drug could help the HTN however it would have no benefit of helping the BPH. (u) C. See B for explanation. (c) D. This drug has antihypertensive properties as well as properties to relieve the symptoms of BPH.

Which of the following physical exam findings suggests worsening or severe aortic stenosis? A. An ejection click preceding the murmur B. A split S2 with respiration variation C. Grade 2/6 systolic murmur radiating to the carotids D. Palpable thrill over the right second intercostal space

(u) A. This finding is typical in mild to moderate AS. (u) B. This can be a normal finding. (u) C. This is the typical murmur for aortic stenosis, but does not suggest the degree of severity by the grade. (c) D. A palpable thrill or LV heave with associated murmur suggests severe AS.

A patient had an acute inferior, transmural myocardial infarction 4 days ago. A new murmur raises the suspicion of mitral regurgitation due to papillary muscle rupture. Which of the following murmur descriptions describes this condition? A. A grade III/VI diastolic murmur heard best at the apex without radiation. B. A grade IV/VI systolic ejection murmur heard best at the base with radiation to the left clavicle. C. A grade II/VI systolic murmur heard best at the apex preceded by a click and without radiation. D. A grade IV/VI systolic murmur heard best at the apex with radiation to the left axilla.

(u) A. This is a classic description of mitral stenosis. (u) B. This is a classic description for pulmonic stenosis. (u) C. This is a classic description for mitral valve prolapse. (c) D. This is a classic description of mitral regurgitation. The papillary muscle rupture is a complication of an acute inferior transmural myocardial infarction, and results in a failure of the mitral valve leaflets to close. The direction of regurgitant flow of blood is toward the left axilla

A 23 year-old male presents to the emergency department with severe right shoulder pain. He is holding his shoulder in internal rotation. Which of the following would increase your suspicion that he has a posterior glenohumeral dislocation? A. History of a direct posterior blow to the humerus B. History of seizure C. Palpable mass in the anterior axilla D. Palpable space beneath the acromion

(u) A. This is a common mechanism for anterior glenohumeral dislocation. (c) B. Tonic-clonic seizures can create severe contraction of the internal rotator muscles of the shoulder. (u) C. This is a classic finding associated with anterior dislocation. The mass is the humeral head outside of the glenoid. (u) D. This is also a classic finding associated with anterior dislocation and the space is where the humeral head should be in a non-dislocated shoulder.

Which of the following results from hypersecretion of growth hormone in a 27 year-old patient? A. Addison's disease B. Myxedema C. Acromegaly D. Cushing's syndrome

(u) A. This is a disease of low cortisol production by the adrenal cortices. (u) B. This occurs secondary to severe deficiency of T3 and T4. (c) C. This disease results from excessive growth hormone production generated by the anterior pituitary or an ectopic source. (u) D. Excessive cortisol production is the hallmark of this process.

A 72 year-old man with a long-standing history of diabetes mellitus, renal insufficiency, and hypertension presents to the clinic complaining of burning and tingling pain in his feet. Which of the following medications would help control pain in this patient? A. Phenobarbital B. Amitriptyline (Elavil) C. Celecoxib (Celebrex) D. Codeine

(u) A. This is an antiepileptic medication that does not help diabetic neuropathy. (c) B. Amitriptyline has been recommended for pain associated with diabetic neuropathy. (h) C. Celecoxib is not indicated for diabetic neuropathy and should not be used if there is decreased kidney function. (u) D. Treatment with codeine puts the patient at risk for dependence and this does not help the actual neuropathic pain.

A 75 year-old female with history of coronary artery disease and dyslipidemia presents for routine follow-up. Physical examination reveals loss of hair on the lower extremities bilaterally with thinning of the skin. Femoral pulses are +2/4 bilaterally, pedal pulses are diminished bilaterally. Ankle brachial index is reduced. Which of the following signs or symptoms is this patient most likely to have? A. Lower extremity edema B. Calf pain with walking C. Numbness of the lower extremities D. Itching of the lower extremities

(u) A. This patient has signs and symptoms consistent with arterial insufficiency. Lower extremity edema is seen in patients with venous insufficiency. (c) B. This patient has signs and symptoms consistent with arterial insufficiency and would most likely complain of intermittent claudication. (u) C. Numbness of the lower extremities would be seen with acute arterial occlusion. (u) D. Itching of the lower extremities may be seen in chronic venous insufficiency because of secondary skin changes, but is not common in arterial insufficiency.

A 26 year-old male who is an avid swimmer has been experiencing right shoulder pain for the past month. On examination, pain is elicited with palpation below the anterior acromion. Anterior shoulder pain is also reported when the patient flexes and extends his arm. Which of the following diagnostic tests is most appropriate at this time? A. Shoulder x-ray B. Shoulder arthroscopy C. Shoulder MRI D. Subacromial lidocaine injection

(u) A. This patient is experiencing mild impingement syndrome and, although plain films may demonstrate bone spurs on the undersurface of the acromion, his age and relatively recent onset of symptoms suggest that this study would likely be normal. (u) B. Arthroscopy is not indicated unless less invasive modalities fail to aid in the diagnosis and symptoms of impingement persist despite therapy. (u) C. While MRI would likely support the diagnosis, it is an expensive modality and not necessary to diagnose mild impingement. (c) D. Subacromial injection of lidocaine leading to a transient but dramatic improvement in pain with shoulder extension makes the diagnosis of impingement highly likely.

A 44 year-old female presents to clinic for evaluation of a syncopal episode that occurred while walking her dog two days ago. She denies amnesia or head trauma. She has had increasing dyspnea on exertion and pedal edema. Physical examination reveals clubbing of her fingers and central cyanosis. Auscultation of the heart reveals tricuspid insufficiency, widely split second heart sound with a palpable P2. Echocardiogram reveals a large ostium secundum atrial septal defect with bidirectional flow. Which of the following is a secondary complication in this patient? A. Left heart failure B. Ebsteins anomaly C. Tricuspid stenosis D. Pulmonary hypertension

(u) A. This patient would more likely have right heart failure than left heart failure. (u) B. Ebsteins anomaly is apical displacement of the septal tricuspid leaflet and not caused by an ASD. (u) C. Tricuspid stenosis is not caused by an ASD. (c) D. Her symptoms and exam findings are consistent with pulmonary hypertension and in her case, Eisenmengers disease, which is a late finding.

A 55 year-old diabetic female presents for a 3 month blood pressure follow-up. At the last visit the BP was 160/90 for the third consecutive visit. She was placed on an ACE inhibitor and educated regarding lifestyle modifications. At today's visit the patient complains of persistent annoying dry cough that has been going on since the last visit. BP today is 120/70. What is the best recommendation to control her BP? A. add a diuretic B. stop the ACE inhibitor and continue lifestyle modifications C. switch patient to an Angiotensin II Receptor Blocker (ARB) D. do nothing and recheck BP in 3 months

(u) A. This patient's blood pressure is controlled; there is no indication at this time to add an additional drug. (u) B. This patient's chronic dry cough is likely secondary to the ACE inhibitor, the medication should be stopped, however the patient needs something for blood pressure control. (c) C. This patient's chronic dry cough is likely secondary to the ACE inhibitor, the medication should be stopped. Angiotensin II Receptor Blockers (ARBs) are similar to ACE inhibitors for BP control, but do not cause cough. (u) D. This patient's chronic dry cough is likely secondary to the ACE inhibitor, the medication should be stopped to encourage compliance.

Which of the following physical findings suggest pernicious anemia? A. Splenomegaly and hepatomegaly B. Petechiae and ecchymosis C. Loss of position and vibratory sensation D. Cheilosis and koilonychia

(u)A. Splenomegaly and hepatomegaly are typically seen in hemolytic anemias. (u) B. Petechiae and ecchymosis are seen in thrombocytopenia. (c) C. Loss of position and vibratory sensation are common neurologic findings in pernicious anemia. (u) D. Cheilosis and koilonychia are seen in iron deficiency anemia.

An 18 year-old sexually active female was seen in the student health clinic 1 week ago for a sore throat. A streptococcal antigen test was positive, and she was given a prescription for oral penicillin. After 3 days, she stopped her medication because she felt better. She now presents with a severe sore throat. On physical examination, she has a temperature of 102.6° F (39.2° C), marked pharyngeal erythema, medial deviation of the soft palate on the left, tender left anterior cervical adenopathy, and a "hot potato" voice. The rest of her history and physical examination are unremarkable. Which of the following is the most likely diagnosis? A. Recurrent streptococcal pharyngitis B. Infectious mononucleosis C. Gonococcal pharyngitis D. Peritonsillar abscess

(u) A. This presentation suggests a complication of an incompletely treated streptococcal pharyngitis rather than recurrent disease. (u) B. Infectious mononucleosis may present with severe sore throat, fever, and cervical adenopathy in this age group, but would not cause deviation of the soft palate or the muffled voice. (u) C. Gonococcal pharyngitis usually follows a more indolent course than this patient's presentation. (c) D. The soft palate deviation and a muffled voice are classic signs of peritonsillar abscess.

A 22 year-old G1P0 female presents at 12 weeks gestation with 24 hours of vaginal bleeding. She complains of continued cramping and bleeding requiring pad change every two hours. Vital signs are normal. Vaginal examination shows bleeding with a dilated cervix. Which of the following is the most likely diagnosis? A. Threatened abortion B. Inevitable abortion C. Incomplete abortion D. Complete abortion

(u) A. Threatened abortion (AB) implies the cervix remains closed with only slight bleeding. (c) B. Abortion is inevitable when cervical effacement, dilatation and rupture of membranes is noted. (u) C. Incomplete AB implies the products of conception have partially passed causing continued bleeding, more common after 10 weeks. (u) D. Complete AB is identified by passage of the entire conceptus.

A 25 year-old female, G2 P1001, presents to your office at 11-weeks gestation with vaginal bleeding, mild lower abdominal cramping, and bilateral lower pelvic discomfort. On examination, blood is noted at the dilated cervical os. No tissue is protruding from the cervical os. The uterus by palpation is 8-9 weeks gestation. No other abnormalities are found. Which of the following is the most likely diagnosis? A. Threatened abortion B. Inevitable abortion C. Incomplete abortion D. Complete abortion

(u) A. Threatened abortion is characterized by bleeding in the first trimester without loss of fluid or tissue. (c) B. Inevitable abortion is the gross rupture of membranes in the presence of cervical dilation. (u) C. Incomplete abortion is when the cervical os is open and allows passage of blood. The products of conception may remain in utero or may partially extrude through the open os. (u) D. Complete abortion refers to a documented pregnancy that spontaneously passes all of the products of conception.

A 60 year-old male has unstable angina, but is otherwise healthy. A 90% lesion is found in the left main coronary artery. Which of the following interventions is most appropriate? A. Thrombolysis with t-PA B. Medical management with nitrates C. Coronary artery bypass graft (CABG) D. Percutaneous transluminal coronary angioplasty

(u) A. Thrombolysis is recommended in acute embolic occlusion, not chronic. (u) B. Medical management is appropriate only for patients who are not surgical candidates. (c) C. CABG is indicated in patients with stenosis of the left main coronary artery and those with three-vessel coronary artery disease. (u) D. Percutaneous transluminal coronary angioplasty is not the management of choice in left mainstem artery disease because of increased potential complications and mortality.

An x-ray taken on a patient complaining of wrist pain after being hit by a baseball reveals a non-displaced mid-shaft ulnar fracture. Which of the following splints is most appropriate for treatment? A. Thumb spica B. Sugar tong C. Cock-up wrist D. Short arm gutter

(u) A. Thumb spica splints are used for scaphoid fractures. (c) B. Sugar tong splints are best used to immobilize the elbow, wrist and forearm. (u) C. Cock-up wrist splints may be useful in some situations unrelated to fractures, such as to immobilize the wrist for tendinitis or to support it in the case of wrist drop due to radial nerve palsy but not to be used in wrist fractures. (u) D. Short arm gutter splints immobilize only the wrist and the ulnar or radial half of the hand.

A 73 year-old obese female with a 20 pack year smoking history presents complaining of chronic productive cough. She states that it has been occurring over the past 3 years more frequently November through February. Which of the following pulmonary function test values would you expect to find decreased? A. tidal volume B. forced expiratory volume in 1 second/forced vital capacity C. residual volume D. total lung capacity

(u) A. Tidal volume is usually unchanged, residual volume and total lung capacity are decreased with a restrictive disease pattern. This scenario presents with bronchitis which is an obstructive disease. (c) B. Forced expiratory volume in 1 second/forced vital capacity is decreased in obstructive lung diseases such as bronchitis. (u) C. See A for explanation. (u) D. See A for explanation.

A 15 year-old male is brought to the emergency department 1 hour after an episode of syncope while running in a 400-meter race. He had a similar episode 2 years ago. His mother and maternal first cousin died suddenly at the ages of 32 and 17 years, respectively. Examination shows abrasions of the face, hands, and knees. Neurologic examination shows no abnormalities. Which of the following is the most appropriate initial test? A. Tilt table test B. CT scan of the head C. Electrocardiogram D. Electroencephalography

(u) A. Tilt table testing should be performed after structural heart disease has been ruled out. (u) B. CT scanning of the head would not be the initial test of choice. (c) C. ECG is the most appropriate to rule out any underlying rhythm abnormalities, Holter or event monitoring would also be included. (u) D. EEG testing routinely would not be helpful as an initial test in this patient.

An 18 year-old young woman comes to the office with a mildly itchy rash on her chest and back for the past two weeks. She noticed an initial area on her back two weeks ago, and now it has spread across her back and chest. On physical examination, she has several salmon-colored plaques with fine scale that follow the cleavage lines of the trunk. Which of the following is the most likely diagnosis? A. Tinea versicolor B. Seborrheic dermatitis C. Psoriasis D. Pityriasis rosea

(u) A. Tinea versicolor is a mild infection characterized by hypopigmented macules on the trunk. This eruption is often asymptomatic-usually appearing during humid, warmer temperatures. It is most often on the upper trunk of older teenagers or young adults. Patients may complain of mild pruritis. (u) B. Seborrheic dermatitis consists of pruritic greasy scales and underlying erythematous patches or plaques on the scalp, central face, presternal areas, and upper back. (u) C. Psoriasis is characterized by chronic, recurring, erythematous papules and plaques with thick silvery white scale on the elbows, knees, and scalp. (c) D. Pityriasis rosea is an acute eruption of fine scaling fawn-colored papules and plaques that are distributed along the cleavage lines of the trunk, often referred to as a Christmas tree pattern. A single plaque, called a herald patch, precedes the secondary eruption by 1-2 weeks.

A patient with preterm labor may be given corticosteroids to A. decrease uterine activity. B. prevent chorioamnionitis. C. enhance fetal lung maturity. D. prevent the development of gestational diabetes.

(u) A. Tocolytics are given to decrease uterine activity with preterm labor. (u) B. IV antibiotics are given to patients with chorioamnionitis. (c) C. Corticosteroids may be given from 24-34 weeks in patients with preterm labor or who have pregnancy complications which may cause premature birth. The corticosteroids enhance pulmonary maturity. (u) D. There are no medications to prevent the development of gestational diabetes, however, patients who have gestational diabetes may be treated with a diabetic diet and/or insulin to manage the condition and decrease complications.

An 8 year-old male presents with episodes described as 20-second lapses of awareness during which he blinks his eyes. After these attacks, he resumes his previous activity. Which of the following is the most likely diagnosis? A. Tonic-clonic seizure B. Myoclonic seizure C. Absence seizure D. Focal seizure

(u) A. Tonic-clonic seizures are associated with a loss of consciousness, tonic-clonic movements, and postictal confusion or sleep. (u) B. Myoclonic seizures are associated with mild myoclonic jerks of the neck and shoulder flexor muscles when falling asleep. (c) C. Absence seizures are characterized by lapses of consciousness associated with postictal confusion. (u) D. Focal seizures are associated with motor or sensory symptoms involving any portion of the body.

: (8) 24. Health Maintenance/Infectious Diseases An 18 year-old college freshman presents to the student health clinic complaining of an outbreak of "athlete's foot" in his dormitory. He has no current signs or symptoms of infection, but is asking for advice on how to prevent contracting this condition. Which of the following is the best piece of preventative advice? A. Apply topical corticosteroids to any affected area B. Avoid excessive moisture and dry the skin carefully after bathing C. Avoid talc or other drying powders D. Apply a topical antifungal preventatively to area of concern

(u) A. Topical corticosteroids are effective for contact dermatitis but will not treat tinea pedis (a fungal infection) and may make the condition worse. (c) B. Personal hygiene is essential for prevention. Avoiding extra moisture and drying carefully between the toes after showering is effective prevention.Wear open toe shoes and change absorbent, nonsynthetic socks regularly. (u) C. Talc and drying agents can assist in keeping the affected area dry and prevent the development of this fungal infection. (u) D. Application of topical antifungal may be helpful to prevent recurrences, but not for preventative use.

A male patient presents with hematuria. Upon further questioning the patient states that the hematuria occurs at the end of his urinary stream. Which of the following is the most likely source of blood? A. renal pelvis B. bladder neck C. anterior urethra D. ureter

(u) A. Total hematuria, blood throughout the urinary stream, suggests a bladder or upper urinary tract source. (c) B. Terminal hematuria, blood at the end of the urinary stream, suggests a bladder neck or prostatic urethral source. (u) C. Presence of blood at the beginning of the urinary stream suggests an anterior (penile) urethral source. (u) D. Hematuria from the kidneys or ureter may be present microscopically or throughout the stream.

A 33 year-old hypertensive patient presents with increasing feelings of worthlessness and hopelessness over the past several months. Other symptoms include frequent crying episodes, loss of appetite, weight loss, insomnia with daytime sleepiness, and thoughts of suicide without any specific plans in place at this time. Which of the following would be the most appropriate first-line medication for this patient? A. amitriptyline (Elavil) B. bupropion (Wellbutrin) C. fluoxetine (Prozac) D. phenelzine (Nardil)

(u) A. Tricyclic antidepressants, such as amitriptyline, are third-line drugs for treating major depression, and are reserved for complicated and/or unresponsive cases. (u) B. Bupropion is a second-line drug for treating major depression and should be avoided in patients with hypertension. (c) C. Selective serotonin reuptake inhibitors are the drugs of first choice for treating major depression. (u) D. Phenelzine, a monoamine oxidase inhibitor, is reserved for treatment failures with supervision by a psychiatrist.

A 35 year-old male patient comes back to the office for a follow-up visit. He remarks that after 5 weeks on fluoxetine (Prozac) 20 mg per day, he still feels depressed but he denies suicidal ideations. What should you do to help this patient? A. Switch to tricyclic antidepressant B. Increase the dose of fluoxetine C. Switch the patient to divalproex (Depakote) D. Admit the patient to the behavioral health unit

(u) A. Tricyclics are second line in the treatment of depression. (c) B. An antidepressant should be raised to the recommended level and maintained at that level for 4-5 weeks, this patient was on too low of a dose. (u) C. Divalproex is a drug that is used to treat seizures and bipolar disorder. (u) D. This patient denies suicidal ideations, and is not in need of an admission to the behavioral health unit.

: (28) 100. Scientific Concepts/Endocrinology A 14 year-old female presents with short stature, growth failure, and physical features consistent with Tanner stage 1. Her skin is dry and pale and her face shows fine wrinkles. She appears apathetic. This patient most likely has a tumor in which of the following endocrine structures? A. Adrenal B. Anterior pituitary C. Neurohypophysis D. Thyroid

(u) A. Tumors of the adrenal gland can precipitate the crisis of pheochromocytoma with the release of epinephrine and norepinephrine. This is an acute phenomenon and a medical emergency. (c) B. The patient exhibits features consistent with panhypopituitarism secondary to lack of hormonal production in the anterior pituitary. Typically young patients present with short stature, growth failure, and delayed puberty. A patient with longstanding hypopituitarism has dry pale, finely textured skin and fine wrinkles on her face along with an apathetic countenance. The likely etiology is a pituitary adenoma. These patients typically require corticosteroid replacement therapy by age 18. (u) C. Lack of hormone production in the neurohypophysis (posterior pituitary) would result in deficiencies in arginine vasopressin and/or oxytocin. Patients may present with diabetes insipidus leading to hyponatremia. (u) D. Thyroid tumors may cause hypo- or hyperthyroidism, but this would not account for the effects secondary to growth hormone and pubertal hormone lack.

: (8) 19. History & Physical/Orthopedics/Rheumatology A 4 week-old female is seen for a well child visit. Birth history is significant for breech presentation and weighing 9 pounds 8 ounces at delivery. Which of the following is indicated to rule out developmental dysplasia of the hip? A. Ultrasound B. Physical examination C. Anterior/posterior radiograph D. Computed tomography

(u) A. Ultrasound is useful for the diagnosis of developmental dysplasia of the hip (DDH). False positive readings prior to 8-10 weeks of age, expense, and comprehensive training needed for interpretation exclude this as a standard evaluation tool. (c) B. A careful physical examination at birth and repeated evaluation at each well visit until the child walks is indicated to identify DDH. A high degree of suspicion is necessary in children with risk factors for DDH, such as positive family history, ligamentous laxity, breech presentation, female gender, large fetal size, and first-born status. (u) C. Radiographs are not helpful because the femoral head is composed of radiolucent cartilage in the newborn. (u) D. Computed tomography is not indicated in a newborn.

Which of the following is a contraindication to prescribing oral hormonal contraceptives? A. history of iron deficiency anemia B. history of dysmenorrhea since age 15 C. positive family history of ovarian cancer D. history of smoking in a patient older than age 35

(u) A. Use of oral hormonal contraceptives in patients with iron deficiency anemia is beneficial in that menstrual periods are shorter with resultant decrease in blood loss. (u) B. Oral hormonal contraceptives are one of the acceptable treatments for primary dysmenorrhea as periods are less painful due to suppression of ovulation. (u) C. Oral hormonal contraceptives have been proven to reduce the risk of ovarian and endometrial cancer. Current recommendations include use in patients with a high risk of ovarian cancer due to family history or BRCA gene status. (c) D. A history of smoking in a patient older than age 35 is considered one of the absolute contraindications to prescribing oral hormonal contraceptives due to the increased risk of thromboembolic events.

213. Clinical Intervention/ENT/Ophthalmology A healthy 13 year-old male presents with a spontaneous unilateral nosebleed. Which of the following is the most appropriate initial treatment? A. Utilization of short acting nasal decongestant B. Direct pressure to nares for 15 minutes C. Cauterization with silver nitrate solution D. Pneumatic nasal tamponade

(u) A. Utilization of short acting nasal decongestant is typically recommended if direct pressure is unsuccessful. (c) B. In most cases uncomplicated anterior nosebleeds are successfully managed by application of direct pressure. (u) C. Cauterization with silver nitrate solution is recommended if direct pressure and/or sympathomimetic treatment is unsuccessful. (u) D. Tamponade/packing is typically reserved for refractory or large volume bleeding.

A 32 year-old male with a history of Tetralogy of Fallot with poor right ventricular function presents for evaluation of sleep apnea. There is no evidence of deviated septum. Polysomnography reveals apneic episodes of 60 seconds in duration. Oxygen saturation falls to low levels. Which of the following is the first-line treatment in the management of this patient? A. Uvulopalatopharyngoplasty B. Nasal septoplasty C. Continuous positive airway pressure D. Antidepressants and oxygen

(u) A. Uvulopalatopharyngoplasty (UVVP) is a surgical procedure with resection of pharyngeal soft tissue and partial amputation of the soft palate and uvula. It is a procedure of last resort in most cases of sleep apnea. (u) B. Nasal septoplasty is performed if gross anatomic nasal septal deformity is present. (c) C. Continuous positive airway pressure prevents hypoxemia and maintains patency of the airway. (u) D. Antidepressants have no role in the treatment of sleep apnea.

A 27 year-old G1P0 female presents complaining of painless spotting since this morning. She is known to be 12 weeks pregnant. Pelvic examination reveals the presence of blood within the vagina with a closed cervical os. The uterus is consistent with a 10-12 week gestation and nontender to palpation. Which of the following is the most likely diagnosis? A. inevitable abortion B. threatened abortion C. incomplete abortion D. complete abortion

(u) A. Vaginal bleeding and cramp-like lower abdominal pain are usually present in an inevitable abortion. The cervical os is also frequently partially open. (c) B. Vaginal bleeding that occurs prior to the 20th week of gestation is classified as a threatened abortion. Pain is usually not a major feature and vaginal examination usually reveals a closed cervical os. (u) C. Vaginal bleeding accompanied by cramp-like pain, cervical dilatation, and passage of some products of conception constitutes an incomplete abortion. (u) D. After all products of conception are passed, the uterus contracts and vaginal bleeding stops. The cervical os closes, but the uterus is smaller than the suspected gestational age following a complete abortion.

A 54 year-old female who has diabetes presents with rubor, absence of hair, and brittle nails of her left foot. She complains of leg pain that awakens her at night. Examination reveals a femoral bruit with diminished popliteal and pedal pulses on the left side. The most appropriate therapy would be A. vasodilator therapy. B. bypass surgery. C. exercise program. D. embolectomy.

(u) A. Vasodilator therapy is not indicated. (c) B. Bypass surgery is indicated in the presence of rest pain and provides relief of symptoms in 80 to 90% of patients. (u) C. While an exercise program is appropriate with claudication, rest pain is a surgical indication. (u) D. Embolectomy is used for acute arterial occlusion.

A patient presents with an acutely painful and cold left leg. Distal pulses are absent. Leg is cyanotic. There are no signs of gangrene or other open lesions. Symptoms occurred one hour ago. Which of the following treatments is most appropriate? A. Vena cava filter B. Embolectomy C. Amputation D. Aspirin

(u) A. Vena cava filters are used in the management of venous thromboembolic disease when anticoagulation cannot be done. (c) B. Embolectomy within 4 to 6 hours is the treatment of choice. (h) C. Amputation is done only when no viable tissue is present. Cutting off a viable limb is never a good idea. (u) D. Aspirin is used in the prevention and treatment of coronary disease and has no role in the treatment of peripheral arterial embolism.

Which of the following diagnostic tests should be ordered initially to evaluate for suspected deep venous thrombosis of the leg? A. Venogram B. Arteriogram C. Duplex ultrasound D. Impedance plethysmography

(u) A. Venogram has been replaced by noninvasive tests due to discomfort, cost, technical difficulties, and complications, such as phlebitis. (h) B. Thrombophlebitis is a venous problem, not an arterial one. Any unnecessary invasive procedure is potentially harmful. (c) C. Ultrasound is the technique of choice to detect deep venous thrombosis in the leg. (a) D. Impedance plethysmography is equivalent to ultrasound in detecting thrombi of the femoral and popliteal veins, but it may miss early, nonocclusive thrombi.

A post-op patient has signs and symptoms highly suggestive of a pulmonary embolism. The results of the CT scan of the lung is nondiagnostic. What is the most appropriate next step in the evaluation? A. Ventilation perfusion (V/Q) scan B. Ultrasound of the legs C. Echocardiography D. D-dimer

(u) A. Ventilation perfusion scans are performed prior to the CT scan of the chest and would not likely add additional information to this clinical scenario. (c) B. In a patient with a high likelihood of pulmonary embolism or an inpatient, as in this case, ultrasound of the legs would be the next diagnostic step after a nondiagnostic CT. (u) C. Although echocardiography may show right ventricular free wall hypokinesis with normal motion of the apex suggestive of pulmonary embolism, more than 50% of patients with a pulmonary embolism will have normal echocardiography. Echocardiography is not used in the diagnosis on inpatients.; (u) D. In a post-op patient, a d-dimer will be positive regardless of the presence or absence of a pulmonary embolism.

Which of the following can optimize quality of life and is an definitive treatment for a patient with refractory heart failure? A. Ventricular assist device B. Intra-aortic balloon counterpulsation C. Cardiac transplantation D. Partial resection of the left ventricle

(u) A. Ventricular assist devices can help to provide temporary circulatory support to those awaiting transplantation. (u) B. IABC is useful in cardiogenic shock, but does not improve quality of life. (c) C. Cardiac transplantation is effective, with survival rates of 80-90% in 1 year, 60-70% over 5 years. It does improve quality of life despite the immunosuppression medications. (u) D. Partial resection of the left ventricle is still experimental and has not shown to have successful results.

An 8 year-old boy is brought to a health care provider complaining of dyspnea and fatigue. On physical examination, a continuous machinery murmur is heard best in the second left intercostal space and is widely transmitted over the precordium. The most likely diagnosis is A. ventricular septal defect. B. atrial septal defect. C. congenital aortic stenosis. D. patent ductus arteriosus.

(u) A. Ventricular septal defect causes a holosystolic murmur rather than a continuous machinery-like murmur. (u) B. Atrial septal defect causes a fixed split S2 rather than a continuous systolic heart murmur. (u) C. Congenital aortic stenosis causes a crescendo-decrescendo systolic murmur heard best in the second intercostal space. (c) D. Patent ductus arteriosus is classically described in children as a continuous machinery-type murmur that is widely transmitted across the precordium.

Acute rebound hypertensive episodes have been reported to occur with the sudden withdrawal of A. verapamil (Calan). B. lisinopril (Prinivil). C. clonidine (Catapres). D. hydrochlorothiazide (HCTZ)

(u) A. Verapamil is a calcium channel blocker and there is no associated rebound hypertension after withdrawal. (u) B. Lisinopril is an ACE inhibitor, which is not associated with rebound hypertension. (c) C. Clonidine (Catapres) is a central alpha agonist and abrupt withdrawal may produce a rebound hypertensive crisis. (u) D. Hydrochlorothiazide is a thiazide diuretic, which is not associated with rebound hypertension.

A 4 year-old boy presents with purulent, foul-smelling nasal discharge for three days. He has not had any other symptoms of respiratory illness, cough, wheeze, or fever. His activity level and appetite has been normal. On exam, he is afebrile. TM's have normal light reflex, canals are clear. Left nare is clear; there is considerable amount of purulent exudate from the right nare, and a bright reflection of light is noticed. Oropharynx is without inflammation or exudate. Neck is supple, without lymphadenopathy. Which of the following is the most likely diagnosis? A. viral URI B. acute sinusitis C. allergic rhinitis D. nasal foreign body

(u) A. Viral URI does not present with foul-smelling nasal discharge. (u) B. Acute sinusitis may present with purulent nasal discharge, but the observation of a bright light reflection suggests a foreign body. (u) C. Allergic rhinitis is seasonal, associated with sneezing and other allergy-related symptoms. (c) D. Nasal foreign body is suggested by unilateral nasal obstruction or discharge

A 22 month-old male infant presents with one day of barking cough preceded by three days of cold symptoms. On physical examination, his axillary temperature is 100.4°F and he has no stridor at rest. Inspiratory stridor is evident when he becomes agitated during the examination. There are no signs of respiratory distress or cyanosis. Which of the following is the most appropriate treatment for this patient? A. Nebulized albuterol B. Nebulized epinephrine C. Oral amoxicillin D. Oral dexamethasone

(u) A. Viral croup is an upper airway disease and there is no role for bronchodilator therapy. (u) B. Nebulized racemic epinephrine is only indicated in the treatment of croup in cases of moderate to severe rest stridor, respiratory distress, or hypoxia. (u) C. Croup is almost always a viral illness and antibiotics have no role in the treatment of this condition. (c) D. Corticosteroids are beneficial in the treatment of croup. Intramuscular administration has shown no benefit over oral administration.

A 35 year-old female presents with fatigue. CBC results reveal the following: WBC: 6,300/microliter Hgb: 9.5 g/dl Hct: 28% MCV: 75 fL MCHC: 32 g/dl MCH: 24 pg Platelets: 550,000/mL Which of the following is the best treatment option for this patient? A. folic acid B. vitamin B12 C. prednisone D. ferrous sulfate

(u) A. Vitamin B12 and folate deficiency present with macrocytic cells and are treated with vitamin B12 and folate respectively. (u) B. See A for explanation. (u) C. Prednisone is used to treat immune-mediated hemolytic anemias which present with normocytic, normochromic red blood cells. (c) D. Iron deficiency anemia is a microcytic, hypochromic anemia and is treated with ferrous sulfate.

A 75 year-old male presents for a routine physical. Vitals are normal with no orthostatic changes. On physical examination, a fine cortical movement with repetitive rubbing of the tip of the thumb along the tips of the fingers is noted at rest. Which of the following is the most likely diagnosis? A. Seizure disorder B. Peripheral neuropathy C. Shy-Drager syndrome D. Parkinson's disease

(u) A. While a seizure may present with fine or gross uncontrolled motor movements, the tremor described is classic pill-rolling tremor noted in Parkinson's disease. (u) B. Peripheral neuropathy presents with loss of sensation not tremor. (u) C. Shy-Drager syndrome is due to autonomic degeneration and typically presents with orthostatic hypotension. (c) D. Parkinson's disease presents with tremor at rest (pill-rolling), bradykinesia, rigidity, and postural instability.

A 22 year-old female complains of worsening pain, swelling, and tenderness in her left heel for 1 week. She sustained a penetrating injury to the heel two weeks ago when she stepped on a nail while running in tennis shoes. Examination reveals a draining puncture wound with surrounding erythema and exquisite tenderness. X-ray of the left foot demonstrates periosteal reaction associated with the wound. Which organism is classically responsible for this infection? A. Escherichia coli B. Streptococcus pyogenes C. Pseudomonas aeruginosa D. Staphylococcus aureus

(u) A. While gram negative enteric organisms can be a cause of osteomyelitis, E. coli is rarely seen outside of the neonatal period. (u) B. Streptococcus pyogenes is an uncommon (~10 %) cause of acute osteomyelitis. The mechanism of injury suggests a different pathogen. (c) C. Pseudomonas aeruginosa is frequently associated with osteomyelitis involving puncture wounds of the foot. This is believed to result from direct inoculation with P. aeruginosa via the foam padding found in tennis shoes. (u) D. Staphylococcus aureus is the most common infecting organism in cases of acute osteomyelitis, however, the mechanism of injury in this case suggests P. aeruginosa.

A mother brings her teenage daughter to the emergency department. The teenager is anxious, tremulous, and in a dysphoric mood. She reports recent nightmares and insatiable hunger. Which of the following diagnostic tests would likely yield the most important information? A. Serum TSH B. Urine drug screen C. Minnesota Multiphasic Personality Inventory (MMPI) D. Electroencephalogram (EEG)

(u) A. While hyperthyroidism can cause tremor and sometimes hunger, it is not associated with a dysphoric mood. It also generally occurs in early adulthood. (c) B. A drug screen would be helpful for many drugs of abuse that might cause these symptoms, but may not be definitive for amphetamines. (u) C. The MMPI is an objective personality assessment instrument. There is no indication for its use here. (u) D. While an EEG might be useful later (assuming the drug screen was negative), there is no history to indicate use of this test early on.

A 27 year-old patient presents with crush injuries to both lower extremities after being involved in an automobile accident. Within six hours of the accident, urine output has decreased to less than 10 cc per hour. Within 24 hours the serum creatinine increased from 0.9 mg/dl to 2.9 mg/dl and serum CPK is now 12,000 U/L. In addition to a fluid challenge with normal saline, which of the following is the most appropriate treatment as this time? A. IV calcium chloride B. Oral captopril (Capoten) C. IV sodium bicarbonate D. Oral sevelamer (Renagel)

(u) A. While hypocalcemia is common in rhabdomyolysis and acute renal failure, treatment is not needed unless clinical signs such as tetany are noted. (h) B. The use of ACE inhibitors, such as captopril, may worsen renal function and are not indicated in this case. (c) C. Treatment of acute renal failure due to rhabdomyolysis is best accomplished with IV fluids and forced alkaline diuresis. (u) D. Sevelamer is a phosphate binder used to treat elevated phosphate levels in patients with end stage renal disease.

A mother brings in her 3 month-old infant and states that she has noticed a rash on her infant's scalp. Physical examination reveals the presence of erythematous and scaling crusty lesions involving the vertex of the scalp. Which of the following is the most appropriate initial intervention? A. selenium sulfide shampoo B. permethrin 1% cream rinse C. warm olive oil compresses D. scrubbing of scalp with hexachlorophene

(u) A. While selenium sulfide shampoos can be used in treatment of scalp seborrheic dermatitis in adults, it is not recommended for use in infants and young children. (u) B. Permethrin 1% cream rinse is utilized in the treatment of pediculosis, not seborrheic dermatitis. (c) C. This infant most likely has scalp seborrheic dermatitis ("cradle cap"). Initial treatment consists of warm olive oil compresses to remove any crusts followed by use of baby shampoo or mild hydrocortisone cream. (u) D. Hexachlorophene is a bacteriostatic skin cleanser which is not indicated in the treatment of seborrheic dermatitis. It is also not recommended for use in infants and young children.

A 45 year-old male presents with complaints of a chronic cough productive of mucopurulent sputum. The cough has been present for the past 3 years, but he attributed it to a "smoker's cough". He has been coughing up a lot of sputum lasting all winter long for the past 2 years. He denies any hemoptysis, weight loss or chest pain. Physical examination reveals a moderately obese male in no acute respiratory distress. Lung fields reveal presence of scattered rhonchi and wheezes. There is 1+ peripheral edema. Which of the following is the most likely diagnosis? A. Lung cancer B. Bronchiectasis C. Chronic bronchitis D. Interstitial lung disease

(u) A. While the respiratory complaints of lung cancer are associated with the location and type of primary tumor, anorexia and weight loss is seen in the majority of patients. Patients will also usually have a new cough or a change in a chronic cough and may complain of hemoptysis and nonspecific chest pain. (u) B. While bronchiectasis presents with a chronic cough productive of copious amounts of purulent sputum, these patients most commonly also have associated complaints of hemoptysis, weight loss and pleuritic chest pain. Examination of the lungs reveals persistent crackles at the bases. (c) C. This patient most likely has chronic bronchitis which is defined as sputum production and cough for at least 3 months of the year for 2 consecutive years which is primarily caused by cigarette smoking. (u) D. Interstitial lung disease is characterized by progressive exertional dyspnea and cough, however sputum production is minimal and the examination of the lungs reveals fine, late inspiratory crackles at the bases in the majority of patients.

A 62 year-old male presents with complaints of vague epigastric abdominal pain associated with jaundice and generalized pruritus. Physical examination reveals jaundice and a palpable non-tender gallbladder, but is otherwise unremarkable. Which of the following is the most likely diagnosis? A. Viral hepatitis B. Pancreatic cancer C. Acute cholecystitis D. Gilbert's syndrome

(u) A. While viral hepatitis may cause jaundice, the liver is enlarged and tender. (c) B. Pancreatic cancer is suggested by the vague epigastric pain with the jaundice resulting from biliary obstruction due to cancer involving the pancreatic head. The presence of a palpable non-tender gallbladder (Courvoisier's sign) also indicates obstruction due to the cancer. (u) C. While acute cholecystitis may present with jaundice and an enlarged gallbladder, the pain is classically colicky and located in the right upper quadrant. On physical examination with deep inspiration and palpation of the right subcostal area increased pain and respiratory arrest (Murphy's sign) is usually seen. (u) D. Gilbert's syndrome is the most common of the hereditary hyperbilirubinemias. It is most often diagnosed near puberty or adult life based on results of a comprehensive metabolic panel.

Which of the following is the most common indication for operative intervention in patients with chronic pancreatitis? A.Weight loss B. Intractable pain C. Exocrine deficiency D. To decrease risk of cancer

(u) A. While weight loss is common with chronic pancreatitis, it is not an indication for surgical intervention. (c) B. Indications for surgical treatment of chronic pancreatitis include severe pain that limits the patient's functioning or intractable pain despite the use of non-narcotic analgesics and absence of alcohol intake. (u) C. While the majority of patients go on to develop diabetes mellitus 25 years after the clinical onset of chronic pancreatitis, this is not an indication for surgical intervention as it would lead to more severe exocrine deficiency. (u) D. While the possible presence of pancreatic cancer is an indication for surgery, there is no indication for prophylactic surgery to decrease the risk of cancer.

Which of the following population groups represent the greatest risk for developing primary hypertension? A. White non-Hispanic B. Hispanic C. Mediterranean D. Black non-Hispanic

(u) A. White non-Hispanic adults have a low risk of hypertension compared to Hispanic and Black non-Hispanics. (u) B. Hispanic adults are lower risk of hypertension than Black non-Hispanic, but not compared to White non-Hispanic. (u) C. Mediterranean adults have a lower risk of hypertension than Black non-Hispanics. (c) D. Black non-Hispanic adults have the highest risk of hypertension.

A 16 year-old G0P0 patient presents complaining of lower pelvic pain that alternates from right to left side of her pelvis. She states that it is related to her cycle and occurs most commonly midcycle. She denies sexual activity. She reports that she has taken ibuprofen at the time of the discomfort with some relief. Her pelvic examination is unremarkable. Which of the following is the most likely diagnosis? A. Endometriosis B. Mittelschmerz C. Functional ovarian cyst D. Pelvic inflammatory disease

(u) A. With endometriosis, the uterus is often fixed and retroflexed in the pelvis. The palpable mass is an endometrioma or "chocolate cyst". The patient with endometriosis also often has dysmenorrhea, dyspareunia, and dyschezia. (c) B. Women may experience pain at the time of ovulation, may alternate side to side. (u) C. Functional ovarian cysts occur from ovulation and are not usually symptomatic. (u) D. Patients with pelvic inflammatory disease often present with fever, pain, and more acute symptoms.

On performing a mental status examination you notice that the patient tends to repeat words and phrases out of context to your questions. This type of thought disorder is defined as which of the following? A. Flight of ideas B. Perseveration C. Circumstantiality D. Derailment

(u) A. With flight of ideas, thoughts move from one topic to another with rapid speech. (c) B. Definition of perseveration is the persistent repetition of words or concepts in the process of speaking. This is often seen in cognitive disorders, schizophrenia, and other mental illness. (u) C. Circumstantiality includes much detail, but information is not relevant. (u) D. Derailment is speech with loose associations and sentences that do not make sense.

A 23 year-old African American female presents with concern regarding "lightening" of the skin in her knees, face, clavicles, and wrists. These areas appear to be increasing in size. She was recently diagnosed with pernicious anemia. In order to confirm her diagnosis, which of the following tests is indicated? A. Wood's light B. KOH prep C. Skin biopsy D. Gram stain

(u) A. Wood's light is primarily used to evaluate macules but there are no pathognomonic fluorescent patterns that would confirm the diagnosis of vitiligo. (u) B. KOH prep would be indicated to search for tinea versicolor by showing a spaghetti and meatball appearance but this patient's condition is consistent with vitiligo. (c) C. Skin biopsy in a patient with vitiligo will show complete absence of melanocytes and will confirm the diagnosis. (u) D. Gram stain is helpful in diseases secondary to a bacterial etiology not vitiligo.

A 60 year-old patient returned from the recovery room to the floor following a subtotal gastrectomy. At 3 AM the next morning, the patient's temperature is 102° F (39° C) and pulse is 112/min. Which of the following is the most likely cause? A. wound infection B. atelectasis C. phlebitis D. shock

(u) A. Wound infection does not present this early. (c) B. Atelectasis is the most common pulmonary complication, affecting 25% of patients with abdominal surgery. It is more common in elderly and overweight patients and occurs within the first 12 to 24 hours postoperatively. (u) C. Phlebitis occurs more commonly after the second postoperative day. (u) D. In shock, the pulse is usually thready and the temperature is not elevated.

Dysmenorrhea would most likely occur in which of the following patients? A. A young teenager who just started having her menses B. A woman on birth control pills C. A marathon runner with one menses per year D. A 35 year-old woman with regular cycles

(u) A. Young teenagers who have just started their menses are unlikely to have dysmenorrhea, because they usually are anovulatory for up to one year. (u) B. Women on birth control pills do not have dysmenorrhea because they do not ovulate on oral contraceptives and OCPs are used to treat dysmenorrhea. (u) C. Marathon runners are often amenorrheic or have oligoamenorrhea. They do not have dysmenorrhea because dysmenorrhea is a function of ovulatory cycles. (c) D. Women with regular menstrual cycles are most likely to have dysmenorrhea. Dysmenorrhea is caused by a excess of prostaglandin F 2 alpha. Prostaglandin production increases under the influence of progesterone, reaching a peak at, or soon after, the start of menstruation

What is the initial treatment of choice for inflammatory bowel disease? A. Ondansetron (Zofran) B. Sulfasalazine (Azulfidine) C. Metronidazole (Flagyl) D. Azothioprine (Imuran)

(u) A. Zofran is an antiemetic and not indicated in the treatment of inflammatory bowel disease. (c) B. Sulfasalazine and other 5-aminosalicylic acid drugs are the cornerstone of therapy in mild to moderate inflammatory bowel disease as they have both anti-inflammatory and antibacterial properties. (u) C. Metronidazole may have a role in Crohn's disease after ileal resection but has no role in the treatment of mild to moderate disease. (u) D. Azothioprine is used in severe, glucocorticoid-dependent inflammatory bowel disease.

A 53 year-old female has a diagnosis of migraine headaches. She had been using sumatriptan (Imitrex) to abort her eadaches, but she is now having one or two headaches per week. The most appropriate preventive therapy is A. zolmitriptan (Zomig). B. promethazine (Phenergan). C. propranolol (Inderal). D. fluoxetine (Prozac).

(u) A. Zolmitriptan, another 5-HT receptor agonist, is used acutely to abort migraine headaches. (u) B. Promethazine is an antiemetic that may be used to alleviate nausea and vomiting from an acute migraine. (c) C. Propanolol is useful in preventing migraine headaches and may be maintained indefinitely. (u) D. Fluoxetine is an SSRI that is used in the treatment of anxiety and obsessive-compulsive disorders.

An 18 year-old male who was struck in the left eye with a baseball presents with ocular pain, periorbital ecchymosis, and restricted upward gaze. Which of the following is the diagnostic study of choice in this patient? A. Zygomatic arch x-ray B. CT scan of the orbit C. Ultrasonography D. Fluorescein staining

(u) A. Zygomatic arch x-ray is not the diagnostic study of choice for a suspected blowout fracture. (c) B. CT scan of the orbit is the study of choice to evaluate a suspected blowout fracture. (u) C. Ultrasound may be helpful if there is a suspected foreign body, however it is not helpful in fracture identification. (u) D. Fluorescein staining is indicated in the evaluation of corneal abrasion, not orbital fracture.

A 25 year-old presents with pain in the proximal ulna after falling directly on the forearm. X-ray shows fracture of the proximal 1/3rd of the ulna. There is an associated anterior radial head dislocation. What is the proper name for this condition? A. Galeazzi fracture B. Monteggia fracture C. Colles' fracture D. Smith fracture

(u)A. A Galeazzi fracture is a fracture along the length of the radius with injury to the distal radioulnar joint. (c) B. A Monteggia fracture is a fracture of the proximal ulna with anterior dislocation of the radial head. (u) C. A Colles' fracture is a fracture of the distal radius with dorsal displacement of the radial head. (u) D. A Smith fracture is a fracture of the distal radius with ventral displacement of the radial head.

Which of the following risk factors is the most predictive for the development of osteoarthritis? A. Age B. Major joint trauma C. Prior inflammatory joint disease D. Repetitive stress

(c) A. Age is the most significant risk factor for osteoarthritis. Prevalence and severity increase with age. Ninety percent of people greater than the age of 40 have degenerative changes of the weight bearing joints. Major joint trauma, prior inflammatory joint disease, and repetitive stress are additional risk factors but not as predictive as age. (u) B. See A for explanation. (u) C. See A for explanation. (u) D. See A for explanation.

In order to prevent the progression of diabetic nephropathy which of the following medications should be instituted? A. Lisinopril (Prinipril) B. Propanolol (Inderal) C. Verapamil (Calan) D. Hydrochlorothiazide (Diuril)

(c) A. All patients should be started on an ACE inhibitor to prevent the progression of proteinuria. ACE inhibitors appear to improve glomerular hemodynamics by decreasing glomerular pressure. (u) B. Beta blockers are not indicated for the treatment of microalbuminuria. (u) C. Calcium channel blockers are not indicated for the treatment of microalubuminuria. (u) D. Thiazide diuretics are not indicated for the treatment of microalbuminuria.

A 59 year-old otherwise healthy female develops acute dyspnea and chest pain one week post total abdominal hysterectomy. Echocardiogram demonstrates normal heart size with normal right and left ventricular function. Lung scan demonstrates two segmental perfusion defects. Which of the following is the next step in the management of this patient? A. Anticoagulation B. Embolectomy C. Thrombolysis D. Inferior vena cava filter

(c) A. Anticoagulation is the treatment of choice in patients with pulmonary embolism with normal ventricular function and no absolute contraindications. (u) B. Embolectomy is not indicated as initial treatment of a pulmonary embolism in patients with normal ventricular function. (h) C. Thrombolysis is contraindicated in patients within 10 days of having major surgery. (u) D. An inferior vena cava filter is considered in patients with contraindications to anticoagulation therapy or failed anticoagulation therapy.

A 54 year-old patient has acute onset of palpitations, tremulousness, profuse sweating, shortness of breath, and numbness and tingling of the extremities. Physical examination reveals a pulse of 104 beats/min and regular, respiratory rate of 30/min, blood pressure of 160/95 mm Hg. Arterial blood gases reveal a low pCO2. ECG shows no acute changes. The most likely diagnosis is A. anxiety neurosis. B. atrial fibrillation. C. depressive neurosis. D. myocardial infarction.

(c) A. Anxiety is characterized by short-lived, recurrent, unpredictable episodes of intense anxiety accompanied by marked physiological manifestations. (u) B. Atrial fibrillation would not account for abnormal respiratory rate or blood gases, and the pulse would be irregular. (u) C. Depressive neurosis may be associated with somatic complaints, but it is not manifested by an acute, abrupt onset of them. (u) D. Even though the patient manifests some signs of a myocardial infarction (sweating and numbness), the ECG is normal.

A 48 year-old female presents to the clinic complaining of hematuria. The patient states that she was found to have hematuria during an insurance physical examination. The patient denies dysuria or frequency. She also denies pain in the abdomen, flank or meatus. She denies any history of previous nephrolithiasis. Urinalysis reveals the urine to be yellow and slightly hazy with a positive dipstick for hemoglobin. Microscopic reveals 5-7 RBCs/HPF without WBCs, bacteria, casts, or crystals. What is the next diagnostic study this patient should undergo? A. CT urography B. Intravenous pyelogram C. Abdominal ultrasound D. Cystoscopy

(c) A. CT urography with and without contrast should be done to evaluate the upper and lower urinary tract for neoplasms, and benign conditions such as urolithiasis. This has replaced IVP for imaging of the upper tracts. Abdominal ultrasound will not help in this scenario and the role of renal ultrasound in evaluation of hematuria is unclear. Cystoscopy will help to assess the bladder and urethra but will not help with evaluation of the upper urinary tract. (u) B. See A for explanation. (u) C. See A for explanation. (u) D. See A for explanation.

Which of the following medication classes is the treatment of choice in a patient with variant or Prinzmetal's angina? A. Calcium channel blockers B. ACE inhibitors C. Beta blockers D. Angiotensin II receptor blockers

(c) A. Calcium channel blockers are effective prophylactically to treat coronary vasospasm associated with variant or Prinzmetal's angina. (u) B. ACE inhibitors are not a treatment for coronary vasospasm. (h) C. Beta blockers have been noted to exacerbate coronary vasospasm potentially leading to worsening ischemia. (u) D. Angiotensin II receptor blockers are not a treatment for coronary vasospasm.

A 24 year-old female presents with complaints of abdominal pain, bloating, and chronic diarrhea. She says she felt worse last month while on vacation in Italy. Despite eating well she lost weight. What would be the most appropriate treatment for this patient? A. Gluten-free diet B. Small bowel resection C. Ciprofloxacin (Cipro) therapy D. Pancreatic enzymes

(c) A. Celiac disease is a diffuse disease of the small bowel that is caused by immunologic malfunction. This disease is active only in the presence of gluten, a constituent of wheat. Avoidance of gluten-containing foods is the treatment of choice. (u) B. Although celiac disease affects the small bowel (particularly the jejunum), resection is too aggressive when dietary manipulation is very effective in the management of this disorder. (u) C. Celiac disease is immune-mediated and is not due to infection so antibiotics are not needed. (u) D. Celiac disease affects absorption in the small bowel and does not involve a dysfunction of pancreatic enzymes.

Which of the following is a major contraindication to curative surgical resection of a lung tumor? A. Liver metastases B. Vagus nerve involvement C. Non-malignant pleural effusion D. Chest wall invasion of the tumor

(c) A. Distant metastases, except for solitary brain and adrenal metastases are an absolute contraindication for pulmonary resection. Other absolute contraindications include MI within past 3 months, superior vena cava syndrome due to metastatic tumor, bilateral endobronchial tumor, contralateral lymph node metastases and malignant pleural effusion. (u) B. See A for explanation. (u) C. See A for explanation. (u) D. See A for explanation.

A 62 year-old male is brought to the emergency department with acute hematemesis. The patient denies a previous history of vomiting. His wife states he has chronic liver disease. Physical examination reveals a distended abdomen without rebound, guarding or organomegaly. There is a fluid wave. Which of the following is the most likely diagnosis? A. Esophageal varices B. Mallory-Weiss tear C. Arteriovenous malformation D. Perforated duodenal ulcer

(c) A. Esophageal varices are dilated submucosal veins that develop in a patient with underlying portal hypertension. The most common cause of portal hypertension is cirrhosis. (u) B. A patient with a Mallory-Weiss tear would have a history of retching but would not have a distended abdomen. (u) C. Most arteriovenous malformations are asymptomatic. If symptomatic they would have symptoms of a slow bleed. (u) D. A patient with perforated duodenal ulcer would have rebound and guarding on examination.

A 22 year-old G0P0 asymptomatic female is seen for her yearly gynecologic examination. She denies performing self-breast exams and any family history of breast or gynecologic cancers. On palpation of her left breast, a solitary 1 cm rubbery, firm, well-circumscribed, non-tender breast mass is noted. In review of her records, similar findings were noted on last year's examination. Which of the following is the most likely diagnosis? A. fibroadenoma B. breast fat necrosis C. fibrocystic changes D. intraductal papilloma

(c) A. Fibroadenoma of the breast most commonly is seen in young females. Typical characteristics include a solitary 1-3 cm firm, painless, freely movable mass that does not change with the menstrual cycle and are slow growing. Most are found as an incidental finding on physical examination or during self-breast examination. (u) B. Fat necrosis of the breast is rare. It is thought to result from trauma to the breast and occasionally is accompanied by ecchymosis. If left untreated, the mass eventually disappears. (u) C. Fibrocystic changes in the breast are most common in females between 30-50 years of age. The disorder is characterized by painful, often multiple and usually bilateral masses that are associated with increased pain and size of breast mass premenstrually. (u) D. Intraductal papillomas are usually non-palpable. The patient presents with complaints of spontaneous onset of bloody, serous, or cloudy nipple discharge.

A 40 year-old female presents with a 1.5 cm well-circumscribed mass noted on mammography in the right upper, outer quadrant. Which of the following procedures is most appropriate and should be done next? A. Fine needle aspiration B. BRCA 1 and BRCA 2 genetic testing C. Serum CA-125 D. Radiation therapy

(c) A. Fine needle aspiration is fairly accurate, easily performed, and has minimal morbidity. (u) B. Although BRCA 1 and BRCA 2 genetic tests are used in the risk assessment for possible breast and ovarian cancer, it would not replace the need to perform a more definitive evaluation of an identified breast mass. (u) C. Serum CA-125 is a tumor marker for ovarian, not breast, cancer. (u) D. Radiation therapy is only indicated after a diagnosis of breast cancer is proven and may be used as adjunctive therapy.

Which of the following is the treatment of choice for benzodiazepine intoxication? A. Flumazenil (Romazicon) B. Naloxone (Narcan) C. Chlordiazepoxide (Librium) D. Clonidine (Catapres)

(c) A. Flumazenil is the treatment of choice for benzodiazepine intoxication. (u) B. Naloxone is used for the treatment of opioid intoxication and withdrawal. (u) C. Chlordiazepoxide is used for alcohol detoxification. (u) D. Clonidine is an alpha blocker that is often used for withdrawal from opioids, alcohol, benzodiazepines, or nicotine.

A 20 year-old female presents with episodes of binge eating, overuse of laxatives, and periods of starvation. Which of the following is the best treatment option for this patient? A. fluoxetine (Prozac) B. gabapentin (Neurontin) C. amitriptyline (Elavil) D. phenelzine (Nardil)

(c) A. Fluoxetine, a SSRI, is the drug of choice for the treatment of bulimia nervosa. (u) B. Gabapentin is used in the treatment of seizure disorders. (u) C. Amitriptyline, a tricyclic antidepressant, is not effective in the treatment of bulimia nervosa. (u) D. Phenelzine, a monoamine oxidase inhibitor, is not indicated in the treatment of bulimia nervosa.

A 22 year-old woman comes to the office because her urine is cola-colored and she has not urinated since yesterday morning. Her past medical history is significant for pharyngitis two weeks ago. Her mother and grandmother have type 2 diabetes. Her blood pressure is 146/92mmHG. On physical examination, she has edema of her face and hands. Which of the following is the most likely diagnosis? A. Glomerulonephritis B. Acute tubular necrosis C. Nephrolithiasis D. Diabetic nephropathy

(c) A. Glomerulonephritis presents with hematuria, cola-colored urine, oliguria, and edema of the face and eyes in the morning. Urinalysis reveals red blood cells, mild proteinuria and red blood cell casts. Glomerulonephritis can occur 1-3 weeks after a strep infection. (u) B. Acute tubular necrosis is caused by acute kidney injury, such as a nephrotoxin, and is associated with uremic symptoms which include nausea, vomiting, malaise, and altered mental status. Granular casts are nonspecific and may be seen in acute tubular necrosis. (u) C. Nephrolithiasis usually presents as a sudden onset of colicky flank pain with associated nausea and vomiting. Urinalysis often reveals gross or microscopic hematuria. (u) D. Diabetic nephropathy is the most common cause of end stage renal disease in the United States. Urine examination reveals albuminuria.

Which of the following is an indication for vaccination against hepatitis A? A. Illicit drug users B. Health care workers C. Renal dialysis patients D. Routine vaccination starting at birth

(c) A. Hepatitis A vaccine is recommended for illicit drug users, anyone living or traveling to endemic areas, sewage workers, food handlers, homosexual and bisexual men, animal handlers, patients with a history of chronic liver disease or a clotting factor disease as well as children and workers in day care settings and institutions. (u) B. Health care workers, renal dialysis patients and routine vaccination starting at birth are some of the recommendations for vaccination against hepatitis B, not hepatitis A. (u) C. See B for explanation. (u) D. See B for explanation.

Which of the following is a staphylococcal infection characterized by a localized red swollen and acutely tender abscess of the upper or lower eyelid? A. Hordeolum B. Uveitis C. Chalazion D. Dacryocystitis

(c) A. Hordeolum (stye) is a staphylococcal infection characterized by a localized red swollen and acutely tender abscess of the upper or lower eyelid. (u) B. Uveitis is an intraocular inflammation involving the uveal tract. (u) C. Chalazion is a granulomatous inflammation of the meibomian gland. (u) D. Dacryocystitis is an infection of the lacrimal sac due to obstruction of the nasolacrimal system.

When performing a pre-participation sports physical in the adolescent population, a murmur with which of the following qualities indicates a risk for sudden death during exercise? A. Increases with the Valsalva maneuver B. Increases with squatting maneuver C. Associated with a mid-systolic click D. Mid-systolic without radiation to the carotids

(c) A. Hypertrophic cardiomyopathy (HCM) is a known cause of sudden death during or just after physical exertion and competitive sports. The murmur associated with HCM is worsened by conditions that cause reduced ventricular volume such as the Valsalva maneuver, sudden standing, and tachycardia. (u) B. Typical systolic flow murmurs will become accentuated with maneuvers which increase venous blood flow to the heart and these murmurs do not place athletes at risk for sudden cardiac death. (u) C. Mitral valve prolapse is the most common type of heart murmur that is associated with a mid-systolic click. Mitral valve prolapse does not place the patient at risk for sudden cardiac death. (u) D. A mid-systolic heart murmur that fails to radiate into the carotids is most commonly associated with a benign systolic flow murmur and does not place the athlete at risk for sudden cardiac death.

An adult presents with a three month history of progressive severe muscle cramps, extremity paresthesias and lethargy which began shortly after a thyroidectomy for a malignant thyroid lesion. Which of the following is the most likely diagnosis? A. Hypoparathyroidism B. Hypothyroidism C. Hyperparathyroidism D. Hyperthyroidism

(c) A. Hypocalcemia secondary to hypoparathyroidism is commonly seen as a complication of thyroidectomy. (u) B. Hypothyroidism is possible without replacement therapy after thyroidectomy but would not result in the tetanylike symptoms and hypocalcemia. (u) C. Hyperparathyroidism will have a different constellation of symptoms and will be typified by elevated serum calcium. (u) D. Hyperthyroidism is not likely status post thyroidectomy though possible with over-aggressive replacement therapy. Symptoms with low serum calcium are the factors that move away from this being the diagnosis.

At the time of ovulation in a normal menstrual cycle, there is a peak in the serum concentration of which of the following? A. Luteinizing hormone B. Prostaglandin C. Progesterone D. Prolactin

(c) A. Luteinizing hormone is responsible for ovulation and, therefore, peaks at that time. (u) B. Prostaglandin is likely associated with the production of vasospasm, vascular necrosis, and menstrual flow, not ovulation. (u) C. The majority of progesterone is secreted by the corpus luteum and, therefore, peaks after ovulation has occurred. (u) D. Prolactin is an anterior pituitary hormone, and although important in reproduction and pregnancy, it is not present in high levels at the time of ovulation.

Which of the following best describes the purpose of intravenous magnesium sulfate in patients with preeclampsia? A. Prevention of convulsions B. Prevention of HELLP syndrome C. Lowering of blood pressure D. Reversal of proteinuria

(c) A. Magnesium sulfate is used to prevent and treat eclamptic seizures. Magnesium sulfate is not sufficient to treat hypertension, therefore antihypertensives must be added. Magnesium sulfate is excreted solely from the kidneys and urine output must be preserved to prevent accumulation of the drug. Magnesium sulfate does nothing to prevent HELLP syndrome. (u) B. See A for explanation. (u) C. See A for explanation. (u) D. See A for explanation.

A 28 year-old woman is complaining of heavy uterine bleeding and pelvic pressure that has progressively worsened over the past year. Evaluation reveals multiple moderate-sized uterine fibroids. The patient desires to have more children. The most appropriate definitive treatment is A. Myomectomy. B. Hysterectomy. C. GnRH agonists. D. Oral progesterone.

(c) A. Myomectomy is the definitive treatment of choice for moderate-sized uterine fibroids in young women who desire to maintain reproductive capability. (u) B. Hysterectomy is indicated as definitive treatment in a patient who does not desire to maintain reproductive capability. (u) C. GnRH agonists are used as an adjunct to surgery for treatment of uterine fibroids. Used alone, they would not be considered definitive treatment. (u) D. Oral progesterone may be used to suppress menorrhagia preoperatively. Used alone, it would not be considered definitive treatment.

A 52 year-old female comes to the office because of black stools for the past 3 days. She is afebrile and she has no pertinent physical examination abnormalities. Which of the following is the most appropriate initial diagnostic study? A. Stool for occult blood B. Stool cultures C. Sigmoidoscopy D. Abdominal CT scan

(c) A. Occult bleeding, as evidenced by the patient's history of black stools, is initially verified by a positive fecal occult blood test. (u) B. Stool cultures are indicated in the evaluation of acute diarrhea and not for the evaluation of acute GI bleeding. (u) C. Melena suggests a source of bleeding that is proximal to the ligament of Treitz, not a lower GI bleed. Sigmoidoscopy is used to evaluate only lower GI bleeding sources. (u) D. Abdominal CT scan is indicated for evaluation of obscure bleeding in order to exclude a pancreatic or hepatic source of bleeding if endoscopy fails to identify the source.

A 24 year-old female presents to your office for a physical examination. She is dressed in a low cut blouse and a short skirt. She is dramatic, emotional and sexually provocative. She complains of difficulty being intimate with men. On further questioning, she seems to overemphasize the severity of her current cold. After a full history and physical examination you suspect what personality disorder? A. Histrionic B. Borderline C. Narcissistic D. Antisocial

(c) A. Patients who are histrionic are attention seekers, and exaggerate their thoughts and feelings, they are often sexually provocative. (u) B. Borderline patients have unstable behavior and mood. They often have feelings of aloneness and selfdestructive behavior. (u) C. Narcissistic patients are grandiose, envious, and have a sense of special entitlement. They lack empathy. (u) D. Antisocial patients are unwilling to conform to social norms and do not learn from prior experiences.

Lab results for a post-operative oliguric patient reveals an increased BUN to creatinine ratio. The patient has a low fractional excretion of sodium (less than 1%). Which of the following is the most likely diagnosis? A. prerenal azotemia B. acute tubular necrosis C. acute glomerulonephritis D. obstructive uropathy

(c) A. Patients who have prerenal azotemia with otherwise normal kidneys will have severe sodium retention in order to help to save fluid. The amount of sodium in the urine is therefore very low. (u) B. Acute tubular necrosis may occur in the post-operative setting but these kidneys are damaged and unable to save sodium. (u) C. Acute glomerulonephritis is a complication of a streptococcal infection wherein the immune complexes damage the glomeruli and lead to hematuria, red blood cell casts, and proteinuria. (u) D. Although patients who undergo abdominal surgery are at risk for damage to the genital urinary system, these patients will not have sodium retention because it is a post renal, not a prerenal injury.

A 52 year-old patient presents with fatigue, complaints of paleness, anorexia, nausea, and weight loss. The patient also complains of numbness in his hands and feet and a recent occurrence of foot drop. He has a past history of diabetes and hypertension. Based on his clinical presentation, which of the following disorders is most likely to be responsible for this clinical picture? A. chronic renal failure B. middle cerebral artery occlusion C. Guillain-Barre syndrome D. Raynaud's phenomena

(c) A. Patients with chronic renal failure will have accelerated atherosclerosis, hypertension, anemia due to lack of erythropoietin production, a tendency toward GI symptoms of anorexia, nausea, and weight loss, and neurological symptoms of peripheral neuropathy that occurs in a stocking and glove distribution along with peripheral motor impairment such as foot drop and restless legs syndrome. Diabetes and hypertension are risk factors for the development of chronic renal disease. (u) B. Middle cerebral artery occlusion would consist of contralateral neurologic impairment rather than bilateral stocking glove neurological impairment (u) C. Guillain-Barre is a condition manifested by an ascending paralysis. (u) D. Raynaud's phenomena is a clinical condition in which patients exhibit pain and pallor of their distal extremities when they are exposed to cold. Upon rewarming, the patient will experience redness as part of the color changing sequence. Raynaud's typically does not cause the other symptoms described in the case scenario.

A 53 year-old man who had previously been in good health presents to the emergency department with a history of passing approximately 300 cc of bright red blood from the rectum 45 minutes ago. From which of the following anatomical sites is the bleeding most likely originating? A. rectosigmoid B. duodenum C. stomach D. esophagus

(c) A. Rectosigmoid is the most common site leading to hematochezia. (u) B. See A for explanation. (u) C. See A for explanation. (u) D. See A for explanation.

A 60 year-old right hand dominant male presents to your office complaining of right shoulder pain and progressively worsening arm weakness. His symptoms are aggravated when working above the shoulder level. On physical examination, there is no evidence of muscle atrophy. He has strong (5/5) adduction of his right shoulder but considerable weakness (1/5) with abduction when raising his arm above his head. Passive range of motion is intact. Which of the following is the most likely diagnosis? A. Rotator cuff tear B. Subacromial bursitis C. Adhesive capsulitis D. Supraspinatus tendonitis

(c) A. Rotator cuff tears are associated with full passive range of motion but have pain and weakness during active abduction. (u) B. Subacromial bursitis causes pain with raising hand above head and has mild degree of weakness. (u) C. Adhesive capsulitis (frozen shoulder) causes significant reduction in both active and passive range of motion. (u) D. Of the four rotator cuff muscles, supraspinatus is most likely to strain causing tendonitis. The symptoms cause limited range of motion due to pain but minimal weakness.

A 36 year-old African American female comes to the clinic for an insurance physical which requires a chest x-ray. She denies any respiratory symptoms. Examination of her chest is negative. X-ray results show marked lymphadenopathy in the right paratracheal region. Angiotensin-converting enzyme (ACE) levels are elevated. Which of the following is the most likely diagnosis? A. Sarcoidosis B. Tuberculosis C. Pulmonary fibrosis D. Lymphoma

(c) A. Sarcoidosis is characterized by paratracheal lymphadenopathy and elevated ACE levels. It is more common in African American patients and may be asymptomatic. (u) B. See A for explanation. (u) C. See A for explanation. (u) D. See A for explanation.

An 18 year-old G1P0 female presents for her 35 week prenatal visit with complaints of headache, blurred vision and right upper quadrant discomfort. Vital signs show BP of 170/100 mmHg and brisk patellar reflexes. Urinalysis shows 3+ proteinuria. Fetal heart tones are 150. What is your next step in the care of this patient? A. Admit to hospital and prepare for delivery B. Admit to hospital with antepartum fetal surveillance and close monitoring of maternal conditions C. Order bed rest at home with daily fetal movement counts and twice weekly antepartum care D. Order bed rest at home with administration of prophylactic magnesium sulfate

(c) A. Severe preeclampsia mandates hospitalization. Delivery is indicated if gestational age is 34 weeks or greater. (u) B. Antepartum fetal surveillance and close monitoring in hospital is appropriate for pregnant female with unfavorable cervix and mild preeclampsia. (u) C. See A for explanation. (u) D. See A for explanation.

Which of the following signs is positive in the presence of ascites? A. shifting dullness B. CVA tenderness C. ballottment sign D. cutaneous hypersensitivity

(c) A. Shifting dullness or the presence of a fluid wave is noted in patients with ascites. (u) B. CVA tenderness is noted in patients with pyelonephritis. (u) C. Ballottment test is used to assess a floating mass in the abdomen. (u) D. Cutaneous hypersensitivity is used to identify a specific zone of peritoneal irritation

Whispered voice test on a patient reveals decreased hearing in the left ear. Which of the following would be most consistent with conductive hearing loss in the left ear? A. Sounds best heard in the left ear on Weber test. B. Air conduction longer than bone conduction in the left ear on Rinne test. C. Sound best heard in the right ear on Weber test. D. Bone conduction longer than air conduction in the right ear.

(c) A. Sound best heard in the ear with decreased hearing on Weber test (in this case, the left ear) is indicative of conductive hearing loss. (u) B. With conductive hearing loss, bone conduction should be heard as long as or longer than air conduction of sound in the effected ear. Air conduction lasting longer than bone conduction of sound would indicate sensorineural hearing loss. (u) C. Sound best heard in the ear with unaffected hearing on Weber test (in this case, the right ear) is indicative of sensorineural hearing loss. (u) D. With conductive hearing loss, bone conduction should be heard as long as or longer than air conduction of sound in the effected ear. The right ear showed normal hearing on physical exam.

Which of the following clinical characteristics is associated with bicipital tendonitis? A. Aggravated by resisted supination of the forearm B. Bulging appearance to the proximal arm C. Weakness of the arm with internal rotation and adduction D. Pain that awakens the patient at night

(c) A. Supraspinatus tendonitis will be aggravated by resisted supination of the forearm. (u) B. Biceps rupture may present with a bulging appearance of the proximal arm. (u) C. Weakness of the arm with internal rotation and adduction is characteristic of pectoralis major rupture or tear. (u) D. Night pain is characteristic of rotator cuff tear or tendonitis.

A 25 year-old male presents to the emergency department for evaluation of a wound on his hand. He cut his hand while tearing down a chicken coop. On examination of his right hand you note a dirty 3cm jagged laceration. The patient is unaware of his tetanus immunization status. Besides cleaning and debriding the wound, what is the recommended clinical intervention in this patient? A. Administer tetanus-diptheria toxoid (Td) and tetanus immune globulin (TIG) B. Administer diphtheria, tetanus, and pertussis vaccine (DTP) C. Administer tetanus toxoid vaccine (TT) and tetanus immune globulin (TIG) D. No tetanus immunization or tetanus immune globulin is necessary in this patient

(c) A. Td or Tdap with TIG is recommended as tetanus prophylaxis in a patient with a contaminated wound and unknown tetanus vaccination status. (u) B. See A for explanation. (u) C. See A for explanation. (u) D. See A for explanation.

A 40 year-old female complains of acute right upper quadrant pain radiating to the back and low grade fever. Laboratory evaluation indicates the presence of urinary bilirubin and an elevation of serum alkaline phosphatase. Which of the following is the most likely diagnosis? A. cholecystitis B. viral hepatitis C. Gilbert's syndrome D. Dubin-Johnson syndrome

(c) A. The presence of urinary bilirubin indicating conjugated hyperbilirubinemia coupled with the elevation of serum alkaline phosphatase suggests biliary obstruction that may lead to cholecystitis. (u) B. Although viral hepatitis presents with conjugated hyperbilirubinemia, aminotransferase elevation would predominate, not alkaline phosphatase. (u) C. Gilbert's syndrome presents as unconjugated hyperbilirubinemia so urinary bilirubin would be absent. (u) D. Although this hereditary disorder presents with conjugated hyperbilirubinemia, liver enzymes would not be elevated.

A 44 year-old female presents for follow-up results of her lipid profile. She is asymptomatic and has a past medical history of hypothyroidism treated with levothyroxine and hypertension controlled with atenolol (Tenormin). She drinks an average of 6 alcoholic beverages a day and smokes 1 pack per day for the last 32 years. Her family history is unremarkable for premature coronary artery disease. Her fasting blood glucose is 98 mg/dL, total cholesterol is 198 mg/dL, LDL cholesterol is 132 mg/dL, HDL cholesterol is 56 mg/dL and triglycerides of 90 mg/dL. Excluding LDL cholesterol levels, how many major risk factors for coronary artery disease does this female possess? A. 2 B. 3 C. 4 D. 5

(c) A. This patient's major cardiac risk factors are smoking and hypertension in addition to the elevated LDL cholesterol. (u) B. See A for explanation. (u) C. See A for explanation. (u) D. See A for explanation.

HIV+ pt presents w/ worsening dementia, fever, HA, & R hemiparesis. Brain MRI reveals 6 lesions throughout the brain that show ring enhancement & surrounding edema. Which is the tx of choice? A. Sulfadiazine and pyrimethamine B. Trimethoprim-sulfamethoxazole C. Radiation therapy D. Ventricular shunt placement

(c) A. Toxoplasmosis is commonly noted in HIV positive patients and presents with multiple ring-enhancing lesions. Treatment of choice for possible toxoplasmosis is sulfadiazine and pyrimethamine. (u) B. Trimethoprim-sulfamethoxazole is used for prophylaxis of toxoplasmosis, but not for treatment of acute infection. (u) C. Radiation therapy is indicated in CNS lymphoma, which typically presents with a single lesion. (u) D. Shunt placement is not indicated in patients with toxoplasmosis.

A 3 year-old girl is diagnosed with atopic dermatitis. Which of the following disorders is this child at risk for in the future? A. Asthma B. Tinea pedis C. Squamous carcinoma D. Systemic lupus erythematosus (SLE)

(c) A. Up to 50% of patients with atopic dermatitis develop asthma and/or allergic rhinitis in the future. (u) B. Patients with atopic dermatitis are more likely to get superimposed viral or bacterial infections such as herpes simplex or staphylococcal, but they are not more at risk for fungal infections. (u) C. Patients with atopic dermatitis are at no greater risk for any skin cancer. (u) D. Lupus is a connective tissue disorder of the immune system, but unrelated to atopic dermatitis.

A 15 year-old girl is hit in the right eye by a golf ball. There is swelling and ecchymosis of the orbit. She complains of double vision. The right eye does not move with downward gaze or right lateral gaze. Which of the following diagnostic tests will provide the most accurate information regarding this injury? A. Facial x-ray B. Schiotz tonometry C. Fluorescein angiography D. Slit lamp biomicroscopy

(c) A. X-ray films may show a blow-out fracture of the orbital floor. Such fractures may lead to oculomotor nerve entrapment or may lead to swelling that impinges on the nerve, causing decreased eye movements. (u) B. Schiotz tonometry is used to evaluate intraocular pressures; elevated pressure indicates glaucoma. (u) C. Fluorescein angiography is used to evaluate retinopathies. (u) D. Slit lamp biomicroscopy is used to evaluate lid contour, lesions, lashes, conjunctiva, sclera, cornea, anterior chamber, and lens.

Pharmacologic treatment of a patient with gestational diabetes should consist of which of the following? A.Oral hypoglycemic agents B. Regular insulin C. Oral corticosteroids D. Glucagon

(h) A. Oral hypoglycemic agents have no role in the treatment of gestational diabetes as these drugs may cross the placenta and harm the fetus. (c) B. Regular insulin is the drug of choice as this will maintain the mother's blood sugar but not cross the placenta. (h) C. Oral corticosteroids have no role in the treatment of gestational diabetes. Corticosteroids will cause the blood glucose to increase. (u) D. Glucagon is given to patients when their blood glucose is abnormally low. Glucagon stimulates gluconeogenesis.

A 43 year-old data entry clerk presents with a one-month history of pain and tingling in the right thumb, index finger, and middle finger. Tinel's sign and Phalen's maneuver are positive. The most appropriate intervention at this time is A. methylprednisolone (Medrol) dose pack. B. splint in neutral position. C. observation. D. surgery.

(u) A. A Medrol dose pack will have no affect on carpal tunnel syndrome. (c) B. Splinting in neutral position relieves impingement of the median nerve, thus improving symptoms of carpal tunnel. (u) C. Observation will not improve symptoms. (u) D. Surgical intervention is reserved for cases unresponsive to conservative therapy.

A 72 year-old farmer comes to the office for evaluation of a pearly ulcerated papule on his right nostril. The papule has been bleeding off and on for the past couple weeks. Which of the following findings would be most concerning on the physical examination of this patient? A. Cherry angioma B. Telangiectasia C. Spider angioma D. Pyogenic granuloma

(u) A. A cherry angioma is a common, violaceous vascular lesion usually located on the trunk. (c) B. Telangiectatic vessels are often visible skin findings with basal cell carcinoma. (u) C. A spider angioma is a benign red papule with radiating telangiectasia usually located on the face, forearms and hands. (u) D. A pyogenic granuloma is a solitary vascular nodule that develops rapidly following minor trauma.

A 23 year-old female with history of asthma for the past 5 years presents with complaints of increasing shortness of breath for 2 days. Her asthma has been well-controlled until 2 days ago. Since yesterday, she has been using her albuterol inhaler every 4 to 6 hours. She is normally very active, however yesterday she did not complete her 30 minute exercise routine due to increasing dyspnea. She denies any cough, fever, recent surgeries, or use of oral contraceptives. On examination, you note the presence of prolonged expiration and diffuse wheezing. The remainder of the exam is unremarkable. Which of the following is the most appropriate initial diagnostic evaluation prior to initiation of treatment? A. Chest x-ray B. Sputum gram stain C. Peak flow D. Ventilation-perfusion scan

(u) A. A chest x-ray should be ordered in an asthmatic patient only if you are concerned about the presence of pneumonia or pneumothorax, neither of which is supported by the H&P findings noted above. (u) B. A sputum gram stain is performed in patients who you suspect have an infectious process, such as pneumonia. (c) C. A peak flow reading will help you to gauge her current extent of airflow obstruction and is helpful in monitoring the effectiveness of any treatment interventions. (u) D. A ventilation-perfusion scan (V/Q scan) is indicated in cases of suspected pulmonary embolism. The patient above does not have any risk factors that would lead you to suspect such a diagnosis.

During an influenza epidemic, a 6 year-old child is seen with fever and a severe sore throat. A throat swab is taken for culture and the child is sent home. The next day, he is reported to have persistent vomiting and increased lethargy. On examination, he is delirious and disoriented. No rash is noted. His reflexes are hyperactive. The liver edge is 3 cm below the right costal margin in the midclavicular line. Which of the following is the most likely diagnosis? A. acute bacterial meningitis B. Guillain Barré syndrome C. Reye syndrome D. measles encephalitis

(u) A. Acute bacterial meningitis presents with fever, malaise, and neck stiffness. The liver is typically normal. (u) B. Guillain Barré syndrome typically presents post-Campylobacter enteritis. Signs and symptoms include an ascending weakness. No hepatomegaly is noted and reflexes are diminished. (c) C. Reye syndrome is typically post-influenza or URI. The patient develops lethargy, drowsiness, and vomiting. Babinski reflex is positive and hyperreflexia is noted. The liver is normal or enlarged. (u) D. Measles encephalitis typically presents one week after the measles rash with ataxia, vomiting, and seizures.

An 85 year-old nursing home resident presents with abrupt onset of cough, sore throat, headache, myalgias, and malaise. On examination the patient's temperature is 102 degrees F; the rest of the exam is unremarkable. Nasal smear is positive for Influenza B. Which of the following is the treatment of choice in this patient? A. Amantadine (Symmetrel) B. Oseltamivir (Tamiflu) C. Acyclovir (Zovirax) D. Nevirapine (Viramune)

(u) A. Amantadine is only used to treat influenza A. (c) B. Oseltamivir is used to treat both influenza A and B. (u) C. Acyclovir is used to treat viral infections due to certain herpes viruses. (u) D. Nevirapine is used to treat infection due to HIV.

A 42 year-old male with unremarkable past medical history is admitted to the general medical ward with community-acquired pneumonia. He has a 20 pack-year history of cigarette smoking. He is empirically started on ceftriaxone (Rocephin). Which of the following antibiotics would be most appropriate to add to his empiric treatment regimen? A. piperacillin (Pipracil) B. vancomycin (Vancocin) C. clindamycin (Cleocin) D. azithromycin (Zithromax)

(u) A. Antipseudomonal penicillins, such as pipercillin, added to an aminoglycoside or fluoroquinolone are indicated for empiric treatment of nosocomial pneumonia. (u) B. Use of vancomycin should be restricted to cases where there is a high index of suspicion of involvement of methicillin resistant Staphylococcus aureus (MRSA). (u) C. Clindamycin is indicated in cases of suspected anaerobe involvement. (c) D. Patients with community-acquired pneumonia who require hospitalization on the general medical ward are treated with an extended-spectrum beta-lactam antibiotic, such as ceftriaxone, with a macrolide, such as azithromycin. Addition of a macrolide is also recommended due to the patient's smoking history and possible involvement of Hemophilus influenzae.

During physical examination an elderly patient is noted to have a painless, brown-colored ulceration in the area of the medial malleolus. Which of the following is the most likely diagnosis? A. Arterial ulcer B. Venous ulcer C. Arterial insufficiency D. Diabetic ulcer

(u) A. Arterial ulcers typically are the last in the sequence of events of peripheral arterial disease; which include decreased or absent pulses distal to the blockage, muscle atrophy, hair loss, thickened nails, smooth and shiny skin, reduced skin temperature, pallor, cyanosis, ulcers, and gangrene. Arterial ulcers typically occur on the feet in the areas of pressure points. (c) B. Venous ulceration develops in the lower extremity secondary to venous incompetence and chronic edema. The medial aspect of the ankle is the most common location. (u) C. Arterial insufficiency is most likely to present with symptoms of claudication prior to the development of skin ulcers. When these ulcers do occur, they are most commonly seen as arterial ulcers and not venous ulcers. (u) D. Diabetic ulcers typically occur due to atherosclerosis and arterial insufficiency along with diabetic peripheral neuropathy. These ulcers are more likely to occur on pressure points on the foot and fail to heal because of poor circulation to these areas.

(u) A. Inflammation in atopic dermatitis results in lichenification of the skin from itching, which leaves the skin dry and scaly when it resolves. The inflammation is the result of elevated T-lymphocyte activation, defective cellmediated immunity, and IgE overproduction. Epidermal hyperplasia is not involved in atopic dermatitis. (u) B. Tinea corporis is a fungal infection and does not have epidermal hyperplasia. (u) C. Ecythma is the result of an infection from group A beta-hemolytic streptococcus and contaminated with staphylococci. Like atopic dermatitis and tinea corporis, ecthyma has no epidermal hyperplasia. (c) D. Psoriasis is characterized by an increased epidermal cell turnover, increased numbers of epidermal stem cells, and an abnormal differentiation of keratin. This leads to the classic scale associated with psoriasis.

(u) A. Asthma is characterized by decreased tactile fremitus, but would have resonant to hyperresonant percussion, not dullness. (u) B. Lobar pneumonia is characterized by dullness to percussion, but would have an increased, not decreased, tactile fremitus. (u) C. A pneumothorax is characterized by decreased to absent tactile fremitus, but would have a hyperresonant percussion note, not dullness. (c) D. Decreased tactile fremitus and dullness to percussion would be found in a pleural effusion.

A 43 year-old woman presents with episodes of facial flushing with increased skin temperature followed by the development of tiny papules and pustules primarily on her cheeks and chin. These symptoms worsen when she eats spicy foods. On examination, telangiectasia are noted along the nasal folds and tiny pustules on both cheeks. Which of the following is the most likely diagnosis? A. Atopic dermatitis B. Rosacea C. Lupus D. Acne vulgaris

(u) A. Atopic dermatitis is characterized by chronic dry skin, and pruritic inflammatory lesions of the flexor surfaces of the body. Repeated rubbing or scratching of the skin causes excoriations and lichenification. Atopic dermatitis is often associated with a personal or family history of allergic rhinitis or asthma. (c) B. Rosacea is chronic, episodic flushing of the face followed by the development of tiny papules and pustules. Eating spicy foods or drinking hot liquids or alcohol can cause exacerbation of the symptoms. (u) C. The rash associated with lupus is commonly found on the cheeks and not on the chin. There are not typically pustules or telangiectasia noted. (u) D. Acne vulgaris does not present with facial flushing and would not be triggered by the consumption of spicy foods.

Hypoglycemia in an elderly patient with diabetes is most likely to manifest as which of the following symptoms? A. Tachycardia B. Diaphoresis C. Intense hunger D. Mental confusion

(u) A. Autonomic responses such as tachycardia, diaphoresis, and hunger are often blunted in the elderly diabetic. (u) B. See A for explanation. (u) C. See A for explanation. (c) D. Manifestations of hypoglycemia in the elderly are mainly from impaired central nervous system function. Manifestations include mental confusion, bizarre behavior, and ultimately coma.

A 5 year-old male presents with a history of recurrent episodes of acute bronchitis, characterized by fever and productive cough. He has no known significant past medical history. His pulmonary examination reveals crackles in the bilateral lower lobes. The remainder of his physical examination is normal. Chest x-ray demonstrates platelike atelectasis and dilated, thickened airways in the middle and lower lungs. Which of the following is the most likely diagnosis? A. Acute bronchitis B. Bronchiectasis C. Pneumonia D. Tuberculosis

(u) A. Barring underlying pulmonary pathology, the chest x-ray in acute bronchitis should be normal. (c) B. Bronchiectasis typically presents as recurrent episodes of acute bronchitis. Platelike atelectasis and dilated and thickened airways, sometimes described as tram lines, are common radiographic findings. (u) C. While the history may suggest pneumonia, the radiographic findings do not support this diagnosis. (u) D. Tuberculosis would present with cavitating granuloma formation more commonly at the apices.

The most reliable sign of acute otitis media (AOM) is A. bulging of the tympanic membrane. B. loss of tympanic membrane mobility. C. reddening of the tympanic membrane. D. air bubbles behind the tympanic membrane.

(u) A. Bulging and air bubbles behind the TM represent OM with effusion. (c) B. Loss of tympanic membrane mobility during pneumoinsufflation is the most reliable sign for diagnosing acute otitis media. (u) C. Reddening of the eardrum is not reliable as it may be due to crying or other vascular changes. (u) D. See A for explanation.

Which of the following medications is most useful in maintaining remission in a patient with ulcerative colitis? A. Oral corticosteroids B. Corticosteroid enemas C. Sulfasalazine D. Macrodantin

(u) A. Corticosteroids, given orally or via enema preparations, are used for acute flare-up of ulcerative colitis but are not effective when given as prophylactic agents. (u) B. See A for explanation. (c) C. Sulfasalazine, olsalazine, and mesalamine are effective in maintaining remission in patients with ulcerative colitis. (u) D. Long-term antibiotic therapy with Macrodantin is used as a prophylactic agent to prevent urinary tract infections and has no role in the treatment or prevention of ulcerative colitis.

A 4 year-old boy presents with pain and irritation of his left ear. Otoscopic examination reveals an insect in the left auditory canal. The tympanic membrane is not completely visualized. Which of the following is the most appropriate management of this patient? A. Debrox insertion with suction removal B. Irrigation with room temperature saline C. Insertion of 2% lidocaine solution with suction or forceps removal D. Polymyxin drop insertion via wick

(u) A. Debrox is used for cerumen impaction not foreign body removal. (u) B. Irrigation with room temperature saline is useful for small particle removal only if the tympanic membrane is well-visualized and without perforation. It is not indicated in the removal of an insect. (c) C. Two percent lidocaine solution will paralyze the insect and provide topical anesthesia for suction or forceps removal. (u) D. Polymyxin B is indicated in otitis externa and administered via a wick when there is significant edema of the auditory canal. It is not indicated in the removal of a foreign body.

6 y/o male presents w/ hemarthrosis of the L knee. Coag studies: PT 12.5s (normal12-14 sec), INR 1.0, aPTT 58s (normal 18-28 sec), platelet 430,000/microliter (normal 150,000-450,000/microliter), & bleeding time 4m (normal 2-12m). Which is the best tx option for this pt? A. Desmopressin acetate B. Corticosteroids C. Vitamin K D. Cryoprecipitate

(u) A. Desmopressin acetate is indicated in von Willebrand's disease, which presents with a prolonged bleeding time. (u) B. Corticosteroids are indicated in immune-mediated thrombocytopenia. (u) C. Vitamin K deficiency will prolong the PT greater than the aPTT. Vitamin K supplement is not indicated in this patient. (c) D.Hemophilia A presents with a prolonged aPTT and normal platelet count and function. Hemophilia A is treated with factor VIII concentrate or cryoprecipitate.

Which of the following is a common symptom associated with laryngotracheobronchitis (viral croup)? A. Drooling B. High fever C. "Hot potato" voice D. Barking cough

(u) A. Drooling and a "hot potato" voice are seen with epiglottitis, not viral croup. (u) B. Fever is usually absent or low-grade in patients with viral croup. (u) C. See A for explanation. (c) D. Viral croup is characterized by a history of upper respiratory tract symptoms followed by onset of a barking cough and stridor.

Which of the following clinical manifestations is most commonly seen in viral croup? A. drooling B. wheezing C. sputum production D. inspiratory stridor

(u) A. Drooling is common in epiglottitis not viral croup. (u) B. Wheezing is noted in asthma. (u) C. Sputum production is noted in bacterial infections. (c) D. Viral croup typically presents with a barking cough and stridor.

A 37 year-old male presents with headaches for the past 2 months. They occur daily and are worse in the morning. In the past week, he has noticed a tendency to drop things from his right hand. On examination, vital signs are normal, and general examination is unremarkable. Neurologic examination reveals mild weakness of the right upper extremity compared to the left. Which of the following is the most likely diagnosis? A. focal seizure disorder B. intracerebral neoplasm C. transient ischemic attack D. amyotrophic lateral sclerosis

(u) A. Focal seizure disorder is not associated with morning headaches. (c) B. Intracerebral neoplasms may present with headaches that are worse in the morning, with improvement during the day. Focal motor or sensory loss depends upon the tumor's location. (u) C. A TIA is characterized by focal findings that resolve completely and spontaneously within 24 hours. (u) D. Amyotrophic lateral sclerosis is a degenerative disorder characterized by motor weakness, but is not associated with morning headaches.

A patient presents with acute eye pain and photophobia after putting in his contact lens. The patient states he removed his contact which he noted to be torn. Which of the following is the diagnostic study of choice in this patient? A. Funduscopic examination B. Applanation tonometry C. Fluorescein staining D. Schirmer Test

(u) A. Funduscopic examination allows for evaluation of the optic disc and retinal circulation. It is not the best diagnostic study for a patient with suspected corneal abrasion. (u) B. Applanation tonometry is used in the evaluation of intraocular pressure in patients with suspected glaucoma. (c) C. Fluorescein staining is done in patients to evaluate for corneal abrasion which commonly presents with severe eye pain and photophobia. (u) D. The Schirmer test is used to evaluate the quantity of tears produced in patient with conditions such as keratoconjunctivitis sicca.

Which of the following laboratory abnormalities is most commonly noted in bulimia nervosa? A. glycosuria B. hypokalemia C. metabolic acidosis D. hyperalbuminemia

(u) A. Glycosuria is associated with diabetes mellitus, not bulimia. (c) B. Episodes of binge eating are followed by purging in the bulimic patient. Vomiting and laxative abuse are the most common methods of purging, leading to hypokalemia. (u) C. A metabolic alkalosis may be noted if potassium losses from purging are great enough. (u) D. Serum albumin levels may be normal or decreased.

Which of the following leukemias is most closely associated with the Philadelphia chromosome? A. hairy cell B. acute lymphocytic C. acute myelocytic D. chronic myelocytic

(u) A. Hairy cell leukemia is a rare lymphocytic leukemia. See D for explanation. (u) B. See D for explanation. (u) C. See D for explanation. (c) D. Philadelphia chromosome is noted most commonly in patients with chronic myelogenous leukemia.

On examination of a pregnant patient the physician assistant notes a bluish or purplish discoloration of the vagina and cervix. This is called A. Hegar's sign. B. McDonald's sign. C. Cullen's sign D. Chadwick's sign

(u) A. Hegar's sign is the softening of the cervix that often occurs with pregnancy. (u) B. McDonald's sign is when the uterus becomes flexible at the uterocervical junction at 7-8 weeks. (u) C. Cullen's sign is a purplish discoloration periumbilical and noted in pancreatitis. (c) D. Chadwick's sign is a bluish or purplish discoloration of the vagina and cervix.

A 23 year-old female complains of fever and genital pain. Examination reveals the presence of lymphadenopathy in the groin and the presence of vulvar vesicles surrounded by erythematous skin. The diagnosis may be confirmed by A. A culture of a vesicle using blood agar medium. B. The presence of similar lesions on the fingers and hands. C. A Gram stain of a scraping from the base of the lesion. D. The presence of giant multinucleated cells on Tzanck smear.

(u) A. Herpes virus will not grow on blood agar medium. (u) B. Herpes genitalis occurs in skin that comes into contact with the herpes virus. Usually the infection is localized. (u) C. A Gram stain will not identify the herpes virus. (c) D. A Tzanck smear is the standard laboratory test to confirm the herpes virus as an etiologic agent of a vesicular lesion on an erythematous base.

A 20 year-old male presents with complaint of brief episodes of rapid heart beat with a sudden onset and offset that have increased in frequency. He admits to associated shortness of breath and lightheadedness. He denies syncope. Electrocardiogram reveals a delta wave prominent in lead II. Which of the following is the most appropriate long-term management in this patient? A. Implantable cardio defibrillator B. Radiofrequency ablation C. Verapamil (Calan) D. Metoprolol (Lopressor)

(u) A. Implantable cardio defibrillators are indicated in the treatment of ventricular arrhythmias, not Wolf-Parkinson-White (WPW) syndrome. (c) B. Radiofrequency ablation is the procedure of choice for long-term management in patients with accessory pathways (WPW) and recurrent symptoms. (u) C. Calcium channel blockers and beta-blockers are not the best options for the long-term management of WPW. They may decrease the refractoriness of the accessory pathway or increase the refractoriness of the AV node in patients with atrial fibrillation or atrial flutter who have an antegrade conducting bypass tract. This may lead to faster ventricular rates. (u) D. See C for explanation.

Otitis externa can be differentiated from uncomplicated otitis media by which of the following physical examination findings? A. Erythematous tympanic membrane B. Mastoid tenderness C. Normal pneumatic otoscopy D. Posterior auricular adenopathy

(u) A. In otitis externa, the erythema is because the lateral surface of the tympanic membrane is ear canal skin, and therefore is often erythematous. (u) B. Mastoid tenderness is not directly associated with otitis media or otitis externa. (c) C. In contrast to acute otitis media, the tympanic membrane moves normally with pneumatic otoscopy in a patient with otitis externa. (u) D. Both otitis media and otitis externa may have posterior auricular adenopathy on examination.

7. Which of the following is a cause of prerenal azotemia? A. Infection B. Renal toxins C. Poor renal perfusion D. Urinary tract obstruction

(u) A. Infection is associated with interstitial nephritis, which is considered a cause of intrinsic renal azotemia. (u) B. This is one of the causes of intrinsic renal azotemia. (c) C. Renal hypoperfusion is the cause of prerenal azotemia, which may be rapidly reversible when renal blood flow and glomerular ultrafiltration pressure are restored. (u) D. Urinary tract obstruction is the cause of postrenal azotemia.

Which of the following is characterized by epidermal hyperplasia and an increase in the epidermal turnover? A. Atopic dermatitis B. Tinea corporis C. Ecthyma D. Psoriasis

(u) A. Inflammation in atopic dermatitis results in lichenification of the skin from itching, which leaves the skin dry and scaly when it resolves. The inflammation is the result of elevated T-lymphocyte activation, defective cellmediated immunity, and IgE overproduction. Epidermal hyperplasia is not involved in atopic dermatitis. (u) B. Tinea corporis is a fungal infection and does not have epidermal hyperplasia. (u) C. Ecythma is the result of an infection from group A beta-hemolytic streptococcus and contaminated with staphylococci. Like atopic dermatitis and tinea corporis, ecthyma has no epidermal hyperplasia. (c) D. Psoriasis is characterized by an increased epidermal cell turnover, increased numbers of epidermal stem cells, and an abnormal differentiation of keratin. This leads to the classic scale associated with psoriasis.

A 65 year-old male presents with multiple lesions on his back. He denies any pruritis. Physical examination reveals the presence of multiple scattered brown plaques with a raised, warty surface that appear to be stuck onto the skin and feel greasy. Which of the following is the most likely diagnosis? A. lentigines B. actinic keratosis C. keratoacanthomas D. seborrheic keratosis

(u) A. Lentigines most commonly are seen on the dorsum of the hand and appear as flat brown spots, often with sharp borders. (u) B. Actinic keratosis usually present as small patches of flesh-colored, pink or yellow-brown lesions often with an erythematous component. The lesions are better felt than seen, having a rough, sandpaper feel and are often tender to palpation. (u) C. Keratoacanthomas usually occur as an isolated lesion on the face appearing as an erythematous, dome-shaped nodule with a central keratinaceous plug. (c) D. Seborrheic keratosis is a common benign plaque in the elderly that characteristically has a velvety or warty surface associated with a stuck on appearance and greasy feel.

Which of the following is a cause of high output heart failure? A. myocardial ischemia B. complete heart block C. aortic stenosis D. thyrotoxicosis

(u) A. Low output heart failure occurs secondary to ischemic heart disease, hypertension, dilated cardiomyopathy, valvular and pericardial disease, and arrhythmia. (u) B. See A for explanation. (u) C. See A for explanation. (c) D. High output heart failure occurs in patients with reduced systemic vascular resistance. Examples include: thyrotoxicosis, anemia, pregnancy, beriberi and Paget's disease. Patients with high output heart failure usually have normal pump function, but it is not adequate to meet the high metabolic demands.

A 25 year-old female presents for a routine gynecological examination. You palpate a 2 cm breast mass in her right breast. Her menstrual period was last week. She has no family history of breast cancer. What is the modality of choice to further evaluate her breast mass? A. Magnetic resonance imaging (MRI) B. Excisional biopsy C. Ultrasound D. Mammography

(u) A. MRI's of the breast are done in patients with prior breast cancer or who have BRCA 1 or 2 positivity. (u) B. Excisional biopsy is not indicated without differentiation of lesion by ultrasound. (c) C. In a 25 year-old an ultrasound is the best choice because of the density of the breast tissue in young women. (u) D. Mammographies are not recommended in women prior to the age of 35 without family history of breast cancer or BRCA positive.

A 33 year-old male presents to your office with a complaint of right knee injury associated with pain and swelling. He states he was running after his loose dog and suddenly stopped, hyperextended his knee, heard a pop and noticed immediate swelling. On physical examination, the Lachman test and anterior drawer test demonstrates joint laxity. Which of the following ligaments is most likely injured? A. Medial collateral B. Lateral collateral C. Posterior cruciate D. Anterior cruciate

(u) A. Medial collateral ligament injuries often occur with rotational injuries or direct impact to the lateral knee. Tenderness medially with laxity with valgus (medial) stress is noted. (u) B. Lateral collateral ligament injury causes pain mostly on the lateral aspect of the knee and patients can experience knee buckling with normal gait. Tenderness laterally with laxity with varus (lateral) stress is noted. (u) C. Posterior cruciate ligament injuries occur with an outside directed force, often a posterior directed force such as a knee striking a dashboard. The patients often do not hear a pop. A posterior drawer test or posterior sag test can be useful in the diagnosis. (c) D. Anterior cruciate ligament injuries occur with sudden deceleration injuries. Patients often hear a pop and the diagnosis is aided by assessing the anterior drawer test and Lachman test. The immediate swelling as well as laxity with anterior drawer test and Lachman test should raise suspicion of anterior cruciate ligamental injury.

Long term use of which of the following drugs may cause a drug-induced lupus-type eruption? A. prednisone B. tetracycline C. procainamide D. oral contraceptives

(u) A. Prednisone is not implicated in drug-induced skin reactions. (u) B. Tetracycline and sulfonamides are known to cause a photosensitive rash on sun exposed areas of the skin. (c) C. Procainamide and hydralazine are the most common drugs that may cause a lupus-like eruption. (u) D. Oral contraceptives may induce erythema nodosum.

Progesterone influence on the breast tissue prior to menstruation causes A. proliferation of the mammary ducts. B. growth of the lobules and alveoli. C. proliferation of Cooper's ligaments. D. increase in the number of glands of Montgomery.

(u) A. Proliferation of the mammary ducts is under the influence of estrogen. (c) B. Growth of the lobules and alveoli is under the influence of progesterone. Prior to menses, the breast swelling that women notice is a result of the progesterone which is secreted from the corpus luteum. During menses, the swelling subsides. (u) C. See B for explanation. (u) D. See B for explanation.

A 25 year-old female presents with a three day history of chest pain aggravated by coughing and relieved by sitting and leaning forward. She is febrile and a CBC with differential reveals leukocytosis. Which of the following physical examination signs is characteristic of her problem? A. Pulsus paradoxus B. Localized crackles C. Pericardial friction rub D. Wheezing

(u) A. Pulsus paradoxus is a classic finding for cardiac tamponade. (u) B. Localized crackles are associated with pneumonia and consolidation, not pericarditis. (c) C. Pericardial friction rub is characteristic of an inflammatory pericarditis. (u) D. Wheezing is characteristic for pulmonary disorders, such as asthma.

Which of the following treatment strategies is most likely to promote optimal health and limit the morbidity and mortality associated with rheumatoid arthritis? A. meningococcal vaccination B. NSAIDs C. methotrexate D. corticosteroids

(u) A. RA patients are at increased risk of infection and should receive influenza and pneumococcal vaccines, not meningococcal. (u) B. NSAIDs provide only symptomatic therapy and do not stop the disease progression of RA. (c) C. Methotrexate is the most commonly prescribed DMARD for RA and has been proven to induce remission in some patients. (u) D. Chronic corticosteroid use is associated with greater toxicity than DMARDs and should be used only for symptomatic therapy.

Which of the following antiarrhythmic drugs can be associated with hyper- or hypothyroidism following long-term use? A. Quinidine B. Amiodarone C. Digoxin D. Verapamil

(u) A. See B for explanation. (c) B. Amiodarone is structurally related to thyroxine and contains iodine, which can induce a hyper- or hypothyroid state. (u) C. See B for explanation. (u) D. See B for explanation.

Which of the following is the treatment of choice for stage one Lyme disease in a patient less than 12 years of age? A. doxycycline B. amoxicillin C. chloramphenicol D. azithromycin

(u) A. See B for explanation. (c) B. Amoxicillin is first line therapy in a patient less than 12 years of age due to the harmful effects of doxycycline on teeth and bones in children. (u) C. Chloramphenicol is only indicated in disseminated disease with known drug resistance. It is not indicated as first line therapy due to its tendency to cause aplastic anemia. (u) D. Azithromycin is not as effective as doxycycline or amoxicillin.

A 56 year-old male with a known history of polycythemia suddenly complains of pain and paresthesia in the left leg. Physical examination reveals the left leg to be cool to the touch and the toes to be cyanotic. The popliteal pulse is absent by palpation and Doppler. The femoral pulse is absent by palpation but weak with Doppler. The right leg and upper extremities have 2+/4+ pulses throughout. Given these findings what is the most likely diagnosis? A. Venous thrombosis B. Arterial thrombosis C. Thromboangiitis obliterans D. Thrombophlebitis

(u) A. See B for explanation. (c) B. Arterial thrombosis has occurred and is evidenced by the loss of the popliteal and dorsalis pedis pulse. This is a surgical emergency. Venous occlusion and thrombophlebitis do not result in loss of arterial pulse. (u) C. See B for explanation. (u) D. See B for explanation.

On examination of a pregnant patient the physician assistant notes the fundal height is at the level of the umbilicus. This corresponds to what gestational age? A. 16 weeks B. 20 weeks C. 24 weeks D. 28 weeks

(u) A. See B for explanation. (c) B. At 20-22 weeks, the fundal height is typically at the level of the umbilicus. (u) C. See B for explanation. (u) D. See B for explanation.

A newborn weighs 8 pounds at birth. Following established growth indices, what would you expect the infant to weigh at 1 year of age? A. 16 pounds B. 24 pounds C. 32 pounds D. 40 pounds

(u) A. See B for explanation. (c) B. Most infants should be expected to triple their birth weight by 1 year of age. (u) C. See B for explanation. (u) D. See B for explanation

A 36 year-old man has a 30 pack-year history of smoking cigarettes and wants to quit. He is otherwise healthy at this time. Which of the following drugs would be appropriate for him? A. Amitriptyline (Elavil) B. Bupropion (Wellbutrin) C. Fluoxetine (Prozac) D. Venlafaxine (Effexor)

(u) A. See B for explanation. (c) B. The only two approved drugs for aiding smoking cessation are nicotine and bupropion. (u) C. See B for explanation. (u) D. See B for explanation.

A 3 year-old boy is seen in the office with a 5-day history of fever, erythema, edema of the hands and feet, a generalized rash over the body, bilateral conjunctival injections, fissuring and erythema of the lips, and cervical adenopathy. Antistreptolysin A (ASO) titer and throat culture are negative. The most serious systemic complication associated with this disorder is A. renal. B. cardiac. C. pulmonary. D. hepatic.

(u) A. See B for explanation. (c) B. The patient most likely has Kawasaki syndrome. The major complication with this disorder is coronary artery aneurysms, which are reported in up to 20% of affected children. The etiology of this disorder is uncertain, although a bacterial toxin with super antigen properties may be involved. (u) C. See B for explanation. (u) D. Children with Kawasaki syndrome may have associated hydrops of the gallbladder, but liver involvement is not part of this disorder.

A 36 year-old female presents for a routine physical. She has no current complaints and her only medication is oral contraceptives. The patient is preparing for a trip to Australia and is worried about the long flight as her mom has a history of deep vein thrombosis after a long trip several years ago. Physical examination reveals BP 110/60 mmHg, HR 66 bpm, regular. Heart is regular rate and rhythm without murmur, lungs are clear to auscultation bilaterally and extremities are without edema. Which of the following is the most appropriate recommendation for your patient? A. Discontinue oral contraceptives B. Recommend walking frequently during the flight C. Begin daily aspirin therapy D. Increase fluid intake 2-3 days prior to the flight

(u) A. See B for explanation. (c) B. The risk of deep vein thrombosis after air travel increases with flight duration. Preventive measures for patients include using support hose and performing in-flight exercises and walking. (u) C. See B for explanation. (u) D. See B for explanation.

A positive direct Coombs' test may be seen in which of the following conditions? A. G6PD deficiency B. Sickle cell anemia C. Hereditary spherocytosis D. Autoimmune hemolytic anemia

(u) A. See D for explanation. (u) B. See D for explanation. (u) C. See D for explanation. (c) D. A positive direct Coombs' test indicates that antibody has attached to an antigen on the RBC which causes agglutination. Autoimmune hemolytic anemia results when the patient has antibodies against their own RBCs. G6PD deficiency does result from an antibody antigen reaction. Sickle cell anemia is caused by a defective hemoglobin which detected by hemoglobin electrophoresis. Hereditary spherocytosis is detected utilizing the osmotic fragility test.

A 48 year-old male with diabetes mellitus presents for routine physical examination. Of note his blood pressure each of his last two follow-up visits was 150/90 mmHg. Today the patient's BP is 148/88 mmHg. The patient denies complaints of chest pain, change in vision, or headache. Which of the following is the most appropriate management for this patient? A. Atenolol (Tenormin) B. Nifedipine (Procardia) C. Hydralazine (Apresoline) D. Lisinopril (Zestril)

(u) A. See D for explanation. (u) B. See D for explanation. (u) C. See D for explanation. (c) D. ACE inhibitors are the first line treatment of choice in a patient with hypertension and diabetes.

A 75 year-old woman presents to the office with complaint of vision loss. Examination reveals a palpable cord in the temporal region. Which of the following is the most helpful initial test to order on this patient? A. Carotid ultrasound B. Chest x-ray C. Complete blood count D. Erythrocyte sedimentation rate

(u) A. See D for explanation. (u) B. See D for explanation. (u) C. See D for explanation. (c) D. The patient is suspected of having temporal arteritis. This disease is most commonly noted in patients over age 50 and should be suspected in patients with sudden vision loss and a palpable cord in the temporal region. Erythrocyte sedimentation rate is almost always increased in this disease.

A 62 year-old male smoker presents to the clinic with the complaint of a chronic cough, hemoptysis, and weight loss. Chest CT shows a mass obstructing the bronchus with hilar and mediastinal lymph node abnormalities. Bronchoscopy with biopsy is performed. On reviewing pathology results you explain to the patient that his type of lung cancer is prone to early hematogenous spread, is rarely amenable to surgical resection and has a very aggressive course. What type of lung cancer is most likely in this patient? A. Squamous cell B. Small cell C. Large cell D. Adenocarcinoma

(u) A. Squamous cell carcinoma, large cell carcinoma, and adenocarcinoma spread more slowly and have the possibility of cure in early stages following resection and chemotherapy. (c) B. Small cell lung cancer is very aggressive with a median survival (untreated) of 6-18 weeks. (u) C. See A for explanation. (u) D. See A for explanation.

A patient presents with chronic back pain. On physical examination testing, the patient is found to have abnormalities of proprioception and vibration discrimination. Which of the following portions of the spinal column are most likely affected? A. Lateral spinothalamic tract B. Ventral spinothalamic tract C. Posterior column D. Transection of the cord

(u) A. The lateral spinothalamic tract affects pain and temperature sensation. (u) B. The ventral spinothalamic tract affects pressures and touch sensations. (c) C. The posterior column affects proprioception (position sense) and vibration sense. (u) D. Patients with transection of the cord will have loss of sensation distal to the area of injury along with paralysis and hyperactive reflexes in the area distal to the transection.

A 3 month-old female presents with her mom for physical examination. The patient's mom denies any complaints. On examination you note a well-developed, well-nourished infant in no apparent distress. There is no cyanosis noted. Heart examination reveals a normal S1 with a physiologically split S2. There is a grade III/VI high-pitched, harsh, pansystolic murmur heard best at the 3rd and 4th left intercostal spaces with radiation across the precordium. Which of the following is the initial diagnostic study of choice in this patient? A. CT angiogram B. Electrocardiogram C. Echocardiogram D. Cardiac catheterization

(u) A. This patient has signs and symptoms consistent with a ventricular septal defect (VSD). CT angiogram and electrocardiogram are not indicated in establishing the diagnosis of a VSD. (u) B. See A for explanation. (c) C. Echocardiogram is the initial diagnostic study of choice in the diagnosis of a VSD. (u) D. Cardiac catheterization may be necessary to accurately measure pulmonary pressures or if a VSD can not be well localized on echocardiogram, but it is not the initial diagnostic study of choice.

A 53 year-old male with history of hypertension presents complaining of recent 4/10 left-sided chest pain with exertion that is relieved with rest. He states the pain usually lasts approximately 4 minutes and is relieved with rest. Heart examination reveals regular rate and rhythm with no S3, S4, or murmur. Lungs are clear to auscultation bilaterally. Electrocardiogram reveals no acute changes. Which of the following is the most appropriate initial step in the evaluation of this patient? A. Cardiac catheterization B. CT Angiogram of the chest C. Echocardiogram D. Nuclear stress test

(u) A. This patient has signs and symptoms consistent with stable angina. Noninvasive diagnostic testing is preferred in this patient. (u) B. CT angiogram may be useful for the evaluation of chest pain, however its role in routine practice has not been established. (u) C. This patient has signs and symptoms of stable angina. There are no signs of valvular heart disease on examination. While an echocardiogram may be performed at some point, it is not the best initial diagnostic step to determine the etiology of the patient's angina. (c) D. Nuclear stress testing is the most appropriate initial diagnostic study in the evaluation of a patient with signs and symptoms consistent with stable angina.

A 68 year-old male with a history of atrial fibrillation treated with warfarin (Coumadin) presents to the emergency department after vomiting large amounts of bright red blood. INR is 3. Which of the following is most appropriate to rapidly lower the patient's INR? A. Discontinue warfarin B. Administer fresh frozen plasma C. Administer protamine sulfate D. Administer heparin sulphate

(u) A. This will not immediately reverse the effects of warfarin. (c) B. Fresh frozen plasma is the most rapid way to lower the patient's INR. (u) C. Protamine sulfate is used to neutralize heparin sulphate not warfarin. (h) D. Heparin administration would increase the bleeding and be harmful.

A 22 year-old patient complains of sudden onset of chest pain accompanied by shortness of breath. The patient appears dyspneic. On examination, the trachea is deviated to the left, breath sounds are faint on the right, and the right chest is hyperresonant to percussion. The preferable treatment for this patient would be A. a tracheostomy. B. insertion of a chest tube with underwater seal, left 2nd intercostal space. C. needle thoracotomy right 2nd intercostal space D. a lung scan for pulmonary embolus and begin heparin sodium (Heparin) therapy.

(u) A. Tracheostomy is indicated for upper airway obstruction, not spontaneous pneumothorax. (h) B. This patient has a tension pneumothorax on the right, and insertion of the chest tube on the left would be life threatening. (c) C. Decreased breath sounds and hyperresonance are noted on the side of the pneumothorax; tracheal deviation to the opposite side indicates development of a tension pneumothorax. Treatment consists of inserting a chest tube on the side of the pneumothorax and connecting to an underwater seal. (u) D. Pulmonary embolus may present with chest pain and dyspnea, but physical examination findings do not include decreased breath sounds with hyperresonance and tracheal deviation.

A 65 year-old patient with steroid-dependent chronic obstructive lung disease presents with a headache that has been increasing in severity over the past week, accompanied by nausea and vomiting. He denies fever, but has had photophobia and a stiff neck. Which of the following is the most likely diagnosis? A. Transient ischemic attack B. Bacterial meningitis C. Migraine headache D. Cryptococcosis

(u) A. Transient ischemic attacks present with focal neurological findings rather than headaches. (a) B. Bacterial meningitis is typically acute in onset and causes fever, but immunocompromised patients may have a slower onset and no fever. (u) C. Migraines generally do not begin in this age group, and are not accompanied by nuchal rigidity. (c) D.Cryptococcus is an opportunistic fungal infection that affects immunocompromised patients, including those with HIV, chronic steroid use, organ transplants, diabetes mellitus, and chronic renal or liver disease. The most common clinical presentation is that of meningitis; fever is present in only about half of patients.

You are called to the nursery to see a male infant, born by uncomplicated vaginal delivery. He weighs 2,600 grams and has one deep crease on the anterior third of each foot. Respirations are 88 breaths/minute with expiratory grunting and intercostals retractions. He is cyanotic on room air and becomes pink when placed on 60% oxygen. Chest x-ray shows atelectasis with air bronchograms. Which of the following is the most likely diagnosis? A. neonatal pneumonia B. congenital heart disease C. hyaline membrane disease D. chronic lung disease of prematurity

(u) A. While tachypnea, grunting, retractions and cyanosis may be signs of neonatal pneumonia, they are primarily late findings of progressive respiratory distress and would not be seen immediately at the time of delivery. A chest x-ray in pneumonia would also most commonly reveal an infiltrate or effusion. (u) B. While congenital heart disease may present with cyanosis, the chest x-ray will reveal a cardiac abnormality, such as cardiomegaly. (c) C. Hyaline membrane disease is the most common cause of respiratory distress in the premature infant. The infant typically presents with tachypnea, cyanosis and expiratory grunting. A chest x-ray reveals hypoexpansion and air bronchograms. (u) D. Chronic lung disease of prematurity is a complication in about 20% of infants with hyaline membrane disease. It is defined as respiratory symptoms, oxygen requirement and chest x-ray abnormalities at 1 month of age so it cannot be diagnosed at this time in this newborn.


Conjuntos de estudio relacionados

463 Part II of Semester Material

View Set

Chapter 10 - Warehousing management

View Set

Abdominal and Musculoskeletal Assessment

View Set

Human Sexuality: Chapter 6 Conception, Pregnancy, & Childbirth

View Set

INTERNATIONAL BUSINESS LAW AND ITS ENVIRONMENT section 1

View Set